General Awareness Banking PDF

You might also like

Download as pdf or txt
Download as pdf or txt
You are on page 1of 195

www.mahendras.

org

Quantitative
Aptitude
SALIENT FEATURES:

## Chapterwise Discussion
## Topicwise Rules and Examples
## Exercise based on previous papers.
## Easy and comprehensible language

Mahendra Publication Pvt. Ltd.


New Delhi

QUANTITATIVE APTITUDE 3
www.mahendras.org

Mahendra Publication Pvt. Ltd.

© Copyright Reserved
##No part of this issue can be printed in whole or in part without the written permission of the
publishers.
##All the disputes are subject to Delhi jurisdiction only.

Registered Office
Mahendra Publication Pvt. Ltd., 103, Pragatideep Building, Plot No. 08, Laxminagar,
District Centre, New Delhi - 110092, TIN-09350038898, w.e.f. 12-06-2014

Branch Office
Mahendra Publication Pvt. Ltd. E-42,43,44, Sector-7, Noida (U.P.)

For queries regarding promotion, distribution & advertisement, contact:-


E-mail-sales@mahendras.org
Ph.: 09208037962

Owned, printed & published by N.K. Jain


103, Pragatideep Building, Plot No. 08, Laxminagar, District Centre, New Delhi - 110092

Writer
Mahendra Publication Team

Please send your suggestions and grievances to:-


Mahendra Publication Pvt. Ltd., CP-9, Vijayant Khand, Gomti Nagar Lucknow - 226010
E-mail:info@mahendras.org

4 QUANTITATIVE APTITUDE
www.mahendras.org

PREFACE
This gives us immense pleasure to present you the latest edition of this book. We thank you from the depth of our
hearts, for the love and affection given by you just from the beginning. Dear friends, change is the law of nature.
We must learn to tolerate the blows of time with patience and learn not only to endure, but also to expect, welcome,
and enjoy both the joys as well as the sorrows of life. We must do what we can to "get success." We have to operate
with the information and skills that are necessary for winning.
Today’s era is governed by technology. The technology has increased the pace of changing the world we see day by
day, and so the pattern of examination and criteria of selection has also changed. As we are aware that interview is
no more in various examinations so the written/objective exams are going to be tougher than earlier.
Every choice you make — including the thoughts you have — has consequences. When you start choosing the right
behaviour and thoughts — which will take a lot of discipline — you'll get the right outcomes.
Be aware of the factors that influence the way you see the world, so that you can deal with them and react against
them. You are your own most important resource for making your life work. Success is a moving target that must
be tracked and continuously pursued.
In this context we have updated this book keeping the forthcoming examination pattern in mind. This edition caters
to need of Quantitative Aptitude that is asked in Preliminary and Mains stages of most of the Banking and Insur-
ance examinations for various posts. This book is thoroughly based on the latest pattern in which time limit is given
for each section. Every chapter in this book describes the concept with the help of various examples and at the
end gives miscellaneous examples to clear the concepts. Examples have been solved with standard as well as short
methods where needed. At the end of each chapter exercises has been given to master the topic by solving variety
of exam oriented question and also include questions asked in recent Banking Exams. So we have now given a
complete focus on concept building and mastering each topic through this book.
We are truly dedicated to provide you the best among the rest. This book is an initiative from our side to make you
perfect in this subject.
We will be highly gratified, if this book helps students in getting selection.
Any suggestions related to the this book shall always be welcomed and we shall endeavor to incorporate them in
our upcoming issues.

Research Team

QUANTITATIVE APTITUDE 5
www.mahendras.org

CONTENTS
(Sr. No.) (Topic) (Page No.)
1. Simplification 7-17

2. Number Series 18-23

3. Percentage 24-31

4. Profit and Loss 32-40

5. Simple & Compound Interest 41-49

6. Average 50-55

7. Ratio & Proportion 56-63

8. Problems on Ages 64-70

9. Partnership 71-77

10. Mixture and Alligation 78-84

11. Time and Work 85-95

12. Pipe & Cistern 96-104

13. Speed, Time and Distance 105-112

14. Problems Based on Trains 113-120

15. Boat and Stream 121-125

16. Inequality 126-134

17. Permutation and Combination 135-144

18. Probability 145-151

19. Data Sufficiency 152-159

20. Data Interpretation 160-183

21. Mensuration 184-194


6 QUANTITATIVE APTITUDE
www.mahendras.org

CHAPTER

1 SIMPLIFICATION

There are 15 prime numbers upto 50 and 25 prime


Types of Numbers
numbers upto 100.
(i) Natural Numbers : Counting numbers are called
(vii) Composite number : Natural numbers which have
natural numbers. Thus 1, 2, 3, 4,5, 6,.......etc. are all
more than two distinct factors are called composite
natural numbers.
numbers eg. (4, 6, 8, 10, 12, 14)
(ii) Whole Numbers : All counting numbers together
Note :
with zero form the set of whole numbers.
1. 1 is neither prime nor composite.
Note :
2. 2 is the only even number which is a prime number.
(a) 0 is the only whole number which is not a natural
number (viii) Co-Prime Numbers : Two natural numbers a and b
are said to be co-prime, if their H.C.F is 1 as-
(b) Except 0 every natural number is a whole
(2, 3), (4, 5), (7, 9), (8, 11), (9, 14)
number.
Note:
Thus 0, 1, 2, 3, 4, 5, 6,........are whole numbers.
it is not necesssary to have a pair consisting prime
(iii) Integers : When negative value of natural number
number to be co prime number.
are included with the whole number these number
are known as integers. in other words numbers which Important Algebric Identities
have no fractional part are integer (a+b)2 = a2 +2ab +b2
e.g. {--&4] &3] &2] &1] 0] 1] 2] 3] 4] 5--}
(a-b)2 = a2-2ab+b2
(a) Positive Integers : All integers greater than 0
a2-b2 = (a+b).(a-b)
are known as positive integers.
(a+b)3 = a3 +b3 + 3ab (a+b)
{1, 2, 3, 4, 5.............}
(a-b)3 = a3 -b3 - 3ab (a-b)
(b) Negative Integers : All the integers less than a3+b3 = (a+b) (a2 +b2 -ab)
0 is known as negative integers.
a3 - b3 = (a-b) (a2 +b2 +ab)
{-1, -2, -3, -4......}
Fraction
Note :
When we divide 50 rupees among 10 people each will get 5.
0 is called null integer as it is neither positive nor
But what will happen if number of people increased by 2?
negative
Every one will get 4 rupees after which 2 rupees will left. If
(iv) Even Numbers : A natural number which is divisible we divide this 2 rupees among 12 people each one will get
by 2 is called an even number. If unit digit a number is some part of rupees i.e. some paise.
0, 2, 4, 6 then the number is divisible by 2 and hence
This is fraction.
even number
5 2
Thus 0, 2, 4, 6, 8, 10........etc. are all even numbers. Example ,23
7
(v) Odd Numbers : A natural number which is not
divisible by 2 is called an odd number.
Classification of Fraction :
Thus 1, 3, 5, 7, 9.......etc. are odd numbers.
(i) Proper fraction - A fraction whose numerator is less than
(vi) Prime Numbers : A natural number which has only denominator or in others words a fraction whose value is less
two factor (one and itself) are called prime number. than one is called proper fraction.
2 is smallest prime number
QUANTITATIVE APTITUDE 7
www.mahendras.org
1 2 15 11 11
e.g., , ?= 29 + = 29
2 5 23 24 24
(ii) Improper fraction - A fraction whose numerator is more
than denominator or in others words a fraction whose value 3 3 1
Ex. 215 + 711 − 500 = ?
is more than one is called improper fraction. 5 11 7
12 100 214
e.g. , , Sol. ?= (215 + 711 -500) + 231 + 105 − 55
5 3 5 385
(iii) Mixed fraction - When a fraction is denoted as 281 281
426 + = 426
combination of integer and fraction is called mixed fraction. 385 385

e.g. 2 2 ,4 5 3 5
3 9 2 1
Ex. 19 - 33 - 26 + 45 = ?
3 2 4 6
Only improper fraction may be converted into mixed fraction.

e.g. 11 ⇒ 3 2 2 1 3 5
3 3 Sol. (19 - 33 - 26 + 45) + ( - - + )
3 2 4 6
Addition & Subtraction  8 − 6 − 9 + 10  3 1
(i) Addition & subtraction of whole Number: ?= 5 +  
 = 5+ = 5 4
12 12
Ex. 8456 + 3891 + 4560 + 9823 = ?
2 1 2
Sol. ? = 26730 Ex. 67 5 + 13 5 - 74 3 = ?
Ex. 94532 - 6754 - ? = 75432 - 2346
2 1 2
Sol. 94532 - 6754 - 75432 + 2346 = ? Sol. ?= (67 + 13 - 74) + ( 5 + 5 - 3 )
? = 14692
 6 + 3 − 10  1
(ii) Addition and Subtraction of Decimal : ?= 6 +   = 6 -
 15 15
To solve the questions based on addition and subtraction of
decimal numbers, first we put zeros after the decimal equal 1 14
to maximum digit after demical present in total numbers and ?= 5 + 1- 15 = 5 15
then we operate addition and subtraction.
Multiplication
Ex. 87.25+125.235-1234.63+14.2+2018 = ?
(i) Multiplication of 2 digit Numbers:
Sol. 87.250+125.235-1234.630+14.200 +2018 = ?
Ex. 48 × 72
? = 1010.055
Step ⇒ 1.
Ex. 266.230 + 934.600 - 16.256 = ? + 6.658
Sol. ? = 1177.916
Addition & subtraction of Mixed Fraction:
To solve questions related to addition and subtraction of
mixed fraction whole numbers are solved seperately and ∴ Unit digit of result = 6
added and subtracted together and then we solve fractions
are solved seperately then both are combined to get answer Step ⇒ 2.

3 1 7 1 1
Ex. 24 + 18 − 13 − 2 = ?− 2
4 2 8 6 4
3 1 7 1 1
Sol. ?= (24 + 18 - 13 -2+2) +  + − − + 
4 2 8 6 4 ∴ Ten's digit of Result = 5

18 + 12 − 21 − 4 + 6 Step ⇒ 3.
?=29+
24

8 QUANTITATIVE APTITUDE
www.mahendras.org
Required result = 329616
Square and Square Roots
Square of (1-50) numbers

Students you should remeber these squares to speed your

calculation
∴ Hundred digit of Result = 4
12 = 1 262 = 676
Result = 3456 22
= 4 27 2
= 729
(ii) Multiplication of 3 digit Numbers: 32
= 9 28 2
= 784
432 × 763 42
= 16 29 2
= 841
Step ⇒ 1. 52 = 25 302 = 900
62 = 36 312 = 961
72 = 49 322 = 1024
82 = 64 332 = 1089
Unit digit of result = 6 92
= 81 34 2
= 1156
Step ⇒ 2. 10 2
= 100 35 2
= 1225
112 = 121 362 = 1296
122 = 144 372 = 1369
13 2
= 169 38 2
= 1444

∴ Ten's digit of Result = 1 14 2
= 196 39 2
= 1521

Step ⇒ 3. 152 = 225 402 = 1600


4 3 2 162 = 256 412 = 1681
17 2
= 289 42 2
= 1764
7 6 3 18 2
= 324 43 2
= 1849
14 +18+12 = 44 192 = 361 442 = 1936

⇒ 44 + 2 = 46 202 = 400 452 = 2025

Hundred digit of Result = 6 21 2


= 441 46 2
= 2116

Step 4. 22 2
= 484 47 2
= 2209
4 3 2 232 = 529 482 = 2304
242 = 576 492 = 2401
7 6 3
25 2
= 625 50 2
= 2500
21 +24 = 45
Type-I. Formula Method:

We know that
∴ Thousand's digit of Result = 9
(a+b)2 = a2 + 2ab + b2 i.e. (a/b)2
Step ⇒ 5.
= a2 / 2ab / b2
Ex. (56)2

= (5/6)2

= 52 / 2 × 5 × 6/ 62

6 3
∴ Ten thousand's digit of Result = 2

=
25 / 60 / 36

QUANTITATIVE APTITUDE 9
www.mahendras.org
= 31 / 3 / 6
Ex. 2
982 = (98-2) 22
= 3136
We break number in two parts i.e. 5 & 6 and follow = 96 / 04
the rule of (a+b)2 = a2 /2ab/ b2 982 = 9604
Ex. (92)2 Ex. 15
(85)2 = (85-15) 152

= (9/2) 2
2
= 81 / 3 6 / 04 = 70 25
= 8464 = 72 / 25
Ex. Find the square of 112 852 = 7225
I. Method : Type-II :
(We break 112 in two parts like that (11/2) (When the number is more than 100)

(11/ 2)2 = 112 / 2 × 11 × 2 / 22 Ex. 8

(108)2 = 108 +8 82

= 125 / 4 / 4 = 116 / 64
12544 (108)2 =11664
II. Method : Since 108 is more than 100 by 8 we add 8 to 108.
(We break 112 in two parts like that 1 / 12) Ex. 3
2 2
(1/ 12)2 = 12 / 2 × 1 × 12 / 122 (103) = 103 +3 / 3

= 1 / 24 / 144 = 106 / 09
= 1 / 25 / 44 (103)2 = 10609
Ans = 12544 Since base is 100 therefore right part always be in two
Ex. Find the square of 211 digit.
(2 / 11)2 = 22 /2 × 2 × 11 / 112 Ex.
12
(112)2 = 112 +12 / 12 2
= 4 / 44 / 121 +1

Ans. 44521 = 124 / 144


In (998) — the above rule is failed
2
(112)2 = 125 44
Now we can use Base-method.......... 200 - Base Method (Near about 200)
100-Base Method (When the number is near about 100). Type-III :
Type-I : (When the number is less than 200)
(When number is less than 100). Ex. 8
2 2
= (192) = 192-8 / 8
Ex. (96)
2

In this example we can use above method but we can


also use 100 base method. = 184 / 64
Let see in this example 96 is less than 100 by 4. ×2
= 368 / 64
Find the square of 4
(192)2 =368 64
Subtract 4 from 96.
Because of 200 base we multiply 184 × 2
4
962 = (96-4) 42 Ex. 2
2 2
(198) = 198-2 / 2
962 = 92 / 16
= 196 / 04
962 = 9216
= 392 / 04
10 QUANTITATIVE APTITUDE
www.mahendras.org
(198)2 = 39204 1
Ex. 15 1
2 2 9
(185) = 185-15 / 15
4
= 170 225 4
8
×2 5 5
= 340 / 225 4
= 342 / 25 6
6
= 34225
3
Similarly we can use 200, 300--------base method. 9
7
Type-IV :
1000 base (When the number is near about 1000) 0 0

Ex. 2 Note: We can see a perfect square number may have 0, 1,


2 2
(998) = 998-2 / 2 4 , 5, 6, 9 at unit place It means if a number has 2, 3, 7, 8
= 996 / 004 at its unit place it will not be a perfect square number

(998)2 = 996004 If a number has 0, 1, 4, 5, 6 or 9 at its unit place it has


a chance to be a perfect square number.
Since base is 1000 therefore Right hand side part one
in three digit. Perfect square :

Ex. 12
If square root of any number is integer then number
2
(1012) 2 = 1012+12 / 12
2 is said to be perfect square number.
Ex. 36 = 6
= 1024 / 144
441 =21
(1012)2 = 1024144
Non-perfect square :
Ex. 2
8
2
(1008) = 1008+8 / 8 If square root of any number is not integer but fraction
or decemal then number is said to be Non-perfect square
= 1016 / 064 number.
(1008)2 = 1016064 5, 200
Special Type :
How to find square root of perfect square number
When unit place of the number is 5.
Ex. Find the 2601
(x5)2 = x (x+1)/25
Ex. (75)2 = 7 × 8 / 52

= 56 / 25
(75) = 5625
2

Ex. (85)2 = 8 × 9 / 52

= 72 / 25
(85)2 = 7225

Ex. (105)2 = 10 × 11 / 52
Ans = 51

= 110 / 25
Rules :
(105)2 = 11025
 For unit place of the answer we see that last digit of
Square Roots: the question i.e. 1 (see the above table)
See the given table : For the ten’s place of the ans. we always take smaller
By oberving unit place of any number we can find number whose square is most nearest to 26.
the unit place of its square root. (i.e. 5)

QUANTITATIVE APTITUDE 11
www.mahendras.org
 In the above process we find two ans. i.e. 51 or 59 113 = 1331 243 = 13824
 We multiply ten digit number to next number 123 = 1728 253 = 15625
i.e. 5 × 6 = 30) 133 = 2197
 now compare 26 to 30 (we find that 26 is less than
How to find cube root of perfect cube Number
30).
While calculating Cube root of any number we will divide
 Therefore in both the ans. 51 and 59 the least ans is
it in two parts.
correct.
Other example. Second part (first three digit of number) will be used to
calculate the unit place of cube root and first part will be use
Ex. to calculate remaining part of the cube root.
Ex. Cube Root of 50653


Unit Place : To find second part will be used. As we can see
the unit place of Second part is 3 which comes at the end of
cube of 7 so the unit place of cube root will be 7.
Since 92 is greater than 90 therefore in both the ans. Remaining part : To find remaining part of cube root first
96 is correct. part will be used. As we can see first part is between cube of
3 and 4 hence cube root will be between 30 and 40.
Ex.
Cube root : Combining above two statements we can say
that required cube root will be 37.
Ex.

Now 11 × 12 = 132
125 is less than 132
∴ 112 is correct answer.

Cube & Cube Root Surds and Indices


Special Note for cube : Learn cube of 1 to 25 only for the An index (plural: indices) is the power, or exponent, of a
competitive exams. number. For example, a3 has an index of 3.

Cube of the numbers: A surd is an irrational number that can be expressed with
5
roots, such as 2 or 19 .
13 = 1 143 = 2744
Important formulas of surds and indices.
2
3
= 8 15
3
= 3375
33 = 27 163 = 4096 Indices Surds
43 = 64 173 = 4913 a × an = am+n
m 1
n
a = an
53 = 125 183 = 5832
am n
ab = n a × n b
63 = 216 193 = 6859 n
= am − n
a
73 = 343 203 = 8000
8
3
= 512 21
3
= 9261
(a )
n n
m
= amn a a
9
3
= 729 22
3
= 10648 n =
b n
b
103 = 1000 233 = 12167

12 QUANTITATIVE APTITUDE
www.mahendras.org
(ab)n = an­bn
( a)
n
n
=a 2 1 2 3
?= ÷ [ 2] = × = 1
n
3 3 3 2
a an m n
a − mn a
b = n Approximation:
  b
Approximation is the way of calculation in which we take
a0 = 1
( a) estaimated value is place of exact or real value. Now a days
m
n
= a n m

approximation question are asked usually in PO exams and


−2 −4
clerk mains exams
Ex. FG 1 IJ ÷ FG 1 IJ = ?
3 3 There is no fix rule to calculate but depends on you which
H 216 K H 27 K technique suits you.

? = b216g ÷ b27g
2/ 3 4/3 In following examples we would try to find the find value by
Sol.
taking nearest value and answer of these questions depend
2 6×6 4 4 upon the choices of given options.
? = b6 g

÷ b3 g
3 , ?=3× ?=
3
9×9 9 What approximate value will come in place of question
Ex. −3 mark (?) in the questions given below?
6 6 × 63 ÷ 6 2 = 6?+2 Ex. 2415 × 655 ÷ 9600 = ?
1 3 1 3
1+ + 3 + 1
Sol. 6?+2 = 6 × × 63 ×
62 62 6?+2 = 6 2 2 Sol. 49 × 26 × = ? , ? ≈ 13
98
? + 2 = 6, ? = 4
Ex. 30.02 % of 261 + 59.98 % of 512 - 104.001 = ?
Simplification
30 510
Sol. 260 × + × 60 – 104 = ? ,
VBODMAS Rule: 100 100

V → Vinculum means bar as (—) ? ≈ 280


B → Bracket- () {} and then [ ] Ex. 249 299 14
× ÷ =?
15 19 99
O → of same as mulipilication but having greater priorty
than divison 250 300 98
Sol. × × = ? , ? ≈ 1850
15 19 14
D → Division [÷]
Ex.
( )
2
M → Multiplication [×] 900.015 – 8.966
=?
( )
2
A → Addition [+] 575.757 – 441.111
S → Subtraction [-]
( )
2

The word ‘VBODMAS’ represents the order of calculation Sol. ? ≈


900 – 9

(30 – 3)2 ≈ 81
( ) (24 – 21)2
2
i.e. order of signs 576 – 441

Ex. 15 ÷ 5 (18 - 15 − 21) = ?


898 × (12.004) + ? = 5000
2

Ex.
Sol. ? = 18 ÷ 5 (18-(-6))
Sol. 900 × 122 + ? = 5000
? = 18 ÷ 5 (24) = 18 ÷ 120
? = 5000–4320 = 680
18 3
?= = Ex. 125% of 8475 + 88 × 14.995 = ?
120 20
2 1  1 1 1 Sol. ? = 10595 + 1320 = 11900 (Approx)
Ex. ÷ − − = ?
3 3  3 6 2  Ex. 721.001 × 7.998 + 6.05 × 8.010 = ?
Sol. ? = 5760 + 48 = 5800 (Approx)
2 1  1 1
?= ÷  + 
3 3 3 3 

QUANTITATIVE APTITUDE 13
www.mahendras.org
EXERCISE
Q.1-30. What will come in place of question mark (?) in the following questions?
Q.1. 18.5(17+18–3)–12.52=?
(1) 465.75 (2) 435.75 (3) 435.25 (4) -425.75 (5) None of these
Q.2. 175% of 480–87.5% of 288=?
(1) 544 (2) 588 (3) 584 (4) 455 (5) None of these

Q.3. 3 1
12 × ÷ + 12 ÷ 30 = ?
4 2
(1) 17.4 (2) 18.5 (3) 18.6 (4) 15.75 (5) None of these
Q.4. 17.5 +19 +12 –13 =?×5
2 2 3 3

(1) 39.35 (2) 37.65 (3) 36.65 (4) 39.65 (5) None of these
Q.5. 225% of 660 – 175% of 224= ?% of 800
(1) 136.625 (2) 136.25 (3) 132.625 (4) 132.25 (5) None of these
Q.6. 196 × 948 ÷ ? = 1176
(1) 148 (2) 158 (3) 128 (4) 160 (5) None of these
Q.7. (91)2 + (41)2 - ? = 9858
(1) 11236 (2) 10816 (3) 10404 (4) 9604 (5) None of these
Q.8. 42% of 445 - 25% of 354 =?
(1) 95.8 (2) 98.4 (3) 102.4 (4) 89.4 (5) None of these
Q.9. 8442 ÷ 576 − ? = 351
(1) 1.50 (2) 0.75 (3) 1.75 (4) 0.55 (5) None of these
Q.10. ?% of 340 + 48% of 480 = 451.40
(1) 95 (2) 85 (3) 130 (4) 65 (5) None of these
Q.11. (75)2 ÷ ? − 113 = 12
(1) 45 (2) 1225 (3) 2025 (4) 35 (5) None of these
Q.12. (3 ) = 19683
? ?

(1) 3 (2) 6 (3) 9 (4) 27 (5) None of these


Q.13. 76% of 845 + 102% of 234 = ?
(1) 862.88 (2) 812.84 (3) 792.82 (4) 880.88 (5) None of these
Q.14. (4.5) × (20.25) ÷ (91.125) = (4.5)
3.7 1.3 1.8 ?+5.2

(1) 6.5 (2) 5.6 (3) 6.3 (4) 5.9 (5) 6


Q.15. (207)2+20% of 200 × 1225 - 25% of 160 = ?
(1) 41409 (2) 42105 (3) 40219 (4) 39319 (5) None of these
Q.16. 24 + 96 + 216 + 384 = 5 2 × ?

(1) 5 3 (2) 3 3 (3) 4 3 (4) 2 3 (5) 3

Q.17. ?% of 800 = 293 – 22% of 750


(1) 16 (2) 18 (3) 20 (4) 24 (5) 25
Q.18. 3
12167 ÷ 46 × 8 + (2 2)6 − 250 = ?
14 QUANTITATIVE APTITUDE
www.mahendras.org
(1) 250 (2) 260 (3) 240 (4) 270 (5) 280
Q.19. (4 × 4)3 ÷ (512 ÷ 8) × (32 × 8) = ( 2 × 2)
4 4 ?+4

(1) 6 (2) 8 (3) 10 (4) 12 (5) 2


2 1 5
Q.20. of of of 12096 = ?% of 1600
7 8 9
(1) 10 (2) 12.5 (3) 15 (4) 17.5 (5) None of these
Q.21. 65% of 8800 + 18% of 720 = ? – 345.25
(1) 6468.50 (2) 6074.25 (3) 6284.85 (4) 6194.85 (5) None of these
Q.22. 625 × 1296 ÷ 324 = ?
(1) 10 (2) 50 (3) 30 (4) 70 (5) 90
13456 × 7396 + (? ) = 7216 + (79 )
2 2
Q.23.
(1) 69 (2) 96 (3) 95 (4) 59 (5) None of these
2
Q.24. 4
×
80
× ? =  23/2  + 4
3
32 560  

(1) 338 (2) 332 (3) 330 (4) 336 (5) None of these
Q.25. 1 1 4
1 + ?− 3 = 1 ÷ 2
3 9 7

2 1 1 2
(1) 1 (2) 2 (3) 3 6 (4) 2 (5) None of these
6 6 3

1
Q.26.
3375 3 × 854 × 514 ÷  454 × 5 × 3−3  = 17?
 

(1) 4 (2) 12 (3) 8 (4) 10 (5) 16


Q.27. 4 × (0.31 ×0.8 × 1.8) = 28.5696
?

(1) 3 (2) 4 (3) 2 (4) 5 (5) None of these

Q.28. ( 70 ) ×12.5 = 850 ×72


2
2
−?

(1) 2 (2) 4 (3) 6 (4) 4896 (5) None of these


Q.29. 33 +44 = ?
2 2 2

(1) 35 (2) 45 (3) 55 (4) 65 (5) None of these


4096 × 59
Q.30. (7414 + 3698 + 1257 + 1869) + =?
759 − 641

(1) 14272 (2) 14274 (3) 14270 (4) 14278 (5) None of these
Q.31-42.What approximate value should come in place of question mark (?) in the questions given below?
Q.31. 0.98 × 10.9 + 48% of 56 – 46.34 = ?
(1) 35 (2) 40 (3) 50 (4) 45 (5) 30
Q.32. 71.8% of 419-11.972=?% of 399
(1) 31 (2) 52 (3) 45 (4) 37 (5) 27

QUANTITATIVE APTITUDE 15
www.mahendras.org
Q.33. 432+532-153=400% of ?
(1) 320 (2) 350 (3) 375 (4) 420 (5) 280
Q.34. 77.92×93.1-44.87×40.996=?
(1) 5200 (2) 5400 (3) 5700 (4) 6100 (5) 4900
Q.35. ?2=479% of 121- 400% of 105
(1) 9 (2) 13 (3) 5 (4) 17 (5) 22
Q.36. 892+452=362+?2
(1) 93 (2) 86 (3) 81 (4) 99 (5) 112
Q.37. 21 -249% of 644=?
3 3

(1) 10 (2) 14 (3) 18 (4) 20 (5) 8


Q.38. 1010 ÷ 36 + 187 × 20.05 = ?
(1) 3770 (2) 3800 (3) 3740 (4) 3700 (5) 3680
Q.39. (559% of 816) + 1449 = ?
(1) 6000 (2) 7000 (3) 5000 (4) 3000 (5) 8000
5
Q.40. 1654×24.62÷ of 15.86 = ?
8
(1) 4100 (2) 4700 (3) 3100 (4) 2700 (5) 2100
Q.41. 3 4 1
4 ×5 ×6 = ?
10 7 3

(1) 120 (2) 180 (3) 150 (4) 295 (5) 157
Q.42. 528.64 + 841.46 = ?
(1) 45 (2) 57 (3) 42 (4) 63 (5) 52
EXERCISE EXPLANATION
Q.1.(2) 592-156.25=? Q.7.(2) (8281 + 1681 – 9858)2 = ?
?=435.75 ? = 1042 = 10816
Q.2.(2) 840-252=? Q.8.(2) ? = 186.9 - 88.5

?=588 ? = 98.4
3 1 Q.9.(2) ? = 351.75 - 351
Q.3.(5) 12 × ÷ + 12 ÷ 30 = ?
4 2
? = 0.75
2
18 + = 18.4 34
5 Q.10.(4) ?× = 45140
. − 230.40
10
Q.4.(4) 306.25+361+1728–2197=?×5
221
?=39.65 ? = 34 × 10
? = 65
Q.5.(1) 1485–392=?×8
75 × 75
?=1093/8 Q.11.(3) = ?
125
?=136.625 ? = 45 × 45, ? = 2025
Q.6.(2) ? = 196 × 948 ÷ 1176 Q.12.(1) (3?)? = (3)9= (33)3
? = 158 ?=3
16 QUANTITATIVE APTITUDE
www.mahendras.org
Q.13.(4) ? = 880.88 28.5696
4? =
Q.14.(1) (4.5) 3.7+ 2 × 1.3 + 3 × 1.8
= 4.5 ?+ 5.2
0.31× 0.8 × 1.8

? = 3.7 + 2.6 + 5.4 - 5.2, ? = 6.5 4? = 64 = 43

Q.15.(1) 42849 + 40 × 35 - 40 = ?, ? = 41409 ?=3

( 4900 − ? )
2
Q.16.(3) 2 6 +4 6 +6 6 +8 6 =5 2 ×? Q.28.(2) ×12.5 = 850 ×72
20 6 850 × 72
( 4900 − ? )
2
?= =4 3 = ,
5 2 12.5
( 4900 − ? )
2
Q.17.(1) 800 × ? = 293 − 750 × 22 = 4896
100 100
4900 - ? = 4896, ? = 4
? = 16
Q.18.(2) 3
12167 ÷ 46 × 8 + (2 2)6 − 256 = ? Q.29.(3) ?2 = 3025
1 ? = 55
23 × × 8 + 512 − 256 = ?, 4 + 512 − 256 = ?
46 64×59
? = 260 Q.30.(3) (7414 + 3698 + 1257 + 1869) + 759 - 641 = ?
Q.19.(1) (4 × 4)3 ÷ (512 ÷ 8)4 × (32 × 8)4 = ( 2 × 2)? + 4 14238 + 32 = ?, ? = 14270
46−12+16 = 4? + 4 Q.31.(4) 11+41 – 7 = 45 (approx)
?=6 Q.32.(4) ?=(302-144)/4
Q.20.(3) 240 = 16×? , ? = 15 ?=37 (approx)
Q.21.(4) 5720+129.6+345.25=?, ? =6194.85 Q.33.(1) 1849+2809-3375=4×?

Q.22.(2) ?= 25×36÷18 = 50 (approx) ?=1283/4


?=320 (approx)
Q.23.(4) 116 × 86 + (?)2 = 7216 + 6241
Q.34.(2) ?=7254–1845=5409=5400 (approx)
(?)2 = 7216 + 6241 - 9976
Q.35.(2) ?2=581–420=161
(?)2 = 3481, ? = 59
?=13 (approx)
Q.24.(4) 3 3 
2
4 80
× × ? =  22  + 4 Q.36.(1) ?2=8650
32 560  
 
?=93 (approx)
(2 + 4) × 560 × 32 Q.37.(4) 9261 – 1610 = ?3
?= = 336
4 × 80

?3 =7651
Q.25.(2) 4 28 7 , 7 4 28
+ ?− = 1× ?= − +
3 9 18 18 3 9 ? = 20 (approx)

7 − 24 + 56 39 13 1 Q.38.(1) ? = 28 + 3740 = 3770 (Approx)


?= = = =2
18 18 6 6
Q.39.(1) ? = 4560 + 1449 = 6000 (Approx)
Q.26.(3) 1
Q.40.(1) 41350 ÷ 9.9125 = ?
3375 3 × 854 × 514 ÷  454 × 5 × 3−3  = 17?
 
? ≈ 4100
4 4 4 4 3
15 × 17 × 5 × 17 × 3 × 3
= 17? , ? =8 43 39 19 31863
154 × 34 × 5 Q.41.(3) × × ⇒ = 151 ≈ 150
10 7 3 210

Q.27.(1) 4? × (0.31 × 0.8 × 1.8) = 28.5696 Q.42.(5) ? = 23+29 = 52

QUANTITATIVE APTITUDE 17
www.mahendras.org

CHAPTER

2 Number Series

Number series is a order of numbers which are not arranged 3. Geometric Series : Under this category, each successive
randomly but follow a pattern. Here in this not we will number is obtained by multiplying (or dividing) the previous
understand how to identify which kind of pattern is following number with a fixed number.
because without this it is next to impossible to have a Ex. 5, 35, 245, 1715, ?
command on number series. Sol. Here previous number is multiplied by 7, hence answer
Types of Series is 12005
Ex. 43923, 3993, 363, 33, ?
There are different type of number series, which is based on
some of the important rules or order. Sol. Here previous number is divided by 11, hence answer is 3.
Ex. 336, 168, 84, 42, 21, ?
1. Pure Series : In this type of number series, the number
itself obeys certain order so that the character of the series Sol. Here previous number is divided by 2,
can be found out. hence answer is 10.5.
4. Two-tier Arithmetic Series : Under this category, the
The number itself may be:
differences of successive numbers form an arithmetic series.
(i) Even Number : Ex. 4, 5, 9, 16, 26, 39, 55 ?
Ex. 2, 4, 6, 8, 10, ? Sol.
Sol. 12 4 5 9 16 26 39 55 74
(ii) Odd Number :
Ex. 1, 3, 5, 7, 9, ? +1 +4 +7 +10 +13 +16 +19
Sol. 11
+3 +3 +3 +3 +3 +3
(iii) Prime Number :
5. Mixed Series: Here, the numbers obeying various orders
Ex. 2, 3, 5, 7, 11, 13, ?
of two or more different type of series are arranged alternately
Sol. 17 in a single number series.
(iv) Perfect square Ex. 7, 15, 32, 67, 138, ?
Ex. 121, 144, 169, 196, 225, ? Sol. ×2 + 1, ×2 + 2, ×2 + 3, ×2 + 4, ×2 + 5,
Sol. 256 Hence answer is = 281
(v) Perfect cube Ex. 8, 15, 42, 141, 580, ?
Ex. 6859, 5832, 4913, 4096, 3375, ? Sol. +7×1, +6×2, +5×3, + 4×4, +3×5,
Sol. 2744 Hence answer is = 2915
2. Difference Series: Under this category, the change in Ex. 7, 4, 5, 9, 20, ?
order for the differences between each consecutive number Sol. ×.5+0.5, ×1+1, ×1.5+1.5, ×2+2, ×2.5+2.5
of the series is found out. Hence answer is = 52.50
Ex. 13, 18, 28, 43, 63, ? Types of Question:
Sol. There are 3 type of questions usually asked in Banking
13 18 28 43 63 88
examination.
+5 +10 +15 +20 +25 1. Missing Number series.
Ex. 1348, 1338, 1318, 1288, 1248, ? 2. Wrong number series
3. Coding and decoding series.
Sol.
1348 1338 1318 1288 1248 1198 1. Missing Number series: In this type of series one number
is missing in given series. Such type of series, find the given
-10 -20 -30 -40 -50 pattern and solved the missing number.

18 QUANTITATIVE APTITUDE
www.mahendras.org
Ex. What value come in place of question mark (?). Ex. 15, 9, 8, 12, 36, 170
Ex. 6 11 21 36 56 ? 19 (a) (b) (c) (d) (e)
Sol. +5 +10 +15 +20 What will come in place of (b) ?
Hence, Answer is = 56 + 25 = 81
Sol.
Ex. 3 7 15 31 63 ?
Sol. +4 +8 +16 +32
Hence, Answer is = 63 + 64 = 127 Similarly,
Ex. 1 6 15 ? 45 66 91
Sol. +5 +9 +21 +25
Hence, Answer is = 15 + 13 = 28
Ex. 504 336 210 120 ? 24 ∴ (b) = 13×2 - (5 × 2) = 26 - 10 = 16
Sol. 83-8 73-7 63-6 53-5 43-4
Ex. 7, 6, 10, 27, 104, 515
Hence, Answer is = 43-4 = 60
9 (a) (b) (c) (d) (e)
2. Wrong number series: In this type of series one number
is odd man out which does not follow the sequence. What will come in place of (d) ?
Ex. Find the wrong number in given series: Sol.
Ex. 2, 3, 7, 22, 89, 440, 2677, 18740
Sol. ×1+1, ×2+1, ×3+1, ×4+1, ×5+1 ......... Similarly,
So, 440 is replaced by 446
Ex. 5, 6, 14, 40, 89, 170, 291
Sol. +12, +32, +52, +72, +92 ............

So, 14 is replaced by 15.
∴ (d) = 39×4 - 4 - 4 = 156 - 4 = 152
Ex. 445, 221, 109, 46, 25, 11,4
Sol. -3÷2, -3÷2.................. Ex. 6, 16, 57, 244, 1245, 7506
So, 46 is replaced by 53. 3 (a) (b) (c) (d) (e)
Ex. 12, 26, 56, 116, 244, 498, 1008 What will come in place of (d) ?
Sol. ×2+2, ×2+4, ×2+6............. Sol.
So, 116 is replaced by 118
Ex. 8, 27, 64, 125, 217, 343
Similarly,
Sol. 23 , 33 , 43 , 53 ..........
So, 217 is replaced by 216
3. Coding decoding series: In the following number series
question a series a given. First you have to find out the pattern
given in the series then followed by this pattern you make
∴ (d) = 196×5 + (5)2 = 980 + 25 = 1005
new series whose first term is given.
Ex. 5, 9, 25, 91, 414, 2282.5 Ex. 8, 9, 20, 63, 256, 1285
3 (a) (b) (c) (d) (e) 5 (a) (b) (c) (d) (e)
What will come in place of (c) ? What will come in place of (e) ?
Sol. Sol.

Similarly, Similarly,


∴ (c) = 17.5×3.5+3.5 = 61.25+3.5 = 64.75 ∴ (E) = 184×5 + (5) = 920 + 5 = 925

QUANTITATIVE APTITUDE 19
www.mahendras.org
EXERCISE
Q.1-25. What should come in the place of question mark (?) in the following number series?
Q.1. 5 25 7 ? 9 19
(1) 23 (2) 22 (3) 25 (4) 32 (5) None of these
Q.2. 7 18 40 84 172 ?
(1) 326 (2) 328 (3) 330 (4) 332 (5) None of these
Q.3. 100 50 52 26 28 ?
(1) 30 (2) 32 (3) 14 (4) 16 (5) None of these
Q.4. 980 392 156.8 ? 25.088 10.0352
(1) 62.72 (2) 63.85 (3) 65.04 (4) 60.28 (5) None of these
Q.5. 113 225 449 ? 1793
(1) 789 (2) 786 (3) 897 (4) 987 (5) None of these
Q.6. 5 12 39 160 805 ?
(1) 4163 (2) 4153 (3) 4181 (4) 4836 (5) None of these
Q.7. 2 10 30 68 ?
(1) 126 (2) 130 (3) 140 (4) 150 (5) None of these
Q.8. 45 46 70 141 ? 1061.5
(1) 353 (2) 353.5 (3) 352.5 (4) 352 (5) None of these
Q.9. 33 321 465 537 573 ?
(1) 600 (2) 591 (3) 585 (4) 498 (5) None of these
Q.10. 2 9 ? 105 436 2195
(1) 25 (2) 27 (3) 30 (4) 33 (5) None of these
Q.11. 4 6 14 44 ? 892
(1) 176 (2) 172 (3) 178 (4) 1821 (5) None of these
Q.12. 126 64 34 20 14 ?
(1) 12 (2) 14 (3) 16 (4) 18 (5) None of these
Q.13. 16 8 12 30 ? 472.5
(1) 100 (2) 105 (3) 205 (4) 300 (5) None of these
Q.14. 25 34 30 39 35 ?
(1) 45 (2) 44 (3) 46 (4) 50 (5) None of these
Q.15. 10 11 13 21 69 ?
(1) 384 (2) 490 (3) 453 (4) 390 (5) None of these
Q.16. 8 4 4 6 12 30 ?
(1) 80 (2) 85 (3) 90 (4) 95 (5) None of these
Q.17. 32 ? 92 134 184
(1) 55 (2) 38 (3) 45 (4) 58 (5) None of these
Q.18. 11 16 26 ? 86
(1) 46 (2) 56 (3) 76 (4) 86 (5) None of these
Q.19. 198 194 185 169 ?
(1) 154 (2) 165 (3) 144 (4) 134 (5) None of these
20 QUANTITATIVE APTITUDE
www.mahendras.org
Q.20. 9050 5675 3478 2147 ?
(1) 3478 (2) 1418 (3) 2428 (4) 3678 (5) None of these
Q.21. 5 17 37 65 ? 145.
(1) 95 (2) 99 (3) 97 (4) 101 (5) None of these
Q.22. 95 115.5 138 ? 189.
(1) 154.5 (2) 164.5 (3) 162.5 (4) 166.5 (5) None of these
Q.23. 3 4.5 9 22.5 67.5 ? 945
(1) 265.25 (2) 236.25 (3) 225.36 (4) 150 (5) None of these
Q.24. 8544 1420 280 ? 18 5
(1) 33 (2) 44 (3) 56 (4) 66 (5) None of these
Q.25. 812 398 ? 90 40 16
(1) 192 (2) 182 (3) 172 (4) 162 (5) None of these
Q.26-35. In the following number series one number is wrong. Find the wrong number.
Q.26. 6072 1000 200 48 14 5
(1) 6072 (2) 1000 (3) 5 (4) 48 (5) 14
Q.27. 198 165 148 117 104 77
(1) 165 (2) 148 (3) 198 (4) 104 (5) 77
Q.28. 445 221 109 53 23 11
(1) 445 (2) 221 (3) 23 (4) 53 (5) 11
Q.29. -1/2 0 1/6 1 3/2 2
(1) 1/2 (2) 3/2 (3) 2 (4) 1/6 (5) -1/2
Q.30. 3 9 39 113 265 577
(1) 3 (2) 113 (3) 39 (4) 577 (5) 265
Q.31. 7 15 42 126 231
(1) 42 (2) 231 (3) 126 (4) 15 (5) 7
Q.32. 8 19 41 65 118
(1) 65 (2) 8 (3) 41 (4) 65 (5) 118
Q.33. 13 13 27 21 33 53
(1) 13 (2) 33 (3) 27 (4) 21 (5) 53
Q.34. 7 8 17 42 90
(1) 90 (2) 7 (3) 17 (4) 42 (5) 91
Q.35. 10 11 24 65 304
(1) 10 (2) 24 (3) 65 (4) 11 (5) 304
Q.36-40. In the following number series questions a series a given. First you have to find out the pattern given in the series
then followed by this pattern you make new series whose first term is given.
Q.36. 2, 15, 92, 463, 1856, 5561
3 (a) (b) (c) (d) (e)
Which of the following numbers will come in place of (c) ?
(1) 670 (2) 672 (3) 673 (4) 675 (5) None of these

QUANTITATIVE APTITUDE 21
www.mahendras.org
Q.37. 512, 256, 128, 64, 32, 16
998 (a) (b) (c) (d) (e)
Which of the following numbers will come in place of (d) ?
(1) 61.275 (2) 62.375 (3) 63.475 (4) 60.275 (5) None of these
Q.38. 3, 5, 12, 38, 154, 772
7 (a) (b) (c) (d) (e)
Which of the following numbers will come in place of (c) ?
(1) 56 (2) 60 (3) 61 (4) 62 (5) None of these
Q.39. 9, 22, 24, 37, 39, 52, 54
11 (a) (b) (c) (d) (e)
Which of the following numbers will come in place of (d) ?
(1) 39 (2) 41 (3) 45 (4) 50 (5) None of these
Q.40. 4, 4, 6, 12, 30, 90
18 (a) (b) (c) (d) (e)
Which of the following numbers will come in place of (e) ?
(1) 405 (2) 403 (3) 404 (4) 402 (5) None of these
Q.41. The number series 1 , 3 , 9 , 31, 129 follow a certain pattern then 5, ______,17, ---------1131. If 1131 is nth term
then the value of n is-
(1) 5 (2) 4 (3) 8 (4) 10 (5) None of these
Q.42. 4, 2, 2, 3, 6, 15,..………., 2835. If 2835 is nth term then the value of n is
(1) 9 (2) 8 (3) 10 (4) 11 (5) 12
Q.43. If one series is 1.5, 6, 22, 93, 471, 2833 and another series which follows same pattern as given series is 2.5, __,
__, __, __, x then x = ?
(1) 2369 (2) 3553 (3) 2961 (4) 4145 (5) 945
Q.44. A series is given as 6, 3, 3, 4.5, 9, 22.5, …. and it’s pth term is 4252.5. Find the value of P.
(1) 10 (2) 11 (3) 9 (4) 8 (5) 12
Q.45. A series is 113, 170, 232, 303, 399, 556, 838. Another series is 93, __, __, __, __, __, m. Which follows same
pattern as given number series. Then m= ?
(1) 808 (2) 443 (3) 626 (4) 818 (5) 909
EXERCISE EXPLANATION
Q.1.(2) Q.5.(3)
5 25 7 22 9 19 113 225 449 897 1793
×2-1 ×2-1 ×2-1 ×2-1
+2 +2
-3 -3
Q.6.(4) ×2+2, ×3+3, ×4+4, ×5+5, ×6+6
Q.2.(5) +11, +22, +44, +88 Q.7.(2) 2 10 30 68 130
Q.3.(3) 100 50 52 26 28 14

(1)3 +1 (2)3+2 (3)3+3 (4)3+4 (5)3+5


÷2 ÷2 ÷2
Q.8.(2) ×1+1, ×1.5+1, ×2+1, ×2.5+1, ×3+1
980 392 1568 62.72 25.088
Q.4.(1) Q.9.(2) +288, +144, +72, +36, +18
Q.10.(3) ×1+(1×7), ×2+(2×6), ×3+(3×5), ×2+(4×4)
×2
5 ×2 ×2 ×2
5 5 5 Q.11.(3) (×1+2) (×2+2) (×3+2) (×4+2) (×5+2)

22 QUANTITATIVE APTITUDE
www.mahendras.org
Q.12.(1) ÷2+1, ÷2+2, ÷2+3, ÷2+4, ÷2+5 Q.38.(4) ×1+2, ×2+2, ×3+2, ×4+2
Q.13.(2) ×0.5, ×1.5, ×2.5, ×3.5, ×4.5 Q.39.(2) +13, +2, +13, +2, +13, +2
Q.14.(2) +9, -4, +9, -4, +9 Q.40.(1) ×1, ×1.5, ×2, ×2.5
Q.15.(3) +1, +2, +8, +48, +384 Q.41.(1) 5×1+2=7
Q.16.(3) ×0.5, ×1, ×1.5, ×2 7×2+3=17
Q.17.(4) +26, +34, +42, +50 17×3+4=55
Q.18.(1) +5, +10, +20, +40 55×4+5=225
225×5+6=1131
Q.19.(3) -22, -32, -42, -52......
Q.42.(1) 4×0.5=2
Q.20.(2) -153, -133, -113, -93...... 2×1=2
Q.21.(4) 22+1, 42+1, 62+1, 82+1, 102+1, 122+1 2×1.5=3
Q.22.(3) +20.5, +22.5, +24.5, +26.5 3×2=6
Q.23.(2) ×1.5 ×2 ×2.5 ×3 ×3.5 ×4 6 × 2.5 = 15
Q.24.(4) ÷6-4, ÷5-4, ÷4-4, ÷3-4, ÷2-4 15 × 3 = 45
45 × 3.5 = 157.5
Q.25.(1) ÷2-8, ÷2-7, ÷2-6, ÷2-5, ÷2-4
157.5 × 4 = 630
Q.26.(2) -12÷6, -10÷5, -8÷4, -6÷3, -4÷2, -2÷1
630 × 4.5 = 2835
Q.27.(3) (142+4), (132-4), (122-4), (112-4), (102-4), (92-4), 10th term = 2835
(82-4) Q.43.(2) 1.5×2+3 = 6 ,6×3+4=22 ,
Q.28.(3) (-224), (-112), (-56), (-28), (-14), (-7) 22×4+5=93,93×5+6 = 471, 471×6+7= 2833
Q.29.(4) (+1/2), (+1/2), (+1/2), (+1/2), (+1/2) Similarly for next series x = 3553
Q.44.(1) 6×0.5 = 3, 3×1=3 , 3×1.5=4.5 ,4.5×2=9 ,
Q.30.(2)
-3 9 41 113 265 577 9×2.5=22.5……945×4.5 =4252.5
113 170 232 303 399 556 838
+12 +32 +72 +152 +312
Q.45.(4) +57 +62 +71 +96 +157 +282
+20 +40 +80 +160
Q.31.(3) 23, 33, 43, 53 +5 +9 +25 +61 +125
Q.32.(4) +11, +22, +33, +44 +4 +16 +36 +64
Q.33.(3) +(02+0), +(12+1), +(22+2), +(32+3) Second Series
93 150 212 283 379 536 818
Q.34.(1) +(1)2, +(3)2, +(5)2, +(7)2
+57 +62 +71 +96 +157 +282
Q.35.(3) ×1+1, ×2+2, ×3+3, ×4+4
Q.36.(3) ×7+1, ×6+2, ×5+3, ×4+4 +5 +9 +25 +61 +125

Q.37.(2) ÷2, ÷2,÷2, ÷2, ÷2 +4 +16 +36 +64

Notes

QUANTITATIVE APTITUDE 23
www.mahendras.org

CHAPTER

3 Percentage

Introduction: Percentage is a fraction whose denominator 100 × 100


is always 100. x percentage is represented by x%. Sol. Ist Method : y = ×x
80
x . ∴ y = 125% of x
I. To express x% as a fraction : We know x% =
100 IInd Method : Let y is 100 then x = 80
20
Thus 20% = .......(means 20 parts out of 100 parts) 100
100 Required % = × 100 = 125%
80
= 1 .............(means 1 part out of 5 parts) Ex. K is what % of N ?
5
20 1 Sol. K K
and 20% = 100 = 5 N
× 100 = %
N
x V. If A is R% more than B, then B is less than A by-
II. To express as a percentage:
y LM R × 100OP%
xFG x IJ
MN b100 + Rg PQ
= × 100 %


We know that
Change in %
y H y K If A is R% less than B, then B is more than A by-

1 F1 I LM R × 100OP%
Thus = GH × 100JK % = 25% MN b100 − Rg PQ
4 4
8 F 8 I Ex. If Akash income is 10% more than that of Vikas
=G × 100J % = 80% income. How much % Vikas income is less than that
10 H 10 K
and 0.8 =
of Akash income?
III. Remember it :
I. Method.
1 = 100%
By using formula-
1 1 1
= 50% = 33 % r 10
2 3 3 less % = × 100 = × 100
100 + r 100 + 10
1 1
= 25% = 20% 10 1
4 5 = 110 × 100 = 9 11 %
1 2 1 2
= 16 % = 14 % II. Method.
6 3 7 7
Since 10% more 100%
1 1 1 1 Two tools
= 12 % = 11 % 110%
8 2 9 9
100
1 1 1 Less % = 10 × ........
= 10% =9 % 110
10 11 11
(To decrease any number, we, multiply with small
1 1 1 9
=8 % =7 % number and divide with large number.)
12 3 13 13
1
IV. Comparison between two values x and y. = 9 11 %
(i) If x is compare to y then we assume always y is Ex. If the income of A is 40% less than that of B, How
equal to 100% much percent B’s income is more than that of A’s?
(ii) When any question ask y is a what percent of x
then x is always write in the denominator 40
Sol. More% = 60 × 100 = 66.66%
Ex. If x is 80% of y, what percent of x is y?
24 QUANTITATIVE APTITUDE
www.mahendras.org
VI. If the price of a commodity increases by R%, then % Ex. If the present population of a town is 72600 and it
reduction in the consumption as not to increase the is decreased by 10% per annum. What will be its
expenditure is- population 2 years hence?
LM R × 100OP% Sol. Population after n years = P  1 −
 R 
n

MN b100 + Rg PQ Here P = 72600


 100 

If the price of a commodity decreases by R%, then R = 10%


the % increase in consumption as not to decrease the n = 2yrs.
expenditure is- Population after 2 years
LM R × 100OP%
MN b100 − Rg PQ
FG
= 72600 1 −
10 IJ 2
= 72600 ×
90
×
90
= 58806

Ex. If tax on a commodity is reduced by 10%, total revenue
H 100 K 100 100

remain unchanged. What is the percentage increase in Trick:


its consumption? 90 90
Population after 2 yrs = 72600 × × = 58806
Sol. Percentage increase 100 100
Note :
10 100 1 These formula is also used for the depreciation value
= × 100 = = 11 %
100 – 10 9 9 of machine.
VII. Result on population : (a) Let the population of a VIII. Net percentage change :
town be P now and suppose increases at the rate of Assume your pocket money is Rs 500 per month. If
R% per annum, then : it is increased by 20 % then what will be your new
pocket money?
1. Population after n years = P 1 +
FG R IJ n
Of course your new pocket money will be 600. But
H 100 K what if it is further decreased by 20% ?
P Will your pocket money will become same as before??
2. Population n years ago = No.Why??
FG1+ R IJ n
This is because of successive process.
H 100 K So next we are going to study is net percentage change.
Ex. Present population of a town 1.21 crore & it increases xy
at 10% per annum. Then find out population after two x+ y+
years as well as before 2 year. 100
Sol. Present Population = 1.21 crore But always remember one thing x and y always come
n 2
with sign. Means if quantity is increased we will take
 10 
Two years after = P  1 + r  = 1.21  1 + + and if it is decreased we will take -.
 100   100  So the answer of the question asked will be-
= 1.21 × 1.21= 1.4641 crore 20 × 20
20 − 20 − = −4%
Two years before = P 100
n
 r  But what if there are three changes??
 1 + 100 
We also have formula for that
1.21 1.21 xy yz zx xyz
= = = 1 crore x+ y+z+ + + −
 10 
2
1.21 100 100 100 10000
 1 + 100  But using this formula is not a great idea instead this
(b) Results on Population : Let the population of a town be P we can apply first formula thrice Like
now and suppose decreases at the rate of R% per annum, then : Ex. Find the net percentage change of 40% increase.25%
decrease and 10% increase?
1. Population after n year Sol. Very important point it doesn’t matter which change is
FG
= P 1− R IJ n
applied first final change will be always same.
H 100 K
40 − 25 −
25 × 40
=5
2. Population n years ago 100
P 5 ×10
= Again 5 + 10 + = 15.5
FG1− R IJ n
100
H 100 K So final change will be increase of 17.5%.
QUANTITATIVE APTITUDE 25
www.mahendras.org
EXERCISE
Q.1. If the income of Ram is 20% more than Shyam’s income. How much percent Shyam’s income is less than that of Ram’s
income?
2
(1) 12% (2) 20% (3) 30% (4) 40% (5) 16 %
3
Q.2. When numerator of a fraction is increased by 40% and denominator increased by 80% the resultant fraction becomes
14
27 . Find the original fraction ?
2 1 4 3
(1) 3 (2) 3 (3) 3 (4) 3 (5) 4
Q.3. A reduction of 5% in the price of tea a person would enable to obtain 5 kg. more for Rs. 2000. Find the reduced
rate per kg.(new rate) of tea.
(1) Rs. 30 (2) Rs. 40 (3) Rs. 20 (4) Rs. 50 (5) Rs. 10
Q.4. Due to increase in the price of sugar by 25%, a man able to purchase 1/2 kg less sugar for Rs.96. Find the original
rate per kg.
(1) Rs.40 (2) Rs.50 (3) Rs.38.4 (4) Rs.52 (5) Rs.30.4
Q.5. In an examination, a student who gets 20% of the maximum marks fails by 5 marks. Another student who scores
30% of the maximum marks gets 20 marks more than the pass marks. The necessary percentage required for
passing is
(1) 32% (2) 23% (3) 22% (4) 20% (5) None of these
Q.6. In a town, 60% registered voters cast their votes in the election. Only two candidates (A and
B) were contesting the election. A won the election by 1860 votes. Had B received 40% more
votes, the result would have been a tie. How many registered votes are there in the town?
(1) 10580 (2) 10880 (3) 10550 (4) 10850 (5) 10250
Q.7. lf A has 4/5 of the number of books that shelf B has. If 25% of the books A are transferred to B and then 25
% of the books from B are transferred to A then the percentage of the total number of books that in shelf A is:
(1) 25% (2) 50% (3) 75% (4) 100% (5)
Q.8. In an election survey, 30% people promised to vote candidate A and remaining promised to vote for candidate B.
If on the day of election x% percent of people who promised to vote for A, voted for B and 40% of people who
promised to vote B voted against him and in the end B lost by 10 votes. What is value of x, if total 250 votes were
there?
(1) 20 (2) 30 (3) 50 (4) 70 (5) None of these
Q.9. Mr. Ram Niwas Singh spent 20% of his monthly income on food and 15% on children's education. 40%
of the remaining, he spent on entertainment and transport together and 30% on the medical. He is left with
an amount of Rs.8775 after all these expenditures. What is Mr. Ram Niwas Singh's monthly income?
(1) Rs.40000 (2) Rs.35000 (3) Rs.42000 (4) Rs.38000 (5) None of these
Q.10. When the price of rice was increase by 25%, a family reduced its consumption in such a way that the expenditure
of rice was only 10% more than before. If 50 kg. were consumed before, find the new consumption?
(1) 44 kg. (2) 45 kg. (3) 43 kg. (4) 46 kg. (5) 42 kg.
Q.11. If the price of salt decrease by 20%, then by what percent consumption should be increase to keep the expenditure
same?
(1) 27 % (2) 26 % (3) 25 % (4) 36 % (5) 30 %
Q.12. A number wrongly divided by 5, instead of multiplied by 5. Find the percent error in result.

26 QUANTITATIVE APTITUDE
www.mahendras.org
(1) 92% (2) 93% (3) 97% (4) 96% (5) 60%
Q.13. If the radius of a circle increased by 20%, then what is the percent change in area?
(1) 44% (2) 45% (3) 43% (4) 46% (5) 42%
Q.14. In an examination 45% student passed mathematics, 50% students passed in english and 15% failed in both subjects.
If 150 students passed in both subjects, find out the total no. of students?
(1) 1200 (2) 1100 (3) 1400 (4) 1500 (5) 1600
Q.15. In a class of 40 students and 2 teachers, each student got toffee that are 10% of the total number of students and
each teacher got toffee that are 15% of the total number of students. How many toffee were there?
(1) 166 (2) 176 (3) 172 (4) 177 (5) 173
Q.16. A 120 ltrs solution of milk and water contains 20% water, what quantity of water must be added with that solution
to get 25% water?
(1) 8 liter (2) 6 liter (3) 3 liter (4) 4 liter (5) 5 liter
Q.17. A person spent 40% of his income. If his income is increased by 20% then his expenditure also increased by 50%.
Find the % change in his saving?
(1) 30% (2) 40% (3) 45% (4) 50% (5) No Change
Q.18. A man income is increased by Rs.5000 and at the same time, the rate of tax to be paid reduced from 10% to 9%.
He now pays the same amount of tax as before. What is his increased income if 20% of his income is exempted
from tax in both cases?
(1) Rs.30000 (2) Rs.60000 (3) Rs.40000 (4) Rs.50000 (5) Rs.20000
Q.19. 75 Litres of mixture contains 20% spirit and rest of water. If 5 ltrs of water be mixed with it, the percent of spirit
in the new mixture.
2 3
(1) 16 % (2) 18 % (3) 15% (4) 25% (5) 26%
3 4

Q.20. Total salary of A and B is Rs. 1500. A spends 90% while B spends 80% of his salary. If ratio of their savings are 3
: 4 then what is the salary of A and B?

(1) Rs. 900, Rs. 600 (2) Rs.600, Rs. 900 (3) Rs.800, Rs. 600 (4) Rs. 700, Rs. 900 (5) Rs. 600, Rs. 800
Q.21. Two numbers are less than the third number by 50% and 54% respectively.By how much percent is the second
number less than the first number?
(1) 13% (2) 10% (3) 12%
(4) Cannot be determined (5) None of these
Q.22. Aman's expense is 30% more than Vimal's expense and Vimal's expense is 10% less than Raman's expense. If the
sum of their expense is Rs. 6447, then what would be the Aman's expense?
(1) Rs. 2,200 (2) Rs. 2,457 (3) Rs. 1,890 (4) Rs. 2,100 (5) None of these
Q.23. Sumitra has an average of 56% on her first 7 examinations. How much she should make on her eighth examination
to obtain an average of 60% on 8 examinations?
(1) 88% (2) 78% (3) 98%
(4) Cannot be determined (5) None of these
Q.24. Two Candidates A and B participate in election. 80% of the total people, cast the votes, in which A received 55%
of the total votes caste. Difference between votes recieved by A and B is 1344. What is the number of registered
votes?

QUANTITATIVE APTITUDE 27
www.mahendras.org
(1) 14200 (2) 18200 (3) 16500 (4) 16800 (5) None of these
Q.25. On republic day, in a class only 45 student appear thus it is decided that each student will get number of chocolate
equal to 40% of the number of student present in class. After distribution of chocolate 45 more student join the
class so it is decided that now the new student will get only number of chocolates equal to the 10% of the total
student present in class. Find the total number of chocolate distributed in class.
(1) 1200 (2) 1210 (3) 1215 (4) 1220 (5) Data inadequate
Q.26. Mr Mishra spent 20% of his monthly income on petrol and household . Out of the remaining he spent 15% on
children’s education ,25% on transport and 20% on entertainment .He is left with an amount of Rs7,200 after
incurring above expences .What is his monthly income?
(1) Rs 1,44,000 (2) Rs 36,000 (3) Rs 7,2,000 (4) Rs 1,14,000 (5) None of these
Q.27. Vijayshree got 273 marks in an examination and got 5% more than the pass percentage. If Subhi got 312 marks,
then by what percentage above the pass marks did she pass the exam ?
(1) 9% (2) 12.5% (3) 20% (4) 25% (5) None of these
Q.28. Fresh fruits contain 75% while dry fruits contain 20% water. If the weight of dry fruits is 300 kg, what was its total
weight when it was fresh ?
(1) 900 kg (2) 850 kg (3) 920 kg (4) 960 kg (5) None of these
Q.29. The monthly income of Shyama and Kamal together is Rs. 28000. The income of Shyama and Kamal is increased
by 25% and 12.5% respectively. The new income of Kamal becomes 120% of the new salary of Shyama. What is
the new income of Shyama ?
(1) Rs. 12000 (2) Rs. 18000 (3) Rs. 14000 (4) Rs. 16000 (5) Rs. 15000
Q.30. Chhaya bought a watch costing Rs. 1404 including sales tax at 8% She asked the shopkeeper to reduce the price
of the watch so that she can save the amount equal to the tax. The reduction of the price of the watch is ?
(1) Rs. 108 (2) Rs. 104 (3) Rs. 112 (4) Rs. 120 (5) None of these
EXERCISE EXPLANATION
Q.1.(5) Method-I. x × 40 
 x × 140
By using formula-  x + 100  14 100 14
= ⇒ =
r 20  y × 80  27 y × 180 27
less % =
100 + r
× 100 =
100 + 20
× 100  y + 100  100

20 2
= × 100 = 16 % x 14 18
120 3 = ×
y 27 14
Method-II.
100% x 2
=
Since 20% more Two tools y 3 Ans.
120%
100 Method-II
Less % = 20 × ....... (To decrease any
110 14
Given fraction →
number, we, multiply with small number and 27
divide with large number.)
14 18 2
2 Original fraction = × =
= 16 % 27 14 3
3
Q.2.(1) Method-I: Q.3.(3) 5% of 2000 = Rs. 100
x Rs.100 is the rate of 5 kg. of tea.
Let the original fraction be then-
y 1 kg of tea = Rs.20 /kg.

28 QUANTITATIVE APTITUDE
www.mahendras.org
25 Amount spent on Food= Rs.2000
Q.4.(3) Since = 96× = 24
100 Amount spent on Children's education= Rs.1500
According to question Amount spent on Entertainment and Transport=
1 Rs.2600
Rs. 24 = kg.
2 Amount spent on Medical= Rs.1950
1
1 kg = Rs. 48= 24 = 48 Rs. Remaining= 10000 – (2000 + 1500 + 2600 + 1950)
2
Hence Original rate 1950 ratio= 8775
100 1 ratio= 4.5
= 48 × = 38.4 Rs.
125 100 ratio= 45000
Q.5.(3) According to the question, Q.10.(1) Expenditure = Price × Consumption
20% + 5 = 30% – 20 new expenditure = 110% of 50
10% = 20 + 5 = 25
So,
100% = 250
Passing marks = 75 – 20 = 55 110 125
× 50 = ×x
100 100
55
Required percentage = ×100
100 = 22%
250 x = 44

Q.6.(4) According to question, So, new consumption is 44 kg.


A – 0.4B = 1.4B R 20
Q.11.(3) % change = × 100 = × 100
A = 1.8B 100 – R 100 − 20
A:B=9:5
Difference (9 – 5 = 4) = 1860 20
= × 100 = 25%
80
14 = 1860 ×14= 6510 Q.12.(4) Let take number = 20
4
5 Actual answer = 20 × 5 = 100
Total registerted vote = 6510 × = 10850
3 20
New Answer = =4
Q.7.(2) Let the number of books in shelf B be 100. 5
∴ Number of books in shelf A = {(100 × 4)/5} = Error = 100 - 4 = 96
80 96
On transferring 25% i.e. of books of shelf A to % error = × 100 = 96%
100
shelf B, the books in shelf B = {100 + (80 × 20 × 20
25)/100} Q.13.(1) % change = + 20 + 20 +
100
B = 100 + 20 = 120 = + 20 + 20 + 4 = + 44%
Books left in shelf A = 80–20 = 60
Increased by 44%
Again, on transferring 1/4th books of shelf B to
shelf A, the books in Q.14.(4) % of students passed in one subject or both
shelf A = {60 + (120/4)} = 90 subjects = 45% + 50% = 95%
Total no of books in A and B = 120 +60 = 180 15% failed in examination,
Required percentage of books in shelf A = So passed % = 100 - 15 = 85%
(90/180) × 100
So, students passed in both subject
= 50%
= (95 - 85) = 10%
Q.8.(1) A = 75 – 20% of (75) + 40% of (175) = 130
So, 10% = 150
B = 175 + 20% of (75) – 40% of (175) = 120
1% = 15
Q.9.(5) Let his monthly income be Rs.10000
100% = 1500
QUANTITATIVE APTITUDE 29
www.mahendras.org
Q.15.(3) Each student got = 10% of 40 = 4 10A 3
savings =
each teacher got = 15% of 40 = 6 20B 4
So, total toffee = 4 × 40 + 6 × 2= 160 + 12 = 172 A 3
=
Q.16.(1) Quantity of water in solution = 20% of 120 B 2

Quantity of milk = 120 - 24 = 96 ltrs 3


A’s income is = × 1500 = Rs.900
Now solution contain 25% water, 5

So, milk % = 100% - 25% = 75% 2


B’s income is = × 1500 = Rs.600
5
So, 75% = 96 ltrs
Q.21.(5) Let the third number be 100.
96
1% = ∴ First number = 50
75
and Second number = 100–54 = 46
96
100% = ×100= 128 ltrs. Decrease = 50 –46 = 4
75
So, water should be added = 128 - 120 = 8 ltrs 4
∴ Required percentage= × 100 = 8%
Short Trick: (by alligation) 50

20% 100% Q.22.(2) Let Vimal's expense be Rs. 100.


∴ Aman's expense = Rs. 130
25
and Raman's expense
75 5 100 1000
15 : 1 = × 100 =Rs.
90 9
15 unit = 120 ltrs. ∴ Ratio of the expenses of Vimal. Aman and
1 unit = 8 ltrs. Raman respectively

Q.17.(5) Income Exp Saving 1000


= 100 : 130 : = 90 : 117 : 100
100 40 60 9
+20 +20% +20 +50% 0% ∴ Aman expense
120 60 60 117
= × 6447
That means no change in his saving. 90 + 117 + 100
Q.18.(4) Since, same percentage of his income is exempted 117
from tax in both cases, this data is not to be = × 6447 = Rs. 2457
307
considered. or
Ratio of interest = 10 : 9 1000
100+130+ = 6447
So, Ratio of income = 9 : 10 9
difference 1 unit = 5000 3070
i.e. = 6447
10 unit = 50000 9
20 6447 × 9 × 130
Q.19.(2) spirit = 75 ×
100
= 15 ltrs ⇒ 130 = = Rs. 2457
3070
New Water = 65 ltrs Q.23.(4) The question cannot be answered because total
marks are not givven.
New mixture = 15 + 65 = 80 ltrs
Total maximum marks obtained in 8 exams
15
% of spirit in new mixture = × 100 = 700+50 = 750
80
75 3 For the average percentage in 8 exams to be 60%,
= = 18 % the total marks obtained should be
4 4
Q.20.(1) Lets A's salary = 100 A 60
= × 750 = 450
B’s salary = 100 B 100

30 QUANTITATIVE APTITUDE
www.mahendras.org
∴ Marks to be obtained in the eighth exam should 312 − 260 52
be = × 100 = × 100 = 20%
260 260
450–392 = 58 Q.28.(4) Quantity of water in 300 kg dry fruits
But, 58 > 50 = (20/100)×300 = 60 kg
Quantity of fruit alone = 300 - 60 = 240 kg.
It means the marks to be obtained would be greater
than the maximum marks. This is impossible. 25 kg fruit piece in 100 kg fresh fruits
For 240 = (100 × 240)/25 = 960 kg.
Q.24.(4) Let the total no. of registered votes =x
Q.29.(5) The monthly income of Shyama and kamal
So total casted votes = 80% of x = 0.8x
S + K = 28000......(1)
Now (55-45)% of 0.8x = 1344 Shyama's income= x; Kamal's income= 28000-x
1344 × 10 K = 120/100× S........(2)
x= = 16800
0.8 S's new income
10 = (28000 -x)× 112.5/100
Q.25.(3) 45×45×+45×90× 100 = 810+ 405
(28000-x)×112.5/100
= 1215
= x × 125/100
Q.26.(5) Remaining salary = 80 × 40%=32
x = 12000
7200 New income of Shyama
100% = × 100 = 22500
32 = 125% of 12000 = Rs. 15000
Q.30.(2) 1.08x = 1404
Total monthly income = Rs 22500
x = 1300
100
Q.27.(3) Passing marks = 273 × = 260 The reduction of the price of the watch
105
Required % = 1404-1300 = Rs. 104

QUANTITATIVE APTITUDE 31
www.mahendras.org

CHAPTER

4 Profit and Loss

If 15% loss accrued on an article, it means :


Profit and loss
→ 100% → represent cost price (CP)
I. Cost price : The price at which an article is purchased
is called its Cost Price and denoted by CP. → 85% → represent selling price (S.P.)
or → 15% → represent loss i.e.,
For any person, Cost Price is an amount or quantity 100% (CP)
which firstly releases from the pocket of that person. i.e. Loss 15%
II. Selling Price : The price at which an article is sold, 85% (SP)

is called its selling price, and denoted by SP.
II. In the case of discount, the relation is generated
or between SP/MP
For any person, Selling Price is, an amount or quantity If 20% discount accrued on an article it means :
which come to the pocket of that person.
→ 100% → represent marked price (MP)
III. Profit or Gain : If S.P. is greater than CP, then Seller
is said to have a profit or gain. i.e. → 80% → represent selling price (SP)

Profit = S.P - C.P → 20%→ discount.

IV. Loss : If S.P.is less than C.P. the seller is said to have 100% (MP)
a loss. i.e. i.e. Discount 20%
80% (SP)
loss = C.P - S.P
V. Profit and loss are always counted on C.P. Ex. A man bought a cycle for Rs. 330. For how much
VI. CP is always 100% in the case of profit and loss. should he sell it so as to gain 10%?
VII. For any transaction there are two persons involved Sol. Profit percentage = x% = 10%
one is called buyer and second one is called seller. Cost Price = 100
VIII. Marked Price : MRP of an article is known as Marked Selling Price = 100 + x = 100 + 10 = 110
Price or labelled price or listed price and denoted by Required answer
MP.
330
IX. Discount always carried on MRP (MP) = × 110 = 363
36.3
100
X. MRP is always 100% in the case of discount.
Ex. A merchant professes to sell his goods at a loss of 10%
Observation: but weighs 750 gm in place of 1 kg. Find his real loss
I. In the case of profit or loss, the relation is generated or gain percentage?
between CP and SP. Sol. Let cost price of 1 kg = 1000
If 25% profit accrued on an article it means: But his giving product worth 750
→100% represent cost price (CP) So it's very clear that he is getting a gain.
→125% represent selling price (S.P.) SP=900
→25% represent profit CP=750

100% (CP) Now gain percent


i.e. Profit 25% = 900 − 750 ×100% = 20%
125% (SP) 750

32 QUANTITATIVE APTITUDE
www.mahendras.org
Ex. A person got 5% loss by selling an article for Rs. 1045. Ex. The percentage profit earned by selling an article for
At what price should the article be sold to earn 15% Rs. 480 is equal to the percentage loss incurred by
profit ? selling the same article for Rs. 240. At what price
Sol. Let CP = 100 SP = 95 should the article be sold to make 20% profit?

New SP = 115 Sol. This question is easy as profit and loss both are same
so CP will lie exactly between then
115 × 1045
Required Answer ⇒ x = = Rs. 1265 SP1 + SP2
95 CP =
2
Short Trick :
480 + 240
CP = = 360
2
120
SP = × 360 = 432
Ex. A person sold an article at profit of 15%. If he had 100
sold it Rs. 480 more, he would have gain 18%. What
Ex. A vendor bought bananas at 5 for Rs. 4 and sold them
is the cost price ?
at 4 for Rs. 5. Find his gain or loss percent.
Sol. Let the CP of an article be x Rs, then
Sol. Suppose, number of bananas bought
115% of x + 480 = 118% of x
= LCM of 5 & 4 = 20
118% of x - 115% of x = 480
FG 4 × 20IJ =16;SP=Rs. FG 5 × 20IJ = 25
3% of x = 480 CP=Rs.
H5 K H4 K
480
x= × 100 = Rs. 16000 25 − 16 9
3 % Profit = × 100 = × 100
Short Trick : 16 16
= 56.25%
Difference in S.P.
CP = Difference in Profit % × 100 Short Trick

480
CP = × 100 = Rs. 16000
3
Ex. If the CP of 15 articles is equal to the SP of 12 articles.
Find the gain or loss %.
Sol. Let the C.P of each article be Rs. 1.
Then CP of 12 articles = Rs. 12
SP of 12 articles = Rs. 15 Special Case: If a'th part of some items is sold at x% loss,
3 then required gain per cent in selling rest of the items
Gain % = × 100 = 25% in order that there is neither gain nor loss in whole
12
Ex. A person bought articles at 20 for a rupee. How many ax
transaction, is %
for a rupee must to sell to gain 25% ? (1 − a )
Sol. CP of 20 articles = Rs. 1, SP of 20 article = 125% of Rs. 1 Ex. A shopkeeper purchased medicines worth Rs. 9000
5 form a company. He sold 1/3 part of the medicine at
For Rs. , article sold = 20 30% loss. On which gain he should sell his rest of the
4
medicines, so that he has neither gain or loss?
4 Here a = 1/3 , x = 30 %
For Rs. 1 article sold = 20 × = 16
5
1
Short Trick : × 30
Required gain % = 3 = 15%
 1
1 − 
Special Case:  3
If 'a' part of an article is sold at x% profit/loss, 'b'
part at y% profit/loss and c part at z% profit/loss

QUANTITATIVE APTITUDE 33
www.mahendras.org
and finally there is a profit/lossof Rs.R, then Cost xy
price of entire article x+y-
100
R
= ×100 10 × 20
ax + by + cz 10 + 20 - = 28%
100
Ex. 33.33% of a commodity is sold at 15% profit,25% is Ex. A tradesman marks his goods at 25% above his
sold at 20% profit and the rest at 24% profit. If the 1
Total profit is Rs. 80 is earned then find the value of cost price and allows a discount of 12 % for cash
2
commodity? (in Rs.)
purchases. What profit does he make?
Sol. Part sold at 24% profit
Sol. Let CP=100
1 1 5
= 1−  +  = MP=125
 3 4  12
80 × 100 SP=87.5% of 125=109.375
Value of commodity = 1 = 400
1 5 Now required answer
×15 + × 20 + × 24
3 4 12
109.375 − 100
Ex. If 2/3 part of an article is sold at 30% profit, 1/4 part = × 100 = 9.375%
at 16% profit and remaining part at 12% profit and 100
finally, there is a profit of Rs.75, then find the cost Ex. A seller gives 2 oranges free on purchase of 10 oranges.
price of the article. Find equivalent discount.
Here a = 2/3 , x = 30 % , b =1/4 , y = 16 % , z = 12 % Sol. Clearly, the retailer get 1 dozen out of 6 dozens free.
and R = 75 Rs
Required CP of article 1
Hence discount% = × 100
6
75
=2 ×100 = 300 2
1 1 =16 %
× 30 + × 16 + × 12 3
3 4 12
Equivalent Discount : When two or more than two discount Short Trick
are given by seller then equivalant discount may be calculated 2
by following formula. × 100%
10 + 2
xy
x+y− 2
100 = 16 %
Note : 3
For more than two discount you can use this formula Ex. A shopkeeper earns a profit of 12% on selling a book
further. at 10% discount on printed price. The ratio of the cost
price to printed price of the book is ?
( order is not important)
Sol. C.P. of the book = Rs. x
Ex. Find the equivalent discount of 10% and 20%
Sol. Let the MP be = Rs. 100 Printed price = Rs. y

90 80 y × 90 x ×112
Then, Net SP = 100 × × = Rs.72 =
100 100 100 100

Required discount = (100 -72)%=28%
x 45
=
Short Trick y 56

34 QUANTITATIVE APTITUDE
www.mahendras.org
EXERCISE
Q.1. If the CP is 60% of the selling price. What is the profit%?
(1) 33. 60% (2) 66.67% (3) 63.27% (4) 60.67% (5) None of these
Q.2. If on selling 12 Notebooks any seller makes a profit equal to the selling price of 4 Note books. What is his percent
profit?
2
(1) 50% (2) 25% (3) 16% (4) Data inadequate (5) None of these
3
Q.3. Arun purchased a house for Rs. 75000 and a site for Rs. 15000 respectively. If he sold the house for Rs. 83000 and
the site for Rs. 10000. Find the resultant% of gain?
1 1
(1) 3% (2) 3 % (3) 30% (4) 33 % (5) None of these
3 3
Q.4. One fifth of the CP, one-seventh of the marked price and one-sixth of the selling price are equal. What is the gain%
or loss% to trader?

2 2
(1) 20% gain (2) 16 % less (3) 14 % profit (4) 10% loss (5) None of these
3 7
Q.5. Shri Ramlal purchased a TV set for Rs. 12500 and spent Rs. 300 on transportation and Rs. 800 on installation. At
what price should he sell it so as to earn an overall profit of 15%
(1) Rs. 14560 (2) Rs. 14375 (3) Rs. 15460 (4) Rs. 153758 (5) None of these
Q.6. A shopkeeper bought two radios for Rs. 1008, he sold one at a loss of 20% and other at a profit of 44%. If he sell
both radios at same price, then find the cost price of the radio which is sold at loss.
(1) Rs. 648 (2) Rs.360 (3) Rs.368 (4) Rs.640 (5) Rs.540
Q.7. A shopkeeper gets 20% more from wholesaler when he buys sugar. He uses a false weight of 750 gram while selling
1 Kg of sugar. Find the profit he earn if he had paid 10000 Rs to the wholesaler.
(1) Rs. 5000 (2) Rs. 7500 (3) Rs. 6000
(4) Cannot be determined (5) None of these
Q.8. A table is sold at a profit of 20%. If both the cost price and the selling price of the table are decreased by Rs. 100,
the profit would be 5% more, original cost price of the table is-
(1) Rs. 600 (2) Rs. 550 (3) Rs. 650
(4) Cannot be determined (5) None of these
Q.9. A shopkeeper marks up his goods by 20% and cheats by 10% while selling it. Find his total profit percent.
1
(1) 30% (2) 32% (3) 33 % (4) 35% (5) None of these
3
Q.10. A man sells an article at a profit of 25% if he had bought it at 20% less and sold it for Rs. 1050 loss, he would have gained
30% find the cost price of the article.
(1) Rs. 10000 (2) Rs. 5000 (3) Rs. 12000 (4) Rs. 10500 (5) None of these
Q.11. Marked price of an article is 60% more than the cost price. A shopkeeper allows X% discount and earns ____%
profit, if a shopkeeper allows 2X% discount and earns ___% profit. Which of the following options satisfies the
two blanks in the question?
A) 44, 28 B) 36, 12 C) 30, 16 D) 20, 12
(1) Only B (2) Only A, B (3) Only C) (4) Only D (5) Only A
Q.12. Divya went to a textile shop. When Divya asked the price of a saree the shopkeeper told her the price by increasing
18% of the original cost. But Divya insisted to decrease the price by 18% so the shopkeeper sold it by decreasing
the price by 18% what is the loss or profit percen of shopkeeper?
QUANTITATIVE APTITUDE 35
www.mahendras.org
(1) 3.25% (2) 2.5% (3) 3%
(4) 3.225% (5) None of these
Q.13. The market price of an article is 40% more than the cost price. There is a profit of x% after giving the discount of
15%. If the cost price of an article is Rs 1160 and its market price is 30% higher, then how many Rs will be the
benefit after giving the discount of x%?
(1) Rs 61.48 (2) Rs 42.25 (3) Rs 85 (4) Rs 48.75 (5) None of these
Q.14. A shopkeeper bought an article marked at Rs. 22800 at two successive discounts of 15% and 20%. He spent Rs.
1064 on transportation and sold the article for Rs. 20710. Then, what was the profit percentage of the shopkeeper?
(1) 35% (2) 30% (3) 25% (4) 20% (5) None of these
Q.15. A fruit seller wants to make a profit of 20%, after giving the discount in cash of 10% and giving 1 dozen banana
free on the purchase of 4 dozen bananas. What percentage above the cost price should he listed on bananas?
(1) 76.47% (2) 75% (3) 65% (4) 83.5% (5) None of these
Q.16. A book seller sells a book at a profit of 10%. If he had bought it at 4% less and sold it for 6 more, he would have
3
gained 18 %. The cost price of the book is-
4
(1) Rs.130 (2) Rs.140 (3) Rs.150 (4) Rs.160 (5) Rs.165
Q.17. A shopkeeper purchased 25 identical pieces of mobile phones at the rate of Rs. 1500 each. He spent an amount of
Rs. 2500 on transport and packing. He fixed the labeled price of each phone at Rs. 2000. However, he decided to
give a discount of 5% on the labeled price. What is percent profit earned by him?
(1) 18.75 (2) 16.75 (3) 14.75 (4) 20.75 (5) None of these
Q.18. A shopkeeper bought 30 kg of wheat at the rate of Rs. 45 /kg. He sold 40% of the total quantity at the rate of Rs.
50/kg. Approximately, at what price/kg should he sell the remaining quantity to make 25 percent overall profit?
(1) Rs. 54 (2) Rs. 52 (3) Rs. 50 (4) Rs. 60 (5) Rs. 56
Q.19. Rajani purchased a mobile phone and a refrigerater for Rs. 12000 and Rs. 10000 respectively. She sold the refrig-
erator at a loss of 12% and mobile phone at a profit of 8%. What is his overall loss/profit?
(1) Rs. 240 Loss (2) Rs. 250 Loss (3) Rs. 240 Profit
(4) Rs. 280 Profit (5) Rs. 360 Loss
Q.20. A trader purchases two watches. He makes the first one up by Rs.200 over the cost price and gives a discount of 20%
on it. The second one he marks up by 50% and gives a discount of Rs. 160. If he gains 15% on both the watches
put together of 8% on the first alone, what is the percent of profit on the second watch?
(1) 21% (2) 22% (3) 18.5%
(4) Can’t be determined (5) None of these
Q.21. A man sold two articles, each for the same price of Rs. 840. He earns 20% profit on the first and 40% profit on the
second. Find his overall profit percent.

1 1 3 3
%
(1) 29 (2) 28 % (3) 29 % (4) 28 13 % (5) None of these
2 2 13
Q.22. An article is listed at Rs. 65. A customer bought this article for Rs. 56.16 and got two successive discount of which
one is 10%. Find the other discount of this scheme.
(1) 4% (2) 5% (3) 3% (4) 6% (5) 8%
Q.23. A shopkeeper sold two article for Rs. 15000 each on first he gains 50% but on the other a loss of 25%. Find the
overall profit percent or loss percent.
2 2
(1) 2% (2) 16 % (3) 25% (4) 63 % (5) No profit no loss
3 5
36 QUANTITATIVE APTITUDE
www.mahendras.org
Q.24. A discount of 15% is given on the marked price of an article. The shopkeeper charges sales tax of 6% on the
discounted price. If the selling price be Rs. 1081.20 What is the marked price of the article?
(1) Rs. 11850.20 (2) Rs. 1250.20 (3) Rs. 302
(4) Rs. 1200 (5) None of these
Q.25. A trader who marks his goods upto 50% and offered discount of 20%. What percent profit the trader makes after
offereing the payment?
(1) 15% (2) 20% (3) 25% (4) 30% (5) 40%
1
Q.26. Article are marked at a price which gives a profit of 25%. After allowing a certain discount the profit reduces 12 %
,then find the discount percent. 2

(1) 10% (2) 15% (3) 20% (4) 18% (5) 25%
Q.27. A person purchased two chairs for Rs. 900. He sold first article at 4/5 of cost price but another at 5/4 of cost price
He gains overall Rs. 90. Find the cost price of Lesser valued chair.
(1) Rs.300 (2) Rs. 400 (3) Rs. 500
(4) Rs. 600 (5) Can’t be determined
Q.28. A property dealer sells a house for Rs.9,00,000 and in this he makes 12.5% profit. Had he sold it for Rs. 6,00,000,
then what percentage of loss or gain have be made ?
1 2
(1) 25% loss (2) 10% loss (3) 12 % gain (4) 16 % gain (5) None of these
2 3
Q.29. A shopkeeper sells one transistor for Rs. 960 at a gain of 20% and another for Rs. 1152 at a loss of 4%. His total
gain or loss percent is :
3 3 2 2
(1) 5 % loss (2) 5 % gain (3) 6 % gain (4) 6 % loss (5) None of these
5 5 3 3
Q.30. Profit earned by selling an article for Rs. 1060 is 20% more than the loss incurred by selling the same article for
Rs. 950. At what price should the article be sold to earn 20% profit ?
(1) Rs. 980 (2) Rs. 1080 (3) Rs. 1800 (4) Rs. 1200 (5) None of these

EXERCISE EXPLANATION
Q.1.(2) Let S.P of the article =x 4x
Hence profit percent = × 100 = 50%
60 8x
C.P. of the article = x
100 Q.3.(2) Total CP = Rs. 75000 + Rs.15000= Rs. 90,000
60 40x Total SP = Rs. 83000 + Rs. 10000= Rs. 93000
Then profit = x− x=
100 100
Hence profit percent Profit = Rs. 93000 - Rs. 90000= Rs. 3000
40x 3000 1
= P × 100 = 100 × 100 Profit percent = ×100 = 3 %
60x
90000 3
CP
100
Q.4.(1) According to question
CP 1 1
200
= 66.67% = MP = SP⇒ CP = 5
3 5 7 6 SP 6
Q.2.(1) Let S.P. of l notebook = x 1
Profit = 5 × 100 = 20%
S.P. of 12 Notebook = 12x
Q.5.(5) Total Cost price
Profit = 4x
= 12500 + 300 + 800 = Rs. 13600
then
115
CP = 12x - 4x = 8x Selling price= 13600 × = Rs. 15640
100

QUANTITATIVE APTITUDE 37
www.mahendras.org
Q.6.(1) 80% of x = 144% of (1008 - x), x = 648 Discount percentage = (160–112)/160 × 100
Q.7.(3) (100 + 20) − (100 − 25) = 30%
P% = × 100 = 60%
100 − 25 This satisfies the given condition
60 Q.12.(5) Loss = x2/100 = 182/100 = 3.24%
P = 10000 × = Rs.6000
100 Q.13.(1) Let cost price of the article is 100.
Q.8.(5) Let the cost price of table be x. Selling price So market price = 140
120x 6x Now discount = 15% of 140 = 21
= =
100 5 Now profit = 140 -21 = 119
So, x% = 19%
(II) Cost price = Rs. (x–100)
Now according to the question,
 6x  x
Profit =  − x  = Rs., Market price of article whose cost is Rs 1160
 5  5
x = 130% of 1160= Rs 1508

Now, 5 × 100 = 25 Then discount = 19% of 1508 = Rs 286.52


x − 100 So selling price = Rs 1221.48
2500
25x – 2500 = 20x, x = = Rs. 500 Now, profit = 1221.48-1160 = Rs 61.48
5
Q.9.(3) C.P. = 100, M.P. = 120, C.P. = 100–10=90 Q.14.(3) Marked price of the article = Rs. 22800
120 – 90 1 Cost price of the article
P% = × 100 = 33 %
90 3  15  80
=  22800 − 22800 × ×
Q.10.(2) Case - I 100  100
C.P S.P. Profit% =Rs. 15504

100 125 25% Actual cost price = 15504 + 1064 = Rs. 16568

80 104 30%
( 20710 − 16568) × 100 = 25%
According to question, Profit % =
16568
21 = 1050 Q.15.(5) Discount percentage when bought 4 dozen
1 = 50, 100 = 5000 bananas = 1/5 ×100

Q.11.(2) Cost price = 100 Marked price = 160 = 20%

Option (A): 1st case: Total discount percentage =28%

SP = 100 + 44/100 × 100 = 144 Let the M.P. of banana = Rs 100

Discount percentage = (160–144)/160 × 100 S.P. = 72


= 10%
100
2nd case: SP = 100 + 28/100 × 100 = 128 C.P. = 72× = 60
120
Discount percentage = (160 – 128)/160 × 100
= 20% 40
required % = ×100 = 66.66%
This satisfies the given condition 60

Option (B): 1st case: MP : CP = 120 : 72 = 5 : 3


SP = 100 + 36/100 × 100 = 136 Hence = 66.66%
Discount percentage = (160–136)/160 × 100 Q.16.(3) Let the C.P. of the book = Rs.x
= 15% 110x
Initial S.P. = Rs. = 1.1 x
2nd case: SP = 100 + 12/100 × 100 = 112 100

38 QUANTITATIVE APTITUDE
www.mahendras.org
New C.P. = Rs.0.96x 840
CP of IInd Article = × 100 = 600
 75  140
New S.P. = Rs. 100 +  % of Rs.0.96x Net Profit Percent
 4
380
475 = × 100 = 29 3 %
= × 0.96x = Rs.1.14x 1300 13
400
Q.22.(1) According to question
= 1.14 x – 1.1 x = 6
90 (100 − x )
0.4x = 6 65 × × = 56.16
100 100
6
x= = Rs.150 100 − x
0.04 58.5 × = 56.16
100
Q.17.(1) Total cost of 25 mobile sets
56.16 × 100
= 1500 × 25 + 2500 = 40000. (100 − x ) =
58.5
40000
Cost price of 1 mobile set = = 1600
25 100 - x = 96
Labeled price = 2000. x = 4%
S.P. 1 mobile = 95% of 2000 = 1900 Q.23.(5) SP of 1st Article = 15000
1900 − 1600 15000
profit % = × 100% = 18.75 CP of 1st Article = × 100 = 10,000
1600 150
Q.18.(4) C.P. of 30 kg wheat = 30 × 45 = Rs.1350
SP of IInd Article = 15000
total S.P. for an overall profit of 25%
15000
125 SP of IInd Article = × 100 = 20000
= 1350 × = 1687.5 75
100 Overall P/L = overall SP - overall CP
S.P of 12 kg of wheat = 12 × 50 = Rs. 600 = 30,000 - 30000 = 0
Expected S.P of 18 kg of remainig wheat that means No profit no loss
= 1687.5-600 = Rs. 1087.5 Short trick
1087.7 When SP1 = SP2
Required S.P. /kg = = Rs. 60
18 Profit percent ≠ Loss Percent
Q.19.(1) Total C.P= Rs (12000 + 10000) Then
= Rs. 22000 Profit or Loss
Total S.P 100 (P − L ) − 2PL 100 ( 50 − 25 ) − 2 × 50 × 25
= = = 0%
 12000 × 108 88  200 + P − L 200 + 50 − 25
=  + 10000 ×
 100 100  Q.24.(4) Let the marked price of the Article be
= Rs. 21760 = x then according to question
Since CP > SP 85 106
x× × = 1081.20
Hence, 100 100
Loss = Rs. 22000 - Rs. 21760 = Rs. 240 1081.20 × 100 × 100
x= = Rs. 1200
Q.20.(4) With this information P% can’t be determined. 85 × 106
Q.21.(3) SP of 1st Article = 840 Q.25.(2) Let CP = 100 MRP = 150
840 80
CP of 1st Article = 120 × 100 SP = 150× Rs. 120
100
= 700 Hence Profit percent = 20%
SP of IInd Article = 840 Short trick

QUANTITATIVE APTITUDE 39
www.mahendras.org
By successive formula 200000 FG
∴Required loss% = 800000 × 100 = 25%
IJ
x+y+
xy
= 50 − 20 +
50 × −20
= 30 - 10
H K
100 100
= 20% FG
100 IJ
Q.29.(2) C.P. of 1st transistor = Rs. 120 × 960
H K
Q.26.(1) Let CP = Rs. 100
= Rs. 800
MRP = Rs. 125
Since proift percent = 12.5% C.P. of 2nd transistor
So SP = 112.5 FG 100 × 1152IJ = Rs. 1200
then required discount %
= Rs.
H 96 K
125 − 112.5 12.5 So, total C.P. = Rs. (800 + 1200) = Rs. 2000
= × 100 = × 100 =10%
125 125
Total S.P. = Rs. (1152+960) = Rs. 2112
Q.27.(1) Let CP of 1st chair is x
FG 112 × 100IJ % = 5 3 %
According to question ∴Gain% =
H 2000 K 5
Then x × 4 + (900 – x ) × 5 = 990 Q.30.(4) Let C.P. be Rs. x
5 4
120
4x 5x Then (1060-x)= 100 (x-950)
– = 990 – 1125
5 4
⇒ 106000 - 100x = 120(x– 950)
16x − 25x
= −135 ⇒ 220x = 220000 ⇒ x = 1000
20

x = Rs. 300 FG 120 × 1000IJ


∴ Desired S.P. = Rs.
H 100 K
FG 100 × 900000IJ = Rs. 800000 =Rs. 1200
Q.28.(1) C.P.=Rs.
H 112.5 K
Notes

40 QUANTITATIVE APTITUDE
www.mahendras.org

CHAPTER Simple &


5 Compound
Interest
Principal: When you first deposit money in a saving acount years it will become 4 times?
or When you borrow some money from another person, bank, Sol. P = x, A = 2x
or any financial institute is known as principal.
x ×R×5
S.I. = x, x = R = 20%
Rate of Interest: The extra money paid by the borrower to the 100
lender for the use of money lent is called Interest. The rate of ...............(1)
interest is the percent charge, or paid, for the use of money. Again P = x
Simple Interest: When principal remains constant A = 4x
irrespective of time period for which interest is being S.I. = 3x
calculated.
x×R× T
This interest is always same for same period at the same rate 3x = 100
of interest. So it is known as simple interest denoted as S.I. 300 = R × T ..............(2)
Formula : From eq. (1) and (2)
P ×R× T T = 15 years
S.I = 100
SHort Trick :
Where P → Principal
n1 − 1 n2 − 1 2 −1 4 −1
R → Rate percent per annum = =
t1 t2 5 t2
T → Number of years

When S.I. is added to principal. It is change into amount. 1 3


=
5 t2
P + S.I. = A
S.I. = A - P t2 = 15 years
Case I : If S.I, R & T are known, By figure :
S.I × 100
P= 5 year 5 year 5 year
R× T P-x 2x 3x +x 4x
+x +x +x
Case II : If S.I., P & T, are known,
Total years = 5 + 5 + 5 = 15 year
S.I × 100
R= Ex. The simple interest is the 9/16 of the principal & the
P×T number of years is equal to rate %. Find the rate %.
Case III : When S.I. P, R are known Sol. Let P = x
S.I × 100 PRT
T= S.I. =
P ×R 100
Ex. Find simple interest on Rs. 5000 at the rate of interest 9 x×R×R
1 x =
4% for 2 years. 16 100
2
900 30 1
5 R2 = ⇒R= = 7 %
5000 × 4 × 16 4 2
Sol. S.I. = 2 = Rs. 500
100 Ex. A sum was invested at a certain rate of simple interest
Ex. A sum of money doubles itself in 5 years. In how many for two year. If the sum were invested at 3% more rate

QUANTITATIVE APTITUDE 41
www.mahendras.org
of interest, it would have fetehed Rs. 72 more. What LMF r IJ n OP
MNGH
is the sum? C.I. = P 1 + −1
Sol. According to question change in interest
100 K PQ
= 3% per annum in years A → Amount
Change in interest in 2 years P → Principal
= 3% × 2 = 6% r → Rate of Interest
that means n → Number of years
6% = 72 *Note
72 Type Rate Time
100% = × 100 = Rs. 1200
6 Yearly R n
Ex. Kumar borrowed a total amount of Rs. 30000 part of Half yearly R/2 2n
it on S.I. rate of 12 p.c.p.a. and remaining on simple
interst of 10 p.c.p.a.. at the end of 2 year he paid in Quaterly R/4 4n
all Rs. 36480 to settle the loan amount. What was the compount intrest may also be calculated by SI formula
amount borrowed at 12 p.c.p.a? and change of principal after each time period.
Sol. Given p = 30000 Note:
A = 36480 We can also use successive percentage net percentage
SI. = A-P = 36480 - 30000 = 6480 method to calculate equivalent simple rate of interest.
Let sum borrowed is x at 12 p.c.p.a xy
x+y+
100
according to question
12 (30000 − x ) × 10 × 2 = 6480 If r = 20%
x× ×2+
100 100 for 2 year equvilant rate of interst
⇒ 24x + 600000 - 20x = 648000 20 × 20
20+20 = 44%
4x = 48000 100
For 3 years
48000
x= = Rs. 12000 44 × 20
4 44 + 20 + = 72.8% and son on.
Compound Interest 100
Ex. Find C.I. on Rs. 16000 at the rate 5% p.c.p.a. for 2
As we discussed the S.I. is same for same time but in the case
years, compounded annually.
of compound Interest this is not happened. In the case of C.I.
Sol. By. Formula
The interest varies according to time eg.
LMF r IJ − 1OP n
Suppose we take 100 Rs. at the rate of compound interest
GMH
C.I. = P 1 + 100 K PQ
10% after one year we can not able to pay amount so, here N
C.I. is imposed on amount rate Ist year. ie. 10% is imposed
on 110 and so on......
L
= 16000 MFG 1 + 5 IJ − 1P
O 2

So. C.I. is equal to S.I. in first year MNH 100 K PQ


but after 1 year C.I. > S.I.
16000 × M
L 21× 21 − 1OP
In ather words we can that principal varies with each time = N 20 × 20 Q
period
Basic Formula :
L 441 O
= 16000 × M 400 − 1P

N Q
A = P 1+
FG r IJ n

= 16000 × 41 = 40 × 41
H 100 K
400
= Rs. 1640

42 QUANTITATIVE APTITUDE
www.mahendras.org
Short Trick : x × 12 × 2 x × 44
5%
800
for Ist year S.I. = C.I + = 11060
P = 16000
1st year 100 100
5% rate of interest
IInd year 800 + 40 24 x × 44
5% x× 100 + 100 =11060
C.I. = 800 + 800 + 40
= Rs. 1640 68
x× = 11060
Ex. Find C.I. on Rs. 10,000 at the rate of interest 6% for, 100
1 11060
1 2 years, compounded annually. x= × 100 = Rs. 16200
68
LMF r I − 1OP n

C.I. = MGH 100 JK


P 1+ Ex. Find the C.I. on Rs. 6000 at the rate of interest 5% for
Sol.
N PQ 3 years, compounded yearly.

LMF1+ 6 I − 1OP LMF R I − 1OP n

C.I. = MGH 100 JK


3/2
P 1+
= 10,000 GH 100 JK Sol.
N PQ

MN PQ
LF 6 I F1+ 6 × 1I − 1OP LMF1+ 5 I − 1OP 3

= 6000 GH 100 JK
1

= 10,000 × MG 1 +
MNH 100 JK GH 100 2 JK PQ MN PQ

L106 × FG1+ 3 IJ − 1PO


F 105 105 105 I
= 6000 GH 100 × 100 × 100 − 1JK
= 10,000 × M

N100 H 100 K Q
L106 × 103 − 1OP = 6000 × M
L 21× 21× 21 − 1OP
= 10,000 × M
N100 100 Q N 20 × 20 × 20 Q
= 6000 × G
F 9261 − 1IJ
= 10,000 × M
L10918 − 10,000 OP = Rs 918
H 8000 K
N 10000 Q F 9261− 8000 IJ
= 6000× G
Short trick :
H 8000 K
Ist Year
6% 6 × 1261 7566
P = 10,000 600 = = = Rs. 945.75
8 8
6% rate of interest
6%
Important Formula
300 + 18
1/2n year If difference between C.I. & S.I. is to find out.
Total = Rs. 918 Let the principal = P
Ex. Mr. Bashu invested a certain sum in Scheme X for 4 Time = T
year. Scheme X offers simple interest at the rate 12 Rate % p.a. = R
p.c.p.a. for the first 2 years and compound interest at
the rate 20% per annum for next two year. The total (1) When time is 2 years
interest earned by him after 4 years is Rs. 11060. What
C.I - S.I = P
FG R IJ 2
was sum invested.
H 100 K
Sol. Let sum invested be - x (2) When time is 3 years
According to question.
FG R IJ × FG 300 + RIJ
2

S.I. + C.I. = 11060 C.I - S.I = P H 100 K H 100 K


C.I for 2 year by successive effect Ex. The difference between the C.I. and S.I. for a period
20 × 20 of 2 years at the rate of 10% per annum is 50. Find
20+20+ = 44% the Principal.
100
Sol. Method I.
QUANTITATIVE APTITUDE 43
www.mahendras.org
By Slab. Method
F r IJ
2P = P G 1 +
15

P=?
10%
1st year
500
for Ist year S.I. = C.I
H 100 K
10% rate of interest
IInd year 500 + 50

F r IJ .............(1)
2 = G1 +
15

10% H 100 K
Hence principal = 5000 Again,
Method II
By formula 8P = P 1 +
FG r IJ n

2
H 100 K
 r 
D = P  
100  8 = 1+
FG r IJ n

2

H 100 K .............(II)
 10 
50 = P  using (I) and (II)
 100 
500000
P= FG1+ r IJ nR|F r I
= 8 = 2 = SG 1 +
15 U| 3

|TH 100 JK V|
100 3
P = Rs. 5000

H 100 K W
Ex. The difference between S.I. & C.I. on a certain sum
of money for 3 years at 10% is Rs. 31. Find the sum FG1+ r IJ = FG1+ r IJ
n 45

of money.
H 100 K H 100 K
Sol. From formula for 3 years.
x = 45 years
F r IJ FG 300 + r IJ
CI. - S.I. = PG
2

H 100 K H 100 K IInd method :

F 10 IJ × 310 2 LMn = bn g OP t2

31 = P GH
1 t1

100 K 100 N 2
Q
100 310 n2, n1 — number of times
31 = P ×
100 × 100 100 t1, t2 — number of years.
P = Rs 1000 t2 t2 3
bg
t2
Short Trick: 8= 2 15 23 = 2 bg 15
15 = 1
10%
t2 = 45 years
Ex. SI of 2 years is 300 and CI is 320. Find
P=? 1st 100 10%
(i) Rate of Interest

2nd + 10 10%
(ii) Principal
Sol. 150
3rd year + 10 + 10 + 1 150 20

20 40 1
(i) r = × 100% = = 13 %
Ex. A sum of money doubles itself at compound interest 150 3 3
in 15 years. In how many years will it become eight
times. 3
(ii) 150 × × 100 = Rs. 1125
40
FG
A = P 1+
r IJ n
Sol.
H 100 K

44 QUANTITATIVE APTITUDE
www.mahendras.org
EXERCISE
Q.1. A sum of Rs 7930 is divided into 3 parts and given an loan at 5% S.I. to A, B and C for 2, 3 and 4 years respect. If
amounts of all three are equal of the their respective periods of loan, A received a loan of -
(1) Rs. 2760 (2) Rs. 3560 (3) Rs. 2660 (4) Rs. 3450 (5) Rs. 2680
Q.2. A sum of Rs. 1550 was lent partly 5% and partly at 8% SI. The total interest received after 3 years is Rs. 300. The
ratio of money lent at 5% to that at 8% is -
(1) 15 : 14 (2) 15 : 13 (3) 16 : 15 (4) 5 : 6 (5) None of these
Q.3. A man invested equal sum of money at 12% per annum on compound interest and simple interest. After three years
the difference between the interest is Rs. 4449.28. Find the total amount he invested.
(1) Rs.11000 (2) Rs.10000 (3) Rs.12000 (4) Rs.12100 (5) Rs.15000
Q.4. If the simple interest on a certain sum of money at the rate of 4% per annum for 3 years is Rs. 1875. What would
be the C.I. at the same rate for the same time?
(1) Rs. 1950 (2) Rs. 1951 (3) Rs. 1975 (4) Rs. 1976 (5) Rs. 1900
Q.5. What is the sum of money at compound interest will give interest Rs. 6238 in 3 years, if the rate of interest is 3%
for the first year, 4% for the second year and 5% for the third year?
(1) Rs. 5000 (2) Rs. 10000 (3) Rs. 25000
(4) Rs. 50000 (5) Cannot be determined.
Q.6. Two partners X and Y invested their capital. X invested Rs. P at 10% per annum, Y invested Rs. (P – 4000) at the
rate of 12% per annum. If they both get total compound interest Rs 6412.8 at the end of two years, then find capital
invested by Y?
(1) 16000 Rs. (2) 12000 Rs. (3) 10000 Rs. (4) 14000 Rs. (5) 18000 Rs.
Q.7. A sum of Rs.7000 is deposited in two schemes. One part is deposited in Scheme A which offers 8% rate of interest.
Remaining part is invested in Scheme B which offers 10% rate of interest compounded annually. If interest
obtained in scheme A after 4 years is Rs.226 more than the interest obtained in scheme B after 2 years, find the
part deposited in scheme B.
(1) Rs.2800 (2) Rs.3850 (3) Rs.3580 (4) Rs.3800 (5) None of these
2
Q.8. The difference between the compound interest and simple interest on a certain sum of money for 3 years at 6
3
% per annum is Rs.184. Then find out the sum.
(1) Rs.13500 (2) Rs.12500 (3) Rs.11500 (4) Rs.10500 (5) None of these
Q.9. What will come at place of blank?
Rs_ is invested at 12.5% compound interest. After 3 years the sum becomes Rs. 5467.5.si?
(1) 3840 Rs. (2) 3520 Rs. (3) 4080 Rs. (4) 4250 Rs. (5) 3675 Rs.
Q.10. A man spends 85% of his monthly income, and the remaining money is deposited in the savings account for 5
years and he gets a simple interest of Rs. 2160 at the rate of 12% per annum. Out of monthly expenditure, he
spends 20% on rent, 15% on food. Out of the remaining he spends 50% on clothes and remaining on education.
Find the 33.33% of money that is spent on food.
(1) 1230 Rs. (2) 1530 Rs. (3) 1020 Rs. (4) 640 Rs. (5) None of these
Q.11. ` 600 becomes ` 708 in 3 years at certain simple rate of interest. If the rate of interest is increased by 5% what
amount will ` 600 become in 3 years?
(1) ` 758 (2) ` 798 (3) ` 804 (4) ` 850 (5) None of these
Q.12. A sum of Rs.31500 was lent partly at 7% and partly at 11% p.a. simple interest. The total interest received after 4
years was Rs.12708. Find the ratio of money lent at 7% to that lent at 11%.
(1) 4: 25 (2) 7: 29 (3) 11: 25 (4) 8: 27 (5) 13: 27
Q.13. Arun invested a sum of money at a certain rate of S.I. for a period of 4 year. Had he invested the same sum for a
period of 6 years, the total interest earned by him would have 50% more than earlier interest amount. what was the
rate of interest per annum?
QUANTITATIVE APTITUDE 45
www.mahendras.org
(1) 4% (2) 8% (3) 5%
(4) can't be determined (5) None of these
Q.14. Mukesh borrowed some money at the rate of 4 p.c.p.a for the first 3 year, at the rate of 8 p.c.p.a. for the next 2 year
and at the rate of 9 p.c.p.a for the period of beyond 5 year. If he pays a total S.I. of Rs. 19550 at the end of 7 years.
How much money did he borrow?
(1) Rs. 39500 (2) Rs. 42500 (3) Rs. 41900 (4) Rs. 43000 (5) Rs. 42000
Q.15. Dhoni investes two equal amounts in two banks giving 10% and 12% rate of interest respectively. At the end of
the year the interest earned is Rs. 1650 /- Find the sum invested in each bank.
(1) Rs. 8500 (2) Rs. 15000 (3) Rs. 7500 (4) Rs. 17000 (5) None of these
Q.16. The simple interest accrued on a certain principal is Rs 6400 in four years at 8 p.c.p.a.What would be the C.I accrued
on that principal at 2 p.c.p.a in 2 years?
(1) Rs 808 (2) Rs 750 (3) Rs 980 (4) Rs 720 (5) None of these
Q.17. What annual installment will discharge a debt of Rs. 2360 due in 4 years. at 12% p.a. simple interest ?
(1) Rs. 400 (2) Rs. 500 (3) Rs. 600 (4) Rs. 300 (5) Rs. 700
Q.18. A sum is invested for 3 years at the rate of 20% p.a. The difference between the simple interest and the compound
interest on same principal is Rs. 408. Find the principal.
(1) Rs. 5525.5 (2) Rs. 2550 (3) Rs. 3187.5 (4) Rs. 3400 (5) None of these
Q.19. The simple interest accrued on an amount of Rs. 40,000 at the end of 3 years is Rs. 12000. What would be C.I.
accrued on the same amount at the same rate in the same period?
(1) Rs. 18765 (2) Rs. 15350 (3) Rs. 21555 (4) Rs. 13240 (5) None of these
Q.20. The compound interest accrued on an amount of Rs. 22000 at the end of two years is Rs. 5596.8. What would be
S.I. accrued on the same amount at the same rate in the same period?
(1) Rs. 5420 (2) Rs. 5360 (3) Rs. 5280 (4) Rs. 5140 (5) None of these
Q.21. Mr. Basant invested Rs. 20,000 with rate of interest at 20 p.c.p.a. If the interest was compounded half yearly for
first year and in the next year it was compounded yearly. What will be the total interest earned at the end of two
years?
(1) Rs. 8800 (2) Rs. 9040 (3) Rs. 8040 (4) Rs. 9800 (5) Rs. 9500
Q.22. A sum of money Rs. 10200 is invested in 2 schemes. Ist scheme has offered compound intrest at the rate 10% for
2 years and IInd scheme has offered S.I. at the rate 5% per annum for 6 years. From both scheme equal interest is
obtained. Find the sum invested at simple interest.
(1) Rs. 4200 (2) Rs. 4800 (3) Rs. 6000 (4) Rs. 5800 (5) Rs. 5000
Q.23. A person invested equal amounts in two schems A and B at the same rate of interest. Scheme A offers S.I. while
scheme B offers compound interest. After two years he got Rs. 1920 from scheme A on interest and Rs. 2112
from scheme B. If the rate of interest is increased by 4%. What will be the total interest after two years from both
schemes?
(1) Rs. 4884.48 (2) Rs.4888.48 (3) Rs.4884.84 (4) Rs. 4384.48 (5) None of these
Q.24. A man has a debt of Rs. 820 which he undertakes to pay back with compound interest at the rate of 5% per annum
in 2 equal yearly installments at the end of first year and second year. What is the amount of each installment?
(approximately)
(1) Rs. 370 (2) Rs. 381 (3) Rs. 390 (4) Rs. 395 (5) Rs. 400
Q.25. The compound interest accrued on an amount at the end of two years at the rate of 12% per annum is Rs. 2862.
What is the amount?
(1) Rs. 11250 (2) Rs. 1220 (3) Rs.13500 (4) Rs. 10000 (5) None of these

46 QUANTITATIVE APTITUDE
www.mahendras.org
EXERCISE Explanation
Q.1.(1) According to question. 21P 25.44 ] -
g
100 + 100 P 4000 = 6412.8
110% a = 115% of b = 120% of C = K
21P + 25.44 P- 101760 = 641280
Hence 46.44 P = 743040
1 1 1 1 1 1 P= Rs. 16000
a:b:c= : : = : :
110 115 120 22 23 24 Capital invested by Y = (16000-4000)
= 23 × 24 : 22 × 24 : 22 × 23 = Rs. 12000
_7000 - x i ×8×4
>d1 + 10 n - 1H + 226
2
=276 : 264 : 253 Q.7.(4) = x
100 100
Loan recieved by
7930 32x 21x
A= × 276 = Rs. 2760 70×8×4 – 100 = 100 + 226
793
53x
Q.2.(3) x × 5 × 3 (1550 − x ) × 8 × 3 2240 - 226 = 100
− = 300
100 100 x = Rs.3800
x = 800 Q.8.(1) According to the question,
2
100 100
P = 184× f 20 p × f 300 + 20 p
800 16
Required Ratio = =
750 15 3 3
By alligation = Rs.13500
100 100 100
Q.9.(1) 5467.5× 112.5 × 112.5 × 112.5 = 3840
Q.10.(3) 5Y=2160
1Y=432
12%=432

= 16 : 15 100%=3600 (saving)
now 15% of income=3600
2 85% of income=20400 (expenditure)
Q.3.(2) d = P  r   300 + r 
 100   100  expenditure on food=15% of 20400=3060
2 33.33% of 3060=1020
12   300 + 12 
449.28 = P     600 × 5 × 3
 100   100  Q.11.(2) S.I. on 5% interest = = ` 90
P = 10000 100
Q.4.(2)
1875
×12.4864 = 1951 Total S.I. = (708–600) + 90= ` 198
12
Total amount = 600+198= ` 798

Q.5.(4) 3× 4 Q.12.(4) Let the sum lent at 7% be x


3+ 4+ = 7.12
100 then,
7.12 × 5 x × 7 × 4 (31500 − x ) × 11 × 4
7.12 + 5 + = 12.476 + = 12708
100 100 100
6238
P= × 100 = Rs.50000 28x (31500 − x ) × 44
12.476 + = 12708
100 100
Q.6.(2) X invested = P Rs.
Y invested = (P-4000) 28x + 31500 × 44 − 44x
= 12708
10×10 100
2 years CI on 10% = 100 = 21%
12×12 - 16x = 1270800 - 1386000
2 year CI on 12% = 12+12 + 100 = 25.44%

QUANTITATIVE APTITUDE 47
www.mahendras.org
115200 2

Q.18.(3) Principal = 408 × 


x= = 7200 100   100 
16   
 20   20 + 300 
Required ratio
408 × 100 × 100 × 100
7200 7200 = = Rs. 3187.5
= = 20 × 20 × 320
(31500 − 7200) 24300 = 8 : 27 Q.19.(4) According to question
Q.13.(4) A ccording to the question. S.I. × 100 12000 × 100
R= = = 10%
P × R × 6 P × R × 4 150 P×T 40000 × 3
= × Now for C.I. We use Slab Method.
100 100 100
10%
This relation give no result.
Q.14.(2) If the amount borrowed be Rs x then. P = 40000 1st 4000 10%
According to question.
2nd 4000 400 10%
x×4×3 x×8×2 x×9×2
+ + = 19550
100 100 100
3rd year 4000 400 + 400 + 40
12x+16x+18x = 1955000

46x = 1955000
CI = Rs. 13240
x = Rs. 42500
Q.20.(3) According to question,
Q.15.(3) Let sum be Rs. x
Rate of interest of C.I. for 2 years
According to question
5596.8
x × 10 × 1 x × 12x1 = × 100 = 25.44%
+ = 1650 22000
100 100
that means
1650 × 100
x= = Rs. 7500 Rate of interest / anum = 12%
22
Now
S.I. for 2 years = 24%
6400 × 100
Q.16.(1) P = = Rs 20000 22000 × 24
8×4 S.I. = = Rs. 5280
100
 2 
2
 Q.21.(2) According to formula
C.I = P  100  − 1
 1 +
   n
 R  
2
10   20 
A = P 1 + = 20000 +  1 +
= 20000 × 0.0404= Rs. 808  100   1
100   100 

Q.17.(2) Let the installment be Rs. 100:


11 11 6
Installments paid at the end of 1, 2, 3 and 4 yers = 20000 × × × = 29040
10 10 5
earn a simple interest at 12% p.a. for 3, 2, 1, 0
C.I = 29040- 20000 = Rs. 9040
year respectively.
Short Trick :
Hence the respective installments amount to -
10%
(100+3×12) + (100+2×12) + (100+1×12) + 100
= 236 P = 20000 1st 2000 10%
136+124+112+100 = 2360
2nd 2000 200 20%
472 = 2360
2360 3rd year 40000 4 0 + 400 + 40

100 = 472 × 100 = Rs. 500


C.I = Rs. 9040
48 QUANTITATIVE APTITUDE
www.mahendras.org
Q.22.(1) Let amount X is invested at C.I. 24 × 24
and amount Y is invested at S.I. again rate of C.I. for 2 year = 24 + 24 +
100
then = 48 + 576 = 53.76%

 2
 Hence
r 
  1
C.I. = P  100 +  − 1 53.76
  New C.I. = 4800 × = 2580.48
100
 10 
2
 So. Total interest = 2304 + 2580.48= Rs. 4884.48.
= x   1 +  − 1 ---- (1)
 100   Q.24.(2) By formula
again D
y×5×6 I=  r  
1
r 
2

S.I. = - (2)  1 + +
  1 + 
100 100   100 
According to question
820
from eq (1) and (2) = 1 2
 5   5 
  y×5×6  1 +  +  1 +
2
10  
x  1 +  − 1 = 100 100 

 100   100 820
x : y = 10 : 7 = 21 21 21
+ ×
Hence 20 20 20

10200 820 820 820 × 400


sum invested at S.I. = × 7 = 4200
17 = 21 441 = 420 + 441 = 861
+
Short Trick : 20 400 400

S.I. for 6 years = 5% × 6 = 30% = 380.95 ≈ 381
10 × 10 Q.25.(1) According to formula
C.I. for 2 year = 10 + 10 × = 21%
100
According to question  r 
n

C.I. = P  100  − 1
 1 +
21% of x = 30% of y  
x : y = 10 : 7  2

12 
10200 × 7 
2862 = P 
1 +  − 1
Hence sum invested at S.I. =  100 
17
= Rs. 4200
 28  2 
Q.23.(1) Difference between S.I. and C.I. for 2 years 2862 = P    − 1
 25  
= 21112 - 1920 = 192
1920 2862 × 625
S.I. for 1 year = = 960 P= = Rs. 11250
2 159
192 Method II
Hence r = × 100 = 20% per annum.
960 By Successive formula, net C.I rate person for 2 years
Now,
xy 12 × 12
= x +y+
S.I. (20%) = 960 100 = 12 + 12 + 100 = 25.44%
960
P (100%) = 20 × 100 = 4800 Now given.

When rate is increased by 4% then 25.44% = 2862

4800 × 24 × 2 2862
New S.I. = = 2304 100% = ×100 = Rs. 11250
100 25.44

QUANTITATIVE APTITUDE 49
www.mahendras.org

CHAPTER

6 Average

Average is equal distribution of the overall value among all Ex. The average score of a cricketer for 10 matches is 38.9
the things or persons present there. So the formula for finding runs. If the average for the first six matches is 42 then
the average is as follows : find the average for the last four matches.

Average =
FG Sum of observations IJ Sol. Required average
H Number of observations K
38.9 × 10 − 42 × 6
= 34.25
4
If there are average is also known as mean observation x1,
x2,x3....xn Ex. The average age of 80 girls was 20 years, the average
age of 20 of them was 22 years and another 20 of them
x1 + x 2 + ... + xn was 24 years. Find out average age of the remaining
Average =
n girls?
Ex. What is average of 40, 42, 38, 48. Sol. Total age of remaining 40 girls
40 + 42 + 38 + 48 =(80 × 20 - 20 × 22 - 20 × 24) = 680 years
Sol. Average = = 42
4 Average = 680 = 17 years
Ex. There are two sections A and B of a class, consisting of 40
10 and 20 students respectively. If the average weight
of sections A is 60 kg and that of section B is 45 kg. Short Trick:
Find the average weight of the whole class (in kg). 22 + 24
Average of 40 girls= = 23 years
60 × 10 + 45 × 20 2
Sol. Average = Average of 40 girls is 3 more than actual average so average
10 + 20
of remaining 40 girls is 3 less than actual average 20 - 3
600 + 900
= 50 = 17 years
30
Ex. A batsman makes a score of 97 runs in the 21th inning Concept of Consecutive Term
and thus increases his average by 2. Find his average (A) To solve consecutive terms problem, we will use exact
after 21th inning. mid concept. Exact mid term is average of given term.
Sol. Let the average after 21th innings = x. Then,  number + 1
Exact mid =   terms
Average after 20 innings = (x - 2)
th
 2
Average 6 +1
Exact mid of 6 terms = = 3.5
= 20 (x-2)+97 = 21x, 20x-40+97 = 21x 2

x = 57 3.5 many in between 3rd and 4th number exact mid of


9 + 1 th
Ex. A student was asked to find the arithmetic mean of the 9 terms = = 5 term
2
numbers 3,11, 7, 9,15,13, 8,19, 17, 21,14 and x. He
found the mean to be 12. What should be the number Ex. Average of 5 consecutive terms is 43.
in place of x? So smallest no is-
Sol. 12=(3+11+7+9+15+13+8+19+17+21+14+x)/12 5 +1
Sol. Exact mid of 5 terms = =3
Number in place of x is 2
1 2 3 4 5
137+x=144 , x = 144-137
41 42 exact mid term = 43
x=7.
Smallest is 41
50 QUANTITATIVE APTITUDE
www.mahendras.org
(B) Average of consecutive terms: Average decreased by = 21 - 20 = 1
1st term + last term Total decreased = 1 × 4 = 4
=
2 That means excluded no is 4 more than average.
Ex. Find the average of all the numbers from 20 to 90 So, excluded number = 21 + 4 = 25
which is divisible by 3.
Ex. The average of 12 no. is 45 when one no is excluded
Sol. First term is 21 and last is 90. then average becomes 46. What is the excluded
21 + 90 number?
So, average = = 55.5
2 Sol. In this case average increased by = (46-45) = 1
Ex. Find the average of all the numbers from 20 to 90
Total increased = 1 × 11 = 11
which is divisible by 8.
That means excluded number is 11 less than average.
Sol. First term 24 and last is 88.
Number is 45 - 11 = 34
24 + 88
So, average = = 56 Ex. Average marks of 35 students is 70. If marks of one
2
Ex. Find the average of all the numbers from 20 to 90 student is excluded then average increased by 2 marks.
which divisible by 6. What is the marks of excluded student?
Sol. First term is 24 and last is 90. Sol. Excluded student marks
24 + 90 = 70 – (35 - 1) × 2= 70 – 68 = 2
So, average = = 57
2 Ex. The average age of 12 persons is decreased by 2 years
Ex. The sum of eight consecutive even number 288. What when one of them having age 39 years is replaced by
is the sum of different set of six consecutive numbers a new person. Then what is the age of new person?
whose lowest number is 10 more than the mean of set
A? Sol. Age of new person = 39 - (12 ×2)
288 = 39 - 24 = 15 years
Sol. Mean of set A = =36
8 Ex. The average weight of 10 persons is increased by 2.5
So, lowest number of other set = 36 + 10 = 46 kg when two of them having weight 60 and 75 kg
Required sum = 46 + 47 + 48 + 49 + 50 + 51 = 291 replaced by two new persons. Then what is the average
weight of new persons?
Concept of addition or removal of items and change in
average: Sol. Sum of age of new persons = (60+75) + (2.5×10) =
The excluded or included item= Average + extra 135 + 25 = 160
(Extra means total change in average) Average age = 160/2 = 80 kg
Ex. Average of 5 no. is 21 one no is excluded then average Ex. The average of 50 numbers is 38. It was found that
becomes 20. What is the excluded no. one number is misread as 89 instead of 39. The correct
Sol. Average is 21. mean is?
So, if the excluded number is 21, then there will be  89 − 39 
no change in average. Sol. New average = 38 -   = 38 - 1 = 37
 50 
But new average is 20.

QUANTITATIVE APTITUDE 51
www.mahendras.org
EXERCISE
Q.1. Find the average of first 10 prime numbers.
(1) 12.1 (2) 11.9 (3) 12.9 (4) 30.25 (5) None of these
Q.2. Out of four numbers, the average of first 3 numbers is thrice the fourth number. If average of all four numbers is
5, what is fourth number?
(1) 4.5 (2) 4 (3) 5 (4) 2 (5) None of these
Q.3. Average age of students in section A of 55 students is 12 and the average age of students in section B of 45 students
is 14 years. Find the average age of students in both section taken together?
(1) 13 years (2) 12.5 years (3) 13.2 years (4) 12.9 years (5) 13.5 years
Q.4. The average monthly income of a family was Rs. 5000 during first 2 months, Rs. 6000 during next 4 months and
Rs. 7000 during last 6 months of the years. If total saving is Rs. 10000, Find average monthly expenditure.
(1) Rs. 550 (2) Rs. 5000 (3) Rs. 5500 (4) Rs. 5550 (5) Rs. 5555
Q.5. In an art exhibition average visitors from Monday to Friday is 620 and 900 in remaining days. Find out average
visitor in per day?
2
(1) 700 (2) 750 (3) 800 (4) 666 (5) None of these
3
Q.6. The average of six consecutive odd no is 48. Then find out largest and smallest no.
(1) 57,47 (2) 53,41 (3) 55, 45 (4) 53, 45 (5) None of these
Q.7. Find the average of numbers lies between 100 and 300 as well as divisible by 7.
(1) 199.5 (2) 200 (3) 205 (4) 203 (5) 2015
Q.8. Set A, in which 6 consecutive odd number, has average 76. The smallest number of Set B in which 6 consecutive
even number, is 33 less than the highest number of set A, what is the average of the numbers of set B?
(1) 53 (2) 67 (3) 93 (4) 89 (5) None of these
Q.9. The average weight of 29 students is 40 kg. If the weight of teacher be included the average weight increased by
400 gms.The weight of the teacher is-
(1) 52 kg (2) 55 kg (3) 50 kg (4) 49 kg (5) None of these
Q.10. A cricketer whose bowling average is 24.85 runs per wicket, takes 5 wickets for 52 runs in last match and there by
decrease his average by 0.85. The number of wickets taken by him before the last match was?
(1) 85 (2) 75 (3) 95 (4) 80 (5) None of these
Q.11. Batting average of Rohit Sharma in three ODI series is 48, 37.5 and 51 respectively. Of the number of matches in
these ODI series are in ratio 3:4:2. Find his overall average in these three series.
(1) 50.5 (2) 49.25 (3) 44.4 (4) 46.75 (5) None of these
Q.12. The average age of a family consisting 15 person is 42 years, when two person aged 36 years and x years is replaced
by 2 new person of average age 21 years of another family then average increases by 1 year. Find the value of x .
(1) 13 (2) 11 (3) 7 (4) 9 (5) None of these
Q.13. Find the average of first 120 natural numbers which is multiple of 7.
(1) 29600 (2) 72800 (3) 92820 (4) 50820 (5) None of these
Q.14. Virat played three matches in a tournament .The respective ratio between the scores of 1st and 2nd match was 6: 5
and that between the scores of 2nd and 3rd match was 3: 2.The difference between the 1st and 3rd match was 128
runs. What was Virat’s average score in all the three matches?
1 1 1 1 1
(1) 223 (2) 226 (3) 229 (4) 230 (5) 231
3 3 3 3 3
Q.15. The average age of a family of 4 member is 25 years. If the present age of youngest member is 7 years, then what
was the average age of the family at the time of birth of the youngest member?
52 QUANTITATIVE APTITUDE
www.mahendras.org
(1) 18 years (2) 24 years (3) 20 years (4) 22 years (5) None of these
Q.16. The average of six number is 40. If average of first three is 35 and last four is 45. Then third number is ?
(1) 35 (2) 40 (3) 45 (4) 50 (5) 60
Q.17. The average of 10 numbers is 43. Average of first four is 40 and last five is 47. Then 5th number is-
(1) 35 (2) 38 (3) 32 (4) 36 (5) 40
Q.18. The sum of 7 consecutive odd numbers is 539, also average of four consecutive even no. is 93. What is sum of
smallest even number and 3rd largest odd number.
(1) 629 (2) 631 (3) 627 (4) 169 (5) None of these
Q.19. The average marks of Rahul, Manu and Sheetal is 73. Rahul's marks is 18 less than Ankit and 12 more than Manu.
If Ankit got 30 marks more than the average marks of Rahul, Manu and Sheetal, what is the sum of the marks of
Manu and Sheetal?
(1) 130 (2) 132 (3) 134 (4) 138 (5) None of these
Q.20. The average marks obtained by 120 students was 37. If the average of passed candidate was 42 and failed candidates
is 18, the number of candidates who failed in the examination is
(1) 85 (2) 70 (3) 75 (4) 95 (5) 25
Q.21. The average score of a class of boys and girls in an examination is 42. The ratio of boys and girsl is 3 : 2. If the
average score of boys is 46, the average score of girls?
(1) 40 (2) 39 (3) 36 (4) 48 (5) 38
Q.22. In the first 10 overs of a cricket game, the run rate was 6.8. What should be the run rate in the remaining 40 overs
to reach the target of 350 runs?
(1) 6.05 (2) 5.05 (3) 7.05 (4) 8.05 (5) 9.05
Q.23. The average salary of all the workers in a workshop is Rs. 8000. The average salary of 12 officer is 15000 and
average salary of non officer is 7000. The total number of non-officers is
(1) 82 (2) 84 (3) 86 (4) 88 (5) 90
Q.24. A number of friend decided to go on a picnic and planned to spend Rs.96 on eatables. Four of them did not turn
up. As a consequence the remaining ones had to contribute Rs. 4 each extra. The number of those who attend the
picnic was-
(1) 12 (2) 16 (3) 6 (4) 8 (5) 24
Q.25. In a school with 300 students, the average age of boys is 16 years and that of the girls is 14 years. If average age
of class is 14 years 8 month then the number of boys in the school is
(1) 60 (2) 80 (3) 100 (4) 120 (5) 140
EXERCISE Explanation
2 + 3 + 5 + 7 + 11 + 13 + 17 + 19 + 23 + 29 9d + d = 20, 10d = 20, d = 2
Q.1.(3) Sum =
10
Q.3.(4) Average = 55 × 12 + 45 × 14
129 55 + 45
Average = = 12.9
10
660 + 630 1290
Q.2.(4) Let the numbers are a, b, c and d. = = = 12.9 years
100 100
According to the question, Q.4.(3) Total income = 5000× 2 + 6000 × 4 + 7000×6
a+b+c = 10000 + 24000 + 42000 = 76000
= 3d, a + b + c = 9d …(I)
3
Total expenditure = 76000 - 10000
a+b+c = 66000
and = 5, a + b + c + d = 20 …(II)
4 66000
From (I) and (II), we get Average expenditure = = 5500
12
QUANTITATIVE APTITUDE 53
www.mahendras.org
Q.5.(1) Total vistors = 620 × 5 + 900 × 2 = 3100 + 1800 50820
= 4900 Now average = =423.5
120
4900
Average visitors = Q.14.(3) Match 1: Match 2 = 6: 5
7 = 700
Match 2: Match 3 = 3: 2
6 +1
Q.6.(5) Exact mid of 6 no= means in between 3rd Therefore,
2
and 4th term
Match 1: Match 2: Match 3 = 18: 15: 10
1 2 3 4 5 6
Now ,
48
18x – 10x = 128
Largest no = 48 + 5 = 53
x = 16
Smallest no. 48 - 5 = 43
Total runs scored in three matches
Q.7.(1) First no is = 105
= 18x + 15x +10x = 43x
Last no is 294
= 43×16 = 688
105 + 294
Average = = 199.5 Required average = (688/3) = 229(1/3)
2
Q.15.(2) Present age sum = 25 × 4 = 100
Q.8.(1) Set A = 71, 73, 75, 77, 79, 81
Sum of age 7 years age = 100 -7 × 4= 72
Set B = 2x-4,2x-2, 2x,2x+2,2x+4,2x+6
72
According to questions- 81 -(2x-4) = 33 Average = = 24 years
3
⇒ x=26
SHORT Trick:
So set (B) = 48, 50, 52, 54, 56, 58
Average of set B = 53 ( 25 − 7 ) × 4
= 24 years
3
 30 × 400 
Q.9.(1) Weight of teacher =  40 + kg. Q.16.(3) Sum of six no. = 40 × 6 = 240
 1000 
= 52kg Sum of 1st three no. 35 × 3 = 105
Q.10.(4) 24.85 52 Sum of last four no. = 45 × 4 = 180
5
24 Sum of 1st three and last four = 105 + 180 = 285
So, 3rd number = 285 - 240 = 45
13.6 0.85
Q.17.(1) Sum of 10 numbers = 43 × 10 = 430
16 : 1
Sum of 1st four numbers = 40 × 4 = 160
Number of wickets before last match .
Sum of last five numbers = 47 × 5 = 235
=
5
× 16 = 80
1 Sum of 1st four and last five = 160 + 235 = 395
3×48×4×37.5 + 2×51
Q.11.(5) Required answer= =44 So, 5th number = 430 - 395 = 35
3+4+2
539
Q.12.(4) 42×15 -_36 + x i + 42 = 43 Q.18.(4) Average of odd number = = 77
15 7
x = 645-636 = 9 So, 3rd largest number = 77 + 2 = 79
Q.13.(5) Sum of first 120 natural numbers which is multiple Average of four even number = 93
of 7,
So, smallest number = 93
7+ 14+21………..+840 = 7 (1+2+3……..+120),
So, smallest number = 93 - 3 = 90
n (n +1) Required sum = 79 + 90 = 169
Sum of first n natural numbers =
2 Q.19.(3) Rahul + Manu + Sheetal = 219 -----(I)
121 Rahul = Ankit - 18 ------------ (II)
So, 7 (1+2+3……..+120) = 7×120× = 50820
2
54 QUANTITATIVE APTITUDE
www.mahendras.org
Rahul = Manu + 12 ------ (III) 6.8 x
Rahul + Manu + Sheetal
Ankit = + 30 ---(IV) 7
3
(x-7) : 0.2
From eqn. (I), (II),(III) and (IV)
Ankit = 103 ( x − 7) 1
= or, 4x - 28 = 0.2
Rahul = 85 0.2 4
Manu = 73 So, x = 7.05
Required sum = 61 + 73 = 134 Q.23.(2) By Alligation
Q.20.(5) Using alligation 15000 7000

Passed : Failed = 19 : 5 8000

1000 7000
Ratio between officer and non - officer= 1 : 7
No. officres is = 12
5 So, non officers = 12 × 7 = 84
So, failed students = 120 × = 25
24 Q.24.(4) Let there were x friends.
96
Boys Girls So, contribution =
Q.21.(3) x x
46
Four friends left
42 96
So, new contribution =
x−4
(42-x) 4 96 96
∴x−4 - = 4, x = 12
x
42 − x 3 So, number of friend who attended the picnic
= =
4 2
= 12 - 4 = 8
x = 36
Q.25.(3)
Q.22.(3) Total run in first 10 over = 6.8 × 10 = 68 Boys Girls
16 14
Required run rate in the remaining 40 overs
44/3
350 − 68
= 40
= 7.05
2 : 4
SHORT Trick: 1 : 2

350 1
Required average of game = =7 So, boys = 300× =100
50 3

QUANTITATIVE APTITUDE 55
www.mahendras.org

CHAPTER
Ratio &
7 Proportion
Comparison of Ratio:
Ratio
Let a : b and c : d, be two ratios, then
Ratio is a comparison of two quantities. The ratio of a to b
is written as- a : b > c : d if ad > bc
a a c
a:b= = a÷b ie. > if ad > bc
b b d
Types of Ratio: Similarly,
(i) Compounded Ratio: When two or more than two a : b < c : d if ad < bc
ratios are multiplied with each other, then it is called a : b = c : d if ad = bc
as Compounded Ratio.
Two Important Results
2 4 6 48 2 4 a
Ex. × × = is the compounded ratio of , If > 1 , it is implied that a > b
3 5 7 105 3 5 b
6. Ist term > 2nd term
7 a
and <1⇒a<b
a c e k b
So × × ........... =
b d f l Important Facts
(ii) Duplicate Ratio: Square of any ratio is known as (a) If A : B = x : y and B : C = p : q then
Duplicate Ratio. (1) A : C = x × p : y × q
Duplicate Ratio of a : b = a2 : b2. (2) A : B : C = (x : y) ×p : y × q
=x×p:y×p:y×q
Ex. Duplicate ratio of 7 : 5 = 49 : 25
Or
(iii) Triplicate Ratio: Cube of any ratio is known as
Triplicate Ratio. A:B =x :y
Triplicate Ratio of a : b = a3 : b3.
B:C=p:q
Ex. Triplicate ratio of 9 : 2 = 729 : 8
x×p:y×p:y×q
(iv) Sub Duplicate Ratio: Square root of any ratio is
known as Sub Duplicate Ratio: (b) If A : B = x : y, B : C = p : q and C : D = m : n, then
(1) A : D = x × p × m : y × q × n
Sub Duplicate Ratio of a : b = a1/2 : b1/2.
(2) A : B : C : D = (xp : yp : yq) ×m:y×q×n
Ex. Sub duplicate ratio of 9 : 16 = 3 : 4
= xpm : ypm : yqm : yqn
(v) Sub Triplicate Ratio: Cube root of any ratio is known
Or
as Sub Triplicate Ratio:
Sub Triplicate Ratio of a : b = a1/3 : b1/3. x:y
Ex. Sub triplicate ratio of 27 : 125 = 3 : 5
p:q
Properties of Ratio :
1. In a ratio, two quantities are compared, so the m:n
quantities must be of the same kind. xpm:ypm:yqm:yqn
2. The ratio of two quantities determines how many times Ex. If A : B = 3 : 4 and B : C = 8 : 9 then A:B:C is
one quantity is contained by the other. Sol. A : B = (3 : 4) × 2
56 QUANTITATIVE APTITUDE
www.mahendras.org
B:C=8:9 Types of Proportion :
A:B:C=6:8:9 (i) Third Proportion :- If a : b :: b : c, then c is called
the 3rd proportional to a and b. c will be calculated
SHORT Trick
as below a : b :: b : c or a : b = b : c
A:B =3:4
⇒ a ×c = b × b
B:C=8:9
b2
A : B : C = 24 : 32 : 36 ∴c =
a
A:B:C=6:8:9
Ex. Find the third proportion of 0.36 & 0.48.
2 1 2 5
Ex. If a : b = : , b : c = : and b2 0.48 × 0.48
= 0.64
9 3 7 14 Sol. Third proportion = c = 0.36
a
7 3 (ii) Fourth Proportion: If a : b :: c : d, then d is called
d:c= : then a:b:c:d is-
10 5 the 4th proportion to a,b and c. d will be calculated
2 1 as below:
Sol. a : b = : = 2:3
9 3 a : b :: c : d or a : b = c:d
2 5 ⇒ a×d = b×c
b:c= : = 4:5
7 14 bc
∴d =
7 3 a
d:c= : = 7:6
10 5 Ex. Find the fourth proportion of 3,6, and 12.
⇒c:d=6:7 bc 6 × 12
Sol. Fourth proportion= d = = = 24
a:b=2:3 a 3

b:c= 4:5 (iii) Mean Proportion: If a and b are two numbers then
mean proportion is ab
c:d= 6:7
if mean proportion between a and b is A then :
a:b:c:d = 2×4×6 : 3×4×6 : 3×5×6 :3×5×7
a:A :: A:b
= 16 : 24 : 30 : 35
⇒ A2 = ab
Ex. If a : b = c : d = e : f = 1 : 2 then (pa+qc + re) : (pb + or
qd +rf) is equal to
a ×b = A ×A
Sol. a c e 1
= = = ∴ A = ab
b d f 2
Ex. Find the mean proportion between 9 and 16.
pa qc re 1
⇒ = = =
pb qd rf 2 Sol. Required mean proportion = ab = 9 × 16 = 12

pa + qc + re 1 (iv) Continued Proportion :


⇒ = or 1 : 2 Three quantities a, b, c of same kind are said to be in
pb + qd + rf 2
continued proportion, when a : b = b : c
Proportion
The middle number b is said to be a mean proportional
A proportion is an expression which states that two to two extreme numbers a & c
ratios are equal.
So, in such case of continued proportion.
3 1
eg. = is a proportion b2 = ac
12 4
(middle number)2 =First number × Last number
It can also be expressed as
Ex. If 3, x, 27 are in continued proportion, then find the
3 : 12 = 1 : 4 or 3 : 12 : : 1 : 4 value of x.
Each quantity in proportion is called a term Sol. Since 3, x, 27 are in continued proportion.
proportional. The first & the last terms are called the x2 = 3 × 27
extreme term whereas the second & the third terms x2 = 81
are called middle term.
x=9
QUANTITATIVE APTITUDE 57
www.mahendras.org
Relation Among the Quantities More than two: x = 2.5
Given a : b = x : y Quantity of alcohol = 4 × 2.5 = 10 litres
b:c=m:n Ex. Cost of a diamond varies directly as the square of its
then these three quantites are related as, weight. A diamond broke into four pieces with their
a:b= x :y
weight in the ratio 1:2:3:4. If the loss in the total value
of the diamond was Rs.70,000, the price of the original
b:c= m:n diamond was-
Sol. Let the weight of the four pieces of diamond be x, 2x,
a : b : c = xm : my : yn
3x & 4x units respectively.
a:c = xm : yn Total weight of the original diamond
Useful Results on Proportion : = x + 2x + 3x + 4x
If four quantities a, b, c and d are said to be proportion = 10x units
if and only if Price of the original diamond
= K (10x)2 = K 100x2
a:b=c:d Where K is a constant
Cost of four pieces of diamond
a×d=b×c = K (x2 + 4x2 + 9x2 + 16x2)
⇒product of extrems = product of means = K.30X2
Loss in the cost of diamond
Componendo and Dividendo:
= K (100x2 - 30x2)
If a : b is equal to c : d
= K.70x2
a c ∴ K.70x2 = 70000
=
b d
Kx2 = Rs. 1000
a+b c+d ∴ Cost of the original diamond
i) Componendo Rule - =
b d
= K (100x2)
a−b c −d
ii) Dividendo Rule - = = 1000 × 100 = 1,00,000 Rs.
b d
Ex. The price of sugar is increased by 20%. If the
iii) Componendo and Dividendo Rule - expenditure is not allowed to increase, the ratio
a+b c+d
=
between the reduction in consumption and the original
a−b c −d consumption is:
Ex. A bag contains 50 P, 25 P and 10P coins in the ratio Sol. When price is increased by 20%
5 : 9 : 4. amounting to Rs. 206. Find the number of then expenditure is decreased by
coins of 50 paise. r
× 100
Sol. Let the number of 50P, 25P and 10 P coins be 5x, 9x 100 + r
and 4x respectively. 20
× 100
5x 9x 4x 120
+ + = 206
2 4 10
2
50x + 45x + 8x = 4120 16 %
3
103x = 4120 Hence required Ratio = 1 : 6
x = 40 Ex. The largest and the smallest angles of a triangle are in
No. of 50 P coins = 5 ×40 = 200 the ratio of 3:1 respectively. The second largest angle
Ex. A mixture contains alcohol and water in the ratio of of the triangle is equal to 440. What is the value of
4 : 3. If 5 litres of water is added to the mixture the 150% of the largest angle of the triangle?
ratio becomes 4 : 5. Find the quantities of alcohol in Sol. Sum of largest and smallest angle
the given mixture. = 1800 - 440 = 1360
Sol. Let the quantity of alcohol and water be 4x litres and 136
3x litres respectively. Largest angle = × 3 = 34×3 = 1020
4
4x 4 150
= ⇒ 8x = 20 150% of largest angle= 102× =1530
3x + 5 5 100

58 QUANTITATIVE APTITUDE
www.mahendras.org
EXERCISE
Q.1. A watermelon is cut into two pieces in the ratio of 3 : 5 by weight. The bigger of the two is further cut in the ratio
of 5 : 7 by weight . Find the ratio of weight of each of the three pieces.
(1) 3 : 5 : 7 (2) 15 : 25 : 56 (3) 36 : 25 : 35 (4) 3 : 2 : 3 (5) None of these
Q.2. If b : (a+c) = 4 : 9 and c : (a+b) = 3 : 7 then find the ratio of a :(b+c)
(1) 53 : 77 (2) 5 : 8 (3) 51 : 79
(4) Can't be determined (5) None of these
Q.3. Divide Rs. 3600 among P, Q, R, in the ratio 7 : 2 : 9. Find the amount received by Q.
(1) Rs. 1000 (2) Rs. 400 (3) Rs. 500 (4) Rs. 1800 (5) Rs. 600
Q.4. What least number must be subtracted from each of the numbers 14, 17 , 34 and 42 so that the remainders may be
proportional ?
(1) 0 (2) 1 (3) 2 (4) 7 (5) 4
Q.5. 20 boys and 25 girls form a group of social workers. During their member ship drive, the same number of boys
and girls joined the group. How many members does the group have now if the ratio of girls to boys is 8:7?
(1) 75 (2) 65 (3) 70 (4) 60 (5) 80
Q.6. What will come at place of blank?
A solution has _ liter milk and 35 liter water. When 25.5 liter milk is poured in solution percentage of milk be-
comes 60%.
(1) 15 (2) 27 (3) 24 (4) 30 (5) 36
Q.7. The value of a diamond is Rs 40000, which is directly proportional to the square of its weight. For some reason
the diamond fall and is broken into two parts in the ratio of 1: 3. Find out the loss percentage on diamond.
(1) 0% (2) 32% (3) 35% (4) 37.5% (5) None of these
Q.8. In a mixture 200 litres of milk and 40 litres of water. ____ litres of mixture is drawn and ____ litres of pure water
is added, the milk in the mixture is 124 litres more than the water. Which of the following options satisfies the two
blanks in the question?
A) 36, 12 B) 24, 8 C) 18, 20 D) 12, 6
(1) Only B (2) Only C (3) Only A (4) Only D (5) Only B) and C
Q.9. A mixture contains milk and water in ratio 5: 1. On adding _______liters of water, the ratio of water to milk be-
come 2 : 3. The quantity of Milk in the mixture is ______liters. Which of the following options is /are possible for
the value of missing place in the same order.
A. 6, 15 B.7,12 C.14, 30 D. 21,30
(1) Only A (2) Only B (3) Any Two (4) Any Three (5) None of these
Q.10. A vessels contains mixture of 3 types of milk A, B, C. The ratio of their quantity is 3 : 4 : 9 respectively. If 10
litres of the mixture is taken out and 3 litres of A, B, C each are filled in the vessels due to which the quantity of
C became twice that of B. Find the initial quantity of mixture?
(1) 48 (2) 58 (3) 40 (4) 50 (5) 68
Q.11. A milkman gives 1.5 lt. milk at first flat and add same amount of water before giving milk at second flat. He follows the
same process with each flat. On the fourth flat, in on purity test it is found that the ratio of milk and water become 343 :
169. If he starts the distribution with pure milk then find the initial quantity of milk.
(1) 12 lit. (2) 15 lit. (3) 18 lit.
(4) 9 lit. (5) Can't be determined
Q.12. The monthly incomes of two persons are in the ratio 2 : 3 and their monthly expenses are in the ratio 5 : 9. If each
of them saves Rs 600 per month then the total of their monthly incomes are -
(1) Rs 4500 (2) Rs 4000 (3) Rs 4800 (4) Rs 6000 (5) None of these
1
Q.13. Two litre of pure milk is added to 6 liter of mixture of milk and water, contains 83 3 % of water. Find the ratio
between water and milk in resultant mixture?
(1) 3 : 5 (2) 5 : 3 (3) 2 : 3 (4) 3 : 2 (5) None of these

QUANTITATIVE APTITUDE 59
www.mahendras.org
Q.14. The incomes of Sonu and Prasant are in the ratio 3:2 and their expenditure are in the ratio 5 : 3. If each saves Rs. 7200.
Then Sonu's income is what percent the expenditure of Prashant?
(1) 100% (2) 200% (3) 300% (4) 350% (5) None of these
Q.15. Number of students in Arts and Science in an institute are in the ratio of 5:8 respectively. If 150 more students join
Arts faculty while 80 more students join Science faculty, the respective ratio becomes 3:4. Originally what was
the total number of students in both faculties together?
(1) 1200 (2) 1400 (3) 1150
(4) Cann't be determined (5) 1170
Q.16. 729 ml of combination has coffee and water in the ratio 7:2. How much more water to be included to get a new
combination of coffee and water in the ratio 7:3?
(1) 162 (2) 81 (3) 75 (4) 360 (5) 400
Q.17. The length and breadth of a rectangle are raised in the ratio 3:4 and 4:5 corresponding. What is the ratio of the
previous area to the new area?
(1) 5:3 (2) 3:5 (3) 5:8 (4) 8:5 (5) 3 : 1
2
Q.18. Rs. 6800 is divided among three person A, B & C in such a way that A received of total shared B & C together.
Find the share of B. 5

(1) 1800 (2) 2000 (3) 2200 (4) C.N.D. (5) None of these
2 3
Q.19. Rs.1540 is distributed among A,B & C such that A receives as much as B and C together and B receives
9 11
as much A & C together. Find the share of C.
(1) 390 (2) 280 (3) 330 (4) 930 (5) 1020
Q.20. The income of A & B are in the ratio of 5:7 & their expenditure are in the ratio of 3:5. If each saves Rs.1500. Find
the income of B.
(1) 4250 (2) 5250 (3) 3550 (4) 4050 (5) 5000
Q.21. An amount of money is distributed among A, B&C in the ratio 6:19:7. If C gives Rs.400 from his share to B, then
new ratio becomes 3:10:3. Find the total amount of money.
(1) Rs.18600 (2) Rs.1450 (3) Rs.12800 (4) Rs.14800 (5) Rs.15800
Q.22. The ratio of A's and B's income last year was 3:4. The ratio of their own income of last year and this year is 4:5
and 2:3 respectively. If the sum of their present income is Rs.6565. Find the present income of A.
(1) Rs.2525 (2) Rs.5025 (3) Rs.2500 (4) Rs.5000 (5) Rs. 2200
Q.23. Rs. 525 is divided among 4 men, 5 women and 6 boys such that share of a man, a woman and a boy may be in the
ratio of 9:8:4. What is the share of a women?
(1) Rs.36 (2) Rs.38 (3) Rs.40 (4) Rs.42 (5) Rs. 48
Q.24. A vessel contains 60 litres of mixture of milk and water in the ratio 7:3 respectively. 8 litres of mixture is replaced
by 12 litres of milk. What is the ratio of milk and water in the resulting mixture?
(1) 67 : 121 (2) 121 : 39 (3) 164 : 123 (4) 165 : 121 (5) 63 : 121
Q.25. Mr. X invested a certain amount in Debit and Equity funds in the ratio of 4:5 respectively. At the end of the one year,
he earned a total dividend of 30% of his investment, after one year he reinvested the amount including dividend in
the ratio of 6:7 in Debit and Equity funds. If the amount reinvested in Equity funds was Rs.94500. What was the
original amount invested in Equity Funds?
(1) Rs.75000 (2) Rs.81007 (3) Rs.60000 (4) Rs.65007 (5) Rs. 63121
Q.26. Three containers have their volume in the ratio 1:4:7. They are full of mixture of milk and water. The mixture
contains milk and water in the ratio (4:1), (3:1) and 5:2 respectively. The contents of all these three containers are
poured into fourth container. The ratio of milk and water in the fourth container is:
(1) 11:15 (2) 15:11 (3) 14:11 (4) 11:4 (5) 13:15
60 QUANTITATIVE APTITUDE
www.mahendras.org
EXERCISE Explanation
5 7 According to the question, Milk in the final
Q.1.(3) 3:5× :5× = 36 : 25 : 35
12 12 solution = 200 – (36 × 5/6) = 170 litres
Water in the final solution = 40 – 36×1/6 + 12
Q.2.(3) b : (a+c) = (4:9) × 10
= 46 litres
c : (a+b) = (3:7) × 13 Difference of milk and water in final solution =
a b c 170 – 46 = 124 litres
51 : 40 : 39 This satisfies the given condition.
Q.9.(5) 5 = 15 liter
51 : 79 1 = 3 liter
Q.3.(2) Amount received by Q M : W = 15 : (3+6) = 15 : 9 = 5 : 3
Hence it is not followed
Its related ratio term
= × Total amount option :- B
Sum of ratio terms 5 = 12 liter
2 1 = 2.4 liter
= × 3600 = Rs. 400 M : W = 12 : (2.4+7) = 12 : 9.4 = 60 : 47
7+2+9
Hence it is not followed.
Q.4.(3) Let no. x is substracted from each number. Hence,
option :- C
14 − x 34 − x
= 5 = 30 liter
17 − x 42 − x 1 = 6 liter
x=2
M : W = 30 : (6+14) = 3 : 2
Q.5.(1) Let x boys & x girls each are added in each group. Hence it is followed
According to question, option :- D
5 = 30 liter
20 + x 7 1 = 6 liter
=
25 + x 8 M : W = 30 : (6+21) = 30 : 27 = 10 : 9
⇒ 160+8x = 175 + 7x Hence it is not followed

⇒ x = 15 Q.10.(2) Suppose, after 10 litre of the mixture is taken out


Hence, total no. of person
Quantity of A = 3x, Quantity of B = 4x
= (20+x)+(25+x)
Quantity of C = 9x
= 20 + 15 + 25 + 15
According to question, 9x+3=2(4x+3), 9x+3
= 75
= 8x+6, x = 3
Q.6.(2)
Mixture in the initial stage
x + 25.5 3
= = 3x+4x+9x+10 = 16×3+10= 58 litre.
x + 25.5 + 35 5
x = 27 Q.11.(1)  1.5 
3
x  1–
Q.7.(4) Let the diamond breaks into the ratio of 1k and  x  343 1.5 343 7
= 1– = 3 =
x 343 + 169 x 512 8
3k
Initially total weight of diamond = total weight x = 8×1.5=12 lit.
of broken diamond=4k Q.12.(2) Let the expenditure of first person and second
According to the question, person is Rs. 5x, and Rs. 9x respectively.
16k2 = 40000, k = 50
The price of the broken diamond = 1k2+9k2 = 5x + 600 2
According to equation, =
10k2 = Rs 25000 9x + 600 3
So loss percentage 15x + 1800 = 18x + 1200, 3x = 600, x
40000 − 25000
× 100 = 37.5% = 200
40000
First person's monthly income = Rs 1600
Q.8.(3) Milk and water ratio initially = 200: 40 = 5: 1
36 litres of mixture is drawn and 12 litres of Second person's monthly income= Rs 2400
water added Total income = Rs. (1600 + 2400)= Rs. 4000
QUANTITATIVE APTITUDE 61
www.mahendras.org
5 1 Required Ratio = 12a2 : 20a2
Q.13.(2) 6 × :6× +2 = 5:3
6 6 =3:5
Q.14.(2) According to question Q.18.(4) CND (because ratio b/w B&C is not known).
3x- 5y = 7200 ... (i) Q.19.(4) According to question,
2x -3y = 7200 ....(ii)
2
On solving (i) and (ii) we get A= (B+C)
9
x = 14400, y = 7200 A : (B+C) = 2 : 9 ..........(1)
Sonu's income = 3 ×14400 = 43200 3
and B = (A+C)
Prashant's expenditure = 3 × 7200 11
= 21600 B : A + C = 3 : 11 ........(2)
Now from eq.(1)
43200 1540
Req. % = × 100 = 200 share of A = × 2 = 280
21600 11
Q.15.(5) According to question,
1540
5x + 150 3 Share of B= × 3 = 330
= 14
8x + 80 4 Hence,
= 24x + 240 = 20x + 600 Share of C= 1540 - (280 + 330)= Rs.930
4x = 360 Q.20.(2) IA : IB = 5 : 7
x = 90 EA : EB = 3 : 5
Original number of students = 5x + 8x = 13x Since saving = Income - Exp.
= 13 × 90 = 1170 and each saves Rs.1500
Q.16.(2) Method-I. Hence change in ratio of both must be save.
729 1500
Coffee = ×7=567 ml So, IB = × 7 = 5250
9 2
729 0 0 0
Water = ×2 = 162 ml Q.21.(3) A : B : C = (6 : 19 : 7) × 1 = 6:19:7
9
Let x lt water be added then, 0 +400 −400
A : B : C = (3 : 10 : 3) × 2
567 7
= +1 −1

162 + x 3 = 6 : 20 : 6
x = 81 ml Since C gives Rs. 400 to B

Method-II (Short Trick): Hence total amount of

C:W=7:2 400
A+B+C = × 32
1
C:W=7:3 = Rs. 12800
729 Q.22.(1) −1 −1
..............(I)
Added Water = 9
×1 = 81 ml A :B = 3: 4
Q.17.(2) According to question −1 0

A : A = 4 : 5 ..............(II)
Real length = 3a
−1 0
Real breadth = 4a B : B = 2 : 3 ..............(III)
Real Area = 3a × 4a = 12a2 From eq. (I) & (II)
Now, 0 3
A= ×5
Increased length = 4a 4
Increased breath = 5a 0 15
New Area = 4a × 5a = 20a 2 A=
4
62 QUANTITATIVE APTITUDE
www.mahendras.org
from eq. (I) & (III) = (36.4 + 12) : 15.6
0 4 = 48.4 : 15.6= 121 : 39
B= ×3
2
Q.25.(1) Let the original amount invested in Debit and
0
B=6 Equity funds are 4x & 5x respectively.

So, Dividend at the end of the year
0 0 15 30 27x
A :B = : 6 = 15 : 24 = 5 : 8 9x × =
4 100 100
Hence, Total investment after 1 year
6565 27x 117x
Income of A = × 5 = 2525 = 9x + =
13 10 10
Q.23.(4) Ratio of 4 men, 5 women & 6 boys According to question, ratio between debit and
= 4 × 9 : 5 × 8 : 6 × 4= 9 : 10 : 6 equity is 6:7.

525 7 117x
∴ × = 94500
Share of 5 women = 25 × 10 = 210 13 10

210 94500 × 13 × 10
Share of a woman = = 42 Rs. x=
5 7 × 117
Q.24.(2) Since 8 litres mixture is drawn. Hence remaining = 15000
mixture = 60 - 8 = 52 Hence, invested amount in equity fund=5x
Now, = 5×15000 = Rs.75000
52 Q.26.(4) According to question:
Milk in remaining mixture= × 7 = 36.4
10 4 3 5
1× +4× +7×
Water in remaining mixture Milk 5 4 7
=
Water 1 1 2
1× + 4 × + 7 ×
52 5 4 7
= × 3 = 15.6 4
10 +3+5
5 44
New ratio of milk & water when 12 ltrs more milk ⇒
1
= = 11: 4
16
is added. + 1+ 2
5

Notes

QUANTITATIVE APTITUDE 63
www.mahendras.org

CHAPTER
PROBLEMs
8 ON AGES
SHORT Trick :
Introduction
A0 . B0 = 3 : 5
Problems based on ages are generally asked in most of
the competitive examinations. To solve these problems, (A+B)0 = 48 years
the knowledge of linear equations is essential. In such A-5 = ?
problems, there may be three situations: B-5 = ?
(i) Age some years ago 48
(ii) Present age A0 = 8 × 3 = 18 years
(iii) Age some years hence A-5 = 18 - 5 = 13 years
Some Important Solved examples : 48
B0 = × 5 = 30
Ex. The ratio of present age of A & B is 2 : 3. The present 8
age of A is 20 years. Find the age of B after 5 years. B-5 = 30 - 5 = 25 years
Sol. Let the age of A is 2x years and age of B is 3x years. Ex. The ratio of ages of A & B before 5 years was 2 : 3. If
2x = 20 the sum of ages of A & B at present is 45 years. Find
the present ages of A & B .
x = 10
Sol. Let the ages of A & B 5 years before are 2x years and
then present age of B is
3x years respectively.
3 × x = 3×10 = 30 years
then, (2x + 5) + (3x + 5) = 45 years
After 5 years, Age of B = 30 + 5
5x + 10 = 45
= 35 years
5x = 35
SHORT Trick :
x=7
A0 : B0 = 2 : 3, A0 = 20 year
The age of A before 5 years = 7 × 2 = 14 years
20
B = 0 × 3 = 30 years At present age of A = 14+5 = 19 years
2
B = 30 +5 = 35 years
+5
The age of B before 5 years= 7 × 3 = 21 years
Ex. The present ratio of age of A & B is 3 : 5. If the sum At present age of B = 21 + 5 = 26 years
of present age of A & B is 48 years. Find the ages of
A & B before 5 years. short trick :
-5 -5
Sol. Let the ages of A & B are 3x years and 5x years A : B =2:3
respectively. (A+B) = 45 years
3x + 5x = 48 (A+B)-5 = 45 - 10 = 35 years
8x = 48 35
x = 6 years A-5 = 5 × 2 = 14 years
Then the present age of A is
A0 = 14 + 5 = 19 years
= 3 × x = 3 × 6 = 18 years
Before 5 years the age of A 35
B-5 = 5 × 3 = 21 years
= 18 - 5 = 13 years
The present age of B is B0 = 21 + 5 = 26 years
= 5 × x = 5 × 6 = 30 years Ex. The ratio of ages of A & B after 5 years will be 3 :
Before 5 years the age of B.= 30 - 5 = 25 years 5. If the sum of ages of A & B at present is 38 years.
Find the ages of A & B before 5 years.
64 QUANTITATIVE APTITUDE
www.mahendras.org
Sol. Let the ages of A & B are 3x years and 5x years Solving eq. (1) & (2)
respectively after 5 years. x = 30 years and y = 36 years
then,
SHORT Trick :
(3x - 5) + (5x - 5) = 38 years
8x - 10 = 38 years A0 : B0 = 5 : 6
8x = 48 years, x = 6 years A-6 : B-6 = 4 : 5
The age of A after 5 years 6
= 6 × 3= 18 years A0 = × 5 = 30 years
1
Then the age of B after 5 years
= 6 × 5 = 30 years 6
B0 = × 6 = 36 years
The age of A before 5 years 1
= 18 - 10 = 8 years Ex. The ratio of age of A & B at present is 6 : 5 & the sum
The age of B before 5 years of their ages is 54 years after 5 years. What will be the
= 30 - 10 = 20 years ratio of their ages after 8 years ?
SHORT Trick : Sol. A0 : B0 = 6 : 5
A+5 - B+5 = 3 : 5 (A+B)+5 = 54
(A+B)0 = 38 years A+8 : B+8 = ?
(A+B)+5 = 38 + 10 = 48 years (A+B)0 = 44
48 44
× 3 = 18 years A0 = × 6 = 24
A+5 = 11
8
A-5 = 18 - 10 = 8 years 44
A+8 = 32, B0 = × 5 = 20
11
48 B = 28
+8

B+5 = × 5 = 30 years
8 A+8 : B+8 = 32 : 28 = 8 : 7
B-5 = 30 -10 = 20 years Ex. If Sita’s mother was 4 times as old as Sita 10 years
Ex. The present ratio of ages of A & B is 6 : 7. After 5 ago. After ten years mother will be twice as old as
years this ratio will be changed into 7 : 8. Find the Sita. How old is mother & Sita at present?
present age of A & B. Sol. Sm–10 : S–10 = [4 : 1]×1 = 4 : 1
Sol. Let the present age of A is x years & B is y years. Sm+10 : S+10 = [2 : 1]×3 = 6 : 3
x 6 ............(1)
= 20
y 7 Sm+10 = × 6 = 60
2
x+5 7 Sm = 50 years
0

= ...........(2)
y+5 8 20
S+10 = × 3 = 30
Solving eq. (1) & (2) 2
We get, x = 30 years S0 = 20 years
Ex. Jayesh is as more younger to Amit as Jayesh is older
y = 35 years
to Prashant. If the sum of the ages of Amit & Prashant
SHORT Trick : is 48 years. What is the age of Jayesh in years ?
A0 : B0 = 6 : 7, A+5 : B+5 = 7 : 8 Sol. Let the age of Jayesh = x years
5 Amit = y years
A0 = 1 ×6 = 30 years Prashant = z years
5 y–x = x–z
B0 = × 7 = 35 years
1 2x = y+z = 48
Ex. The ratio of ages of A & B is at present 5 : 6. 6 years before age of Jayesh (x) = 24 years
this ratio was 4 : 5. Find the present age of A & B. Ex. Neeraj is as younger to Gopal as he is older to Deepak.
Sol. Let present age of A is x and B is y years. If the sum of the ages of Gopal & Deepak is 58 years.
x 5 What is Neeraj’s age ?
Then, y = 6 ....................(1) Sol. G – N = N–D, G+D = 2N
x−6 4 58 = 2N
Again, = ................(2)
y−6 5 N = 29 years
QUANTITATIVE APTITUDE 65
www.mahendras.org
EXERCISE
1
Q.1. A man's age is 125% of what it was 10 years ago, but 83 % of what it will be after 10 years. After how many
3
years his age will become 64 years?
(1) 12 years (2) 16 years (3) 15 years (4) 14 years (5) 10 years
Q.2. The ratio of the ages of the husband and the wife five years ago was 11: 9 whereas at the same time the ratio of
the ages of the husband and the son was 5: 1. Five years hence the ratio of the ages of the husband and the wife
will become 13 : 11. What is the sum of the present ages of all the three persons of the family?
(1) 128 years (2) 127 years (3) 126 years (4) Data inadequate (5) None of these
Q.3. The ratio of sum and the difference of the present age of Vivek and Arvind are 2 : 1. Four years ago, this ratio was
3 : 2. What will be the ratio of their ages after 12 years.
(1) 9 : 5 (2) 5 : 9 (3) 8 : 3 (4) 4. 5 (5) None of these
Q.4. The ratio of ages of Ravi 5 years ago and that of Rohan's age after 5 years is 2:3. The ratio of their present age is
3:4. What is the age of Ravi before 3 years and that of Rohan's age after 7 years?
(1) 39 : 49 (2) 75 : 10 (3) 72 : 107 (4) 3 : 5 (5) None of these
1
Q.5. A man's is 125% of what it was 10 years ago, but 83 % of what it will be after 10 years. What is his present age?
3
(1) 40 years (2) 50 years (3) 60 years (4) 30 years (5) 45 years
Q.6. The ratio of present age of A and B is 11:13 and sum of their ages is 72 years. Then find the present age of C who's
age is twice that of A?
(1) 56 years (2) 12 years (3) 78 years (4) 66 years (5) None of these
Q.7. The ratio of present age of Manoj and Waseem is 3 : 11. Waseem is 12 years younger than Rehana 7 years hence
Rehana will be 85 years old. What will be the present age of Manoj's father who is 25 years older than Manoj?
(1) 42 years (2) 43 years (3) 44 years (4) 45 years (5) 46 years
Q.8. Rohan's present age is 3 years less than twice his age 12 years ago. Also the ratio of present ages of Mohan and
Rohan is 4:9. What will be the Mohan's present age.
(1) 4 years (2) 12 years (3) 8 years
(4) 16 years (5) Cannot be determined
Q.9. Ratio of present ages of A and B is 2:3. B's age after 16 years will be twice the age of A after 4 years. Find the
difference between present ages of A and B?
(1) 7 years (2) 9 years (3) 11 years (4) 2 years (5) 8 years
Q.10. The average age of Ram and his two sons is 17 years. While the average age of Ram's wife and his two son's is 16
years. If the age of Ram is 33 years, find the age of Ram's wife.
(1) 31 years (2) 32 years (3) 35 years (4) 30 years (5) None of these
Q.11. The average age of two boys and their fathers exceed by 3 years than the average age of those two boys and their
1
mother. The average of those 4 persons is 19 years. If the average of these two boys is 5 2 years, then find the age
of father and mother?
(1) 37 years & 28 years (2) 47 years & 38 years (3) 50 years & 41 years
(4) 35 years & 32 years (5) None of these
Q.12. If 6 substracted from the present age of A and the remainder is divided by 18, the resultant is the age of B. C's
age is 5 years and B is 2 years younger than C. What is the present age of A?
(1) 60 years (2) 48 years (3) 84 years
(4) 64 years (5) None of these
Q.13. The average age of a family of five members is 24. If the present age of the youngest member is 8 years, what was
the average age of the family at the time of the birth of the youngest member?
66 QUANTITATIVE APTITUDE
www.mahendras.org
(1) 20 years (2) 16 years (3) 12 years (4) 18 years (5) 21 years
2
Q.14. Yashika got married 8 years ago Today her age is 1 times her age at the time of her marriage At present her
7
daughters age is one-sixth of her age What was her daughter’s age 3 years ago?
(1) 2 years (2) 3 years (3) 4 years (4) 5 years (5) None of these
Q.15. The ratio of the present ages of a son and his father is 1 : 5 and that of his mother and father is 4:5. After 2 years
the ratio of the age of the son to that of his mother becomes 3:10. What is the present age of the father?
(1) 30 years (2) 28 years (3) 37 years (4) Data inadequate (5) None of these
Q.16. The respective ratio between the present age of Manisha and Deepali is 5 : X. Manisha is 9 years younger than
Parineeta. Parineeta's age after 9 years will be 33 years. The difference between Deepali's and Manisha's age is
same as the present age of Parineeta. What will come in place of X?
(1) 23 (2) 39 (3) 15
(4) Cannot be determined (5) None of these
5
Q.17. Sush was thrice as old as Poonam 6 years back. Sush will be times as old as Poonam 6 years hence, How old
is Poonam today? 3
(1) 20 (2) 14 (3) 12 (4) 15 (5) None of these
Q.18. The sum of the ages of a father and his son is 4 times the age of the son. If the average age of the father and the
son is 28 years, what is the son’s age?
(1) 14 years (2) 16 years (3) 12 years (4) Data inadequate (5) None of these
Q.19. Radha's present age is three years less than twice her age 12 years ago. Also, the ratio of Raj's present age to
Radha’s present age is 4 : 9. What will be Raj’s age after 5 years?
(1) 12 yrs (2) 7 yrs (3) 21 yrs
(4) Can’t be determined (5) None of these
Q.20. Neeraj's age is 1/5 of his father's age. Neeraj's father's age will be twice Vinod’s age after 10 years, if Vinod’s
eighth birthday was celebrated two years ago, then what is Neeraj’s present age ?
(1) 14 years (2) 6 years (3) 30 years
(4) 10 years (5) None of these
Q.21. After 5 yrs, the age of a father will be thrice the age of his son, whereas five years ago. he was 7 times as old as his
son was. What are their present ages?
(1) 30 yrs (2) 40 yrs (3) 50 yrs
(4) 60 yrs (5) None of these
Q.22. In a family, a couple has a son and daughter. The age of the father is three times that of his daughter and the age
of the son is half of his mother. The wife is nine years younger to her husband and the brother is seven years older
than his sister. What is the age of the mother?
(1) 40 years (2) 45 years (3) 50 years
(4) 60 years (5) None of these
Q.23. Abhay’s age after six years will be three-seventh of his father’s age. Ten years ago, the ratio of their ages was 1 :
5. What is Abhay’s father’s age at present?
(1) 30yrs. (2) 40yrs. (3) 50yrs. (4) 60yrs. (5) None of these
Q.24. Tanya’s grandfather was 8 times older to her 16 years ago. He would be 3 times of her age 8 years from now. Eight
years ago, what was the ratio of Tanya’s age to that of her grandfather?
(1) 1:2 (2) 1:5 (3) 3:8 (4) 11:53 (5) None of these
Q.25. Eighteen years ago, a father was three times as old as his son. Now the father is only twice as old as his son. Then
the sum of the present ages of the son and the father is:
(1) 54 years (2) 72 years (3) 105 years (4) 108 years (5) None of these
QUANTITATIVE APTITUDE 67
www.mahendras.org
EXERCISE Explanation
Q.1.(4) Let his present age be x years Q.5.(2) Age 10 years before = 100%
According to the question, Present Age= 125%
x = 125% of (x – 10) 25% ................... 10 years

4x = 5x – 50 10
125% .............. × 125 = 50 years
25
x = 50 years OR
Required number of years = 64 – 50 = 14 years Age 10 years after = 100%
Q.2.(3) Five years ago, 1
H : W = 11 : 9 Present Age = 83 %
3
and H : S = 5 : 1 2
16 % ................ 10 years
Hence, H : W : S = 55 : 45 : 11 3
1 10 × 3 250
55k + 10 13 83 % ........ × = 50 years
= 3 50 3
45k + 10 11 Q.6.(4) A0 : B0 = 11 : 13
Five years hence, Sum of A + B = 72 years
k=1 24 parts ....... 72
Required value = 55 + 45 + 11 + 15 = 126 years 72
Q.3.(1) Let the present age of Vivek and Arvind be x & 11 parts ....... 24 ×11 = 33 years
y. According to question Present age of A = 33 years
x+y 2 Present age of C = 2×33 = 66 years
x − y = 1 , x = 3y.......(I) Q.7.(2) Ratio of Age of Manoj : Waseem = 3 : 11
4 years ago their ages will be (x - 4) & (y- 4) Waseem present age = 78-12 = 66 years
Again according to question
66
( x − 4) + (y − 4) 3 Age of Manoj =
11
×3
= ( x − 4) − ( y − 4) = 2 Present age of Manoj's father = 18+25= 43 years
or 2x + 2y - 16 = 3x - 3y Q.8.(2) Let Rohan's is present age be = x years
5y - x = 16 ...............(II) According to question:
On solving eq. (i) & (ii) x = 2 (x - 12) - 3
x = 24 x = 2x - 24 - 3
y = 8, Age of Vivek after 12 years x = 27
= 24+12 = 36 years Rohan's present age = 27 years
Age of Arvind after 12 years= 8+12 = 20 years 4
Mohan's present age = 27× = 12 years
Required Ratio = 36 : 20 = 9 : 5 9
Q.9.(5) Let the present ages of A and B be 2x & 3x.
Q.4.(3) Let the present age of Ravi and Rohan be 3x and
4x respectively. A's age after 4 years = (2x + 4)
According to question: B's age after 16 years = (3x + 16)
(3x + 16) = 2 (2x + 4)
3x − 5 2
= 3x + 16 = 4x + 8
4x + 5 3
4x - 3x = 8
9x - 15 = 8x + 10 x = 8 years
x = 25 Required Difference = 3x - 2x = 3×8-2×8
Present age of Ravi = 3x = 3×25 = 75 = 24 - 16 = 8 years
Present age of Rohan = 4x = 4×25 = 100 Q.10.(4) Total age of Ram and his son = 17×3
Ravi age before 3 years = 72 = 51 years ........ (i)
Rohan age after 7 years 107 Total age of Ram's wife and his son
Required ratio = 72 : 107 = 16×3 = 48 years ........ (ii)
68 QUANTITATIVE APTITUDE
www.mahendras.org
So (i) – (ii) 5
(Ram + son)-(Ram's wife + son) = 3 years 5x = z
4
33 - Ram's wife = 3 years
Ram's wife = 30 years 4x = z................(3)
Q.11.(1) Let the ages of boys be P and Q and that of mother x+2 3
and son be x and y respectively. After 2 years, =
z + 2 10
According to question:
Y +P+Q X+P+Q 10x + 14
− = 3 .........(I)
3 3 10x + 20 = 3z +6 ⇒ = z.......(4)
X+ Y+P+Q 3
Also = 19 .......... (II)
4 from 3 & 4
P + Q 11 10 x + 14
Also 2 = 2 .................. (III) 4x = ⇒ x=7
3
On solving eq. (II) and (III)
x + y = 65 ....................... (IV) Putting x = 7
On solving eq. (I) and (III) y = 35
y - x = 9 ........................ (V) Q.16.(5) Let age of Manisha = M, Deepali = D, Parineeta
On solving equation (IV) & (V) =P
x = 28, y = 37 M 5
= ........(1)
D x
Q.12.(1) According to question
A−6 M = P - 9.......(2)
=B
18 P + 9 = 33
A = 18 B + 6 .................... (I) P = 24
B = C - 2 = 5 - 2 = 3 years.......(II) Putting P = 24 in eq. (2)
Putting value of B in eq. .... (I) M = 24 - 9 = 15
A = 18×3 + 6= 54 + 6 = 60 years D - M = P ........(3)
Q.13.(1) Total age of 5 person = 24 × 5 = 120 years D - 15 = 24
Before 8 year 5 × 8 = 40 years D = 39
120 - 40 = 80 years Putting M = 15 and D = 39 in eq. (1)
Now the family member is 4 = 80/4= 20 years 15 5
=
Q.14.(2) Yashika's age today = x 39 x
Yashika's age of marriage = x-8 5 × 39
⇒x= = 13 ⇒ x = 13
9
x = ( x − 9) 15
7 Q.17.(3) Let Present age of Sush= S and Poonam = P
7x = 9x - 72 Þx = 36 years (S-6) = 3 (P-6)
1 S = 3P - 12.......(1)
Daughter's present age = 36 × =6
6 5
3 years age = 6 - 3 = 3 years (S + 6) = (P+6)
3
Q.15.(5) Let present age of son = x, father y, mother = z
35 + 18 = 5P + 30.........(2)
x:y=1:5
From (1) and (2)
5x = y.........(1)
3 (3P-12) + 18 = 5P + 30
z 4 9P - 36 + 18 = 5P + 30
z:y=4:5= =
y 5 P = 12 years
5z = 4y ..........(2) Q.18.(1) Let present age of father = F and Son = S

QUANTITATIVE APTITUDE 69
www.mahendras.org
F + S = 4S.......(1) M 
3  - 7 = M+ 9
F+S 2 
= 28
2
3M
- 21= M+ 9 , M = 60 years
F + S = 56..........(2) 2
4S = 56 Q.23.(3) 3
S = 14 years ( A + 6) = ( F + 6) .....(1)
7
Q.19.(5) Let present age of Radha = x and Roy = y
1
x = z (x-12) - 3 ( A − 10) = ( F − 10) .......(2)
5
x = 27 years.......(1)
From (1) and (2)
y 4
= .......(2) 3F 18 F
x 9 + -6= - 2 +10
From (1) and (2) 7 7 5
y 4
= 3F F 18 8F
27 9 ⇒ - = 14 - ⇒
7 5 7 35
y = 12 years
y + 5 = 12 + 5 = 17 years F = 50 years
Q.20.(2) Let present age of Neeraj = N, Father = F,. Vinod Q.24.(4) (G - 16) = 8 (T-16)
=V G = 8T - 112.......(1)
F (G+8) = 3 (T+B)
N= ⇒ 5N = F..........(1)
5 G = 3T + 16.......(2)
(F+10) = 2(V+10) ÞF = 2V +10..........(2) From 1 and (2)
(V-2) = 8 Þ V = 10 8T - 112 = 3T + 16
Putting V = 10 in eq.....(2) 5T = 128
F = 2 × 10 + 10 = 30 128
T= .....(A)
Putting F = 30 in eq. (1) 5
5N = 30 3 ×128
N = 6 years G= + 16
5
Q.21.(2) (F+5) = 3 (S+5) 384 + 80 464
F = 3S + 10.......(1) G= = ......(B)
5 5
(F-5) = 7 (S-5)
F = 75 - 30.......(2) T-8
From (1) and (2) =?
G-8
S = 10 Years, F = 40 years
From (A) & (B)
Q.22.(4) F = 3D.......(1)
128
M -8
S = ......(2) T-8 128 - 40
2 = 5 =
G-8 464 464 - 40
-8
F = M + 9..........(3) 5
S = D + 7.......(4) T-8 11
=
M=? G-8 53
From (1) and (3)
Q.25.(4) (F-18) = 3 (S-18), F = 3S - 36.........(1)
3D = M + 9.........(A)
F = 25.............(2)
From (2) and (4)
2S = 3S - 36, S = 36 Putting S = 36 in (2)
D + 7 = M/2.........(B)
F = 72
From (A) and (B)
F+S = 36 + 72= 108 year
70 QUANTITATIVE APTITUDE
www.mahendras.org

CHAPTER

9 PARTNERSHIP

Ex. A,B and C start a business each investing Rs.20,000. After


Partnership
5 months A withdraws Rs.5000, B withdraws Rs.4000 and
When two or more than two persons run a business then they C invested Rs.6000 more. At the end of the year, a total
are called partners and when they agree to share profit and profit of Rs.69900 was recorded. Find the share of each.
losses, the deal is known as Partnership.
Elements of Partnership: Sol. Ratio of the capital of A, B and C
Capital: The sum of money invested by the partners to start A : B : C
any business is known as capital. (20000×5+15000×7):(20000×5+16000×7):(20000×5+26000×7)
Equivalent Capital: When the capital invested by the 205000 : 212000 : 282000
partners is multiplied by the time of investment, the product 205 : 212 : 282
thus obtained is known as Equivalent Capital.
Categorisation of Partnership:  69900 
A’s share= Rs.  205 + 212 + 282 × 205  = Rs. 20500
Case 1 : When the investment and the time of investment is  
equal, the profit and loss is distributed equally.  69900 
Ex. A, B and C are three partners. Each invested Rs.20,000 B’s share=Rs.  × 212 = Rs. 21200
 205 + 212 + 282 
in a business. After a year the profit of the business
was 90,000. Then find the share of each in the profit.  69900 
Sol. Ratio of shares of A, B and C = Ratio of their investment C’s share = Rs.  × 282 = Rs. 28200
 205 + 212 + 282 
A : B : C
20000×12 : 20000×12 : 20000×12 Case 4: When capital and time are given in form of ratio.

1 : 1 : 1 Ex. A, B and C become partners in a business. A contributes


Now Profit share of A 1 1 1
rd of the capital for th of the time B contributes th
3 4 5
 90000  90000 ⇒ 1
=  1 + 1 + 1 × 1 ⇒ 30000/- of the capital for th of the time and C the rest of the
  3 6
Similarly B and C will also get = 30000/-each. capital for the whole time. If the profit is Rs. 1820,
then the A's share in profit is :
Case 2: When the investment is different but the time of
investment is same. Then the profit and loss is shared in the Sol. 1 1 1 1 7 , 1 1 7
× : × : ×1 : :
ratio of their capital. 3 4 5 6 15 12 30 15
Ex. P, Q and R started a business each investing Rs. 20,000,
5 : 2 : 28
30,000, 50,000 respectively. After 1 year the profit
recorded by the business was 1,50,000. Then find the 1820
Sahre of A = × 5 = 260
share of each in the profit? 35
Sol. A : B : C 1 1 7
Ex. A, B and C share profit in the ratio of : : if C retires,
20000×12 : 30000×12 : 50000×12 4 6 12
2 : 3 : 5 they share the profit of C in the ratio of 4 : 5 respectively.

150000 The new profit shareing ratio of A and B will be :


P's share in profit = × 2 =30000/
2+3+5 1 1 1
Sol. A : B : C = : : =3:2:1
Q's share in profit = 45000/- 4 6 12
R's share in profit = 75000/- 1× 4 1 31 23 = 31 : 23
A:B:C= 3+ :2+ ×5 = :
Case 3 : When the investment in a business are different then 9 9 9 9
the profit & loss is shared in the ratio of their equivalent capital.
QUANTITATIVE APTITUDE 71
www.mahendras.org
EXERCISE
Q.1. Three men A, B and C start a business together. They invest Rs. 30000, Rs. 24000, and Rs.42000 respectively in
the beginning. After 4 months, B took out Rs.6000 and C took out Rs.10000. They get a profit of Rs.11960 at the
end of the year. B's share in the profit is-(approximate value)
(1) Rs. 2700 (2) Rs. 2600 (3) Rs. 2800 (4) Rs. 2500 (5) Rs. 2900

Q.2. A starts a business with an investment of Rs.16000. After 3 months B joins the partnership with an investment equal
5
to 8 th of A's investment and A withdraws Rs.2000 from the business. After 3 months more C joins the partnership
with an investment of Rs.9000. In the end of the year, C gets Rs.1026 as his share in profit. Find A's share in the

profit.
(1) Rs. 3300 (2) Rs. 3306 (3) Rs. 3406 (4) Rs. 3506 (5) None of these
Q.3. A,B and C started a business with an investment in the ratio 5:6:8 respectively. After one year C withdraw 50%
of his capital and A increased his capital by 60% of his investment. After two years in what ratio should the earned
profit be distributed among A,B and C respectively?
(1) 2 : 3 : 3 (2) 4 : 3 : 2 (3) 13 : 12 : 12
(4) Cannot be determined (5) None of these
Q.4. Prakash, Sunil and Anil started a business jointly investing Rs.11 lakh, Rs.16.5 lakh and Rs.8.25 lakh respectively.
The profit at the end of 3 years was Rs.19.5 lakh. What will be the 50% of Anil's share in the profit?
(1) Rs.4.50 Lakh (2) Rs.2.25 Lakh (3) Rs.2.50 Lakh (4) Rs.3.75 Lakh (5) None of these
Q.5. Sonu invested 10% more than Mona. Mona invested 10% less than Raghu. If the total sum of their investment is
Rs.5780. How much amount did Raghu invested?
(1) Rs.2010 (2) Rs.2000 (3) Rs.2100 (4) Rs.2210 (5) None of these
Q.6. Gita invests Rs.48000 to start a business. Four months later Shreya joins her by investing Rs.62000 and another
two months later Deepika joins then both by investing Rs.80000. At the end of one year the business earn a profit
of Rs.20661. What is Deepika's share in the profit?
(1) Rs.7668 (2) Rs.6603 (3) Rs.7240 (4) Rs.6390 (5) None of these
Q.7. Three friends Naimish, Ranveer and Arjun together started a business by investing Rs.250, Rs. 400 and Rs. 300
respetively. After 4 months Naimish invested Rs. 150 more and after 6 months Ranveer withdraw Rs. 100. While
after 8 months Arjun invested Rs. 200 more . At the end of the year, a profit of Rs. 1280 is obtained. Find the share
of Ranveer?
(1) Rs. 360 (2) Rs. 380 (3) Rs. 395 (4) Rs. 420 (5) Rs. 440
Q.8. Mr. Shiv Kumar started a business investing Rs.25000 in 1996. In 1997 he invested an additional amount of Rs.10000
and Mr. Rakesh joined him with an amount of Rs.35000. In 1998, Mr. Shiv Kumar invested another additional
amount of Rs.10000 and Mr. Suresh joined them with amount of Rs.35000. What will be Rakesh's share in the
profit of Rs.150000 earned at the end of three years from the start of the business in 1996?
(1) Rs.70000 (2) Rs.50000 (3) Rs.45000 (4) Rs.75000 (5) None of these
Q.9. Mrs. Raksha invested an amount of Rs.60,000 to start a software business. After six months, Kamal joined here
with an amount of Rs.90,000. After one year from the commencement of business, Raksha put in an additional
amount of Rs.20,000. At the end of three years, they earned a profit of Rs.71,20,000. What is Raksha's share in the
profit?
(1) Rs.35,20,000 (2) Rs.26,40,000 (3) Rs.27,00,000 (4) Rs.3,84,00,000 (5) None of these
Q.10. X, Y and Z are sharing profit in the ratio 4 : 3 : 2. Y retired from the firm and X and Z decide to share profit in the
ratio of 3:2. Calculate the gaining ratio.
(1) 7 : 8 (2) 5 : 9 (3) 4 : 7 (4) 5 : 8 (5) None of these

72 QUANTITATIVE APTITUDE
www.mahendras.org
Q.11. A, B and C started a business with their investment in the ratio 1:3:5. After 4 months. A invested the same amount
as before and B as well as C withdraw half of their investment. The ratio of their profit at the end of the year was:
(1) 5 : 6 : 10 (2) 6 : 5 : 10 (3) 10 : 5 : 6 (4) 4 : 3 : 5 (5) Can't be determined
Q.12. A, B and C are partners in a business. A, whose money has been used for 4 months, claims 1 /8 of the profit. B,
whose money has been used for 6 months, claims 1/3 of the profit. C had invested Rs. 1560 for 8 months. How
much money did A and B contribute together?
(1) Rs. 720 (2) Rs. 1560 (3) Rs. 2000 (4) Rs. 1280 (5) None of these
Q.13. A and B started a business by investing Rs. 20000 and Rs. 25000 respectively. After 4 months B left and C joined
by invested Rs. 15000. At the end of the year, there was a profit of Rs. 23000. What is C’s share?
(1) Rs. 8000 (2) Rs. 9000 (3) Rs. 6000 (4) Rs. 12000 (5) None of these
7 4 6
Q.14. A, B and C enter into a partnership in the ratio : : . Alter 4 months, A increase his share 50%. If the profit
3 3 5
earned by C at the end of one year be Rs. 21,600; then find the total profit.
(1) Rs. 102100 (2) Rs. 102400 (3) Rs. 103600 (4) Rs. 104000 (5) None of these
Q.15. Avinash, Manoj and Arun started a business in partnership investing in the ratio of 3:2:5 respectively. At the
end of the year they earned a profit of Rs. 45,000 which is 15% of their total investment. How much did Manoj
invest?
(1) Rs. 60,000 (2) Rs. 1,80,00 (3) Rs. 30,000 (4) Rs. 90,000 (5) None of these
Q.16. A and B started a business by investing Rs. 35,000 and Rs. 20,000 respectively. B left the business after 5 months
and C joined the business with a sum of Rs.15,000. The profit earned at the end of the year is Rs. 84,125. What is
B’s share of profit?
(1) Rs.14133 (2) Rs. 15,000 (3) Rs.13,460
(4) Cannot be determined (5) None of these
Q.17. Ninad. Vikas and Manav enter into a partnership. Ninad invests some amount at the beginning. Vikas invests
double the amount after 6 months and Manav invests thrice the amount invested by Ninad after 8 months. They
earn a profit of Rs. 45,000 at the end of the year. What is Manav’s share in the profit?
(1) Rs. 25,000 (2) Rs. 15,000 (3) Rs. 12,000 (4) Rs. 9,000 (5) None of these
Q.18. A invests Rs. 64,000 in a business. After few months B joined him with Rs. 48,000. At the end of year, the total
profit was divided between them in the ratio 2:1. After how many months did B join?
(1) 7 (2) 8 (3) 4 (4) 6 (5) None of these
Q.19. Geeta press makes a profit of 9,00,000, 20% of which is paid as taxes. If the rest is divided among the partners P
1
Q and R in the ratio of 1 : 1 : 2, then shares of P, Q and R are respectively.
2
(1) Rs. 2,40,000, Rs. 3,20,000, Rs. 1,60,000 (2) Rs. 3,20,000, Rs. 2,40,000, Rs. 1,60,000
(3) Rs. 2,60,000, Rs. 3,20,000, Rs. 2,40,000 (4) Rs. 1,60,000, Rs. 2,40,000, Rs. 3,20,000
(5) None of these
Q.20. A, B and C started a business with their investments in the ratio 1 : 2 : 4. After 6 month A invested the half amount
1
more as before and B invested same the amount as before which C withdrew th of the their investments. Find
the ratio of their profits at the end of the year. 4
(1) 5 : 12 : 13 (2) 5 : 11 : 14 (3) 5 : 12 : 14 (4) 5 : 6 : 11 (5) None of these
Q.21. A, B and C started a business by investing Rs. 24,000, Rs. 32000 and Rs. 18000 respectively. A and B are active
partners and get 15% and 12% of total profit and remaining profit is to be distributed among them in the ratio of
their investment. If C got total Rs. 65700 as a profit, what was the total amount of profit ?
(1) Rs. 375000 (2) Rs. 367000 (3) Rs. 350000 (4) Rs. 360000 (5) Rs. 370000

QUANTITATIVE APTITUDE 73
www.mahendras.org
1
Q.22. X, Y and Z enter into partnership. X invest part of total capital for one-fourth of the time. Y contributes one fifth
4
of the capital for half of the time. Z contributes the remaining capital for the whole time. How should they divided
a profit of Rs. 1140 ?
(1) Rs. 100, Rs. 160, Rs. 880 (2) Rs. 110, Rs. 140, Rs. 860
(3) Rs. 120, Rs. 150, Rs. 840 (4) Rs. 140, Rs. 170, Rs. 830
(5) None of these
1
Q.23. A, B and C are three partners in a business A, whose money has been used for 4 months, claims of the profit,
1 8
B whose money has been used for 6 months, claims 3 of the profit. C had invested Rs. 1560 for 8 months. C had

invested Rs.1560 for 8 months. How much money did A and B contribute ?
(1) Rs. 740, Rs. 1250 (2) Rs. 730, Rs. 1240 (3) Rs. 720, Rs. 1280
(4) Rs. 750, Rs. 1260, (5) Rs. 750, Rs. 1200
1 1 1
Q.24. A, B and C started a business in partnership and invested in the ratio of : : . After 4 months A withdraw half
4 3 6
1
of his investment and after its 2 months B withdraw of its investment. If the total earned profit, at the end of
3
year is Rs. 14000. Find the share of thier profit.
(1) Rs. 2500, Rs. 2450, Rs.2145 (2) Rs. 3000, Rs. 4500, Rs.2100
(3) Rs. 4000, Rs. 3500, Rs.1254 (4) Rs. 4200, Rs. 5600, Rs.4200
(5) None of these
Q.25. A and B started a business in partnership by investing Rs. 10000 and Rs. 4000 respectively. condition of partnership
is that B got Rs. 100 per month for management of the business. After paying 5% interest on the capital, annual
profit has distributed in the ratio of their investment, Find the share of their profit, if the annual profit is Rs. 4000.
(1) Rs. 3000 each (2) Rs. 2500 each (3) Rs. 1500 each (4) Rs. 2000 each (5) None of these
Q.26. A and B started a business together, A invested 36,000 and B invested 60,000. A is active partner and B is sleeping
partner. A received 10% of the profit for managing the business, and remaining is divided in proportion to their
capitals. If A total received Rs 7000 of profit, then find the share of B in profit.
(1) Rs 10,000 (2) Rs 12,000 (3) Rs 8,000 (4) Rs 9,000 (5) None of these

EXERCISE Explanation
Q.1.(5) Ratio of capital month.
A : B : C =(16000×3+14000×9) : (10000×9) : (9000×6)
30000×12 : (24000×4 : (42000×4 =(48000+126000) : (90000) : (54000)
+18000×8) +32000×8) 29 : 15 : 9
45 : 30 : 53 Where,
Now, 9 ratio = 1026
11960 1026
B's share= × 30 = 2917.073 ∴ 29 ratio = × 29
40 + 30 + 53 9
or Rs. 2900 (Approx.) A's share of profit = Rs. 3306
16000 × 5 Q.3.(3) For profit sharing ratio
Q.2.(2) B's Investment = = 10000
8 A : B : C
Now,
(5×12+8×12) : (6×24) : (8×12+4×12)
Ratio of equivalent capital of A, B and C for 1
74 QUANTITATIVE APTITUDE
www.mahendras.org
(60+96) (144) : (96+48) 1  5
= 90,000×2 ⇒ 90000 ×  = Rs. 225000
156x 144x 144x 2  2
= 13 : 12 : 12 Raksha : Kamal
Q.4.(2) Profit sharing ratio = 44 : 45
Prakash Sunil Anil Now, Raksha's share
=[1100000×3] : [1650000×3] : [825000×3] 7120000
= × 44 = Rs.3520000/-
= 4 : 6 : 3 44 + 45
1950000 3 4 2 2
Anil's share = × 3 ⇒ 4,50,000/- Q.10.(1) =  −  :  − 
4+6+3 5 9 5 9
Hence, 50% of Anil's share = 2.25 lakh
 27 − 20   18 − 10 
Q.5.(2) Capital Ratio : =  : = 7 : 8
 45   45 
Sonu Mona Raghu Q.11.(1) Let the initial investment of A,B and C be x, 3x,
99 : 90 : 100 5x
5780 Now,
Raghu's share in investment= 99 + 90 + 100 × 100
A B C
= Rs. 2000
3 5
Q.6.(4) Ratio of the profit of Gita, Shreya and Deepika (x×4+2x×8) :(3x×4+ x×8): (5x×4+ x×8)
2 2
= 48000 × 12 : 62000 × 8 : 80000 × 6 = 20x : 24x : 40x
∴ Deepika's share in profit = 5 : 6 : 10
20661 Q.12.(3) Ratio of their profit
= × 30 = Rs.6390
36 + 31 + 30
1 1   1 1 
Q.7.(2) Ratio of the profit = : : 1 −  +  
8 3   8 3 
(250×4+400×8) :(400×6+300×6) : (300×8+500×4)
⇒ 21 : 21 : 22 1 1 13
= : :
1280 8 3 24
∴ Ranveer share = × 21 = Rs.420
64 = 3 : 8 :13
Q.8.(2) Shiv Kumar's equivalent capital Now, for A & C
=(25000×3) + (10000×2) + (10000×1) A × 4 : C × 8 = 3 : 13
= Rs.105000 A × 4 : 1560 × 8 = 3 : 13
Rakesh's equivalent capital =(35000×2) 1560 × 8 × 8
A= = Rs.720
= Rs.70000 4 × 13
Suresh's equivalent capital= (35000×1) For B & C,
= Rs.35000 B × 6 : C × 8 = 8 : 13
Shiv Kumar : Rakesh : Suresh B × 6 : 1560 × 8 = 8 : 13
3 : 2 : 1 1560 × 8 × 8
Hence, Rakesh's share in the total profit B= = Rs.1280
6 × 13
15000
= 3 + 2 + 1 × 2 = Rs.50000/- Contribution by A & B together

Q.9.(1) Equivalent capital of Raksha = 720 + 1280= Rs. 2000

=Rs. (60000×1+80000×2) = Rs. 2,20,000 Q.13.(3) A : B : C

Equivalent capital of kamal = 20000×12 : 25000×4 : 15000×8

QUANTITATIVE APTITUDE 75
www.mahendras.org
= 12 : 5 : 6 Profit of R = 4x = 4 × 80,000= Rs. 3,20,000
23000 Q.20.(3) Initial capital = 2x : 4x : 8x
Share of C = × 6 = Rs.6000
23 Total capital invested by A
= (2x × 6 + 3x ×6) = 30x
7 4 6
Q.14.(5) A : B : C = : : = 35 : 20 : 18 Total capital invested by B
3 3 5
= (4x × 6 + 8x × 6) = 72x
Ratio of profit
Total capital invested by C= (6×8x + 6x × 6)
= [35×4+49×8]:20×12:18×12
= (48x + 36x) = 84x
532 : 261 : 216
New ratio of capitals :
21600
Total profit= × 1009 = Rs. 100900 A:B:C
216
Q.15.(1) Total investment Capital — 30x : 72x : 84 x
Profit— 5 : 12 : 14
= 100 × 45000 = Rs.300000
15 Required ratio of their profit = 5 : 12 : 14
Avinash : Manoj : Arun = 3 : 2 : 5 Q.21.(5) 12 : 16 : 9
Investment of Manoj 65700 37
× × 100 = Rs. 370000
 2  9 73
=  × 300000 = Rs.60000
 10  Q.22.(1) Let the total time = 8 years
Q.16.(3) A : B : C = 35000 ×12 : 20000×5 : 15000 × 7 Let the total capital = 20 units
84 : 20 : 21 X :Y : Z
Capital— 5 : 4 : 11
84125
Share of B = × 20 = 13460 ¯× ¯× ¯×
125
Time— 2 : 4 : 8
Q.17.(2) x × 12 : 2x × 6 : 3x × 4 Profit— 10 : 16 : 88
1:1:1 5 : 8 : 44
45000 According to the question,
Share of Manav = × 1 = Rs.15000
3 (5+8+44) units = Rs. 1140
Q.18.(3) 64000 × 12 : 48000 × x = 2 : 1 57 units = Rs. 1140
16 2 1140
= 1 units = Rs. = Rs. 20
x 1 57
x = 8 month Profit of x = 20 ×5 = Rs. 100

After = 12 - 8 = 4 month Profit of Y = 20 × 8 = Rs. 160

Q.19.(4) P : Q : R Profit of Z = 20 × 44 = Rs. 880

3 Q.23.(3) Let total profit = 24 units


Capital — 1 : :2
2 1
Profit of A = × 24 = 3 units
Profit— 2 : 3 : 4 8
Profit = (2x + 3x + 4x) = 9x units 1
Profit of B = × 24 = 8units
According to the question, 3
8 A:B:C
9x = 9,00,000 ×
100 Capital— x : y : 1560
9x = 7,20,000 Time— 4:6:8
Profit of P = 2x = 2 ×80,000 = Rs. 160,000 Profit — 3 : 8 : 13 [24-(8+3)]
Profit of Q = 3x = 3 × 80,000 = Rs. 240,000
76 QUANTITATIVE APTITUDE
www.mahendras.org
We know Profit of B = 1400 × 4 = Rs. 5600
Capital × Time = Profit Profit of C = 1400×3 = Rs. 4200
Pr ofit Q.25.(4) B's profit share in 1 years = 12×100
= Capital
Time = Rs. 1200
13 units = 1560
∴ 10000 × 5 × 1
8 Interest of A = 100 = Rs. 500
1 unit = Rs. 960
4000 × 5 × 1
960 × 8 Interest of B = = Rs. 200
y= 100
6
Total profit of A and B = 1200 + 500 +200
y = Rs. 1280 = Rs. 1900
3 Remaining profit = 4000 - 1900
x = × 960 = Rs. 720
4
= Rs. 2100
Capital of A = Rs. 720
A : B
Capital of B = Rs. 1280
Capital 10000 : 4000
1 1 1
Q.24.(4) A : B : C = : : 5 : 2
4 3 6
Ratio of shares of A, B and C 2100
Share of A in remaining profit = ×5
A:B:C ( 5 + 2)
= Rs. 1500
Capital 3x : 4x : 2x
Share of B in remaining profit
Total capital invested by A in 1 year
2100
3x × 4 + 1.5x × 8 = 24 x = m (5 + 2) × 2 = Rs.600
Total capital invested by B in 1 years
Total profit of A
4x
= 4x × 6 + 3 × 6 = 32x = 500 + 1500 = Rs. 2000
Total capital invested by C in 1 year Total profit of B = 1200 + 600 + 200
= 2x × 12 = 24x = Rs. 2000
A:B:C Q.26.(4) Ratio of their profit share =36000: 60000 =3: 5
Capital 24 x : 32x : 24x Let total Profit = 80
3x : 4x : 3 x 3
A will get 10% of 80 + of remaining profit
8
According to the question,
(3x + 4x + 3x) = 14000 3
8 + 72 × = 35
10x = 14000 8

x = 1400 Now 35 = 7000
Hence, Profit of A = 1400×3 = Rs. 4200 45
so 45 = 7000× = 9000
35

QUANTITATIVE APTITUDE 77
www.mahendras.org

CHAPTER
MIXTURE and
10
ALLIGATION
Sol. Cost price of 1 kg of mixture
General Rules
100
I. Alligation : It enables us to find the ratio in which two = 68.20×
110
or more ingredients at the given price must be mixed
to produce a mixture of a desired price. Applying Rule of Alligation
Ist type IInd type
II. Mean price : The cost price of a unit quantity of the
mixture is called the mean price. Rs.60 Rs.65
III. Basic Formula : If two ingredients are A and B are (Mixture)
Rs.62
mixed of price x and y respectively, then the price of
resultant mixture is M mean price. The ratio in which
ingredients are mixed is given by rule of alligation
3 2
M− y
R = Required Ratio = 3 : 2
x −M
Ex. 729 Litres of mixtures containing milk and water in
The above formula can be respented as the ratio 7 : 2. How much more water should be added
Ingredient A Ingredient B so that the new mixture contains milk and water in the
x y ratio 7 : 3 ?.
2
Sol. 729 litre quantity of water =
9
mean price M Pure water = 1
3
Quantity of water in mixture =
10
(M-y) : (x-M) Applying Alligation:
2
Thus the required ratio is R 1
9
M− y y −M
= = 3
x −M M− x
10
Ex. In what ratio must rice costing Rs.8.50 per kg be mixed
with rice costing Rs.13 per kg so that the mixture be
3 3 2
worth Rs.10 per kg? 1
10 10 9
Sol. One type of Rice 2nd type of Rice 7 7
Rs.8.50 Rs.13.00 10 90
Required Ratio = 9 : 1
10 Rs. 9 = 729 litre
729
1= ×1= 81 Litres
3.00 1.50 9
IV. Mixture of more than two elements:
2 : 1 These question may seems to be a little tricky at first,
but similar concept is applied repeatedly in order
Ex. In what ratio must a grocer mix two varities of sugr to calculate final ratio of ingredients when mixture
costing Rs.60 per kg and Rs.65 per kg, so that on contains more than two ingredients.
selling the mixture at Rs.68.20 per kg he may gain
1. Take two ingredients in such away that first ingredient
10%?
is lower than the mean value and the other one is
78 QUANTITATIVE APTITUDE
www.mahendras.org
higher than mean value. in cask to that of the water is 16 :65. How much wine
2. Calculate the ratio of ingredients. did the cask hold originally ?
3. Repeat for all possible pairs.
Sol. Let initial quantity of wine in a cask = x lit
4. Final ratio is the ratio obtained from step 2 (if an
ingredients is common in the ratios, add value for this
So, After 4 process,
particular ingredient)
Ex. Three types of Rice of Rs. 1.27, Rs. 1.29 and Rs. 1.32 L F 8I 4 OP
per kg are mixed together to be sold at Rs. 1.30 per 4 = M xGH 1 − x JK
kg. in what ratio should this rice be mixed.

MN PQ lit.
Sol. I III II III
FG
x 1−
8 IJ 4

127 132 129 132



H x K =
16
⇒ 1−
8 FG IJ = FG 2 IJ
4 4

FG x − 8 IJ = 2
130 130 x 81 x H K H 3K H x K 3
2 3 1 2 ⇒ 3x - 24
Hence final ratio is 2 : 1 : 3 + 2 = 2 : 1 : 5 = 2x ⇒ x
V. Mixture containing 4 ingredients:
= 24 lit.
Ex. How much a shop owner mixture 4 types of rice worth
Rs. 95, Rs. 60, Rs. 90 & Rs. 50 perkg so that he can Ex. A shopkeeper has two types of article. The CP of 1st
make the mixture of these rice worth Rs. 80 perkg. article is 20 Rs./Kg. and other article is X Rs./kg. He
Sol. II III has quantity of 1st article is 10 kg and other article
I II
is 20 kg. He sold the mixture of these article at Rs.
95 60 60 90
39/kg. with a profit of 30%. Find the value of X ?
Sol. 20 x
80 80
30

20 15 10 20 30-x : 10
4 : 3 1 : 2 Again
III IV 30 − x 10
90 50 =
10 20
80 On solving above equation
x = 25 Rs./Kg
10 Ex. The ratio of milk and water in a mixture is 5 : 3.
30 How much fraction of the mixture must be drawn
3 : 1 off and substituted by water so that the ratio of milk
I : II : III : IV = 4 : 4 : 5 : 1 and water is the mixture becomes 1 : 1 ?
Sol. 3 1
VI. Concept of replacement:
8 1
Suppose a container contains a - units of liquid from
which b -units are taken out and replaced by water- 2
after k-operations, the quantity of pure liquid
1 1
:
 k

= a  1 − b   units.
2 8
  a   4 : 1

Ex. 8 litres are drawn from a cask full of wine and is then 1
Removed mixture =
filled with water. This operation is performed three 5
more times. The ratio of the quantity of wine now left

QUANTITATIVE APTITUDE 79
www.mahendras.org
EXERCISE
Q.1. What quantity of tea costing Rs.8 per kg be mixed with 15 kg of tea costing Rs.20 per kg so that mixture be worth
Rs.12 per kg.?
(1) 15 kg (2) 20 kg (3) 25 kg (4) 30 kg (5) 40 kg
Q.2. Two alloys are both made up of copper and tin. The ratio of copper and tin in the first alloy is 1 : 3 and in the second alloy
is 2 : 5. In what ratio should the two alloys be mixed to obtain a new alloy in which ratio of tin and copper be 8 : 3?
(1) 7 : 5 (2) 5 : 7 (3) 4 : 7 (4) 7 : 4 (5) 3 : 4
Q.3. A mixture contains milk and water in the ratio 3 : 2 and another mixture contains that of in the ratio 4 : 5. How
many litres of the latter must be mixed with 3 ltrs of the former so that the resulting mixture may contain equal
quantities of milk and water?
3 2 4 2 2
(1) 5 ltr. (2) 5 5 ltr. (3) 1 5 ltr. (4) 2 ltr. (5) 4 ltr.
5 5 5
Q.4. A mixture contains two liquids ‘A’ and ‘B’ are in the ratio 4:1. If 10 litres of mixture is withdrawn and replaced
with 10 litres of ‘B’, then the ratio becomes 2:3. What was the initial quantity of A?
(1) 16 ltr. (2) 20 ltr. (3) 25 ltr. (4) 15 ltr. (5) 18 ltr.
Q.5. A Jar contains a mixture of milk and water in the ratio 5:1, 18 litre of mixture is taken out from jar and 6 litre pure
water is added in the jar. If the resultant ratio between milk and water in the jar is 3:1, what was the initial quantity
of mixture in jar before the replacement?
(1) 54 ltr. (2) 36 ltr. (3) 60 ltr. (4) 72 ltr. (5) 76 ltr.
Q.6. Ajay purchased two different kinds of vinegar. In the first mixture, vinegar is 75% and in the second mixture, vin-
egar is 83.33%. If he mixes, the two given mixtures and makes a third mixture in which the percentage of vinegar
is 80%, what is the quantity of the first mixture that is required to make 20 litres of the third kind of mixture?
(1) 8 litres (2) 9 litres (3) 6 litres (4) 7 litres (5) None of these
Q.7. Two vessel contain petrol and kerosene in the ratio 7 : 3 and 1 : 4. In what ratio should both vessel is mixed so that
the mixture contain petrol and kerosene in the ratio 3 : 2?
(1) 4 : 1 (2) 1 : 4 (3) 2 : 3 (4) 3 : 2 (5) None of these
Q.8. If a mixture contains 60% of milk and the remaining is water. Another mixture contains 75% milk, 20% water.
The new mixture is formed by mixing these two mixtures such that the water contains 25%. In the new mixture
the remaining is honey. What is the % of honey in the new mixture?
(1) 11.25% (2) 15% (3) 3.75% (4) 12% (5) None of these
Q.9. Two tank contain petrol and diesel. In the first tank the ratio of petrol to diesel is 7:4 and in the second tank the ratio
is 4 : 5. A third tank of capacity 38L is filled with these two tanks so that the mixture contains the ratio of petrol to
diesel is 5 : 4. How many litres are taken from the second tank?
(1) 11L (2) 12L (3) 13L (4) 16L (5) None of these
Q.10. 1050 rupees are distributed among 1400 males and females such that each male got Rs.1 and each female got 50
paise. Find the number of females among the group.
(1) 600 (2) 700 (3) 800 (4) 550 (5) 640
Q.11. Weight of two friends Ram and Shyam are in the ratio of 4:5. Rams weight increases by 10% and the total weight of
Ram and Shyam together becomes 82.8 kg, with an increase of 15%. By what % did the weight of Shyam increase?
(1) 17% (2) 18% (3) 19% (4) 20% (5) 21%
Q.12. Mukesh earned Rs.4000 per month. From the last month, his income is increased by 8%. Due to rise in prices, his
expenditure also increased by 12% and his saving decreased by 4%. Find his increased expenditure.
(1) Rs.3000 (2) Rs.3200 (3) Rs.3360 (4) Rs.3060 (5) Rs.3150
Q.13. A man travelled a distance of 90 km in 9 hours, partly on foot at 8 km/hr and partly on bicycle at 17 km/hr. Find
the distance travelled on foot.
80 QUANTITATIVE APTITUDE
www.mahendras.org
(1) 50 km (2) 55 km (3) 60 km (4) 56 km (5) 75 km
Q.14. A cricketer whose bowling average is 24.85 run per wicket takes 5 wickets for 52 runs and there by decrease his
average by .85. The number of wickets taken by him before the last match was-
(1) 64 (2) 72 (3) 80 (4) 96 (5) 92
Q.15. A man lent Rs. 1500 partly at 6% and other at 5% simple interest. If the total interest earned at the end of 1 year is
Rs.85, then find the amount lent out at 5% simple interest?
(1) Rs.550 (2) Rs.500 (3) Rs.540 (4) Rs.640 (5) Rs.555
Q.16. Jaya purchased 150 kg of wheat at the rate of Rs.7 per kg. She sells 50 kg at a profit of 10%. At what rate should
she sell the remaining quantity so that she gains an overall profit of 20%?
(1) Rs.8 (2) Rs.8.25 (3) Rs.8.40 (4) Rs.8.75 (5) Rs.9.50
Q.17. The ratio of land to water on the whole of the earth is 1:2 and it is 2:3 on the northern hemisphere. The ratio of land
to water on the southern hemisphere-
(1) 11 : 4 (2) 4 : 11 (3) 15 : 4 (4) 4 : 15 (5) 11 : 15
Q.18. A mixture contains milk & water in the ratio 5 : 3. If 16 litres of mixture is replaced by 16 litres of water the ratio
of milk and water becomes 3:5. How much milk was there in the mixture initially?
(1) 8 ltr. (2) 16 ltr. (3) 24 ltr. (4) 25 ltr. (5) 28 ltr.
Q.19. A vessel contains mixture of spirit and water. The percent of spirit in the mixture is 18%, 6 litres of mixture are
replaced with pure water, if the vessel contains 15% spirit now, the capacity of vessel is-
(1) 25 ltr. (2) 30 ltr. (3) 36 ltr. (4) 40 ltr. (5) 45 ltr.
Q.20. How many kg of salt at 42 paisa per kg must be a man mixed with 25 kg of salt at 24 paisa per kg., so that 25%
gain on outlay on selling the mixture at 40 paisa per kg ?
(1) 15 kg (2) 16 kg. (3) 25 kg. (4) 30 kg. (5) 18 kg.
Q.21. A man has 90 pens. He sells some of these at a profit of 15% and the rest at 9% profit. On the whole transaction he
gets a profit of 11%. How many pens did he sell at 9% profit ?
(1) 60 (2) 50 (3) 40 (4) 70 (5) 30
Q.22. A mixture of a certain quantity of milk with 15 ltr of water is purchase at 100 paisa per ltr. If pure milk be worth
Rs. 1.15 ltr, then how much milk is there in the mixture ?
(1) 80 ltr (2) 90 ltr (3) 100 ltr (4) 110 ltr (5) 120 ltr
Q.23. Ram covered a distance of 200 km in 10 hrs. The first part of his journey is covered by auto, then he hired a car. The speed
of the auto and car is 15 km/hr. and 30 km/hr. respectively. Find the ratio of distance covered by auto and that of car.
(1) 3 : 4 (2) 2 : 1 (3) 1 : 1 (4) 2 : 3 (5) None of these
Q.24. Two brands of detergents are to be combined detergent A contains 40% bleach and 60% soap. While detergent B
contains 25% bleach and 75% soap. If the combined mixture is to be 35% bleach. What % of the final mixture
should be detergent A ?
(1) 30% (2) 45.64% (3) 20% (4) 32.5% (5) 66.67%
Q.25. 12 litres of water is drawn out from a container full of water and replaced by milk. Again 12 litres of mixture are
drawn and the container is again filled with milk. The ratio of final quantity of water to milk in the container is 25
: 11. How much did the container hold ?
(1) 60 litres (2) 65 litres (3) 72 litres (4) 39 litres (5) None of these
Q.26. Tea worth Rs. 126 per kg and Rs. 135 per kg are mixed with a third variety of the ratio 1 :1 : 2 If the mixture is
worth Rs. 153 per kg, the price of the third variety per kg will be :
(1) 175.50 Rs./kg (2) 195.50 Rs./kg (3) 165.50 Rs./kg (4) 135.50 Rs./kg (5) None of these
Q.27. In what ratio must a person mix three kind fo tea each of which has a price of 70,80 & 120 rupees per/kg in such
a way that the mixture cost 100 Rs./kg
(1) 2 : 3 : 4 (2) 2 : 1 : 2 (3) 5 : 2 : 2 (4) 2 : 2 : 5 (5) 3 : 2 : 1

QUANTITATIVE APTITUDE 81
www.mahendras.org
EXERCISE Explanation
Q.1.(4) Required Ratio = 1 : 1
One type of Tea 2nd type of Tea
1 = 10 litre
Rs.8/kg Rs.20/kg
2 = 20 litre
12 Rs./kg (Mixture) 20
Quantity of A = ×4 = 16 litre
5
1
8 4 Q.5.(4) Quantity of water in the Jar =
6
2 : 1 (Ratio) Quantity of water added = 1
1 Part = 15 kg 1
Quantity of water in the mixture = 4
2 Part = 15×2 = 30 kg.
1
3 6 1
Q.2.(3) Quantity of Tin in Ist alloy =
4 1
5 4
Quantity of Tin in 2 alloy =
nd
7
Quantity of Tin in the mixture = 3 1
Apply rule of Alligation: 4 12

3 5 Required Ratio = 9 : 1
4 7 1 =.6 litre
8 9 = 54 litre
11
Now = 54 + 18 litre = 72 litre
Q.6.(1) By allegation rule
8 5 3 8
11 7 4 11
3/4 5/6
1 1 4/5
77 44 1/30 1/20=2:3
Required Ratio = 4 : 7
Required answer =20×2/5 = 8 litres
3
Q.3.(2) Quantity of milk in the Ist mixture= Q.7.(1) 30% 80% Kerosene in first mixture
5
4 = 3/10×100%= 30%
Quantity of milk in the IInd mixture =
9 40% Kerosene in final mixture
1
Quantity of milk in the final mixture= = 2/5×100%= 40%
2
Applying Alligation: 40% 10% Kerosene in second mixture
3 4 = 4/5×100%= 80%
5 9
4 : 1
1
2 Hence, option(1) is correct.
Q.8.(3) Taking percentage of water
1 4 3 1 40 20
2 9 5 2 25
1 1
18 10 5 15
Required ratio = 5 : 9 1:3
5 Part = 3 litres Now let first mixture be 100 ltr. and second
3 27 2 mixture be 300 ltr.
9 Part = 5 ×9 = 5 = 5 5 litres Quantity of Milk = 5% of 300 = 15 ltr.
Q.4.(1) 15
1 Percentage of Milk in final mixture = ×100
5 1 400
= 3.75%
3 Q.9.(4) 7/11 4/9
5
5/9
2 2 1/9 : 8/99
5 5 11 : 8
82 QUANTITATIVE APTITUDE
www.mahendras.org
Amount taken from the first tank = 11/19×38 In this case we get ratio of time
= 22L Hence,
Amount taken from the second tank = 8/19×38 9 = 90 hrs.
= 16L
7 = 7 hrs.
Hence, option(4) is correct.
D = S × T = 8 × 7 = 56 km/hrs.
Q.10.(2) Amount received by each man = 1 Rs.
Amount received by one women = 50 paise Q.14.(3) New Average = 24 run per wicket
Amount received by each person Old Average = 24.85
1050 52
= = 75 paise Average of last match = = 10.4
1400 5
Applying Alligation :
Applying alligation :
24.85 10.4
1 .50
24
.75

13.6 .85
.25 .25
Required Ratio = 16 : 1
Required Ratio = 1 : 1 1 = 5 wicket
2 = 1400 16 = 80 wicket
1400 Q.15.(2) Interest on 1500 at 6% = 90 Rupees
1= 2 ×1 = 700 womens.
Q.11.(3) % increase in Ram’s weight = 10% Interest on 1500 at 5% = 75 Rupees
% increase in Shyam’s weight = x% 85 × 100 17
Overall Interest = = %
% increase in overall weight = 15% 1500 × 6 3
Ram Shyam on Rs.1500/-= 85 Rupees
10% x%
90 75 6% 5%

17%
15% 85
Or 3

2 1
: 10 5
x-15 5 3 3
2 1 2 1
x − 15 4
= 1
5 5 Required Ratio = ×1500 = 500 Rs.
3
x = 19% Q.16.(4) S.P. at 10% Profit = 7.70
S.P. at 20% Profit = 8.40
Q.12.(3) % increase in Exp % decrease in Saving
12 -4 (50 kg) (100 kg)
7.7 X
8 % increase in income
8.4

12 4 2
1
Required Ratio = 3 : 1
8.4 − 7.7 2
3 =
Expenditure = × 4000 =3000 Rs. x − 8.4 1
4
112 x = 8.75 Rs.
Increase Expenditure = 3000 × = Rs.3360
100
Q.13.(4) Foot On cycle Q.17.(2) By Alligation:
2
8 km/hr 17 km/hr x
5
1
10 km/hr Overall speed 3
= 90/9= 10 km.
1 1
7 : 2 2 2

QUANTITATIVE APTITUDE 83
www.mahendras.org
2 1 1 6 − 5 1 − 3x Distance ratio = 15×2 : 30×1 =1 : 1
⇒ − = − x = =
5 3 3 15 3 Q.24.(5) A B
40 25
1 = 5-15x
4 = 15x
4 35
x=
15
Required Ratio of land to water
10 : 5
= 4 : (15 - 4) = 4 : 11 2 : 1
Q.18.(4) By Alligation: Therefore, % of detergent in A
5 0 = (2/3)×100 = 66.67%
8
2
3
Q.25.(3) x  1 − 12  = 11
8  
x 25
3 2
8 8 x = 72 liter
3:2 ⇒ 5 Q.26.(1) Since first and second varieties are mixed in equal
2 Ratio = 16 Litre proportions, so their average price
5 Ratio = 40 Litre FG 126 + 135 IJ =Rs. 130.50

5
Initial quantity of Milk = × 40 = 25 Litre
= Rs. H 2 K
8 So, the mixture is formed by mixing two varieties,
Q.19.(3) 18 0
one at Rs. 130.50 per kg and the other at say, Rs.
15 x per kg in the ratio 2 : 2, i.e., 1 : 1. We have to
15 : 3 find x.
5 : 1 By the rule of alligation, we have :
Now, 130.50 x
1=6
6 = 36 litre.
153
40
Q.20.(2) Cost Price of mixture= × 100 = 32 Rs. / Kg
125
42 24 2 2
32 1 : 1
8 : 10 153 − 130.5 1
Hence, =
4 : 5 x − 153 1
5 = 25 x = 175.50 kg.
4 = 16 Q.27.(4)
I
III
Q.21.(1) 15 9 70 120
11
100
2 : 4
Þ 1 : 2 20 30
Hence pen at 9% profit = 2/3 × 90 = Rs.60
Similarly
Q.22.(3) 115 0
II pair III pair
100 80 120
100 : 15
100
20 : 3
3 = 15 litre, 20 = 100 litre 20
20
Q.23.(3) 15 30 Thus find ratio
20
= I : II : III
10 : 5
= 20 : 20 : (30 + 20) = 2 : 2 : 5
2 : 1
84 QUANTITATIVE APTITUDE
www.mahendras.org

CHAPTER

11 TIME AND WORK

In most of the problems on time and work, one of the


Important Formulas
following basic parameters is to be calculated:
(a) Time : Time needed by one or more than one (i) If A can do a piece of work in X days and B
person to complete a job or time for which a person(s) can do the same work in Y days, then both of
actually worked on the assigned job. them working together will do the same work
(b) Alone time : Time needed by single person to
in 
XY 
complete a job.  days
X+Y
(c) Work : The amount of total work (assigned) or
the part of total work actually done. (ii) A,B,C while working alone can complete a work
in X,Y and Z days respectively then they will
Basic Concepts together complete the work in
(1) Total amount of a complete job (or assigned job) = 1,
XYZ
always, unless specified. days
XY + YZ + ZX
(2) If any person ‘M’ completes a job alone in t days, then
alone time for ‘M’ = t Ex. A can do a piece of work in 6 days and B in 9 days. How
many days will both take together to complete the work?
(3) 1 day’s work by any person
Sol. Method I:
F 1 IJ
= G
th

H alone timeK × part of total work Here A = 6 and B = 9

(4) The reciprocal of 1 day’s work gives the alone time i.e. They together complete work in 1 days

1 = 1/6 + 1/9
Alone time ∝
1day' s work 3+2 5
=
(5) When more than one person working on the same = 18 18
piece of work, then their combined 1 day’s work = 18
sum of 1 day’s work by each person. i.e., If A, B and They together complete the whole work=
5
C are three persons working on a job, then (A+B+C)’s = 3.6 days
1 day’s work = A’s 1 day’s work + B’s 1 day’s work +
C’s 1 day’s work. Method II (L.C.M. method) :
(6) It is the application of concept (4) for more than one
person.

The reciprocal of combined 1 day’s work gives the
time for completion by the person working together. 18
They together complete the whole work=
5
i.e., time for completion = 3.6 days
1 Formula Method:
=
combined 1day' s work They together complete the whole work
It implies that.  XY  9×6
=  X + Y  = 9 + 6 = 3.6 day
If three persons, say, A, B and C are working together
on a job, then Ex. If man A complete a piece of work in 12 days and the
Time for completion by them same work is completed by B and C in 15 days and
20 days respectively. In how many days the work is
1
= completed if they work together ?
b g
A + B + C ' s 1 day' s work
QUANTITATIVE APTITUDE 85
www.mahendras.org
Sol. METHOD I : FORMULA METHOD :
1 1 1 by formula
Work/ day of A,B and C = + +
12 15 20  XY  8×5 1
5+4+3 1 = X − Y  = = 13 days
= =   8−5 3
60 5
3. If A and B working together, can finish a piece of work
Total days = 5 days in X days, B and C in Y days, C and A in Z days then.
METHOD II (L.C.M. method) : A, B and C working together will finish the job in
5w/d A 12 days 2XYZ
4w/d B 15 days L.C.M.= 60 work XY + YZ + ZX days
3w/d C 20 days Ex. A and B can complete a piece of work in 8 days B
Total work/day by (A+B+C) = 12 work and C can do it in days 12 days, C and A can do it in
8 days, A B and C together can complete it in-
Total work is done by (A+B+C)
60 1
= =5 days Sol. Work done by A and B in one day = 8
12
FORMULA METHOD :
1
They together complete the whole work work done by B & C in one day =
12
xyz
= 1
xy + yz + zx work done by C and A in 1 day = work done by
A,B and C together in 1 day 8
12 × 15 × 20
= = 5 days 1 1 1 1 1
12 × 15 + 15 × 20 + 20 × 12 =  + +  =
2  8 12 8  6
2. Two person A and B working together, can complete a
piece of work in X days. If A alone can complete the A,B and C together complete the whole job =6 days
work in Y days, then B working alone will complete METHOD II (L.C.M. METHOD) :
 XY  3 w/d (A+B) 8d
the work in   days 2 w/d (B+C) 12 d
X−Y
3 w/d (C+A) 8d
Ex. A and B together can do a piece of work in 5 days and 8 w/d 2(A+B+C)
A alone can do it in 8 days. B alone can do the same 4 w/d (A+B+C)
one day work
piece of work in
Time taken by A, B & C together to complete the work
Sol. METHOD I :
= 24/4 = 6 days
Here (A+B) = 5 days, A = 8 days
FORMULA METHOD:
B alone can finish the work in one day
A + B and C can do finish the whole work
1 1 8−5
= − = part
5 8  40  =
2 × 8 × 12 × 8
=
2 × 8 × 12 × 8
= 6 day
40 8 × 12 + 12 × 8 + 8 × 8 96 + 96 + 64
B finish the whole work = days
3 Ex. A can complete a piece of work in 20 days & B can
1 complete the same work in 25 days. If they start the
= 13 days work together but After 5 days A left the work. In how
3
many days the total work would be finished?
METHOD II. (L.C.M. METHOD) :
Sol. METHOD I:
8 w/d A+B- 5 day
5 w/d A - 8 day Total work Work/day of A and B

3 w/d B 1 1 9
= + =
(One day work) 20 25 100
5×9
B finish the whole work A and B’s 5 day’s work = = 9/20
100
40 1
= = 13 days Remaining work = 1 - 9/20 = 11/20
3 3
86 QUANTITATIVE APTITUDE
www.mahendras.org
Since A left after 5 days hence remaining work done 1
by B = 11/20 Total days So total days = 13 +3
5
11/ 20 11 25 13 3 1
= × = days = 16 days
1 20 1 4 5
25 Ex. A can complete a piece of work in 9 days and B in 12
3 3 days respectively. If they work for a day alternately,
So, total days = 5 + 13 = 18 days
4 4 In how many days the work would be finished, If A
METHOD II: begins the work?
5w/d A 20 days Sol. 4w/d A - 9 days
100 work LCM = 36 work
4w/d B 25 days
3w/d B - 12 days
9w/d A+B
3 Now,
A+B A B 55 =13 days A B A B
5 days 4 4
7w
5×9 = 45w 55w 4w 3w 4w 3w
100 w 2 days 2 days

3 3 7 w → 2 days
Total Time = 5+13 = 18 days
4 4 5x 7w → 2 days ×5
Ex. A and B can complete a piece of work in 24 days &
36 days respectively. They start the work but 3 days 35w → 10 days
before completion of work, A left. In how many days Remaining work = 36 -35 = 1 w
will the total work be completed. 1w will be done by A (because A starts the work)
Sol. METHOD I :
A’s 1 day’s work = 1/24 1
∴ 1w → 4 days----------- (done by A)
B’s 1 days’ work = 1/36
∴A left before 3 days

FG
∴ Total time = 10 +
1 IJ
days
∴For last 3 days B’s work =
3
=
1
1
H 4 K
36 12 = 10 days
4
1 11 4. Work efficiency of any worker inversely proportional
Remaining work = 1 − =
12 12 to time taken by him.
Hence remaing work done by both so total days 1
i.e, work efficiency ∝ Time
11 11
12 12 1 Ex. A is twice as good a work man as B & together they
= = = 13 finish a piece of work in 18 days. In how many days
1 1 5 5
+ will A alone finish the work ?
24 36 72
Sol. A : B = 2 : 1
1 1
So total days = 13 5 +3 = 16 5 days Time taken by A and B = 18 days

METHOD II: work done by A and B = 18 × 3 = 54 work


3 w/d A−24 day 54
2 w/d B −36 day A will do alone = = 27 days
Total work 2
5 w/d A+B Ex. If the ratio of work efficiency of A & B is 6 : 5 & that
(One day work) of B & C is 6 : 5. If A can complete the piece of work
1 in 2 days, In How many days the same work can be
66/5= 13 5 days A completed by B separately ?
A+B B Sol. A : B = [6 : 5]
(3 days) Note : Here B : C is not required, this is given to
66 work 3×2=6 confuse you,
work
72 work time taken by A = 2 days
QUANTITATIVE APTITUDE 87
www.mahendras.org
work done by A = 2 × 6 = 12 work ×
a men or b women or c children d days
12 2
time taken by B = = 2 days
5 5
5. MDH FORMULA:
x men & y women & z children ? days
(i) More men can do more work. i.e. M ∝ W.
a×b×c ×d
1 D=
(ii) More men means less working hours. i.e. M ∝ H . xbc + yac + zab
(iii) More men can do same work in less number of Ex. 3 Men or 4 Women can do a piece of work in 43 days.
1 In how many days 7 men & 5 women can do the same
days. i.e. M ∝ .
D work ?
W Sol. 3m or 4w = 43 days
M∝
DH 7m + 5w = ?
MDH From formula : M1 D1 = M2 D2
= Constant
W 3 m × 43 = (7m + 5w) × D2
(iv) If M1 can do W1 work in D1 days working H1 hr/ 15
day for Rs. R1 and M2 man can do W2 work in 129 m = (7m + m) × D2
4
D2 days working H2 hr/day for Rs. R2 then
43m
ForMULA is given as: 129 m =
4
× D2 ⇒ D2 = 12 days
M1D1H1 M2D 2H2
= SHORT Trick :
W1R1 W2R2
a×b×d
D=
Here, R = wages (in Rs.) xb + ya
Ex. If 10 men complete half work in 12 days when they 3 × 4 × 43
work 8 hours per day, In how many days 18 men D= = 12 days
28 + 15
complete the full work when they work 6 hours/day ?
Ex. 1 man or 2 boys or 3 girls can do a piece of work in
Sol. By formula : 88 days. In how many days one man, one boy and one
M1D1H1 M2D2H2 girl can do the same work ?
=
W1 W2 Sol. 1m = 2b = 3g = 88 days

(1m + 1b + 1g) = ?
10 × 12 × 8 18 × D2 × 6
= By formula : m1 d1 = m2 d2
1/ 2 1
3g × 88 = (1m + 1b + 1g) × d2
10 × 12 × 8 × 2 = 18 × D2 × 6 3
160 7
3g × 88 = (3g + g + 1g) d2
D2 = = 17 days. 2
9 9
3g × 88 = FG 6g + 3g + 2gIJ d
6. (i) If a men or b women can do a piece of work in
d days then x men and y women together finish

H 2 K 2

the whole work- 11


3g × 88 = g × d2
a men or b women d days 2
×
+ d2 = 48 days
x men & y women ? days
SHORT Trick :
a×b×d a×b×c ×d
D= D=
xb + ya xbc + yac + zab

(ii) If a men or b women or c children can do a piece of 1 × 2 × 3 × 88


D= = 48 days
work in d days then x men, y women and z children 6+2+3
together finish the whole work-
88 QUANTITATIVE APTITUDE
www.mahendras.org
EXERCISE
Q.1. A man can do a job in 15 days. His father takes 20 days and his son finishes it in 25 days. How long will they take
to complete the job if they all work together ?
15 17 18 12
(1) 5 days (2) 5 days (3) 6 days (4) 6 days (5) None of these
47 23 47 47
Q.2. A man can do a piece of work in 5 days, but with the help of his son, he can do the same work in 3 days. In what
time can the son do it alone ?
1 1
(1) 6 days (2) 7 days (3) 7
days (4) 8 days (5) None of these
2 2
Q.3. Two workers A and B are engaged to do a work. A working alone takes 8 hours more to complete the job than if both
1
worked together. If B worked alone, he would need 4 hours more to complete the job than they both working
2
together. What time would they take to do the work together ?
(1) 4 hours (2) 5 hours (3) 6 hours (4) 7 hours (5) None of these
Q.4. A and B can do a piece of work in 72 days; B and C can do it in 120 days while A and C can do it in 90 days. In
what time can A alone do it ?
(1) 80 days (2) 100 days (3) 120 days (4) 150 days (5) None of these
Q.5. A and B can do a piece of work in 5 days; B and C can do it in 7 days; A and C can do it in 4 days. Who among
these will take the least time if put to do it alone ?
(1) A (2) B (3) C (4) Data inadequate (5) None of these
Q.6. A can do a piece of work in 4 hours; B and C together can do it in 3 hours, while A and C together can do it in 2
hours. How long will B alone take to do it ?
(1) 8 hours (2) 10 (3) 12 hours (4) 24 hours (5) None of these
Q.7. A can do a certain work in the same time in which B and C together can do it, A and B together could do it in 10
days and C alone in 50 days, then B alone could finish it in :
(1) 15 days (2) 20 days (3) 25 days (4) 30 days (5) None of these
Q.8. A does half as much work as B in three-fourth of the time. If together they take 18 days to complete the work, how
much time shall B take to do it ?
(1) 30 days (2) 35 days (3) 40 days (4) 20 days (5) None of these
Q.9. A can finish a work in 24 days, B in 9 days and C in 12 days. B and C start the work but are forced to leave after
3 days. The remaining work was done by A in :
1
(1) 5 days (2) 6 days (3) 10 days days (4) 10 (5) None of these
2
Q.10. X and Y can do a piece of work in 20 days and 12 days respectively. X started the work alone and then after 4 days
Y joined him till the completion of the work. How long did the work last ?
(1) 6 days (2) 10 days (3) 15 days
(4) 20 days (5) None of these
Q.11. What will come at place of blank?
Aman completes a work in _ days. Brijesh and Dheeraj completes the same work in 22.5 days working together.
Brijesh is 33.33% more efficient than Aman whereas Dheeraj is 20% less efficient than Aman.
(1) 24 (2) 36 (3) 45 (4) 48 (5) None of these
Q.12. A person can complete a job in 91 days. He works alone on Day 1. On Day 2, he is joined by another person who
also can complete the job in exactly 91 days. On Day 3, they are joined by another person of equal efficiency. Like
this, everyday a new person with the same efficiency joins the work. How many days are required to complete the
job?
(1) 12 (2) 15 (3) 13 (4) 14 (5) 11
QUANTITATIVE APTITUDE 89
www.mahendras.org
Q.13. Ram, Lakhan and Krish are ready to do a work. Ratio of time taken to complete the work by Lakhan and Ram is
9 : 4 and ratio of time taken by Ram and Krish to finish the work is 3:2. If they all complete the work in 27 days
than find the number in which Krish will complete the work?
(1) 56 days (2) 46 days (3) 53 days (4) 37 days (5) None of these
Q.14. If 5 men or 9 women can do a piece of work in 19 days. In how many days will 3 men and 6 women do the same
work?
(1) 10 days (2) 12 days (3) 13 days (4) 15 days (5) None of these

Q.15. A can cultivate th of a land in 6 days and B can cultivate rd of the same land in 10 days. Working together A

and B cultivate th of a land in:


(1) 4 days (2) 5 days (3) 8 days (4) 10 days (5) 12 days
Q.16. A certain number of men can complete a job in 30 days. If there were 5 men more, it could be completed in 10 days
less. How many men were in the beginning?
(1) 10 (2) 15 (3) 20 (4) 25 (5) None of these
Q.17. A men undertakes to do a certain work in 150 days. He employes 200 men. He finds that only a quarter of the work is done
in 50 days. The number of additional men that should be appointed so that the whole work will be finished in time is:
(1) 50 (2) 75 (3) 100 (4) 125 (5) 300
Q.18. In a fort, there was sufficient food for 200 soliders for 31 days. After 27 days, 120 soliders left the fort. For how
many extra days will the food last for the remaining soliders?
(1) 10 days (2) 6 days (3) 4 days (4) 12 days (5) None of these

Q.19. 12 men take 36 days to do a work while 12 women complete th of the same work in 36 days. In how many days

10 men and 8 women together will complete the same work ?


(1) 6 days (2) 27 days (3) 12 days (4) Data inadequate (5) None of these
Q.20. 12 men can complete a piece of work in 36 days. 18 women can complete the same piece of work in 60 days. 8
men and 20 women work together for 20 days. If only women were to finish the remaining work in 4 days, how
many total women would be required?
(1) 70 (2) 28 (3) 66 (4) 40 (5) None of these
Q.21. A alone can do a piece of work in 6 days and B alone in 8 days. A and B undertook to do it for Rs. 3200. With
the help of C they completed the work in 3 days. How much is to be paid to C?
(1) Rs. 375 (2) Rs. 400 (3) Rs. 600 (4) Rs. 800 (5) None of these
Q.22. Six men earn as much as seven women, two women earn as much as three boys, four boys earn as much as five
girls. If a girl earns Rs. 16 a week, what does a man earn per week?
(1) Rs. 35 (2) Rs. 20 (3) Rs. 40 (4) Rs. 30 (5) None of these
Q.23. It 8 men, working 9 hours per day can build a wall 18 metres long, 2 metres wide and 12 metres high in 10 days.
How many men will be required to build a wall 32 metres long, 3 metres wide and 9 metres high by working 6
hours a day in 8 days?
(1) 25 men (2) 18 men (3) 30 men (4) 16 men (5) None of these
Q.24. A can do a piece of work in 36 days. B in 54 days and C in 72 days All the three began the work together, but A
left before 8 days and B 12 days before the completion of the work. In how many days, the work was finished from
the day it started?
(1) 24 (2) 28 (3) 36 (4) 48 (5) None of these
Q.25. Two men and 7 boys can do a piece of work in 14 days, 3 men and 8 boys can do it in 11 days In how many days
can 8 men and 6 boys do a work 3 times as much as the first ?
(1) 21 (2) 27 (3) 28 (4) 35 (5) None of these
90 QUANTITATIVE APTITUDE
www.mahendras.org
Q.26. Kaveri takes twice as much time as Kanti and thrice as much as Kalpana to finish a place of work. They together
finish the work in one day. Find the time taken by by Kaveri to finish the work.
(1) 2 (2) 4 (3) 6 (4) 8 (5) None of these
Q.27. 1 man or 2 women or 3 boys can do a work in 44 days. Then in how many days will 1 man, 1 woman and 1 boy
do the work?
(1) 24 (2) 48 (3) 36 (4) 40 (5) None of these
Q.28. Two friends take a piece of work for Rs. 960. One alone could do it in 12 days, the other in 16 days with the
assistance of an expert they finish it in 4 days How much remuneration the expert should get ?
(1) Rs. 400 (2) Rs. 640 (3) Rs. 520 (4) Rs. 300 (5) None of these
1
Q.29. A,B and C working together can complete a piece of work in 7 days. If A is twice as fast as B, B is thrice as fast
2
as C then in how many days B & C can complete this work together.
3 3
(1) 17days (2) 18 days (3) 17 days (4) 18 days (5) None of these
4 4
Q.30. Ram can do a piece of work in 6 days while Shyam can do the same piece of work in 8 days. Ram and Shyam work
together for 2 days and then Ram leaves the work while Shyam continues to work till the work gets completed. On
which day from the beginning, will the work be completed?
(1) 6th (2) 5th (3) 7th (4) 4th (5) None of these
Q.31. A group of certain number of men can complete a job in 5 days. On every alternate day starting from second day,
3 men are withdrawn from the job and on every alternate day starting from third day, 2 men are added to the job.
If the job is completed in 6 days, what is the number of persons who started the job?
(1) 10 (2) 12 (3) 15
(4) Cannot be determined (5) None of these
Q.32. Ashok can complete one-fourth of a piece of work in 6 days, Shiva can complete one-third of the same work in 8 days
and Rahul can complete one-fifth of the work in 4 days. Ashok, Rahul, and Shiva start working on the work together
for 5 days after which Ashok and Rahul stop working. How many days more would Shiva take to complete the work?
(1) 5 (2) 6 (3) 7 (4) 8 (5) None of these
Q.33. The ratio of efficiencies of Mohan and Shyam is 2:3. The ratio of the time taken by Shyam and Anil to complete
some work is 1: 3. If all three work together to finish a particular work, the work is completed in 24 days. How
much time will it take to finish the same work if Mohan and Anil work together?
(1) 30 (2) 24 (3) 48 (4) 36 (5) None of these

EXERCISE Explanation
Q.1.(3) 1 day’s work of the three persons Q.3.(3) Let A and B together take x hours to complete the
work. Then,
FG 1 + 1 + 1 IJ = 47
=
H 15 20 25 K 300 A alone takes (x+8) hrs and B alone takes x +
FG
9 IJ
So, all the three together will complete the work
300

hrs to complete the work
H
2 K
in 18 days
≈ 6
47 47 1 1 1
+
Q.2.(3) Formula
b x+8 g FG x + 9 IJ =
x
Son’s 1 day’s work H 2K
bS + Fgno.of days × father no.of days 1 2 1
=
(father's 1day work − both work together) ⇒ x + 8 + 2x + 9 = x
b g b g

F 1 1I 2
Son’s 1 day’s work = GH − JK = ⇒ x(4x+25) = (x+8)
3 5 15
⇒ 2x2 = 72 ⇒ x2 =36 ⇒ x = 6 hours
The son alone can do the work in 15 = 7 1 days
2 2
QUANTITATIVE APTITUDE 91
www.mahendras.org

9 1
Short trick, × 8 = 6 hour Q.7.(3) (A+B)’s 1 day’s work = ,
2 10
1 1
Q.4.(3) (A+B)’s 1 day’s = C’s 1 day’s work = .
72 50
1 (A+B+C)’s 1 day’s work
(B + C)’s 1 day work =
120
1
FG 1 + 1 IJ = 6 = 3
(A + C)’s 1 day’s work = 90
=
H 10 50 K 50 25
Adding, we get : 2 (A+ B +C)’s 1 day’s work Also, A’s 1 day’s work = (B +C)’s 1 days work.


F 1 1 1 I 12 1
= GH 72 + 120 + 90 JK = 360 = 30
Then, we get : 2 × (A’s 1 day’s work) =
3
25
1 ⇒ A’s 1 day’s work = 3 .
⇒ (A + B +C)’s 1 day’s work = 50
60 1 3FG 2 1 IJ
FG
1

1 IJ
1 ∴B’s 1 day’s work= −
10 50
=
H =
50 25 K
So, A’s 1 day’s work =
H
60 120
=
K
120
So, B alone could do the work in 25 days.
∴ A alone can do the work in 120 days.
Q.8.(1) Suppose B takes x days to do the work.
Q.5.(1)
1
(A + B)’s 1 day’s work = ,
∴ A takes 2 ×
FG
3 IJ
3x
5
1

4Hx =
K2
days to do it

(B + C)’s 1 day’s work = 7 , 1


(A + B)’s 1 day’s work =
18
1 1 2 1
(A+C)’s 1 day’s work = . ∴ + = or x = 30
4 x 3 x 18
Adding we get : 2 (A +B +C)’s 1 day’s work So, B can do the same work in 30 days.
F 1 1 1I 83
= G + + J= FG 1 + 1 IJ = 7

H 5 7 4 K 140 Q.9.(3) (B + C)’s 1 day’s work =
H 9 12 K 36
83
(A +B + C)’s 1 day’s work = Work done by B and C in 3 days
280
FG83 1IJ 43 FG 7 × 3IJ = 7
A’s 1 day’s work =
H
280 7

K=
280
, =
H 36 K 12
FG
83 1 IJ 13 Remaining work = 1 −
FG 7
=
IJ
5
B’s 1 day’s work =
H
280 4
− =
K 280
, H 12 12K
1
C’s 1 day’s work = G
F 83 − 1IJ = 27 Now,
24
work is done by A in 1 day.
H 280 5 K 280 So,
5 FG
work is done by A in 24 ×
IJ
5
=10 days.
Thus time taken by A,B, C is
280
days,
12 H 12K
280 280
43
Q.10.(2) Work done by X in 4 days= G
F 1 × 4IJ = 1

13
days and
27
days respectively. H 20 K 5
Clearly, the time taken by A is least.
Remaining work = 1 −
FG 1IJ
=
4

Q.6.(3)
1
A’s 1 hour’s work = ’(B+C)’s 1 hour’s work
H 5 K 5
4
(X+Y)’s 1 day work=
FG 1 + 1 IJ = 8 = 2

1 1
= ’ (A+C)’s 1 hour’s work = . H 20 12 K 60 15
3 2 2
So, work is done by X and Y in 1 day.
(A+B+C)’s 1 hour’s work =
FG 1 + 1IJ = 7 . 15
4
H 4 3 K 12 So, work is done by X and Y in
5
F 7 − 1IJ = 1
B’s 1 hour’s work = G
FG
15 4
×
IJ

H 12 2 K 12
H2 5 K
= 6 days
∴ B alone will take 12 hours to do the work. Hence, total time taken = (6 +4) days = 10 days.
92 QUANTITATIVE APTITUDE
www.mahendras.org
Q.11.(4) A:B=3:4 (eff) Q.17.(3) Days remaining after 50 day's work
A : D=5 :4 (eff) = 150–50=100 days

A:B:D=15:20:12 (eff) 1
50 men has completed work in 50 days.
4
32
Required answer= 22.5 × = 48 days 1 3
15 Remaining work = 1– =
Q.12.(3) 91= n/2(2+(n-1)) 4 4
which is equal to triple of work already completed.
on solving n=13
Men Days Work
Q.13.(3) Efficiency ratio of Lakhan: Ram: Krishna 8:18:27
200 50 1
Number of days Ratio of Lakhan: Ram: Krish -
27:12:8 x 100 3
Sum of one day work of Lakhan,Ram and Krish 50 3
∴ Men required = 200 × × = 300 men
1 1 1 1 100 1
+ + = ∴ Additional men required = 300–200 = 100 men
27k 12k 8k 27
Q.18.(2) After 27 days, the remaining food is sufficient for
Number of days of Krish = 538×8538×8 = 53 days 200 soliders for 31-27=4 days.
Q.14.(4) 5 m or 9 w ⇒ 19 days ⇒ 5 m=9m =19 days Remaining soliders = 200–120=80 soliders
3m and 6 w ⇒ ? ⇒ 3m + 6w ⇒ ? Soliders Days
Formula - m1d1 = m2 d2
200 4
5m × 19 = (3m + 6w)×d2

80 x
5

5m × 19 = (3m+ ×6)×d2 Less soliders, provision will last for more days,
9
FG
27m + 30m IJ i.e. inverse proportion

95m = H 9 K
×d2
∴ The food will be sufficient for 4 ×
200

95m × 9 = 57m × d2 80
=10 days ⇒ Extra days = 10–4 = 6 days
95 × 9 Q.19.(2) The number of persons and number of days are

= d2 ⇒ 15 days
57 same in both the cases.
2 1 1
Q.15.(3) One day work of A = × = . 3
5 6 15 ⇒ 1 woman's work = man's work ⇒ 8
1 1 1 4
One day work of B = × = 3
3 10 30 women's work = 8 × =6 men's work
One day work of A and B 4
∴ 10 men + 8 women = 10 men + 6 men= 16 men
1 1 3 1
= + = = 12 men can do the work in 36 days.
15 30 30 10
12
4 4 ⇒16 men can do the work in 36× = 27 days
∴ Days taken to do th work= × 10 =8 days 16
5 5 Q.20.(1) (12×36) men's work= (18×60) women's work
Q.16.(1) Let men employed originally = x. Then
30x=20(x+5) ⇒ 10x=100 ⇒ x = 10 men 18 × 60 5
⇒ 1 man = = women
Alternative Method : 12 × 36 2
5
Ratio of days = 30 : (30–10) = 3 : 2 8 men + 20 women = 8 × + 20 women
2
⇒ Ratio of workers=Inverse ratio of 3 : 2 = 2 : 3 = 40 women

Difference = (3–2) ratio = 1 ratio Total work days = 18×60 woman days

= 5 men ⇒ Men (originally) = 2 ratio = 1080 woman days


Work already done =40×20 = 800 woman days
= 2×5 = 10 men.

QUANTITATIVE APTITUDE 93
www.mahendras.org
Remaining work = 1080-800 = 280 woman days M1D1H1 M2D2H2
Q.23.(3) =
Required number of women = 280÷4= 70 women W1 W2
Alternative method :
8 ×10 × 9 x×8×6
(12×36) men's = (18×60) women's =
18 × 2 × 2 18 × 2 ×12
⇒ 2 men = 5 women x = 30 men
8men + 20 women Q.24.(1) Let the total time taken be x days.
2 According to the given condition
= 8 men + 20× men = 16 men
5 x −8 x − 12 x
12 men can do the work in 36 days. ⇒ + + =1
36 54 72
∴ 16 men work for 20 days
6 ( x − 8) + 4 ( x − 12) + 3x
66 × 20 20 ⇒ =1
= 12 × 36 = 27 216

20 7 6x − 48 + 4x − 48 + 3x 13x − 96
⇒ =1⇒ =1
Remaining work = 1 – 27 = 27 216 216
7
Woman-days required to finish th of the work Þ13x - 96 = 216 Þ 13x = 216 + 96 = 312
7 27
= ×18×60=280 312
27 x= = 24
Women required to finish work in 4 days 13

= 280 ÷ 4 = 70 women. Q.25.(1) 2 men + 7 boys in 14 days


3 1 ⇒ 28 men + 98 boys in 1 day
Q.21.(2) A's work for 3 days = = .
6 2 3 men + 8 boys in 11 days
1 3
B's work for 3 days = 3 × = ⇒ 33 men + 88 boy s in 1 day
8 8
1 3 1 28 men + 98 boys - 33 men + 88 boys
C's work 1 – – =
2 8 8 2 boys = 1 man
1
C's share = × Rs.3200=Rs. 400 Now. 2 men + 7 boys = 11 boys; 8 men + 6 boys
8
= 22 boys
Alternative Method :
3 More boys, fewer days, more work, more days
A's wages = × Rs.3200= Rs.1600. B's wages Boys
6
3 Boys Days Work
= × Rs.3200 = Rs.1200 11 14 1
8
C's wages = 3200–(1600+1200)= Rs. 400
22 x 3
Q.22.(1) 6 men - 7 women; x 11 3
∴ = ×
2 women = 3 boys; 14 12 1

4 boys = 5 girls; ∴ Number of days = 21 days


Now earning of 1 girls per week = Rs. 16. Q.26.(3) More, the alone time of Kaveri is related to the
5 alone times of to other two persons, so assume (be
Earning of 1 boy = × 16 = Rs. 20
4 alone time of kaveri) =-x.
x x
3 Then, alone time of Kanti= and of Kalpana=
Earning of 1 women = × 20 = Rs. 30 2 3
2 Kaveri’s 1 day work + Kanti’s 1 day work +
7 Kalpana's 1 day
Earning of 1 man = × 30 = Rs. 35.
6 work = combined 1 days work

94 QUANTITATIVE APTITUDE
www.mahendras.org
1 1 1 1 Req. Answer = 2 + 4 = 6th day
⇒ + + = ⇒x=6
x x/2 x/3 1 Q.31.(3) Let the number of men who worked on day-1 be
‘n’.
∴ Alone time for Kaveri = 6 days, for
The number of people working each day starting
Kanti = 6/2 = 3 days,
from day-1 will be,
Kalpana = 6/3 = 2 days.
n, n-3, n-1, n-4, n-2, n-5.
Q.27.(1) Number of required days
This work is equal to the work done by ‘n’ men in
1 44 × 1 × 2 × 3 5 days.
= =
1 1 1 6+3+ 2
+ + So, 6n-15 = 5n => n = 15
44 × 1 44 × 2 44 × 3
Q.32.(4) Let the total amount of work be 120 units.
= 24 days
Ashok completed 1/4rd of the work i.e. 30 units
4 1 of work in 6 days. Thus, the amount of work
Q.28.(1) First friend’s 4 day’s work = (Since, the
12 3 Ashok can complete in 1 day = 30/6 = 5 units
work is finished in 4 days, when expert assists) The amount of work Shiva completed in 8 days
4 1 = 120/3 = 40 units
Second friends s 4 day ’s work =
16 4 Thus, the amount of work done by Shiva in 1 day
 1 1 5 = 40/8 = 5 units
The expert’s 4 day’s work= 1 −  +  =
3 4 12 Similarly, the amount of work done by Rahul in
1 day = 24/4 = 6 units
Now. total wages of Rs.960 is to be distributed
among two friends and the expert in proportion The work done by all three of them in 5 days
to the amount of work done by each of them = (5 + 5 + 6) x 5 = 80 units
So, 960 is to be divided in the proportion of Thus, amount of work remaining= 120 - 80
1 1 5
: : or 4 : 3 : 5 = 40 units
3 4 12
Thus, the number of days taken by Shiva to
5 complete the remaining work = 40/5 = 8 days
Share of expert = × 960 = Rs. 400
12 Q.33.(3) The ratio of efficiency of Mohan and Shyam is
2:3. So let Mohan does 2 units of work in a day
Hence, the expert should get Rs. 400
while Shyam does 3 units of work in a day.
Q.29.(2) 6 + 3 + 1 = 10
The ratio of time taken by Shyam and Anil is 1:3,
Total work = 7 × 10 = 75 so the ratio of efficiency of Shyam and Anil is 3:1
(B & C) one day work = 3 + 1 = 4
=> Anil does 1 unit of work in a day
75 3
Total time = = 18 => So when all 3 are working together, the total
4 4 work done in one day = 3 + 2 + 1 = 6 units
Q.30.(1) R 6 4
24 Total work which they do= 6×24= 144 units
S 8 3
Work done = 2(4+3) = 14 Now if only Mohan and Anil are working, then
they will finish 2+1 = 3 units of work in one day.
Remaining work = 10
So, Total time required for them to finish the
10 work = 144/3 = 48 days.
Remaining time = = 3.33
3

QUANTITATIVE APTITUDE 95
www.mahendras.org

CHAPTER

12 PIPE & CISTERN

Pipe : Pipes are connected to a tank or cistern and are used + 3 l/m A +20m
to fill or empty the tank, there are of two types. 60 l
+ 2 l/m B +30m
Inlet : A pipe connected with a tank or cistern that fills it is
known as inlet, means nature of pipe is positive. + 5 l/m A+B
Outlet : A pipe connected with a tank or cistern emptying it 60
Time taken by ( A + B) =
= 12 min
is known as outlet, means nature of pipe is negative. 5
Important Points Method iii: by formula-
Both Pipe A and B together fill the tank in
(1) Chapter Pipe & cistern is more similar to chapter Time
& Work. 20 × 30 600
= = = 12 min.
(2) If an Inlet can completely fill the empty tank in x hours, (20 + 30) 50
the part of the tank filled in 1 hour = 1/x. Ex. If pipe 'A' can fill the tank in 8 hr. and pipe 'B' can
empty the tank in 16 hr. When both pipes are opened
(3) If an outlet can empty the full tank in y hours, the part simultaneously, how much time will be taken to fill
of the tank empty in 1 hour = 1/y. the tank ?
(4) If both inlet and outlet are open, net part of the tank Sol. Method i:
1 1
filled in 1 hour = − . Here A = 8 hour, B = 16 hour
x y
Part of cistern fill by A & B in 1 hour
(5) If a pipe A alone can fill the tank in x hours and pipe
B can fill or empty the tank in y hours. If both pipe 1 1 2 −1 1
are working simultaneously then tank to fill or empty = − = =
8 16 16 16
 xy  Both pipe fill the tank = 16 hours
by  hours.
 x ± y 
Method iI:
For filling pipe, we take ‘+’ sign.
+ 2 l/h A = +8 h
16 l
For empty or drain pipe we take ‘-’ sign. B = -16h
- 1 l/h
Ex. Two pipes A & B can fill a tank in 20 min. and + 1 l/h A+B
30 min. respectively. If both the pipes are opened
simultaneously. How much time will be taken to fill Time taken when A & B both are opened,
the tank ?
16
A+B= = 16 hour.
Sol. Method i: 1
Here A = 20 min, B = 30 min Method iii: by formula-

1 1 16 × 8
∴ Part of the tank filled by A & B in 1 min. = + Both pipe fill the tank = = 16 hours
16 − 8
20 30
5 1 (6) Three pipes A,B,C can fill the tank in x,y and z hours
= = respectively.If all three pipes opened simultaneously
60 12
the time taken to fill the cistern is given by -
Both Pipes A and B together fill the tank in 12 min.
xyz
Method ii: xy + yz + zx

96 QUANTITATIVE APTITUDE
www.mahendras.org
Ex. Three taps A,B,C, can fill the tank in 4,6 and 12 hour Method iI:
respectively. How long would these three taps take to A +30m
+ 6 l/m
fill the tank if all of them are opened together? 180 l
+ 3 l/m B +60m
Sol. Method i: + 4 l/m +45m
Here A = 4 hours, B = 6 hours and C = 12 hours -5 l/m C
All together fill tank in 1 hour 180
C can empty the tank= min. = 36 min
1 1 1 5
= + +
4 6 12
Method iII: by formula-
3 + 2+1 1
= = part, C can empty the tank
12 2
30 × 60 × 45
All fill the tank in 2 hours. =
30 × 45 + 60 × 45 − 30 × 60
Method iI:
+4h 81000
+ 3 l/h A = = 36 min.
12 l 1350 + 2700 − 1800
+ 2 l/h B +6h
+ 1 l/h C +12h (8) One inlet pipe A is k times faster than the other inlet
pipe B.
+6 l/h A+ B+C
(a) If B can fill a cistern in x hrs., then the time in
12 which the cistern will be full, if both the inlet
All fill the tank = hours = 2 hours
6
pipes are opened together, is hrs.
Method iII: by formula-
All together fill the tank (b) If A can fill a cistern in y hrs., then the time in
which the cistern will be full, if both the inlet
4 × 6 × 12 4 × 6 × 12
= =  k 
4 × 6 + 6 × 12 + 12 × 4 24 + 72 + 48 pipes are opened together, is  y hrs.
 k + 1
4 × 6 × 12
= = 2 hours (9) One fill pipe A is k times faster and takes x minute
144 less time than the other inlet pipe B, then
(7) Two pipes A & B can fill the tank in X, Y respectively.
There is also an outlet pipe C. If all three pipe opened (a) The time taken to fill a cistern, if both the pipes
simultaneously tank will full in Z hours the time taken  kx 
are opened together is  2 mins.
by C to empty the full tank is given by -  (k − 1) 
XYZ
 x 
XZ + YZ − XY (b) A will fill the cistern in  min.
k − 1
Ex. Two taps A & B, can fill the tank in 30 min and 60
 kx 
min respectively. There is a third exhaust pipe C at (c) B will fill the cistern in  min.
the bottom of the tank. If all taps are opened together  k − 1
then tank will be full in 45 minutes. In what time Ex. One inlet pipe A is 9 times faster than second inlet pipe B.
can exhaust tap C empty the cistern when tank is If B can fill a cistern in 40 mins, then find the time when the
completely full? cistern will be full if both inlet pipes are opened together.
Sol. Method i: Sol. Here k = 9 and x = 40
Here A = 30 min, B = 60 min and A+B-C = 45 min.  x 
∴ Cistern will be full in = 
C can empty the tank in one minute
 k + 1 mins
1 1 1 6+3−4 1  40 
=  mins or 4 mins.
= + −
30 60 45
= = part.  9 + 1
180 36
Ex. One inlet pipe A is 3 times faster than second inlet
All fill the tank in 36 minutes. pipe B. If A can fill a cistern in 16 minute, then find
QUANTITATIVE APTITUDE 97
www.mahendras.org
the time when the cistern will be full if both inlet pipes Method iI: by formula-
are opened together.
Both pipes fill the tank
Sol. Here k = 3 and x = 16
 x 
= 
∴ Cistern will be full in =  x  y minute  k + 1
 k + 1
Here both the pipes fill the tank = 36 mins
 3 
=  16 = 12 minute.
 3 + 1 k = 3, so slower pipe x fill the tank
Ex. One inlet pipe A is 3 times faster than second inlet pipe
B. If together can fill the tank in 36 min, then in how = 36 × (3 + 1) = 144 mins
much time slower pipe can fill the tank. Ex. One inlet pipe A is 5 times as fast as second inlet pipe
Sol. Method i: B and takes 32 min less than the inlet pipe B. When
will the cistern be full if both inlet pipes are opened
Let time taken by faster pipe be x min then slower
together?
pipe take = 3x
1 1 1 Sol. Here k = 5 and x = 32.
∴ + =
x 3x 36  kx 
∴ Cistern will be full in =  2 min.
x=
36 × 4
= 48 minute  (k − 1) 
3
The time taken by slower pipe to fill the tank 5 × 32 20
= min. = min.
= 3×48 = 144 min. (5 − 1)
2
3

Notes

98 QUANTITATIVE APTITUDE
www.mahendras.org
EXERCISE
Q.1. Pipe A can fill a tank in 3 hrs while another pipe B can empty the same tank in 6 hrs. If both the pipes are opened
simultaneously. In what time will the tank be completely full?
(1) 4 hrs (2) 3 hrs (3) 2 hrs (4) 1.5 hrs (5) 6 hrs
Q.2. A vessel can be completely filled by a pipe in 16 hrs but due to a leak in the vessel, the vessel is now filled in 24
hrs. If the vessel is completely full, in what time the leak will empty the vessel completely.
(1) 48 hrs (2) 36 hrs (3) 44 hrs (4) 42 hrs (5) None of these
Q.3. Two pipes A and B can separately fill in a cistern15 and 10 minutes respectively and a waste pipe can discharge 7
litres per minute. If all the pipes are opened when the cistern is full, it in emptied in 2 min. How many litres does
the cistern hold?
(1) 21 ltr (2) 30 ltr (3) 40 ltr (4) 35 ltr (5) 45 ltr
Q.4. Two fill taps A and B can separately fill a tank in 45 and 40 min respectively. They started to fill tank together but
fill tap A is turned off after few minutes and fill tap B fill the rest part in 23 minutes. After how many minutes was
tap A turned off?
(1) 7 min (2) 8 min (3) 11 min (4) 15 min (5) 9 min
Q.5. Two pipes A and B can fill a tank in 15 hours and 20 hours respectively A third pipe C can empty the full tank in
25 hours. All the three pipes are opened in the beginning. After 10 hours, C is closed. Find in how much time will
the tank be full.
(1) 12 hrs (2) 8 hrs (3) 10 hrs (4) 14 hrs (5) 16 hrs
Q.6. Two taps A and B can fill a cistern in 15 min and 20 min respectively. Then both are turned on at the same time. If
the tap A is turned off after some time, after that cistern is filled in 12 min. After how much time tap A is closed?

3 3 2 1 3
(1) 4 min (2) 3 7 min (3) 5 min (4) 8 min (5) 7 4 min
5 5 9
Q.7. A, B and C can fill a tank in 6, 8, 12 hours respectively. If the pipes are opened in order at 10 am, 11 am, 12 am.
When will the cistern be filled? (approximately)
(1) 1 P.M. (2) 1.30 P.M. (3) 1.50 P.M. (4) 2 .00 P.M. (5) 2.20 P.M.
Q.8. The inlet to the tank can fill it in 4 hours while the outlet can empty it in 5 hr. Both the pipes were opened at 9 am
but after some time the outlet closed and it is observed that the tank was full at 5 pm. At what time was the outlet
closed?
(1) 12 P.M. (2) 2 P.M. (3) 3 P.M. (4) 8 P.M. (5) 6 P.M.

Q.9. Two pipes A and B can fill a tank in 30 min and 36 min respectively. Both pipes are open in the empty tank but due
5 9
to dust material present in the pipes A flows of original quantity and pipe B flows parts of original quantity.
6 10
31
After some time pipe is cleaned, then tank is filled in the min so after how much time pipes are ready to fill the
2
tank properly?
(1) 1 min (2) 2 min (3) 1.5 min (4) 5 min (5) 5.5 min
Q.10. A tank is filled by three pipes with uniform flow. The first two pipes operating simultaneously fill the tank in the
same time during which the tank is filled by the third pipe alone the second pipe fills the tank 5 hours faster than
the first pipe and 4 hours slower than the third pipe. The time required by the first pipe is:
(1) 11 h (2) 15 h (3) 4 h (4) 12 h (5) 18 h
Q.11. A tank is filled in 5 hours by three pipes P,Q & R. The pipe R is twice as fast as Q and Q is twice as fast as P. How
much time will pipe P alone take to fill the tank?
(1) 35 hr (2) 55 hr (3) 20 hr (4) 18 hr (5) 15 hr
Q.12. Three pipes P,Q and R can fill a tank in 6 hours. After working at it together for 2 hrs R is closed & P and Q can fill
the remaining part in 7 hours. The number of hours taken by R alone to fill the tank is-

QUANTITATIVE APTITUDE 99
www.mahendras.org
(1) 10 (2) 12 (3) 13 (4) 14 (5) 16
Q.13. 7 pipes attached with a tank out of which some are inlets and outlets. Every inlet can fill tank in 10 hours and every
30
outlet can empty tank in 15 hours when all the pipes are opened simultaneously the tank filled up hours. Find
the no. of inlets & outlets- 11

(1) 5,2 (2) 2,5 (3) 3,4 (4) 4,3 (5) None of these
Q.14. Two pipes A and B can fill a tank in 20 minutes and 40 minutes. If both pipes are opened simultaneously, after how
much time should B be closed  so that the tank is full in 18 minutes?
(1) 10 min (2) 9 min (3) 8 min (4) 7 min (5) 4 min
Q.15. Three pipes P, Q and R connected to a cistern. The first pipe (i.e) P can fill 1/2 part of the tank in one hour, second
pipe, Q can fill 1/3 part of the cistern in one hour. R is connected to empty the cistern. After opening all the three
pipes in a cistern which is 7/12 part filled. Then how much time required to empty the cistern completely?
(1) 2 hours (2) 3 hours (3) 4 hours (4) 5 hours (5) None of the Above
Q.16. In a tank there is a pipe which can be used for filling the tank as well as for emptying it. The capacity of the tank
is 1200 m³. The emptying of the tank is 10 m³ per minute higher than its filling capacity and the pump needs 6
minutes lesser to empty the tank than it needs to fill it. What is the filling rate of the pipe?
(1) 20 m³ / min. (2) 40 m³ / min. (3) 50 m³ / min. (4) 60 m³ / min. (5) None of the Above
Q.17. In what time would a cistern be filled by three pipes whose diameters are 1cm, 2 cm and 3 cm running together,
when the largest pipe alone can fill the tank in 21 minutes. The amount of water flowing through the pipe is directly
proportional to the square of its diameter?
(1)10.5 minutes (2) 11.5 minutes (3) 12.5 minutes (4) 13.5 minutes (5) None of these
Q.18. A pipe can empty a tank in 60 minutes alone. Another pipe whose diameter is twice the diam-
eter of first pipe is also opened. Now find the time in which both pipe will empty the tank together.
(1) 8 min (2) 10 min (3) 12 min (4) 14 min (5) None of These
Q.19. A fill pipe can fill the tank in 15 minutes and a drain pipe can drain the tank in 30 minutes. If a system of ‘x’ pipes
(includes both fill and drain pipes) fills the tank in 3 minutes. Find the possible value of ‘x’.
(1) 10 (2) 16 (3) 12 (4) 14 (5) None of these
Q.20. An inlet pipe can fill a tank in 8 hours. An outlet pipe can empty the same tank in 12 hours. Initially the tank is
completely empty. At this point of time, the inlet pipe is connected to the tank. After 3 hours, the outlet pipe is also
connected to the tank. After how much time from the beginning, will the tank be full?
(1) 15 hours (2) 18 hours (3) 20 hours (4) 24 houre (5) 22 hours

EXERCISE Explanation
1 1 part/hr ← A+B
Q.1.(5) Part filled by pipe A in 1 hr. part
3 6
Time taken to fill the tank = 1 = 6 hr
1
Part empty by pipe B in 1 hr part
6 1
Q.2.(1) Part filled by inlet pipe A =
Part empty by pipe (A+B) in 1hr 16
1 1 2 −1 1 1
= − = = Part filled by inlet pipe A+B = 24
3 6 6 6
Time taken by both pipes to fill the tank = 6 hr. where B is leakage.
AlternatIVE Method: so accordingly
2 part/ hr 3 hr A 1 1 1
+ =
6 A B A +B
3 part/ hr 6 hr B

100 QUANTITATIVE APTITUDE


www.mahendras.org
1 1 1 x x + 23
+ = ∴ + =1
16 B 24 45 40

1 1 1 1 −2 1 8x + 9x + 207
= − , = =– =1
B 24 16 B 96 48 360
Time taken by B to empty the tank= 48 hrs 17x = 153, x=9 min
AlternatIVE Method : AlternatIVE method :
6 part/ hr 16 hr (A) part/min 8 45 min (A)
96 360
4 part/ hr 24 hr (A+B) part/min 9 40 min (B)

-2part/hr B A+B B
– 23min–
96
Time taken by B to empty the tank= = 48 hr B worked for 23 mins
2
Q.3.(1) According to question Work done by B in 23 mins = 23 × 9= 207parts
1 1 1 1 Remaining part = 153 parts
+ − =
A B C A +B+C 153
A+B = = 9 min
1 1 1 1 17
+ + =− Pipe A was turned off after 9 min
15 10 C 2
 1 1 1  23
1 1  1 1 Q.5.(1) Tank fill in 10 hrs =  + − =
+ = − − +  15 20 25  30
C 2  15 10 
 23  7
1 15 − 2 − 3 Remaining part =  1 − 30  = 30
+ =−  
C 30
work done by (A+B) in 1 hr
1 −10
+ =  1 1  7
C 30  15 + 20  = 60
 
1 1
=− 7
C 3 Now part filled by (A+B) in 1 hr.
60
Full tank is empty in 3 min
In 1 min ....... 7 lit is empty 7
∴ part is will be filled by (A+B) in
30
In 3 min ..... 7 × 3 = 21 lit
AlternatIVE Method :  60 7 
=  ×  hrs =2 hours
2 part/min 15 min (A)  7 30 
3 part/min 30 10 min (B) ∴ Total time = 10+ 2 = 12 hours
15 part/min 2 min (A+B+C)
AlternatIVE method :
10 part/min C
20 part/hr 15 hrs (A)
30 15 part/hr 300 20 hrs (B)
C will empty full tank in = = 3 min 12 part/hr 25 hrs(C)
10
In 1 min ......... 7 lit is empty C
In 3 min .... 7 × 3 = 21 lit. A+B+C A+B
– 10 hr –
Q.4.(5) Let the tap A be turned off after x minutes
∴Tap B is opened for (x+23) minutes 10×(20+15-12)
Work done by A+B+C in 10 hr = 10 × 23

QUANTITATIVE APTITUDE 101


www.mahendras.org
= 230 parts Water filled by pipes A & B in next 1 hour
Remaining part = 300 - 230 = 70  1 1
 6 + 8  liter
70  
Remainig part filled by A + B in = = 2 hr 4+3 7
35 = =
liter
Total time = 10 + 2 = 12 hrs 24 24
1 and water filled by pipes A,B & C in 1 hours
Q.6.(2) Water filled by tap A in 1 min =
15 1 1 1 4+3+2
= + + =
6 8 12 24
1
Water filled by tap B in 1 min = 9
20 = l
24
Water filled by tap A and tapB in 1 min
Now remaining water to be filled by A,B & C
1 1 4+3 7 together
= + = =
15 20 60 60
Hence
According to question, 1 7 
1−  + 
Let after x min pipe A is closed Remaining time =  6 24 
9
7x 1
So, + 12 × =1 24
60 20
4 + 7 11
1−   1−
7x 3  24  = 24 = 13 × 24 13
+ =1 = = hours
60 5 9 9 24 9 9
24 24
7x 3 2
= 1− = 4
60 5 5 = 1 hours
9
2 60 cistern will be filled
x= ×
5 7 that means at 12 + 1.5 = 1 : 30 pm
AlternatIVE method: Q.8.(2) +5l/h A + 4h
A 20
4 l/m 15 min - 4l/h B - 5h
60l
3 l /m B 20 min Given total time = 9 am to 5 pm = 8h

Let after x min tap A is closed then according to Since both pipes are opened at 9 am
question Let of tap 't' time B pipe closed then

x min A B 12 min t+(8-t) × 5 = 20


A+B 60l t+ 40 - 5t = 20

4t = 20
7x + 3 × 12 = 60
t = 5h
7x = 60 - 36 = 24
Hence
24 3
x= = 3 min Required answer = 9 am + 5 hours = 2 pm
7 7
Q.7.(2) Since A,B & C are opened in order at 10 am, 11 Q.9.(1) 6l/m A 30 min
am & 12 am 180
5l/m B 36 min
Hence, According to question
1 5
Water filled by pipes A in 1 hour = liter efficiency of A = 6 × =5 l/m
6 6
Again,

102 QUANTITATIVE APTITUDE


www.mahendras.org
9 2
effienciency of B = 5× = 4.5 l/m (P+Q)'s 1 hour work =
10 21
efficiency of A + B = 9.5 l/m Therefore, R's 1 hour work
again = (P+Q+R)'s 1 hour work
Let after x min dust particle is cleaned 1 2 = 1
(P+Q)'s 1 hour work = −
So, 6 21 14
31 therefore R alone will fill the tank = 14 hours
9.5x + 11 × = 180
2 Q.13.(1) +3 A (Inlet type) +10 min
-2 B (Outlet type) -15 30
9.5 + 170.5 = 180
9.5x = 180 - 170.5 30
But tank fill in 11 hrs
9.5x = 9.5
30
x = 1 min So, work done in one hour = = 11
30
Q.10.(2) According to question 11
Let the first pipe alone takes x hours to fill the With these total pipes
tank then second pipe takes Total inlets = (5) = 5 × 3 = 15 w/h
= x - 5 third pipe takes = x - 9 Total outlets = (2) = 2 × (2) = -4 w/h
1 1 1 11w/h
So, + =
x x−5 x−9 Q.14.(5)   A fill the tank in 1 minute (20×2 = 40)
x−5+ x 1 = 2 units
=
x ( x − 5) x − 9 B fill the tank in 1 minute (40×1 = 40)

(2x - 5) (x-9) = x (x-5) =  1units

x2 - 18x + 45 = 0 For 18 min(A) = 18×2 = 36 units

(x-15) (x-3) = 0 Remaining = 40 – 36 = 4 units

x = 15 hours. Time for B to be closed  so that the tank is full in

Q.11.(1) Suppose pipe p alone takes x hour to fill the tank 4


18 minutes = = 4 min
x x 1
Then pipes Q & R will take & hours Q.15.(3) In 1 hour, P can fill = 1/2 Part
respectively 2 4
Time taken to fill the Cistern by Pipe P = 2
1 2 4 1 hours
So, + + = In 1 hour, Q can fill = 1/3 Part
x x x 5
Time taken to fill the Cistern by Pipe Q = 3
7 1 hours
=
x 5 [1/2 + 1/3 – 1/R] = 7/12
1/R = 1/4
x = 35 hours Time required to empty the Cistern = 4 hours
2 1 Q.16.(2) 1200/x – 1200/(x+10) = 6
Q.12.(4) Part of tank filled in 2 hr = =
6 3 200/x – 200/(x+10) = 6
x2 + 10x – 2000 = 0
1 2
Remaining capacity = 1 − = x = 40
3 3
Q.17.(4) More the diameter more will be the water flowing
2 through it and less will be the time taken.
therefore, (P+Q)'s work =
3 Means bigger pipe will take less time to fill the
tank

QUANTITATIVE APTITUDE 103


www.mahendras.org
21× 32 21× 9 So the correct option to choose is 4.
t= =
1 +2 +3
2 2 2
14 Q.20.(2) Let the volume of the tank be 48. The inlet pipe

fills 6 litres in 1 hour
So total time = 13.5 minutes
while the outlet pipe empties 4 litres in 1 hour.
Q.18.(3) Time taken by pipe to empty the tank is inversely
proportional to cross-sectional area. For the first three hours only inlet pipe is
connected, so it will fill
So, time taken by second pipe to empty
= 6×3 = 18 litres in those 3 hours.
= 60/4 = 15 min
Now after this both the pipes are operating
Both pipe can empty the tank
simultaneously.
1 1 1
= + = = 12 min So, effectively only 2 litres will be filled in 1
60 75 12
hour. We have 30 litres to fill. Hence time
Q.19.(4) In a given time, one fill pipe cancels out two drain required = 30/2
pipes, and we require 5 more fill pipes to fill the
= 15 hours.
tank in 3 minutes. So the number of pipes would
be in the form of 3x+5. Total time from the beginning to fill the tank
Among the given options only ‘14’ is in the form = 3 + 15 = 18 hours
of 3x+5.

Notes

104 QUANTITATIVE APTITUDE


www.mahendras.org

CHAPTER
Speed, Time AND
13
Distance
Speed 1
S∝ → at constant distance
The distance covered in a unit time interval is known as speed. t
It is obtained by dividing the distance covered by an object
Ex. In covering a certain distance the speed of A and B are
by the time it takes to cover that distance.
in the ratio of 3 : 4. A takes 30 minutes more than B
Dis tance Travelled to reach the destination. The time taken by A to reach
Speed (S) =
Time Taken the destination is-
Distance (d) = Speed ×Time Sol. Let the distance of destination be D km
Dis tance Let the speed of A= 3x km/hr.
Time (t) =
Speed The speed of B = 4x km/hr.
Unit conversion : According to the question
1. km/hr to m/sec conversion: D D 1
− = 30 =
3x 4x 2
A km/hr = A ×
FG 5 IJ
H 18 K
m/sec. D 1 D
= , =6
2. m/sec to km/hr conversion: 12x 2 x
D 6
F 18 I
A m/sec = G A × J
Time taken by A = = = 2 hours.

H 5K km/hr. 3x 3

Ex. A plane is moving with the speed of 180 km/hr. Its short Trick :
speed in metre per second is- Speed ratio = 3 : 4
Time ratio = 4 : 3
5
Sol. 180 × = 50 m/s 1 = 30 min
18
4 = 120 min =2 hours
Important Points :
Ex. If a person walks at 14 km/hr instead of 10 km/hr, he
1. If the time taken is constant, then the distance travelled would have walked 20 km more. The actual distance
is directly proportional to the speed i.e. more speed, travelled by the person is:
more distance can be travelled in the same time.
Sol. Let the actual distance travelled be x km.
S ∝ d → at constant time x x + 20
Then, = ⇒ 14x = 10x + 200
10 14
2. If the speed is constant, the distance travelled is
directly proportional to the time taken, i.e. more the ⇒ 4x = 200 ⇒ x= 50km.
distance travelled, more the time taken at the same short Trick :
speed.
20
(14 - 10) =
d ∝ t → at constant speed T
T = 5 hrs.
3. If the distance travelled is constant, the speed is
inversly proportional to the time taken, that is more Distance = 10 × 5 = 50 km
speed, less the time taken for the same distance to beEx. Excluding stoppages, the speed of a bus is 54 kmph
travelled. and including stoppages, it is 45 kmph. For how many
minutes does the bus stop per hour?
QUANTITATIVE APTITUDE 105
www.mahendras.org
Sol. Due to stoppages, it covers 9 km less.
Relative Speed :
Time taken to cover 9 km I. If two objects are travelling in the same direction at s1
FG 9 × 60IJ min = 10 min km/hr. and s2 km/hr respectively such that s1 >s2 then
= H 54 K s1-s2 is called the relative speed.

Average Speed  A


s 1 km/hr
→  B
s 2 km/hr

The average speed of an object is a measure of the distance
covered by that object in a set period of time. Relative Speed = s1-s2 (s1 >s2)
Total Distance A
Average Speed = ←
s km/hr 
Total Time Taken 1

(a) If A covers a distance d1 km at S1 km/hr and then
B
d2 km at S2 km/hr. then the average speed during ←
s km/hr 
whole journey is given by
2

S1S2 ( d1 + d2 ) Relative Speed = s1-s2 (s1 >s2)


Average Speed =
S1d2 + S2 d1 II. If two objects are travelling in the opposite direction
at s1 km/hr and s2 km/hr, respectively, then s1 +s2 is
(b) A person goes certain distance (A to B) at speed
called their relative speed.
of S1 and return (B to A) at speed of S2 km/hr. If
he takes T hrs in all, then
 A
s 1 km/hr
→ ← B
s 2 km/hr

2S1 × S2
Average speed =
S1 + S2 Relative Speed = s1+s2
Distance between A and B is
A
←
s km/hr   B

 S1 × S2  1 s2 km/hr

 t × S + S 
1 2
Relative Speed = s1+s2
(c) If a person travelled three equal distance by three Ex. A thief is noticed by a policeman from a distance of
different speed S1, S2 and S3 then 200 m. The thief starts running and the policeman
3S1 × S2 × S3 chases him. The thief and the policeman run at the
Average speed = rate of 10 km/hr and 11 km/hr. respectively. What is
S1S2 + S2S3 + S3S1
the distance between them after 6 minutes.
Ex. A man goes from place A to B at a speed of 12 km/
hr. and return at the speed of 18 km/hr. The average Sol. P 11 km/hr. T
10km/hr.
speed for the whole journey is-
200m
2 × 12 × 18 2 × 12 × 18
Sol. Average Speed = Relative speed of police= 11-10 = 1 km/hr.
(12 + 18) = 30
5
2 = m/sec
= 14 km/hr. 18

5 ∴ Distance decreased in 6 minutes
Ex. One-third of a certain journey is covered at the rate of
5
25 km/hr. one fourth at the rate of 30 km/hr. and the = × 6 × 60 = 100 m
rest at 50 km/hr. The average speed for whole journey. 18

Sol. Let total journey = x km ∴ Distance remained between them

So, according to the question = 200-100 = 100 m

x x 5x 3x Important Points
Total time = + + =
3 × 25 4 × 30 12 × 50 100
a
I. If the new speed is of the original speed, then the
x 1 b
Average Speed = = 33 km/hr.
3x 3 change in time taken to cover the same distance is
100 given by
106 QUANTITATIVE APTITUDE
www.mahendras.org
Sol. Let Distance = x
Change in time =  -1 ×original time
b

a  x x 12
So, − =
4 5 60
II. If a man travels at the speed of x km/hr. reach t1 hr.
late and if he travels at the speed of y km/hr. reach t2 x 1
=
hr. early the travel distance. 20 5

xy x= 4 km
y−x
( t1 + t 2 ) km short Trick :

III. If two persons A and B start at the same time from 4 × 5 12
Distance = × = 4km
two points P and Q towards each other and after 1 60
crossing they take T1 and T2 hrs in reaching Q and P
respectively, then Ex. Two places A and B are placed at a distance of 150
km on a highway. A car moves from A and another car
A's Speed T2
= moves from B at the same time. If both car travelled
B' s Speed T1
in same direction with the different speed then they
meet after 5 hours. If they travelled in opposite
3
Ex. If a man travels th of his original speed then he directions then they meet in 1 hour speed of the fast
4 car is-
reaches 20 minutes late. Find his actual time :
Sol.
Sol. Let the actual speed be x km/hr and the actual time be
t minutes When they are travelled in same direction (x>y)

1 1 20 then (x-y) × 5 = 150


− =
3x x 60 ⇒ x - y = 30 ......... (i)
4 When they are travelling in opposite directions
4x 1 1 (x + y) = 150 ....... (ii)
− =
3x x 3 Adding eq. (i) and (ii)
x = 1 km/hr.
2x = 180
1 x = 180/2
Actual time = = 1 hr. = 60 min.
1
x = 90 km/hr.
short Trick : Ex. Ram arrives at a Bank 15 minutes earlier than shedules
time if drives his car at 42 km./hr. He arrives 5 minutes
Actual time = 3 late if he drives his car at 35 km./hr. The distance of
× 20
( 4 − 3) the Bank, from his starting point is:
= 60 min. Sol. Difference in time = (15+5) minutes = 1/3 hour

Ex. Ritu goes at the speed of 4 km/hr. reaches her school ∴ Required distance
6 minutes late and next day she goes at the speed of 42 × 35 1
5 km/hr. reaches her school 6 minutes earlier find the
= × = 70 km.
7 3
distance between her home and school ?

QUANTITATIVE APTITUDE 107


www.mahendras.org
EXERCISE
Q.1. A bus moves 270 km at a speed of 45 km per hour and there after increases its speed upto 60 km/h. to travel another,
480 km. Find the average speed of the bus.
4 4 3 2 8
(1) 53 km/hr. (2) 55 km/hr (3) 58 km/hr. (4) 51 km/hr. (5) 47 9 km/hr.
7 7 7 8
Q.2. Walking 5/7 of his usual rate a boy reaches his school 6 min late. Find his usual time to reach school.
(1) 15 minutes (2) 12 minutes (3) 18 minutes (4) 20 minutes (5) 22 minutes
3
Q.3. Two friends x and y walk from A to B at a distance of 78 km at 1.5 km per hr and 1 per hour respectively. y
4
reaches B, returns immediately and meet x at C. Find the distance from A to C.
(1) 75 km (2) 90 km (3) 72 km (4) 80 km (5) None of these
Q.4. A person leaves a place A to place B at 6 am and reaches place B at 10 am. Another person leaves B at 8 am and
reaches A at 11.30 am. They will meet each other at.
(1) 8:56 am (2) 7:45 am (3) 8:20 am (4) 8:40 am (5) None of these
Q.5. It takes 4 hours for a 300 km journey of 60 km is covered by train and the rest by car. it takes 20 min more if 100
km is covered by train and the rest by car, the ratio of the speed of train to that of the car is-
(1) 4 : 3 (2) 3 : 4 (3) 2 : 3 (4) 3 : 2 (5) 5 : 3
Q.6. Ram and Shyam starts simultaneously from point A towards B with speed of 7.5 m/sec and 33 km/hr respec-
tively. When one of them reaches B, the person has to return back towards A immediately. If he meet the other
person after 2 hours of total journey. Find the distance between point A and B.
(1) 120 km. (2) 100 km. (3) 60 km.
(4) 50 km. (5) Cannot be determined
Q.7. Two trains P and Q start from Delhi and Mumbai towards Mumbai and Delhi respectively. After passing each
other they take 19 hours 30 minutes and 8 hours 40 minutes to reach Mumbai and Delhi respectively. If the train
from Delhi is moving at 80 km/hr then find the speed of the train from Mumbai?
(1) 110 kmph (2) 100 kmph (3) 90 kmph (4) 120 kmph (5) None of these
Q.8. A bus crosses a certain distance with out stoppage with the speed of 60 km/hr. The bus covers the same distance
with the stoppage speed of 40 km/hr. What is the time of stoppage of the bus per hour (in minutes)
(1) 20 (2) 15 (3) 12 (4) 40 (5) 18
Q.9. A monkey climbs 20 m. in a minute and slips down 6 m in next minute How much time he will reach upto a 60 m
high pole in this way.
(1) 6 min 59 sec (2) 6 min 54 sec. (3) 6 min 58 sec (4) 6 min 50 sec (5) 6 min 45 sec.
Q.10. A bus started from A to B then after covering two third distance some fault occurs. Driver mentioned the fault quickly
and bus will move again. But now the speed of the bus is reduced by one third and for this reason bus reaches at B
late by 1 hour what was the actual time in going B from A.
(1) 6.30 hr (2) 7 hr (3) 6 hr (4) 6.45 hr (5) 7.30 hr
Q.11. A bus from A to B started with the speed of 60 km/hr. at 6 am another bus from A to B started with the speed of 80
km/hr at 8 am. At which time will second bus catch the first bus ?
(1) 2 p.m. (2) 1 p.m. (3) 12 a.m. (4) 3 p.m. (5) 2.30 p.m.
Q.12. A man travels with the speed of 40 km/hr. then he reaches the particular place 10 min late but if he travels with the
speed of 50 km/hr. then he reaches 5 minutes before the stimulated time. Then how much time will he take to reach
the place.
(1) 1 hr. 10 min. (2) 1 hr. 15 min. (3) 1 hr. 12 min. (4) 1 hr. 18 min. (5) 1 hr. 5 min.

108 QUANTITATIVE APTITUDE


www.mahendras.org
Q.13. A bus started its journey from Ramgarh and reached Devgarh in 88 minutes with its average speed of 100 km/hr.
If the average speed of the bus is increased by 10 km/hr. How much time will it take to cover the same distance?
(1) 60 minutes (2) 70 minutes (3) 75 minutes (4) 80 minutes (5) 72 minutes
Q.14. A faster train takes 2 hrs less than a slower train in a journey of 192 km. If the average speed of the slower train is
16 km/hr less than that of the faster train then the average speed of the faster train is-
(1) 36 km/hr. (2) 64 km/hr. (3) 36 km/hr. (4) 48 km/hr. (5) None of these
Q.15. Two guns were fired from the same place at an interval of 12 minutes. A person approaching the firing place in his
car, hears the second shot 11 minutes after the first. If the speed of sound is 330 m/s, find the speed of the car.
(1) 50 km./h. (2) 72 km./h. (3) 90 km./h. (4) 108 km./h. (5) None of these
Q.16. A man covered a certain distance at some speed. Had he moved 3 km per hour faster, he would have taken 40 minutes
less. If he moved 2 km per hour slower, he would have taken 40 minutes more. The distance (in km) is:

(1) 20 (2) 35 (3) 36 (4) 40 (5) None of these


Q.17. A thief running at 8 km/hr is chased by a policeman whose speed is 10 km.hr. If the thief is 100 m ahead of the
policeman, then the time requiared for the policeman to catch the thief will be-
(1) 2 min (2) 3 min (3) 4 min (4) 6 min (5) None of these
Q.18. A person started his journey in the morning. At 11 a.m., he covered 3/8 of the journey and on the same day at 4.30
p.m., he covered 5/6 of the journey. He started his journey at:
(1) 6 a.m. (2) 5.30 a.m. (3) 7 a.m. (4) 6.30 a.m. (5) None of these
Q.19. Two guns were fired from the same place at an interval of 10 minutes and 30 seconds but a person in the train ap-
proaching the place hears the second shot 10 min. after the first. Find the speed of the train supposing that sound
travels at 330 m/s.
(1) 111.8 kmph (2) 59.4 kmph (3) 58.6 kmph (4) 19.8 kmph (5) None of these
Q.20. In a race of 600 m. A can beat B by 50 m. and in a race of 500 m, B can beat C by 60 m. By how many metres
will A beat C in a race of 400 m ?
(1) 16 m (2) 50 m (3) 77 m (4) 84 m (5) None of these
Q.21. Two cars 'M' and 'N' are running in the same direction, car 'M' had already covered a distance of 60 Kms. when Car
'N' started running. The cars meet each other in 3 hours after Car 'N' started running. What was the speed of Car 'M'?
(1) 40 kmph (2) 45 kmph (3) 60 kmph (4) Data inadequate (5) None of these
Q.22. Sandeep, Deepak and Naimish drive with a speed of 30km/hr, 40 km/ hr and 60 km/hr respectively. If they all start
from a same point but not together, Deepak start two hours after Sandeep and cross Sandeep at the same instance
as Naimish does. After how many hours of Sandeep Naimish start?
(1) 6 (2) 4 (3) 8 (4) 3 (5) 2
Q.23. P, Q and R starts from the same place and travel in the same direction at speeds 30 km/hr, 40 km/hr and 60 km/hr
respectively. Q starts 2 hours after P, but Q and R overtakes P at the same instant. How many hours after P did R start?
(1) 1 (2) 2 (3) 4 (4) 6 (5) 8
Q.24. A car and a truck start travelling towards each other from city A and city B respectively. After crossing each other
car takes 6 hours 40 minutes to reach at B and truck takes 3 hours and 45 minutes to reach city A. If speed of truck
is 36 Km/hr. then find the distance between city A and B.
(1) 225 Km (2) 320 Km (3) 350 Km (4) 315 Km (5) None of these
Q.25. After travelling 150 km car met with an accident and its speed reduced by 25% hence reached 45 min late to its
destination. Had this accident occoured 60 km after, it would have reached its destination only 30 minutes late.
Find the length of total journey.
(1) 300 Km (2) 270 Km (3) 330 Km (4) 420 Km (5) None of these
QUANTITATIVE APTITUDE 109
www.mahendras.org
EXERCISE Explanation
Q.1.(1) Here distance speed are known. Time is not Speed = 7 : 8, Distance = 7×4=28
known we use relation. Distance Travelled by first person till 8 A.M. = 14
270 14
⇒ = 6 hr. hr = 56 minute
45 Required time =
7+8
480 So required answer = 8 : 56 am
⇒ = 8 hr.
60 Q.5.(1) Let the speed of the train be x km/hr and car be
y km/hr.
Total time = 14 hr.
60 240
750 Then + = 4 hr.......(I)
Total distance ⇒ x y
14

= 53
4
km/hr. and 100 + 200 = 25 ...... (II)
7 x y 6
Q.2.(1) Speed 5 : 7
60y + 240x 6 (100y + 200x )
Time 7 : 5 ∴ =
4 25
Speed and time are proportional to each another.
= 250y + 1000x = 400y + 800x
Difference in time 2 = 6
20x–16x=8y–5y
1 = 3 minutes
4x = 3y
3 × 5 = 15 minutes
x:y=3:4
Q.3.(3)
Q.6.(3) Let the distance between A and B = D
Total distance covered by them = 2D
18
Relative Speed = 7.5 × + 33 = 60
5
When y meets x at C. y has walked the distance Time = 2 hrs.
AB + BC and x has walked the distance AC. 2D = 60 × 2
So, both x and y have walked together a distance D = 60
= 2 × AB = 2×78 = 156 km t1
Q.7.(4) Speed of train Q = Speed of train P ×
The ratio of speed of x and y is 1.5 : 1.75 t2

So that 39 / 2
Speed of train Q = 80 ×
150 : 175 = 6 : 7 26 / 3
Hence the distance travelled by Speed of train Q = 80 × 3/2 = 120 kmph
6 Q.8.(1) x = first speed without stoppage
x = AC = × 156
6+7 y = speed with stoppage
6 x−y
× 156 = 72 km. stoppage time =
13 x
60 − 40
Q.4.(1) Ist 6 am.................. 10 am = 40
8 am ................. 11.30 am
20 1
= ⇒ × 60 = 20 minute
60 3
Q.9.(2) Height covered in first minute= 20 metre
Decrease in height in next minute

= 6 metre
Time = 4 : 3.5 = 8 : 7
110 QUANTITATIVE APTITUDE
www.mahendras.org
So, monkey climbs up (20 - 6)m in 2 minutes Dis tance
Required time =
14×3 = 42 m. Speed
440 60
Monkey climbs up 42 m in 6 min = × = 80 minutes
3 110
2×3 =6 min Q.14.(4) Let the speed of the faster train be x km/hr.
Remaining height = 60 - 42 = 18 ∴ Speed of the slower train
20 m. climb up in 60 sec. = (x - 16) km/hr.
60 192 192
18 m climb up in = × 18 = 54 second. ⇒ = −2
20 x x − 16
6 min + 54 sec = 6 min 54 sec x = 48 km/hr.
1 Q.15.(4) The person hears the second fire in 11 minutes
Q.10.(3) Let Speed= 1- = 2/3, Speed = 2 : 3
3 instead of 12 minutes since the car is moving
towards the firing place.
Time = 3 : 2
∴ The distance travelled by car in 11 minutes is equal
⇒ 1 = 1 × 2 = 2 hr.
to distance covered by sound in 12–11=1 minutes.
2 1
Distance 1- ⇒ = 2 hr. ∴ Distance covered by car in 1 second
3 3
So, total time is = 6 hr. 330 × 60
= =30 m.
11 × 60
Q.11.(1) 18
∴ Distance convered by car in 1 hour = 30 ×
= 108 km./hr. 5
After 2 hrs.
Q.16.(4) Let total distance be D km and the normal speed
be x km/h.

Time difference = 8 - 6 = 2 hrs D D 2 D D 2


Then – = and – =
x x+3 3 x–2 x 3
Distance moved by first bus before the second
bus starts] ⇒
D

D
=
D D 1
– ⇒ –
1
=
1

1
x x+3 x–2 x x x+3 x–2 x
= 60×2 = 120 km
Relative speed= 80 - 60 = 20 km/hr. ( x + 3) – x x – (x – 2) 3 2
⇒ = ⇒ =
x ( x + 3) x ( x – 2) x ( x + 3 ) x ( x – 2)
120
Time = 20 = 6 hr. ⇒ 3(x–2)=2(x+3) ⇒3x–6=2x+6
Time taken by second bus to catch first train- 8 ⇒x=12 (km/hr.)
a.m. + 6 = 2 p.m.
Now, D – D = 2 ⇒ D = 2 ⇒ D = 2 × 60 = 40 km.
Q.12.(5) Let he takes x minutes to reach at exact time 12 15 3 60 3 3
Q.17.(2) Policeman gains = (10–8) km/hr. = 2 km/
50 (x - 5) = 40 (x + 10) hr. ⇒ He will gain 100 metre in 3 minutes.
50x - 250 = 40x + 400 Q.18.(4) Journey covered between 11 a.m. and 4.30 p.m.
10 x = 650 5 3 11
= – =
6 8 24
x = 65 1
Time taken = 4.30 p.m.–11 a.m. = 5 hours
Exact time is 1 hr. 5 minutes. 2
Q.13.(4) Distance between Ramgarh and Devgarh 11
= hours.
2
100 × 88 440
= = km
60 3 11 11
⇒ of journey is covered in = hours.
New speed = 110 km/hr. 24 2

QUANTITATIVE APTITUDE 111


www.mahendras.org
3 Let T be the time taken by Naimish to cross
∴ of journey is covered in Sandeep with Deepak 
8
40×6 = 60×T
11 24 3 9 1 T = 4 hours
× × = hours = 4 hours
2 11 8 2 2 So Naimish start after 2 hours of Deepak and 4
1 hours (2+2) of Sandeep
Journey was started at 11 a.m.– 4 hours Q.23.(3) Let the time taken by P when he was overtaken
2
= 6.30 a.m. by Q and R be T hours.
Q.19.(2) Speed of train Time taken by Q
Distance Travel by sound = T – 2 hours
= Time taken by train to travel same distance Since, distance travelled is same by the time Q
overtakes P
330 x 30
= 30(T)= 40(T-2)
10 x 60
T = 8 hours
= 33 m/s
2 Distance travelled by P = 30×8
= 33 x 18 km / h = 240 km 
2 5
R takes 4 hours to cover a distance of 240 km.
= 59.4 km / h So, he starts 8 - 4 = 4 hours after P.

Q.20.(5) In race of 600m. Q.24.(4) SC t
= B
A travels 600m. SB tC
B travels 550 m.
SC 225 3
in race of 500m. = =
SB 400 4
B travels 500 m.
C travels 440 m. now 4=36   so   3=27

∴ in race of 400 m. required distance

440 20 15
C travels = x 400 = 352m. = 27 × + 36 × = 315 Km
500 3 4

Q.25.(3) 3
A travels = 400m. S 4S

150
A beat C by = 400 - 352 = 48m. 60
Speed = 4 : 3, Time = 3 : 4
Q.21.(4) Data inadequate
Difference 1 = 45 So, 3 = 135 m.
We only find the ratio of the speed of both cars
but we can't find the actual speed. Again Difference 1 = 30 So, 3 = 90 m.
Q.22.(2) Let t be the time taken by Deepak to cross Sandeep In 45 minute car travel 60 km
30×(t+2)=40×t So Speed = 80 km
t = 6 hours Required Answer =150+ 2.25×80 = 330 km

112 QUANTITATIVE APTITUDE


www.mahendras.org

CHAPTER
PRoblemS
14 on Trains
This type of questions are totaly based on the concept of exception are there when any vertical object is taken
time & distance & speed. Mainly problems based on trains then distance of that object is considered as zero.
have two objects. Ex. Length of pole, person and tree is zero.
1. Object I: Those object who cross another object is III. Concept of Time : Time is always constant (The time
considered as object I (primarly trains are considered which is taken by object I to cross object II)
as Ist object but some times person also considered
as 1st object when person crosses another object like Some important points
stationary train or platform) 1. When a moving train crosses a standing person i.e.
Ex. train, bus, person etc.
L1
2. Object II : Those object who are crossed by object I S1 =
T
train, platform, over bridge, tunnel, person, pole tree,
wall etc. Ex. Find the time taken by a train 150m long, running at
54 km/h in crossing an electrical pole.
General Rule
L
Concept of speed : Sol. S = (because pole has no breadth)
T
I. When one object is stationary and another is still then L 150m 150
only speed of moving object is considered as S1=x (let) T= = 54 × 5 = = 10 sec
S 15
(S2 =0) m/s
18
(1) When both object are moving in same direction
2. When a moving train crosses a moving person in the
In this case relative speed is difference between same direction
speed of both objects that means relative speed
= S1 ~S2 L1
S1 − S2 = and L2 = 0
T
A B

s km/hr → 
s km/hr → Ex. A train 180m long is running at a speed of 65 km/h.
1 2

In what time will it pass a man who is running at a
speed of 5 km/h in the same direction ?
Relative Speed = s1-s2 (s1 >s2)
L1
A B Sol. S1 - S2 =
←
s km/hr  ←
s km/hr  T
1 2

L1 180
(2) When both object are moving in opposite T= = m/s= 10.8 sec
S1 − S2 5
direction. 60 ×
18
In this case relative speed is sum of both speed 3. When a moving train crosses a moving persons in the
of both object means relative speed = S1 + S2 opposite direction

← B
 L1
 A
→ s 2 km/hr S1 + S2 =
s 1 km/hr
T

Relative Speed = s1+s2 Ex. A train 150metres long is moving at a speed of 85


km/hr. It will cross a man coming from the opposite
A
←  B
→ direction at a speed of 5 km./hr. in:
s km/hr 
1 s2 km/hr
Sol. According to question
II. Concept of distance: Distance is always added of L1
both object with irrespective of direction but in some S1 + S2 =
T

QUANTITATIVE APTITUDE 113


www.mahendras.org
5 150 L1 + L 2

(85 + 5) × =

S1 + S2 =
18 T T
T = 6 sec. Ex. Two trains are running in opposite directions with the
4. When a moving train crosses a platform: same speed. If the length of each train is 120 metres
and they cross each other in 12 seconds, the speed of
L1 + L 2 , S = 0 each train is :
S1 = 2
T Sol. Let the speed of each train be x m/sec.
Ex. A train 140 m long is running at 60 km/h. In how
much time will it pass a platform 260m long? 120 + 120
Then, = 12
x+x
Sol. S = L1 + L 2 , t = L1 + L 2
t s 20  20 18 
∴x= m/sec. =  ×  km/hr.
2  2 5
140 + 260
= = 36 km/hr.
5
60 × 8. When a moving train crosses a person sitting in another
18
train:
= 24 sec Ans.
L1
5. When a moving train crosses another stationary train: S1 ± S2 =
T
L1 + L 2 L2 = 0
S1 =
T S1+S2 (according to direction)
S2 = 0 Ex. A goods train 150 m. long moving with speed 54 km/h.
A man is sitting in a passanger train which is moving
Ex. A train 180 m long taken 20 sec. in crossing a
with 18 km/hr. in same direction. In how many seconds
stationary train 220 m long. The speed of the train
the goods train crosses the man sitting in a passanger
is - (in km./h.)
train ?
L1 + L 2 180 + 220 400
Sol. S = = = Sol. Let the length of the goods train is L.
t 20 20
L
18 S1 - S2 = (because man has no breadth)
t
= 20 × 5 = 72 km./h.
5 150m
6. When a moving train crosses another moving train (54 - 18) =
18 t
along same direction:
L1 + L 2 5 150m
S1 − S2 = ×36 =
T 18 t

Ex. Two trains travelling in the same direction at 54 km/ t = 15 sec
hr. and 36 km/hr. completely pass one another in 1 9. When a moving train crosses two different object:
minute. If the length of the first train is 125 metres,
Ex. A train running at a speed of 30 km/h takes 18 sec to
the length of the second train is:
pass a railway platform and it takes 15 sec to pass a
Sol. According to question man who is running at a speed of 6 km/h in the same
L1 + L 2 direction. Find the length of platform ?
S1 − S2 =
T Sol. Let the length of the train is L.
and the length of the platform is P.
5 125 + L 2
(54 − 36) × = L +P
18 60 S1 =
t
5×60 = 125+L2
5 L +P
L2= 175 metres. × 30 =
18 18
7. When a moving train crosses another moving train
coming from opposite direction: L + P = 150 ---- (I)

114 QUANTITATIVE APTITUDE


www.mahendras.org
Again x' s rate
Sol. = Time taken by Y to reach A
S1 - S2 = L y' s rate Time taken by X to reach B
T
5 L 40 FG 10 × 5 IJ = 5

18
× ( 30 - 6) = 15 ⇒
Y's rate
=
H 3 24 K 6
L = 100 meters

F 40 × 6 IJ km/hr. = 48 km/hr.
∴Y’s rate = G
Length of train L = 100 m H 5 K
Length of platform = 150 - 100
Ex. A train starts from x and travel towards y at 10:00 am.
= 50 m. Another train B start from y and travel towards x at
Ex. A train travelling at 36 km/hr. completely crosses 11:00 am if they reach their destinations at 2:00 pm
another train, having half its length and travelling in and 5:00 pm. At what time will they meet?
the opposite direction at 54 km/hr. in 12 seconds. If it Sol. Time taken by A = 4 hour, Time taken by B = 6 hours.
1 Let the distance = 4×6 = 24 km
also passes a railway platform in 1 minutes, the
2 and speed of A and B are 6 km/h & 4 km/h
length of the platform is: 24 km
A B
Sol. Relative speed = (36+54)km/hr. = 90 km/hr.
6 4


FG
= 90 ×
5 IJ
m/sec. = 25 m/sec.
Remaining distance = 24 - 6 = 18 km
H 18 K 6+4 =
18
T
Let l metre be the length of slower train.
T = 1.8 hour
Fl+ l I

G J
Now, G 2 J =12 or l = 200 m


T = 1hour 48 min
12:48 pm
GH 25 JK Ex. A train overtakes two persons walking along a railway
Also, speed of slower train track. The first one walks at 4.5 km/hr. the other one
walks at 5.4 km/h. The train needs 8.4 and 8.5 seconds

FG
= 36 ×
5 IJ
m/sec. = 10 m/sec. respectively to overtake them. What is the speed of the
H 18 K train if the persons are walking in the same direction
as the train?
Now, if x metre be the length of platform, then
Sol. Length of train
x + 200

10
=90 or x = 700 metres. (S train − 4.5) × 5
× 8.4 ---- (1)
=
18
10. When two trains start from two points X & Y towards
Length of train
each other at the same time and after crossing they
5
take p & q second to reach Y & X respectively then = (S Train - 5.4) × 18 × 8.5 ---- (2)
ratio of
Speed = q: p (S − 4.5) × 5 × 8.4
18
Ex. Two trains x and y start from stations A and B
towards B and A respec­tively. After passing each (S − 5.4) × 5 × 8.4
other, they take 4 hours 48 minutes and 3 hours 20 =
18
minutes to reach B and A respec tively. If train x is
S = 81 km/h
moving at 40 km/hr., the speed of train y is : -

QUANTITATIVE APTITUDE 115


www.mahendras.org
EXERCISE
Q.1. A train running at 40 km/h crosses a pole in 36 second. Find the length of train.
(1) 300 m (2) 200 m (3) 400 m (4) 500 m (5) 450 m
Q.2. A train running at the speed of 20 m/s takes 10 second to pass a man who is running at the speed of 5 m/s in the
same direction. Find the length of the train.
(1) 150 m (2) 200 m (3) 250 m (4) 300 m (5) None of these
Q.3. A train 200 m long takes 5 second to pass a standing man. Find the time taken by train in crossing a railway platform
of 240 m long.
(1) 18 sec. (2) 13 sec. (3) 11 sec. (4) 26 sec. (5) None of these
Q.4. A man is standing on railway bridge which is 100 m long. He finds that train crossed the bridge in 5 second but
himself in 3 sec. Find the length of the train.
(1) 100 m (2) 150 m (3) 200 m (4) 250 m (5) None of these
Q.5. A train running at a speed of 25 km/h takes 18 sec to pass a railway platform and it takes 9 sec to pass a man who
is running at the speed of 5 km/h in the same direction. Find the length of the platform.
(1) 77 m (2) 70 m (3) 75 m (4) 71 m (5) None of these
Q.6. Two trains are moving in opposite directions at speeds of 80 km/h and 70 km/h and their length is 1.20 km and
0.80 km respectively. Find the time taken by the slower train to cross each other.
(1) 48 sec (2) 50 sec (3) 42 sec (4) 44 sec (5) None of these
Q.7. Two trains each 200 m long move towards each other on parallel track with velocity 22 km/h and 28 km/h respectively.
What is the time taken when they meet each other?
(1) 20 sec (2) 24.8 sec (3) 28.8 sec (4) 30 sec (5) None of these
Q.8. Two trains are running 40 km/h and 20 km/h respectively in the same direction. The faster train completely passes
a man sitting in the slower train in 18 sec. Find the length of faster train.
(1) 105 m (2) 100 m (3) 90 m (4) 110 m (5) None of these
Q.9. A train with a speed of 90 km/h crosses a bridge in 36 seconds. Another train 100 m shorter crosses the same bridge
at 45 km/h. What is the time taken by the second train to cross the bridge?
(1) 32 sec (2) 64 sec (3) 40 sec (4) 50 sec (5) None of these
Q.10. A train moves pass a telegraph pole and a bridge 264 m long in 8 sec and 20 sec respectively what is the speed of
the train?
(1) 15m/s (2) 20 m/s (3) 25 m/s (4) 18 m/s (5) 22 m/s
Q.11. If a train runs at 40 km/h, it reaches its destination late by 11 minutes but if it runs at 50 km/h, it is late by 5 minutes
only. Find the correct time for the train to complete its journey.
(1) 19 minutes (2) 20 minutes (3) 21 minutes (4) 18 minutes (5) None of these
Q.12. Two trains, Train A and Train B crosses each other completely in 18 sec while travelling in opposite directions,
speed of train A is 72 km/hr and speed of train B is 54 km/hr. Length of train A is 170 m less than the train B. From
the statement given in the above question which of the following can be determined?
A) Length of train A
B) Length of train B
C) Time taken by train B to cross a 130 m length of bridge
D) Time taken by train A to cross a 70 m long canal
(1) All A), B), C) and D) (2) Only B) and C) (3) Only D) and B)
(4) Only A (5) Only C
Q.13. Train A started from station P towards station Q. At the same time, train B started from station R towards station Q. All
the three stations are in a straight line such that station Q is between station P and station R. Station Q is equidistant

116 QUANTITATIVE APTITUDE


www.mahendras.org
from station P and station R. Distance between station P and station Q is 440 Km. If the speed of train A and train B is
_______Km/hrs and ______Km/hrs respectively, the distance between both the trains after five hours is 580 km.
Which of the following satisfies the two blanks given in the questions?
I. 28 km/hrs, 32 km/hrs II. 40 km/hrs, 20 km/hrs III. 25 km/hrs, 35 km/hrs
(1) None (2) Only II (3) Only III (4) Only I and II (5) All I, II and III
Q.14. A goods train and a passenger train are running on parallel tracks in the same direction. The driver of the goods
train observes that the passenger train coming from behind overtakes and crosses him completely in 80 seconds.
Whereas a passenger on the passenger train observes that the goods train crosses in 40 seconds. If the speeds of
the trains be in the ratio 1:3. Find the ratio of lengths of passenger train to goods train.
(1) 1:1 (2) 2:1 (3) 1 : 3 (4) 1:2 (5) 3∶1
Q.15. A train overtakes two person who are walking in the same direction in which the train is going, at the speed of 2
km/h and 4 km/h and passes them completely in 9 second and 10 sec respectively. Find the length of the train.
(1) 200 m (2) 50 m (3) 150 m (4) 250 m (5) None of these
Q.16. Two train X and Y start from station A and B towards B and A respectively after passing each other, they take 4
hours 48 minutes and 3 hours 20 min to reach B and A respectively if train X is moving at 40 km/h, Find the speed
of train Y.
(1) 60 km/h (2) 54 km/h (3) 64.8 km/h (4) 48 km/h (5) None of these
Q.17. A train travels a certain distance without stoppages with an average speed of 90 km/h. but with stoppages at an
average speed of 75 km/h. Find the number of minutes per hour that the train stops.
2
(1) 8 min (2) 10 min (3) 6 min (4) 15 min (5) 12 min
3
Q.18. Two trains are moving in the opposite directions on parallel tracks at the speeds of 63 km/hr and 94.50 km/hr
respectively. The first train passes a pole in 6 seconds whereas the second train passes a pole in 4 seconds.Find the
time taken by the trains to cross each other completely?
(1) 4.80 sec. (2) 4.40 sec. (3) 3.80 sec.
(4) Can't be determined (5) None of these
3
Q.19. A train after travelling 48Kms. meets with an accident and then proceeds at of its former speed and arrives at
4
its destination 92 minutes late. Had the accident occured 72 km, further, it would have reached the destination only
80 minutes late. Find the speed of the train ?
(1) 100 Km. ph (2) 120 Km. ph (3) 90 Km. ph (4) 150 Km. ph (5) None of these
Q.20. Two places A and B are 162 kms. apart. A train leaves A for B and at the same time another train leaves B for A.
The two trains meet after 6 hours. If the train travelling from A to B travels 8 kmph faster than the other, what is
the speed of the faster train ?
(1) 17.5 Kmph. (2) 18 kmph (3) 18.5 Kmph (4) Data inadequate (5) None of these
Q.21. Two trains at the same times from two stations and proceed towards each other at the rate of 30 km/h & 35 km/h respectively.
When they meet it is found that one train has travelled 60 km. more than the other . What is the distance between two station?
(1) 880 (2) 720 (3) 780
(4) Can't be determined (5) None of these
Q.22. Two trains (X and Y) start from 'A' and 'B' respectively, towards each other. After they meet at point 'C' the train
1
'X' takes two hours to reach B while the trains 'Y' takes 4 hours to reach A. If the distance between places A
2
and B is 450 kms. How far should 'C' be from 'A' ?
(1) 180 km. (2) 270 km. (3) 225 km.
(4) 360 km. (5) None of these
QUANTITATIVE APTITUDE 117
www.mahendras.org
EXERCISE Explanation
Q.1.(3) When train crosses the pole L1 + L2 = 125

L (because pole has no breadth) L1 18


S= 9= ×
T 25 − 5 5
40 × 5 L
= L1 = 50, L2 = 75m
18 36
L = 400 m Length of train = 50 m
Q.2.(1) When train crosses a man travelling in same Length of platform = 75 m
direction
Q.6.(1) When they cross each other
L
S1-S2 = T L1 + L 2
S1 + S2 =
L T
(20 -5) =
10 (80 + 70) × 5 = 1200 + 800
L 18 T
15 =
10
T = 48 sec
L = 150 m
Q.7.(3) When they are moving in opposite direction
Q.3.(3) According to question
L L1 + L 2
S= S1 + S2=
T T

S=
200
= 40 m/s (22 + 28) × 5 = 200 + 200
5 18 T
when train crosses the platform 50 × 5 400
=
L1 + L 2 18 T
S=
T
T = 28.8 sec
210 + 240
40 = L
T Q.8.(2) S1 - S2 =
T
440
40 =
T ( 40 − 20) × 5 = L
18 18
T = 11 second
L + 100 L L = 100 m
Q.4.(2) S= ........ (I) S=
5 3
According to question Q.9.(2) S = L+B
T
L + 100 L
= 90 × 5 L + B
5 3 =
18 36
3L + 300 = 5L
2L = 300 L+B = 900
L = 150m 45 × 5 L − 100 + B
Q.5.(3) When it crosses platform 100 = T
L1 + L 2 L1 45 × 5 900 − 100
S= , S −S =
T T M 100 T
25 × 5 L1 + L 2 45 × 5 800
= =
18 18 18 T
118 QUANTITATIVE APTITUDE
www.mahendras.org
T = 64 sec = 880 - 300 = 580 Km
L This is satisfies the given condition.
Q.10.(5) S = T when crosses a pole From III:
Distance between station P and station R
L = 440×2 = 880 Km
S= ------ (1)
8 Distance travelled by train A in 5 hours
L + 264 = 25×5 = 125 Km
S=
20 Distance travelled by train B in 5 hours
= 35×5 = 175 Km
L L + 264
⇒ 8= Required distance = 880 – (125 + 175)
20
= 880 - 300 = 580 Km
5 L = 2L + 528 This is satisfies the given condition.
3L = 528 Q.14.(2) Let the speed of passenger train be 3v and that
L = 176 m of goods train be v. as the ratio of their speeds be
176 1:3.
S= = 22 m/s Also, suppose that the lengths of passenger train
8
3x 6x be P and goods train be G.
Q.11.(1) − =6
2 5 Then,
15x − 12x P
=6 2v =
10 80
x = 20
From equation (I) G
3x Again 2v =
= t + 11 40
2
3 × 20 On division
= t + 11
2
P 80
t = 30 – 11=19 minutes =
G 40
Q.12.(1) LA +LB = (72+54)×5/18×18=630 P:G=2:1
LB - LA = 170
LB = 400 m. LA = 230m Q.15.(2) (S − 2) × 5 = L ..........(1)
All A, B , C and D 18 9
Q.13.(5) From I:
Distance between station P and station R
(S − 4 ) × 5 =L
..........(2)
= 440×2 = 880 km 18 10
Distance travelled by train A in 5 hours 9S − 18 = 10S − 40
= 28×5 = 140 km S = 22 km / hr
Distance travelled by train B in 5 hours
= 32×5 = 160 km 5 L
Required distance = 880 – (140 + 160) 20 × =
18 9
= 880 - 300 = 580 Km
This is satisfies the given condition.
L = 50m
From II:
Distance between station P and station R time1
= 440×2 = 880 km Q.16.(4) Speed of train Y = ×speed of train x
time2
Distance travelled by train A in 5 hours
= 40×5 = 200 km 240 + 48
Distance travelled by train B in 5 hours = × 40
180 + 20
= 20×5 = 100 km
Required distance = 880 – (200 + 100)
QUANTITATIVE APTITUDE 119
www.mahendras.org
2x = 27- 8
288 12
= × 40 = × 40 = 48 km/h
200 10 x = 19 = 9.5 km / hr.
2

90 − 75 15 × 60 Speed of faster train = 9.5 +8 =17.5 km/h


Q.17.(2) × 60 = = 10 min
90 90 Difference of the distance
Q.21.(3) Time=
Q.18.(1) Length of Ist Train = S x T Difference of Speed
= 63 x 5 x 6 = 105 metre 60
18 = 12 hrs
Length of IInd Train = 94.5x 5 x4 = 105 Metre 35 − 30
18
Time Taken to cross each other Train Reliance speed = 30+35 = 65 km/hr.
Total length
= Total distance = Reliance speed + time
Relative speed
105 + 105 = 65 x 12 = 780 km.
=
e +94.5jx18
63 5
Q.22. (2) According question
= 210
43.75 Train x ← d2 → ← 2 hours → Train y
= 4.80 second
A.....................................B
Q.19.(2) 120 km/hr.
← 4 1/2 hours → c ← d1 →
difference of 12 minute time comes when accident
occured after 1st accident point in 12 minute d
1=
T
1
train travel 24 km, hence speed of train d T
2 2
= 24 x 60
d
12 1 = 2 x2
d 9
= 120 km/hr. 2
d
Q.20.(5) 17.5 km/hr. 1=2
d 3
Let the speed of slower train is x km/hr. 2
then speed of faster train is x + 8 km/hr. Total Distance value of ( 2 + 3 )
Formula used = 450 km.
Relative Speed = Distance
Time 450
Distance A to C value of (3) = x3
5
162 = 270 km.
(x + x + 8) = 6

Notes

120 QUANTITATIVE APTITUDE


www.mahendras.org

CHAPTER

15 Boat & Stream


Concept of Boat and Stream Depends on Concept of Speed, Bs km/hr.
Time and Distance mainly it has 4 facts.
Boat
(1) Still water : If the speed of the water in the river or
pond is zero it is called still water. It is denoted as Bs.

Bs km/h Cs km/hr.
Boat
Bs-Cs = Us
Ex. If the speed of the boatman in still water is 10 km/hr
Still water and speed of the water (stream) is 4 km/hr then the

speed of the boatman in upstream.
(2) Stream: If the water of the river is flowing at a Sol. Us = Bs - Cs
constant speed it is called as stream. It is denoted as
Cs. Hence Us = 10-4 = 6 km/h
Important formulas:
(1) Speed of the boat in still water
Cs km/hr. down stream speed + upstream speed
= 2
(3) Down Stream Speed: If a boat or swimmer goes along
the stream is called down stream speed. Ds + Us
Bs = 2
Down stream speed = speed of the boat + speed of the
current Ex. A man can goes 32 km/hr upstream & 36 km/h
downstream. Find the speed of man in still water.
Ds = Bs + Cs
Sol. Speed of man in still water
Bs
Boat Ds + Us 32 + 36
(Bs) = =
Bs+Cs = Ds 2 2

Cs = 34 km/h

(2) Speed of the current
Ex. If the speed of the boatman in still water is 10 km/hr
and speed of the water (stream) is 4 km/hr then speed down stream speed - upstream speed
=
of the boatman in downstream. 2

Sol. Since Ds = Bs + Cs Ds − Us
Cs =
Hence Ds = 10 + 4 = 14 km/hr. 2
Ex. A man can go 36 km/hr upstream & 48 km/h
(4) Up stream speed: If a boat or a swimmer moves downstream. Find the speed of current.
against the stream it is called upstream speed.
Ds − Us
Upstream speed = speed of the boat - speed of the Sol. Speed of current Cs =
2
current
48 − 36
Us = Bs - Cs = 6 km/hr
2

QUANTITATIVE APTITUDE 121


www.mahendras.org
EXERCISE
Q.1. A boat can travel a distance 10 km in an hour against the stream whereas 20 km in an hour with the stream. Hence
find the speed of boat in still water.
(1) 15 km/h (2) 22 km/h (3) 20 km/h (4) 44 km/h (5) 25 km/h
Q.2. The distance between two points is 36 km., boat rows in still water at 6 kmph. It takes 8 hour less to cover this
distance in downstream in comparison to that in upstream. The rate of stream is:-
(1) 3 kmph (2) 2 kmph (3) 2.5 kmph (4) 4 kmph (5) None of these
Q.3. Ram goes downstream with a boat to some destination and returns upstream to his original places in 6 hours. If the
speed of the boat in still water and the stream are 12 km/hr and 5 km/hr respectively, then find the distance of the
destination from the starting position.
(1) 25km (2) 26.67km (3) 33km (4) 29.75km (5) 20km
Q.4. When a person is moving in the direction of the current is 20km/hr, speed of the current is 5km/hr. Then find the
speed of the person against the current?
(1) 10km/hr. (2) 15km/hr. (3) 30km/hr. (4) 25km/hr. (5) 11km/hr.
Q.5. There are two places A and B which are separated by a distance of 100km. Two boats starts form both the places
at the same time towards each other. If one boat is going downstream and the other one is going upstream, if the
speed of A and B is 12km/hr. and 13km/hr. respectively. Find at what time will they meet each other.
(1) 10 hrs. (2) 4 hrs. (3) 8 hrs. (4) 6 hrs. (5) 7 hrs.
Q.6. A girl was travelling in a boat, suddenly wind starts blowing and blows her hat and started floating back downstream.
The boat continued to travel upstream for 12 more minutes before she realized that her hat had fallen off. She
turned back downstream and she caught her hat as soon as she reached the starting point. If her hat flew off exactly
2km from where she started. What is the speed of the water?
(1) 12 km/hr (2) 8 km/hr (3) 5 km/hr (4) 9 km/hr (5) 10 km/hr
Q.7. The ratio of the speed of the motor boat to that of the current of water is 36:5 . The boat goes along with the current
in 5 hours 10 minutes. How much time it will take to come back .
(1) 45/2 hr (2) 41/6 hr (3) 55/3 hr (4) 38/7 hr (5) 52/8 hr
Q.8. There is a road beside a river. Two friends started from a place A, moved to a temple situated at another place B and
then returned to A again. One of them moves on a cycle at a speed of 6 km/hr, While the other sails on a boat at a
speed of 8 km/hr. If the river flows at the speed of 6 km/hr, which of the two friends will return to place A first?
(1) Cyclist (2) Sailor (3) Both come at same time
(4) Anyother (5) None
Q.9. The different between downstream speed and upstream speed is 2 kmph and the total time taken during upstream and
downstream is 2 hours. What is the upstream speed, if the downstream and upstream distance are 2 km each? (approx)
(1) 5.2 kmph (2) 3.7 kmph (3) 2.8 kmph (4) 1.4 kmph (5) 1.2 kmph
Q.10. A boat travels downstream from point A to B and comes back to point C half distance between A and B in 18 hours.
If speed of boat is still water is 7 km/hr and distance AB = 80 km, then find the downstream speed.
(1) 15 km/hr (2) 18 km/hr (3) 12 km/hr (4) 10 km/hr (5) 6 km/r
Q.11. A boat can row 18 km downstream and back in 8 hours. If the speed of boat is increased to twice its previous speed,
it can row same distance downstream and back in 3.2 hours. Find the speed of boat in still water.
(1) 9 km/hr (2) 5 km/hr (3) 4 km/hr (4) 8 km/hr (5) 6 km/hr
Q.12. A ship of length 300 m traveling from point A to B downstream passes a Ghat along the river in 18 sec, while in
return it passes the same Ghat in 24 sec. If the rate of current is 9 km/hr. Then what is the length of the Ghat?
(1) 50 m (2) 60 m (3) 80 m
(4) 100 m (5) Cannot be determined
Q.13. A boat takes 25 hours for travelling downstream from point A to point B and coming back to point C midway
between A and B. If the velocity of the stream is 5 km/hr and the speed of the boat in still water is 10 km/hr, what
is the distance between A and B?
122 QUANTITATIVE APTITUDE
www.mahendras.org
(1) 100 km (2) 122 km (3) 146 km (4) 178 km (5) 150 km
Q.14. A boat running at the certain speed downstream covers a distance of 4.8 kms. in 8 minutes. The same boat running
upstream covers the same distance in 9 minutes. What is the speed of the current?
(1) 2.4 kmph (2) 3 kmph (3) 2 kmph (4) 3.2 kmph (5) None of these
Q.15. The speed of a boat in still water is 6 kmph and that of current is 3 kmph. The boat starts from point A and rows to
point B and comes back to point A. It takes 12 hours during this journey. How far is point A from point B?
(1) 27 km (2) 25 km (3) 20 km (4) 30 km (5) None of these
Q.16. The speed of a boat in still water is 17.5 kmph and that of current is 2.5 kmph. The boat goes from X to Y in
downstream and returns to point Z. The whole journey takes 429 minutes. If the distance between Z and Y is 2/5
of the distance between X and Y. Find the total distance covered by the boat. (approximate to the nearest integer).
(1) 130 km (2) 140 km (3) 160 km (4) 120 km (5) 150 km
Q.17. The speed of the boat in still water is 16 kmph. and speed of the current is 2 kmph. It takes a total time of 6.5 hours
to row upstream from point A to point B and downstream from point B to C. The distance from point A to B is
two-third the distance between point B and C, what is the total distance travelled by the boat (Both upstream and
downstream)?
(1) 112 km (2) 98 km (3) 124 km (4) 90 km (5) 105 km
Q.18. A boy takes 3 hours 45 min to cover a distance of 15 km along the stream and 2 hours 30 min to cover the distance
fo 5 km against the stream. Find the speed of the current in km/hr.
(1) 2 km/h. (2) 1 km/h. (3) 3 km/h. (4) 4 km/h (5) 1/2 km/h
Q.19. A boy swims 40 km upstream and 55 km downstream in 13 hours. and he can also swims 30 km upstream and 44
km downstream in 10 hours. Then-
1. Find the speed of man.
2. Find the speed of stream.
(1) 8 kmph, 3 kmph (2) 16 kmph, 6 kmph (3) 11 kmph, 5 kmph
(4) 10 kmph, 6 kmph (5) 8 kmph, 2 kmph
Q.20. A man takes double time to swim upstream a certain distance as compared to the time taken to cover the same
distance in downstream. The ratio of speed of boat in still water to that of the speed of current is:
(1) 2 : 1 (2) 3 : 1 (3) 3 : 2 (4) 4 : 3 (5) 1 : 2
Q.21. A boat can travel 4.2 km upstream in 14 min. If the ratio of the speed of the boat in still water to the speed of the
stream is 7:1. How much time will the boat take to cover 17.6 km downstream?
(1) 42 minute (2) 48 minute (3) 40 minute (4) 44 minute (5) None of these

EXERCISE Explanation
Q.1.(1) Speed of boat upstream 36 36
10km ∴ − =8
= = 10 km/h (6 − Cs) (6 + Cs)
1h
 6 + Cs − 6 + Cs 
Speed of baot downstream ⇒ 36  =8

20km  (6 − Cs)(6 + Cs) 
= = 20 km/h
1h 36 × 2Cs
=8
Speed of boat in still water (6 + Cs)(6 + Cs)

20 + 10
= = 15 km/h Cs2 + 9Cs - 36 = 0, Cs = 3 kmph.
2
2xD 119× 6
Q.2.(1) Speed of current = Cs kmph. Q.3.(4) T = ⇒D= = 29.75km
x2 - y2 2×12
downstream speed =(6+Cs) kmph.
upstream speed = (6-Cs) kmph. Q.4.(1) Speed of the person = 20 – 5 = 15km/hr Speed of
the person against the current = 15 – 5 = 10km/hr.
According to question.
Q.5.(2) Downstream = (12+x)km/hr
T1 - T2 = 8 hours.
QUANTITATIVE APTITUDE 123
www.mahendras.org
Upstream = (13-x)km/hr D D/2
Distance So, + = 25 , D = 150 km
Time = 15 5
Relative Speed Q.14.(3) Let the speed of boat in still water = Bs kmph.
∴ The speed of current = Cs kmph
Relative speed = 12 + x + 13 – x
= 25 km/hr Hence, Ds = Bs + Cs
100 4.8 4.8 × 60
Time = = 4 hours = ⇒ = 36 kmph.
25 8 8
60
Q.6.(3) Distance = 2 km and
Time = 2 × 12 = 24 mins 4.8 4.8 × 60
Us = Bs - Cs = = = 32 kmph
2 9 9
Req. Answer = × 7 = 5 km / hr
24 60
Now, for speed of current
S1 T1 36 + 5 31
Q.7.(2) = -5 = x Ds − Us 36 − 32
S2 T2 36 6 = =
2 2
41
x= = 6 hours 50 minutes Speed of current (Cs) = 2 kmph
6
Q.15.(1) Downstream rate (Ds) = 6 + 3 = 9 kmph
Q.8.(1) Average speed of the cyclist =6 km/hr
Upstream rate (Us) = 6-3 = 3 kmph
Average speed of sailor
If the required distance be x km, then
2 × 2 × 14
= = 3.5 km / hr x x x + 3x
2 + 14 + = 12 = 12
9 3 9
Q.9.(4) 2 2
+ =2 4x = 12 × 9
x x+2 12 × 9
x2 − 2 = 0 x=
4
x = 1.414 kmph x = 27 km.
Q.16.(1) Downstream speed of boat = 17.5+2.5
Q.10.(4) A to B is 80, so B to is 80/2 = 40 km 20 kmph
Let speed of current = x km/hr So 80/(7+x) + 40/ Upstream speed of boat = 17.5-2.5
(7-x) = 18 Solve, x = 3 km/hr
15 kmph.
So downstream speed = 7 + 3 = 10 km/hr
Distance between X and Y = X km
Q.11.(5) Let speed of boat = x km/hr and that of stream
2X
= y km/hr Distance between Y and Z = km.
5
18 18
+ =8 3
x+y x−y Total time = 429 minutes ⇒ 7 hours.
20
when speed of boat becomes 2x km/hr:
143  Dis tance 
18 18 or = hours.  = Time
+ = 3.2 20  Speed 
2x + y 2x − y
x 2x 143
x = 6 km/hr + =
20 5 × 15 20
Q.12.(2) (S+9)×18 = (S-9)×24; S =63
15x + 8x 143
300+x = 72×5/18×18; x =60 =
60 4
Q.13.(5) Downstream speed = 10+5 = 15 Upstream speed 23x = 143×15
= 10-5 = 5
143 × 15 ≈
Now total time is 25 hours If distance between A x= 93 km.
23
and B is d, then distance BC = d/2
2x 7x
Now distance/speed = time, Total Distance= x + =
5 5
124 QUANTITATIVE APTITUDE
www.mahendras.org
7 × 93 30 44
= ≈ 130 km. + = 10 ------- (2)
5 x y
Q.17.(5) Distance between B to C = x km Now, on solving from eq. (1) and (2)
2x We get
Distance between A to B = km
3 y = 11 kmph., X = 5 kmph.
Downstream speed
(i) Speed of boy
= 16+2 ⇒ 18 kmph.
Ds + Us x + y
Upstream speed = 16-2 ⇒ 14 kmph. = =
2 2
According to the question
11 + 5
2x x = = 8 kmph
⇒ + = 6.5 2
3 × 14 18
(ii) Speed of current
⇒ = 6.5 Ds − Us x−y
= =
2 2
13x = 6.5 × 126
11 − 5
6.5 × 126 = 3 kmph
x= 2
13
Q.20.(2) Let man's rate upstream be x kmph.
x = 63
Man's rate downstream be 2x kmph.
2x 5x 5 × 63
Total distance = +x= = (Speed of still water) : (Speed of Stream)
3 3 3
= 105 km.
 2x + x   2x − x 
Q.18.(2) downstream speed (Ds) =  :  =3:1
 2   2 
15
= km/h = 4 km/h. Q.21.(4) Let the speed of the boat in still water and the speed
3
3 of the stream is 7x and x
4
upstream speed (Us) Then, Downstream speed= 8x, upstream speed
= 6x
5
= km/h = 2 km/h. According to the question-
1
2 Upstream speed
2
4.2 ×60 = 18 kmph
Speed of current
14
Ds − Us 4 − 2
= = = 1 km/h. u:v =7:1
2 2
so D : U = 8 : 6
Q.19.(1) Let the upstream speed = x kmph. Now 6 = 18
Let the downstream speed = y kmph. so 8 = 24
40 55 17.6
+ = 13 ------- (1) Required answer = ×60 = 44 min.
x y 24

QUANTITATIVE APTITUDE 125


www.mahendras.org

CHAPTER

16 INEQUALITY

Now, Multiplying Equation (2) by 2, we get,


Introduction
2(2x - 5y = -5) = > 4x - 10y = -10....(iii)
In this chapter, we will study about the problems based on
equations and methods of comparing their values. Inequality Now, from (i) and (iii)
basically deals with few signs. (<) "Less than"; (>) "greater 4x - 3y = 4
than"; (=) equal to; (<) "less than or equal to"; (>) "greater 4x - 10y = -10
than or equal to".
- + +
Pattern of questions asked in exams is as follows:-
7y = 14
In the following question two equations are given. You have
to solve both equations and- y=2
Give the answer- ∴ x = 2.5
(i) x >y (ii) x > y (iii) x < y Hence x > y
(iv) x < y (v) x = y or the relation can’t be established. Quadratic equation: A quadratic equation is a second -
Variable : Variables are the values which do not remain fixed order polynomial equation in a single variable because it is a
and thus are denoted by English alphabets second-order polynomial equation, the fundamental theorem
of algebra identities that it has two solutions. These solutions
Ex. (a,b,c,m,n,o,p,q, r,x,y,z, etc.)
may be both real or both complex.
Constant : Constant are the values which remain fixed.
Quadratic formula :
Ex. (1,2,3,4 ........ and so on).
For the standard equation ax2 + bx +c= 0
The power of any variable is always one and the variable has
x = −b ± b − 4ac
2
one or more than one is known as linear equation.
2a
Types of Equation:
(i) Linear Equation: A linear equation is an equation in Ex. I. x2 + x - 6 = 0
which each term is either a constant or the product of
x2+3x-2x-6=0
a constant and the first power of a single variable.
x(x+3)-2 (x+3)=0
(a) Linear Equation with one variable:
(x+3) (x-2) = 0
Ex. (I) 6x + 5 = 4x - 15
x = - 3, 2
6x - 4x = - 20
II. y2 + 10y + 24 = 0
2x = - 20
y2 + 6y + 4y + 24 = 0
x = - 10
y(y+6) + 4 (y+6) = 0
(II) 4y - 4 = 5 + y
(y+6) (y+4) = 0
4y - y = 5 + 4
y = -6, - 4
3y = 9
Now, on comparing the values of x and y we get,
y=3
x y
which means that x < y.
- 3 > -6
(b) Linear Equation with two variables
- 3 > -4
Ex. (I) 4x - 5 = 3y -1
2 > -6
(II) 2x + 3 = 5y-2
2 > -4
4x - 3y = 4...... (i)
Hence, we can say that x > y
2x - 5y = -5.....(ii)
126 QUANTITATIVE APTITUDE
www.mahendras.org
Example: y2 = 36
x2 + 2x = 7 y = +6, -6
x2 + 2x - 7 = 0 x>y
Ex. I. 2x+3y=14
a = 1, b = 2 c = -7
II. 4x+2y=16
With quadratic formula:
Sol. I. (2x+3y = 14)× 2 ⇒ 4x+6y = 28
−b ± b2 − 4ac 4x + 6y = 28
x=
2a
4x + 2y = 16
( 2) − 4 (1)( −7) (−)(−)(−)
2
−2 ±
x=
2 (1)
0 + 4y = 12

x=
−2 + (16)
2

= =
(
−2 ± 2 2 −1 ± 2 2 ) y=3
2 2 2 x = 2.5

= -1 ±2 2 x<y
Ex. I. 20x2–x–12=0
D = b2 - 4ac
II. 20y2+27y+9=0
Discriminant Rule:
Sol. I. x = + 0.8, – 0.75
(1) If b2 - 4ac is less than 0, there is no real solution. II. y = – 0.6, –0.75
(2) If b2 - 4ac is equal to 0, then the equation has two equal 0.8 > -0.6
real solution.
- 0.75 = - 0.75
(3) If b2 - 4ac is greater than 0, then the equation has two
-0.75 < -0.6
different real solution.
Hence no relationship can be established.
Important Examples :
Ex. In the following questions two equation numbered I 12
Ex. I. = x
and II are given. You have to solve both equestions x
and -----------
5

II. y 2 – (8) = 0
Give answer (1) if x >y 2

Give answer (2) if x ≥ y y
Give answer (3) if x < y
12
Give answer (4) if x ≤ y Sol. I. = x , 12=x
x
Give answer (5) if x = y or the relationship can not
be established. 5
Ex. I. x2–24x+144=0
II. y 2

(8 ) 2 1 5

= 0 , y × y 2 – .8 2 = 0
2

II. 2y2–52y+338=0 y
Sol. I. (x–12) = 0, x = +12
2

II. y2 – 26y+169=0 5 5
(y–13) = 0
2 y =8 2 2 , y=8
y = +13, x<y x>y
Ex. I. x + 18 = 144 − 49 3 7 12 25
Ex. × x= × y
10 4 6 15 24
12 × 4 3× 4 7
II. - = y 7 5
4 4 Sol. x= y
y7 y7 8 6
21x = 20y
Sol. I. x+18=52
x=25–18=7 x<y
14 Ex. I. x + y = 16
II. 48–12=
y7 II. x2 + y2 + xy = 192

QUANTITATIVE APTITUDE 127


www.mahendras.org
Sol. I. (x+y)2 = 256 3y (2y-1) - 1 (2y -1) = 0
x2 +y2 + 2xy = 256 ....... (I) 1 1
From eq. (I) and (II) y= ,
2 3
xy = 64
x<y
(x-y)2 = (x+y)2 - 4xy
Ex. I. 2x – 1 – x + x = 5
= 256 - 4 × 64
II. 3y – y – y – 10 = – 15
=0
Sol. I. 2x – 1 = 5
x-y = 0
2x = 6
∴ x = y
x=3
Ex. I. 4x2 + 8x = 4x + 8
II. 3y – 2y = – 5
II. y2 + 9y = 2y - 12
y=–5
Sol. I. 4x2 + 4x - 8 = 0
x2 + x - 2 = 0 x>y
x2 + 2x -x - 2 = 0 Ex. I. x2 = (38)2 × 52
x (x+2) - 1 (x+2) = 0 II. y2 = (38×5)2
x = -2,1 Sol. I. x = + 64
II. y2 + 7y + 12 = 0 II. y = + 7
y2 + 4y + 3y + 12 = 0 So, relationship can not be established.
y (y+4) +3 (y +4) =0 Ex. I. x 2 = 4096
y = - 3, - 4 2
II. y 2 =  3 343 
x > y  
Ex. I. 2x2 + 40 = 18x Sol. From equation (I) and (II),
II. y2 = 13y - 42 x = +8
Sol. I. x2 - 9x + 20 = 0 y=7
x2 -5x - 4x + 20 =0 Relation can't be established
x (x-5) - 4 (x-5) =0 Ex. I. x – y = 9
x = 4,5
II. x + y = 7
II. y2 - 13y + 42 = 0
Sol. I. x = 8
y2 - 6y - 7y + 42 = 0
y (y-6) -7 (y-6) = 0 y=1
y = 6,7 x>y
x<y Ex. (I) x where: 3x2 + 2x – 8=0
1 7 (II) y: where: 3y2 + 5y -12 =0
Ex. I. 6x 2 + = x
2 2
4 4
II. 12y2 + 2 = 10y Sol. x= -2 and ; y= -3 and ;
3 3
Sol. I. 12x2 - 7x + 1 = 0 So no relation
12x2 -4x -3x + 1 =0 Ex. (I) x where: 4x2 – 16x + 15 = 0
4x (3x-1) -1 (3x-1) =0 (II) y where: 2y2 + y -6 = 0
1 1 3 5
x= , Sol. x= and
3 4 2 2

II. 6y2 - 5y + 1 = 0 3
y= -2 and
6y2 - 3y - 2y + 1 = 0 2

128 QUANTITATIVE APTITUDE


www.mahendras.org
Ex. It takes (X-2) men to do a work in 2Y days and it takes
New Pattern Inequality
(X+6) men to do the same work in Y days.
Ex. In each of the following question, a question is
I: X II: Y
followed by information given in two statement
named as Quantity-I (Q1) and Quantity –II (Q2). You Sol. As the time is becoming half so means number of
have to study the information along with the question people have doubled; So (X+6)=2×(X-2)=>X=10
and compare the value derived from Quantity-I and Using M1D1=M2D2 the value of Y cannot be found. Y
can take any value.
Quantity-II and give answer-
(1) Quantity I > Quantity II So we cannot determine a unique value of Y

(2) Quantity I ≥ Quantity II Ex. A hemisphere of radius 4 cm is to be shipped in a


shipping box of rectangular shape. The dimension of
(3) Quantity II > Quantity I
the box are consecutive odd numbers.
(4) Quantity II ≥ Quantity I
I: Minimum volume of box required to have the
(5) Quantity I = Quantity II or Relation cannot be shipment possible.
established
II: 960 cm3
Ex. 90
0
A
D Sol. I: The minimum length of any side of rectangular box
0
50
should be greater than the diameter of the hemisphere.
Diameter of hemisphere=8; So for odd number,
B 0
x0 minimum length= 9, next side =11, next side=13
y 0
0
20 C Volume=9×11×13= 1287 cm3
40
E Ex. (I) No. of days in which A will work alone, given
I. angle x II. 1100 A and B can complete work in 8 days, B and
C can complete work in 12 days, C and A can
Sol. In triangle BCE, angle y= 180-(40+20)= 120
complete work in 8 days.
Sum of exterior angle of a quadrilateral is 360 degree
(II) No. of days in which A will work alone, given
So in quadrilateral ABCD; A and B can complete work in 18 days, they
50 + y +x+90=360 [y=120] started work together and after working for 6
days A left and B completed remaining work
x=1000
in 24 days.

Q1<Q2
Sol. LCM = 24
Ex. AB is parallel to DC and AD is parallel to BC. BD is
4 cm more than AC. AC and BD are the diagonal with So
O as point of intersection. 24
A A+B= =3
B 8
900
24
o 6 cm B+C= =2
12
D C
24
I: AB II: 9 cm C+A= =3
8
Sol. I: ABCD is a parallelogram as opposite sides are
parallel. In a parallelogram the diagonal are divided 2 ( A+B+C) = 3+2+3 = 8
in equal part at the point of intersection. A+B+C = 4
So AC = 2 OC = 12 BD=16 cm (A+B+C) – (B+C) = 4 – 2 = 2
means OB=8 and OA= 6 24
So A = = 12 days
in triangle AOB; use Pythagoras theorem to find 2
AB=10 cm
A = 36 days
Q1>Q2
QUANTITATIVE APTITUDE 129
www.mahendras.org
EXERCISE
Q.1-20. In each of the following question, two equations Q.14. (I) 18x + 13y = 8
(I) and (II) are given. You have to solve them and (II) 27x + 12y = 12 
Give answer-
Q.15. (I) 204x2 + 59 x + 2 = 0
(1) If x>y (2) If x>y
(II) 300y2 - 20y - 1 = 0
(3) If x<y (4) If x<y
(5) If x=y or relationship can not be established Q.16. (I) y2 + 6x - 40 = 0 
Q.1. (I) (8x2 + 21) – (6x2 + 23) = 0 (II) y2 + 45y + 476 = 0
(II) (7y2 – 14) – (10y2 – 17) = 0   Q.17. (I)  x = 25
Q.2. (I)  2x2 – 17x + 36 = 0 4

(II) 15y2 – 16y + 4 = 0 (II) 16y2 - 40y + 25 = 0


Q.3. (I) 7x2 – 41x + 30 =  0 Q.18. (I) 4x2 + 9 - 12x = 0           
(II)  7y2 – 26y + 24 = 0 (II) 64y2- 80y + 25 = 0
1 Q.19. (I) 34x2 + 11x = 3  
Q.4. (I)
(196)2 x + 324 = 172
(II) 51y2 - 77y + 12 = 0 
1
(II) (441) 2 y 2 − 288 = 1413 Q.20. (I)

Q.5. (I)    x –  2401 = 0


(II)  y – 7 = 0  (II)
Q.6. (I) 4x 2 + 4 3x + 3 = 0
Q.21-40. In each of the following question, a question is
(II) 3y 2 + 4 3y + 4 = 0 followed by information given in two statement
Q.7. (I) 16x2 – 8x + 1 = 0            named as Quantity-I (Q1) and Quantity –II (Q2).
You have to study the information along with
(II)  100y2 – 20y +1 = 0 the question and compare the value derived from
Q.8. (I) 14x2 - 53x + 14 = 0 Quantity-I and Quantity-II and give answer-
(1) Quantity I > Quantity II
(II) 4y2 - 25 = 0
(2) Quantity I ≥ Quantity II
Q.9. (I) (x + 3)2-25=0 (3) Quantity II > Quantity I
(II) y -14y + 49 =0
2 (4) Quantity II ≥ Quantity I
(5) Quantity I = Quantity II or Relation
Q.10. (I) x2 -10x+25=0
cannot be established
(II) y2 + 26y -169 =0 
Q.21. (I) The age of teacher, if the average age of 36
Q.11. (I) x2 - 6.9x + 11.7 = 0 students is 14. When teacher’s age is included
(II) y2-10.2y + 10.8 = 0 the average increases by 1.
(II) The age of teacher, if the average age of 19
1 students is 35. When teacher’s age is included
Q.12. (I) 2x 2 + 20% of 15x − of 2.5 = 0 the average increases by 0.5.
50%
Q.22. (I) Profit Percentage , if Some articles were bought
1 at 6 articles for Rs. 5 and sold at 5 articles for
(II) 2y2- 50% of 36y + of 10 = 0
25% Rs. 6.
(II) Profit Percentage, if 100 toys are bought at the
Q.13. (I) x 2 − 11 11x + 110 = 0 rate of Rs. 350 and sold at the rate of Rs. 48
per dozen.
(II) y 2 − 13 13y + 156 = 0
130 QUANTITATIVE APTITUDE
www.mahendras.org
Q.23. (I) On selling 17 balls at Rs. 720, there is a loss Q.32. Find the original consumption if
equal to the cost price of 5 balls. The cost price (I) After increasing price by 20% a family now
of a ball is: gets 5 kg less on that price.
(II) A man buys a cycle for Rs. 1400 and sells it at (II) After increasing price by 25%, a family in-
a loss of 15%. The selling price is: creases its expenditure by 10% and gets 6 kg
Q.24. (I) A and B together can do a piece of work in 4 less than original consumption
days. If A alone can do the same work in 6 days, Q.33. Find the present age of A.
then B alone can do the same work in? (I) Three years before, the ratio of ages of A and
(II) A can do a piece of work in 4 hours; B and C B was 5:6. Three years hence this ratio will
together can do it in 3 hours, while A and C become 6:7.
together can do it in 2 hours. How long will B (II) Eleven years before the ratio of ages of A and
alone take to do it? B was 1:3 and eleven years hence the ratio will
Q.25. (I) A man on tour travels first 160 km at 64 km/hr become 1:2.
and the next 160 km at 80 km/hr. The average Q.34. Find the sum.
speed of the tour is:
(I) If the Compound Interest for 2 years at 20%
(II) A went from P to Q with the speed of 60km/ rate of interest is Rs 1,320.
hr. and return back with the speed of 90km/hr.
(II) If the amount on a sum for 2 years in which
Find the average speed.
rate of interest for 3 years makes a sum of 125
Q.26. (I) The ratio between the speeds of two trains is 7 to amount 216 is Rs 2880.
: 8. If the second train runs 400 km in 4 hours,
Q.35. Find the distance if-
then the speed of the first train is:
(I) A man covers a distance in 15 hours. He covers
(II) Find the speed of a train which passes a tree in
first half at 12 kmph and second half at 15 kmph.
12 seconds. The length of the train is 264m.
(II) Two buses moves towards each other at a speed
Q.27. (I) A and B started a business by investing Rs.
of 30 kmph and 40 kmph respectively. When
20000 and Rs. 35000 respectively. Find the
they meet it is found that faster bus covers 30
share of B out of an annual profit of Rs. 3520.
km more than slower one.
(II) X and Y invested in a business. Their profit ratio
Q.36. (I) Selling price, if cost price is Rs 24,000 and
is 2:3. If X invested Rs. 4000. Find the amount
profit is 20%
invested by Y?
(II) Selling price, if cost price is Rs 24,000 and
Q.28. (I) The age of P is twelve times that of her daughter
shopkeeper gained 16 2/3% after giving dis-
Q. If the age of Q is 3 years, what is the age of P?
count of 25%
(II) The ratio between the present ages of A and B
Q.37. (I) Volume, if diameter of sphere is 14 cm
is 2:3. 4 years ago the ratio between their ages
was 5:8. What will be A’s age after 7 years? (II) Volume, if side of cube is 8 cm
Q.29. (I) The difference between SI and CI compounded Q.38. If a & b are natural numbers and 6 ≥ a > b > 0.
annually on a certain sum of money for 2 years (I) 5a2 b
at 8% per annum is Rs. 12.80. Find the principal (II) 6ab2
(II) A sum fetched a total simple interest of Rs. 800 Q.39. (I) The shopkeeper sold an article at 10 % discount on
at the rate of 8 %per annum in 5 years. What is marked price and he gains 20%. If the marked price
the sum? of the article is Rs.400, then find the cost price.
Q.30. There are 5 Brown balls, 4 Blue balls & 3 black balls (II) The shopkeeper marks the price of the Pen
in a bag .Four balls are chosen at random Rs.350 and his profit % is 15%. Find the cost
(I) The probability of their being 2 Brown and 2 price of the Pen, if he allows a discount of 8%?
Blue ball. Q.40. (I) Rakesh can swim at 10 km/hrs in still water.
(II) The probability of their being 2 Brown, 1 Blue The river flows at 6 km/hrs and it takes 6 hours
& 1 blacks more upstream than downstream for the same
Q.31. If the quantity of milk in mixture is 10 litre then find distance. How far is the place?
the quantity of water if (II) A boy can row 20 km/hrs in still water and the
river is running at 8 km/hrs. If the boy takes 4
(I) After selling it at Cost price, milkman saves 25%
hours to row to a place and back, how far is the
(II) If the ratio of Milk and water is 5:1 place?
QUANTITATIVE APTITUDE 131
www.mahendras.org
EXERCISE Explanation
Q.1.(5) I. 2x2 - 2 = 0 If, y2-10.2y+ 10.8 = 0
2x2 = 2 Then, y = 1.2 and 9
x2 = 1 So, x>y and y>x
x = ± 1 Q.12.(3) On solving we get
II. -3y2 + 3 = 0 x = -2.5, 1
y2 = 1 y= 5 , 4
y = ±1 x<y
Hence , Relationship Cannot be established. Q.13.(5) If x2- 11 11x + 110 = 0
Q.2.(1) x= 4.5, 4 Then x = 10 11 and 1 11
y = 0.67, 0.4 If, y 2 − 13 13y + 156 = 0
x>y
Q.3.(5) I. 7x2 - 35x - 6x + 30 = 0 Then, y = 12 13 and 13
7x (x-5) - 6 (x-5) = 0 So, x > y and x < y
6 Q.14.(3) x = 2/15 ; y = 47/135
x = 5,
7 x<y

II. 7y2 - 14y - 12y + 24 = 0 Q.15.(3) x =   -0.25, -0.039

12 y =   +0.1, -0.033


7y (y - 2) - 12 (y - 2) = 0, y = 2, x< y
7
Q.16.(1) x = -10, 4 ; y = -28, -17
\ Relation can't be established.
Q.4.(1) I. 14x + 18 = 172 Q.17.(1) x = 2.5 ; y = 1.25
14x = 154, x = 11 x>y
II. 21y2 = 1701, y2 = 81, y = +9 Q.18.(1) 33 55
\ x > y x= ;y =
22 88
Q.5.(5) I. x = 2401, x = 49
x>y
II. y = 7, y = 49 Q.19.(4)
3 1 4 3
\x=y x= , − ;y = ,
17 2 3 17
3 3 2 2
Q.6.(3) x= , ; y =  , x<y
2 2 3 3
x<y Q.20.(1) I. , x = 80
Q.7.(1) x = 0.25, 0.25 ; y = 0.1, 0.1
x>y II. , y = 26
Q.8.(5) 7 2 5
x= , ,y = ± x>y
2 7 2
Q.21.(1) I. 14+ (37×1) =14+37=51yrs.
Q.9.(3) x=2 ,-8,     y= 7 ,7
II. 35 + (20×0.5) =35+ 10=45yrs.
So, x<y
11
Q.10.(1) x= 5, 5,     y= -13, -13 Q.22.(1) I. Profit %ge = ×100 =44%
25
So, x>y
0.5 2
Q.11.(5) If, x² - 6.9x + 11.7 = 0 II. Profit %ge ×100 = 14 %
3.5 7
Then x = 3.9 and 3
132 QUANTITATIVE APTITUDE
www.mahendras.org
Q.23.(3) I. C.P. of 12 balls = S.P. of 17 balls = Rs.720. 4000 2
II. =
720 y 3
CP of 1 ball = =Rs. 60
12
y = 6000.
II. SP =85% of 1400 Q.28.(1) I. Ratio P:Q 12:1
85 1=3
= ×1400 =Rs. 1190. 12 = 12×3 =36 years.
100
1 1 2 5x + 4 2
Q.24.(5) I. B work= - = ⇒ 12days =
II.
4 6 24 8x + 4 3
15x+12 = 16x+8
1 x=4.
II.  A’s 1hr work
. A’s age 4 yrs ago 5×4=20
4
Then A’s age after 7yrs is
1 20+4+7=31yrs.
(B+C’s) 1 hr work .
3
Pr 2
1 Q.29.(5) I. SI-CI= .
(A+C’s) 1hr work . 1002
2
P × 82
1 1 7 2
= 12.8
A+B+C 1hr work = + = . 100
4 3 12
7 1 1 64P2
− = = 12.8
B’s work = = 12 hours. 2
12 2 12 100
160 160 9 P=Rs2000.
Q.25.(3) I. Total time taken = + = hrs
64 80 2 Pnr
II. SI=
320 100
Then avg speed = ×2
9
P×8×5
2 800=
= 320 × = 71.11km/hr. 100
9
2 × 60 × 90 800 × 100
II. =72km/hr. P= =Rs.2000.
150 40
Q.26.(1) I. Let the speed of two trains be 7x and 8x. 5
c2 ×4 c2 60 4
Q.30.(3) I. = =
400 12
c4 495 33
= 100
4 5
8
c 2 × 4 c1 × 3 c1 120 8
8x=100Þx=12.5. II. 12
= =
c4 98
33 33
Then speed of first train =7×12.5
=87.5km/hr. Q.31.(1) Quantity I.
II. Length of the train = 264m. Milk : Water
Time taken to pass the tree = 12 seconds. 100    :   25
10=4  : 1=2.5
264 In Quantity II:
Speed of the train = m/sec = 22
12 5=10 ; 1=2
18 Quantity 1> Quantity 2
m/sec = 22 × km/hr = 79.2 km/hr.
5 Q.32.(3) Quantity I: 20% Increase= 1/5 
Q.27.(3) I. Ratio 20:35=4:7 1
11 = 3520
×T = 5
1+ 5
7 ?ÞRs2240. T=30 Kg
QUANTITATIVE APTITUDE 133
www.mahendras.org
Quantity II:  D= 3× (30+40) =210 Km 
                100 II > I
110                        125
125-110=15 120
Q.36.(1) I. SP = × 2400 = 2880
100
15
×T = 6
125 7
T=50 Kg II. SP = × 2400 = 2800
6
II > I
Q.33.(5) Quantity I:            A                B 4
Q.37.(1) I. Volume of sphere = × π × r3
3 years before       5                6 3
3 years after          6                7 R=7; Volume= 1437.33
Difference in both case 6-5=1 and 7-6=1
II. V=a3 = 83 = 512
1=6
6=30 I > II
Present age =30+3=33 Q.38.(2) Dividing Quantity I and Quantity II we get
Quantity II:                  A                B I 5a
11 years before             1                3 =
II 6b
11 years after                1                2
Difference A=1-1=0 ; B= -1 Now b has to be > 0 and a has to be > b
To make the difference same; multiply equation If a = 2, b = 1, I > II
2 by 2; we get Similarly we will get I > II in all cases
Quantity II:                  A                B But a = 6 and b = 5
11 years before             1                3 Then
11 years after                2                4
a=b
Difference A=2-1=1; B=4-3=1
1=2 Quantity I > Quantity II
11=22 Q.39.(1) Quantity I:
Present age= 22+11=33 Selling price of the article = 400×(90/100)
Q.34.(1) CI for 2 years on 20%=1/5 = Rs.360
5             6 Cost price of the article = 360×(100/120)
5             6 = Rs.300
5×5=25   6×6=36 Quantity II:
CI=36-25=11 =1320 Selling price of the Pen = 350×(92/100)
Sum=25=3000 = Rs.322
Quantity II: Find Rate
Cost price of the Pen = 322×(100/115)
Cube root(125): Cube root (216) 
= Rs.280
5:6
1 Quantity I > Quantity II
6−5
rate= × 100 =20% = Q.40.(3) x/(10-6) - x/(10+6) = 6
5 5
5             6 x/4 - x/16 = 6
5             6 3x/16 = 6
5×5=25   6×6=36
x = 32 km
36=2880; hence 25=2000
I > II Quantity II:
Speed of still water (x)
Q.35.(3) I. D = 2 × 12 × 15 × 15 = 200
27 = 20 km/hrs, Speed of stream (y) = 8 km/hrs
Quantity II: D = t×[(x2- y2)/2x]
Speed difference for 1 hour =40-30=10 = 4×[(202 - 82)/(2×20)]
Kmph; means in 1 hour faster bus will = 4×(400 - 64)/40
cover 10 km more than slower one; hence
to cover 30 km more it will take 3 hours. = 4×(336/40) = 33.6 km
Distance= Relative Speed × Time Quantity I < Quantity II
134 QUANTITATIVE APTITUDE
www.mahendras.org

Permutation &
CHAPTER

17 Combination
The study of permutations and combinations is concerned 6! 6! 6 × 5 × 4!
with determining the number of different ways of arranging (i) 6
P2 = = = = 30
(6 − 2)! 4 ! 4!
and selecting objects out of a given number of objects, 7! 7! 7 × 6 × 5 × 4!
(ii) 7
P3 = = = = 210
without actually listing them. There are some basic counting
(7 − 3)! 4 ! 4!
techniques which will be useful in determining the number
of different ways of arranging or selecting objects. Note : Number of all permuations of n-things, taken all
Factorial at a time = n!

Factorial denoted by n or n ! Circular Permutations : There are two cases of circular-


permutations:-
Where n! = n. (n-1) (n-2)...............3.2.1
(a) If clockwise and anti clock-wise orders are different,
eg. (I.) 5! = 5.4.3.2.1 = 120 then total number of circular-permutations is given by
(II) 6! = 6.5.4.3.2.1 = 720 (n-1)!
from (i),and (II) we get (b) If clock-wise and anti-clock-wise orders are taken
6! = 6.5.4.3.2.1 as not different, then total number of circular-
permutations is given by (n-1)!/2
6! = 6.5!
in general n! = n (n-1) !
Combination
(selection, committee, group)
(III) 1! = 1
(IV) 0! = 1 Each of the different groups or selections which can be
formed by taking some or all of a number of object called
n! = n ×(n-1)! a combinations.
n is greater than 0 Suppose we want to select two out of three persons A, B,C.
1! = 1 ×(1-1)! Then possible selections are :
1! = 1×(0)! AB, BC, CA
1 = 0! Note : AB and BA represent the same selection.
Permutation : (Arrangement) Suppose we want to select three out of three person A, B, C.
The different arrangements of a given number or things by Then possible selections are ABC.
taking some or all at a time, are called permutation. Note : ABC and BAC and CAB are the same selection.
A permutation is an arrangement of objects in a definite order. Number of combinations : The number of combinations of
All permutations (or arrangements) made with the letters n things, taken r at a time is :
of ‘abc’, taking two at a time are : (ab ba, ac, ca, bc, cb). n!
n
Cr =
All permutation made with the letters a,b,c taking all at a
time are :
b g
r !× n − r !

(abc, acb, bac, bca, cab, cba) Note : (i) If (n=r), n Cr = 1 and n C 0 = 1
Number of Permutations : Number of all permutations of Important points
n things, taken r at a time, is given by : n
C r = n C bn − r g ,
n n!
Pr =
b g
n−r !
16 16 !
eg. C13 =16 C16 −13 =16 C 3 =
e.g. 3 !× 13 !
QUANTITATIVE APTITUDE 135
www.mahendras.org
16 × 15 × 14 × 13 ! 6 × 5 × 4 × 3 × 2×1
= = 560 = = 180
3 × 2 × 1 × 13 ! 2 × 1× 2 × 1
Trick :
Conclusion : The basic difference between permutation &
6! 6 × 5 × 4 × 3 × 2×1
combination. = = 180
2 !× 2 ! 2 × 1× 2 × 1
Permutation
Ex. In how many different ways can the letters of the
There are three letters a,b,c and permutation made with them
word ELEPHANT be arranged so that vowels al-
by taking two at a time- a,b,c
ways occur together?
ab
ac Sol. Vowels = E, E and A.
bc 3!
ba 6-different ways They can be arranged in= i.e. 3!/2! Ways
2!
ca
cb 6! × 3!
So total ways = = 2160
* Here ab, is possible then ba is also possible in permutation. 2!

Combination : There are three person A, B, C we want to Ex. In how many different ways can the lettes of the
select two out of three. The number of combination are : ABC word ‘ROSE’ be arranged so that vowels never come
AB together ?
AC Sol. Ways (never)
BA
BC 3-different ways
= Ways (total) - Ways (always)
CA
CB = 4! - 3! × 2! = 24 -12 = 12

* Here AB is possible, then BA is not possible in combination Ex. In how many different ways can the word
because they represent same combination. ‘TOFFEE’arranged so as, vowels never come to-
gether ?
Result : In the above discussion, we observe that if AB is
possible and BA is also possible then we apply permutation Sol. Ways (never) = ways (Total) - ways (always)
and when AB is possible and BA is not possible then we
apply combination. 6! 4 !× 3 !
= − = 180 - 36= 144
2 ! × 2 ! 2 !× 2 !
Permutation :
Ex. How many words can be formed by re-arranging
Ex. In how many different ways can the letters of the
the letters of the word CURRENT such that C and
word ROSE be arranged ?
T occupy the first and last position respectively?
Sol. n = 4, r = 4
Sol. 5!
By Formula : = 60
2!
4 4! 4! 4 × 3 × 2 × 1 Ex. In how many ways 4 Indians, 5 Africans and 7 Japa-
P4 = = = = 24
(4 − 4)! 0 ! 1 nese be seated in a row so that all person of same

nationality sits together?
Trick :
Sol. 4! × 5! × 7! × 3!
4! = 4 × 3 × 2 × 1 = 24
Ex. In how many ways 5 Americans and 5 Indians be
Ex. In how many different ways can the letters of word seated along a circular table, so that they are seated
‘Toffee’ arranged ? in alternative positions?
Sol. Here, ‘F’ and ‘E’ repeated two times therefore- Sol. 4!x5! =2880
6! Ex. How many necklace of 12 beads each can be made
6
P6 (6 − 6)! 6!
= = from 18 beads of different colours?
2
P2 × 2 P2 2! 2! 2 !× 2 !
× Sol. Here clock-wise and anti-clockwise arrangements are same.
(2 − 2)! (2 − 2)!
Hence total number of circular–permutations:
136 QUANTITATIVE APTITUDE
www.mahendras.org
18
P12 18! Sol. = 10c3 × 5c1
= 10 5 10 5
2 × 12 6 × 24 = × = ×
3× 7 1 4 37 4
Ex. In how many ways can  15  people be seated
around two round tables with seating capacities 10 × 9 × 8
= × 5= 120 × 5 = 600
of 7 and 8 people? 3×2

Sol. 15
C8×6!×7! + 15C7×6!×7! Ex. From a group of 10 men & 5 women. 4 persons are
to be selected such that either 4 men or 4 women in
Combination the group. Find the different number of ways.
Ex. Find the value of 5c2. 10 5
Sol. c4 + 5c4 =
4 6+ 4 1
10

Sol. c2 = 2 5 = 2 5 =
5 5×4
= 10 10 × 9 × 8 × 7
5-2 ×3 2 = + 5= 210 + 5 = 215
4×3×2
Ex. Find the value of n when nc2 = 105 ?
Ex. A committee of 5 members is to be formed out of 4
Sol. c2 = 105
n
men and 5 women.
n
2 n-2 = 105 (i) In how many ways can a committee consisting
n (n- 1) = 210 { 142 < 210 < 152} of at least 1 woman be formed?
n = 15 Sol. 5
c1 × 4c4 + 5c2 × 4c3 + 5c3 × 4c2 + 5c4 × 4c1 + 5c5 × 4c0
Ex. There are 15 persons in a group. They shake hand = 5 × 1 + 10 × 4 +10 × 6 + 5 × 4 + 1 = 126
with each other. Find the different no. of hand shake. (ii) In how many ways can a committee consisting
Sol. By formula of 3 men and 2 women be formed ?
15 15 15 × 14 4 5
c2 = 2 15–2 =
15
= = 105 Sol. 4
c3 × 5c2 = 3 1 × 2 3
2 13 2
Ex. From a group of 10 men & 5 women 4 persons are to
be selected to form a committee. Find the different 4×5×4
= = 40
number of ways for selection. 2
15 15 Ex. There are 6 men and 7 women. In how many ways
Sol. 15
c4 = 4 15-4 = 4
11 a committee of 4 members can be made such that a
15 × 14 × 13 × 12 particular man is always to be excluded?
= = 105 × 13 = 1365
4×3×2 Sol. Here are total 13 people, a particular man is to be
Ex. From a group of 10 men & 5 women. 4 person is excluded, so now 12 people are left to chosen from
to be selected such that 3 men and 1 woman in the and 4 members to be chosen. So ways are 12C4 = 495.
group ?

Notes

QUANTITATIVE APTITUDE 137


www.mahendras.org
EXERCISE
Q.1. Find the number of different words that can be formed from the word 'SUCCESS'.
(1) 360 (2) 480 (3) 420 (4) 5040 (5) None of these
Q.2. In how many different ways can the letters of the word ‘LEADING’ be arranged in such a way that the vowels
always come together?
(1) 360 (2) 480 (3) 720 (4) 5040 (5) None of these
Q.3. In how many different ways can the letters of the word ‘CORPORATION’ be arranged so that the vowels always
come together?
(1) 810 (2) 1440 (3) 2880 (4) 50400 (5) 5760
Q.4. In how many diffirent ways can the alphabets of word "Equation" can be arranged taking 5 letters at a time
with the condition that the letters can be repeated?
(1) 32768 (2) 34569 (3) 37268 (4) 21724 (5) None of these
Q.5. In how many ways the alphabets of the word "CREATION" can be arranged so that all the consonants are alternate?
(1) 1052 (2) 952 (3) 1152 (4) 852 (5) None of these
Q.6. In how many ways the letters of the word "WOMEN" can be rearranged so that when constant occupies odd places,
vowel comes on even places ?
(1) 10 (2) 11 (3) 12 (4) 13 (5) 14
Q.7. In how many ways can 5 men, 3 women and 2 girls can be seated in a row so that the people of same gender are
not seated together ?
(1) 3718640 (2) 3729123 (3) 3628800 (4) 3620160 (5) None of these
Q.8. In how many ways can 7 boys and 5 girls be seated in a row so that no two of the girls can be together ?
(1) 9676800 (2) 9266420 (3) 9352980 (4) 3456780 (5) None of these
Q.9. A round table meeting is held between eight persons A, B, C, D, E, F G and H. In how many ways can they seated
so that A and D always sits diagonally opposite to each other?
(1) 1220 (2) 1330 (3) 1110 (4) 1440 (5) None of these
Q.10. How many ways can 4 apples be given in such a way to 3 girls if one girl can recieve 1 apple.
(1) 64 (2) 360 (3) 6 (4) 24 (5) None of these
Q.11. A group photograph of a family having 6 females and 20 males is to be taken the first row consist of women and the second
row consist of boys with 2 tallest boys standing at the extreme corners of the second row. Find the number of arrangement.
(1) 2 × 18! × 6! (2) 18! × 6! (3) 3 × 6! × 18! (4) 9! × 14! (5) None of these
Q.12. Find the total number of employee code that can be formed by using two alphabets followed by 2 numbers and the
letters should be distinct.
(1) 61000 (2) 54000 (3) 42000 (4) 63000 (5) None of these
Q.13. 12 points lie on a circle. How many cyclic quadrilaterals can be drawn by using these points ?
(1) 595 (2) 495 (3) 394 (4) 295 (5) 410
Q.14. Two boxes A and B contain 5 balls each. We have to choose 6 balls in all of which at least 2 should be from Box A
and atleast 2 from Box B. In how many ways the selection can be made?
(1) 150 (2) 180 (3) 165 (4) 200 (5) None of these
Q.15. There are 5 boys and 4 girls, In how many ways 4 boys and 2 girls can be seated on 6 chairs?
(1) 6400 (2) 12500 (3) 21600 (4) 34300 (5) None of these
Q.16. There are 5 professor, 8 lecturer and 7 teacher. 6 people to be selected to make an interview board. In how many ways
can they selected if it contain equal number of professors, lecturers and teachers?
(1) 5880 (2) 4770 (3) 3450 (4) 2180 (5) None of these
Q.17. A committee of 5 persons is to be formed from 6 men and 4 women. In how many ways can this be done when at
most 2 women are included?
(1) 124 (2) 132 (3) 186 (4) 174 (5) None of these

138 QUANTITATIVE APTITUDE


www.mahendras.org
Q.18. A football team of 11 players is to be selected out of 16 players. 16 players consists of 2 goal keepers and 5 defenders
and rest forwards. In how many ways can it be selected so that it consist of 1 goal keeper and at least 4 defenders?
(1) 992 (2) 1100 (3) 1092 (4) 999 (5) None of these
Q.19. In a cricket tournament, there are 153 matches played. Every two team played one match with each other. The
number of team participating in the tournament is-
(1) 12 (2) 11 (3) 18 (4) 14 (5) 16
Q.20. A college has 10 volleyball players and 6 captain.Make a team of 6 members including captain. How many different
selections can be made ?
(1) 1134 (2) 1100 (3) 1300 (4) 1000 (5) None of these
Q.21. The total number of committee of 7 people is to be formed from 9 boys and 6 girls such that the boys are in the
majority and it has atleast 1 girl-
(1) 4914 (2) 2072 (3) 2076 (4) 3426 (5) None of these
Q.22. How many number plates of 3 digit can be formed with four digits 1,2,3 and 4 ?
(1) 27 (2) 24 (3) 8 (4) 20 (5) 16
Q.23. How many numbers of five digits can be formed with the digits 1,3,5 7 and 9 no digit being repeated ?
(1) 120 (2) 240 (3) 720 (4) 360 (5) 5040
Q.24. How many different 5 - digit numbers can be formed by using the digits of the number 713628459?
(1) 15210 (2) 15120 (3) 15180 (4) 45360 (5) 30240
Q.25. How many numbers of five digits can be formed with the digits 0,2,4,6 and 8 ?
(1) 24 (2) 48 (3) 96 (4) 120 (5) None of these
Q.26. How many even numbers of three digits can be formed with the digits 0,1, 2, 3, 4, 5 and 6 ?
(1) 30 (2) 105 (3) 75 (4) 180 (5) 150
Q.27. A round table conference is to be held between delegates of 15 companies. In how many ways can they be seated
if delegates from two MNCs may wish to sit together ?
(1) 14! (2) 14!×2! (3) 14!/2! (4) 14!×2!-1 (5) None of these
Q.28. A person has 12 friends out of which 7 are relatives. In how many ways can he invite 6 friends such that at least 4
of them are relatives ?
(1) 460 (2) 426 (3) 432 (4) 464 (5) 462
Q.29. How many words can be made of letters COURTESY which began with C and end with Y ?
(1) 120 (2) 540 (3) 420 (4) 720 (5) 1040
Q.30. Of 8 balls how many arrangement can be made in which 3 given balls are not all together ?
(1) 8!-6!×2! (2) 6!-8!×2! (3) 8!-6!×3! (4) 6!-8!×3! (5) None of these
Q.31. From a group of 7 men and 6 women, five persons are to be selected to form a committee so that at least 3 men are
there in the committee. In how many ways can it be done?
(1) 564 (2) 645 (3) 735 (4) 756 (5) None of these
Q.32. Out of 7 consonants and 4 vowels, how many words of 3 consonants and 2 vowels can be formed?
(1) 210 (2) 1050 (3) 25200 (4) 21400 (5) None of these
Q.33. A box contains 2 white balls, 3 black balls and 4 red balls. In how many ways can 3 balls be drawn from the box,
if at least one black ball is to be included in the draw?
(1) 32 (2) 48 (3) 64 (4) 96 (5) None of these
Q.34. In how many different ways can the letters of the word ‘DETAIL’ be arranged in such a way that the vowels occupy
only the odd positions?
(1) 32 (2) 48 (3) 36 (4) 60 (5) 120
Q.35. How many 4-letter words with or without meaning, can be formed out of the letters of the word, ‘LOGARITHMS’,
if repetition of letters is not allowed?
(1) 40 (2) 400 (3) 5040 (4) 2520 (5) 13
Q.36. The Indian Cricket team consists of 16 players. It includes 2 wicket keepers and 5 bowlers. In how many ways can a cricket
eleven be selected if we have to select 1 wicket keeper and at least 4 bowlers?

QUANTITATIVE APTITUDE 139


www.mahendras.org
(1) 1029 (2) 1072 (3) 1040 (4) 1092 (5) None of these
Q.37. In a small village, there are 87 families, of which 52 families have at most 2 children. In a rural development
program 20 families are to be chosen for assistance, of which at least 18 families must have at most 2 children. In
how many ways can the choice be made?
(1) 52C19×35C1+50C19×37C1+52C20 (2) 52C18×35C2+52C19×35C1+52C20
(3) C18× C2+ C19× C1- C20
52 35 52 35 52
(4) 52C18×35C2-52C19×35C1+52C20
(5) None of these
Q.39. How many arrangements can be made out of the letters of the word COMMITTEE, taken all at a time, such that
the four vowels do not come together?
(1) 216 (2) 720 (3) 1260 (4) 4320 (5) None of these
Q.40. In how many ways can the letters of the word EDUCATION be rearranged so that the relative position of the
vowels and consonants remain the same as in the word EDUCATION?
(1) 4!×5! (2) 5!×3! (3) 4!×4! (4) 5!×5! (5) None of these
Q.41. 3 boys and 2 girls are to be seated in a row in such a way that two girls are always together. In how many different
ways can they be seated?
(1) 12 (2) 24 (3) 72 (4) 36 (5) 48
Q.42. If the letters of the word CHASM are rearranged to form 5 letter words such that none of the word repeat and the results
arranged in ascending order as in a dictionary what is the rank of the word CHASM ?
(1) 24 (2) 32 (3) 60 (4) 48 (5) 120
Q.43. How many arrangements of the letters of the word ‘BENGALI’ can be made if the vowels are to occupy only odd
places.
(1) 576 (2) 567 (3) 625 (4) 720 (5) None of these
Q.44. In how many ways 3 mathematics books, 4 history books, 3 chemistry books and 2 biology books can be arranged
on a shelf so that all books of the same subjects are together.
(1) 42472 (2) 41472 (3) 44725 (4) 45722 (5) 46427
Q.45. There are four bus routes between A and B; and three bus routes between B and C. A man can travel round-trip
in number of ways by bus from A to C via B. If he does not want to use a bus route more than once, in how many
ways can he make round trip?
(1) 72 (2) 44 (3) 32 (4) 19 (5) 24
Q.46. What is the total number of ways in which Dishu can distribute 99 distinct gifts among his 88 distinct girlfriends
such that each of them gets at least one gift?
(1) 172x8! (2) 144x8! (3) 36x8! (4) 72x8! (5) None of these
Q.47. From a total of six men and four ladies a committee of three is to be formed. If Mrs. X is not willing to join the
committee in which Mr. Y is a member, whereas Mr.Y is willing to join the committee only if Mrs Z is included. 
How many such committee are possible?
(1) 138 (2) 128 (3) 112 (4) 91 (5) None of these
Q.48. The number of ways which a mixed double tennis game can be arranged amongst 9 married couples if no husband
and wife play in the same is:
(1) 1514 (2) 1512 (3) 3024 (4) 3028 (5) None of these
Q.49. A basket contains 6 blue, 2 red, 4 green and 3 yellow balls. If 5 balls are picked at random find the number of ways
for selecting the balls such that at least one is blue?
(1) 2877 (2) 2651 (3) 2400 (4) 2600 (5) None of these
Q.50. There are eight boxes of chocolates, each box containing distinct number of chocolates from 1 to 8. In how many
ways four of these boxes can be given to four persons (one boxes to each) such that the first person gets more
chocolates than each of the three, the second person gets more chocolates than the third as well as the fourth persons
and the third person gets more chocolates than fourth person?
(1) 70 (2) 72 (3) 40 (4) 80 (5) None of these

140 QUANTITATIVE APTITUDE


www.mahendras.org
EXERCISE Explanation
Q.1.(3) No. of Permutation = n! / p! × q!, where p = 3628800 - 8640
= of one type , q = ( of another type ). = 3620160 ways
No. of Permutation = 7!/ 3! × 2! Q.8.(1) _ B _ B _ B _ B _ B _ B _ B _
No. of Permutation = 420 Boys = 7!
Q.2.(3) Then, we have to arrange the letters LNDG (EAI). Girls = 8p5
Now, 5 (4 + 1 = 5) letters can be arranged in 5! 7 ! × 8p5 = 9676800 ways
= 120 ways. Q.9.(4) A
The vowels (EAI) can be arranged among
themselves in 3! = 6 ways.
 Required number of ways = (120 x 6) = 720.
Q.3.(4) In the word 'CORPORATION', we treat the vowels D
OOAIO as one letter.
A and D can be seated in 2! ways
Thus, we have CRPRTN (OOAIO).
and others can be seated in 6! ways
This has 7 (6+1) letters of which R occurs 2 times
and the rest are different. Total number of ways = 2! × 6! = 1440 ways
Number of ways arranging these letters Q.10.(4) G1 G2 G3
7! ↓ ↓ ↓
= = 2520
2! 4 × 3 × 2 = 24 ways

Now, 5 vowels in which ) occurs 3 times and the Q.11.(1) Difference = 2 × 18! × 6!
5! Q.12.(5) Out of 26 alphabets two distinct letters can be
rest are different, can be arranged in = 20 day
3! arranged in 26p2 ways.
The first digit of the number in 10 ways
Required number of ways = (2520×20) = 50400
The second digit of the number in 10 ways
Q.4.(1) The word formed will be of 5 letters and the letters
can be repeated. So, total number of ways
_ _ _ _ _ = 26p2 × 10 × 10 = 65000
8×8×8×8×8 Q.13.(2) Any set of 4 points we get a cycle combination
quadrilateral number of ways of choosing 4 points
85 = 32768 ways
out of.
Q.5.(3) In one arrangement consonants will come on even
12 points is 12c4 = 495
places and in are arrangement they will come on
odd places Q.14.(4) Part A Part B
therefore 2 × 4! × 4! = 1152 ways So,
Q.6.(3) Consonants comes on 1, 3, 5 place (2 and 4), (3 and 3), (4 and 2),
So, total arrangement = 3! = 6 = 5c2 × 5c4 +5c3 × 5c3 + 5c4 × 5c2
Vowels comes on 2, 4 place
= 10×5 + 10 × 10 + 5 × 10
So, total arrangement = 2! = 2 = 50 + 100 + 50 = 200 ways
So, total arrangement = 6 × 2 = 12 ways Q.15.(3) 4 boys can be selected out of in 5c4 ways
Q.7.(4) Never together = All - together = 2 girls can be seated out of 4 in 4c2 ways
= 10 ! - 3! × 5! × 3! × 2! Number of selecting = 5c4 × 4c2 = 30

QUANTITATIVE APTITUDE 141


www.mahendras.org
Ways of arranging = 6! Q.23.(1) the no. of digits = 5
Total number of arrangement Required no. = 5P5 = 5! = 120
= 30 × 720 = 21600 Q.24.(2) Pr = n! / (n-r)!
n

Q.16.(1) 5c2 × 8c2 × 7c2 = 5, 880


P5 = 9! / (9-5)!
9

Q.17.(3) When at most 2 women are included, the committee


P5 = 9! / 4!
9

many consist of
P5 = 15,120 
9

3 men & 2 women or Q.25.(3)


4 man 1 women
ten thou- thousand hundreds tens units place
or sand place place place place
5 men & 0 women
So, 4c2 × 6c3 + 4c1× 6c4 + 6c5
in 4P1 i.e. After flling up ten thousands place we are
120 + 60 + 6 = 186 ways. 4 ways left with 4 digits including 0 and the number
Q.18.(3) According to the condition the team can be selected (any one of blank places is 4. So, in 4P4 = 4! = 24 ways
in the following ways : of 2/4/6/8
1 goalkeeper and 4 defender and 6 forward Q.26.(2) Case (i) : When 0 occurs at units place :
or Hundreds tens place units place
1 goal keeper and 5 defender and 5 forward Place
6
P2 = 6×5 = 30 ways Only o, i.e. in 1
So, 2c1 × 5c4 × 9c6 + 2c1 × 5c5 × 9c5
ways

840 + 252
Total such numbers = 30 × 1 = 30
1092 ways
Case (ii) : When 0 does not occur at unit place :
Q.19.(3) nc2 = 153
After filling of After filling up any one of in 3
Let n be the number of team participating
unit place we units place and ways
n! are left with hundreds place
= 153
n! (n − 2)! 6 digit but 0 we are laft with

cannot occur at 5 digits ( includ-
On solving n = -17 and n = 18 hundred? place. ing O ; so in 5
n = 18 We are finally ways.
left with d digits,
Q.20.(1) 5 members can be selected from 10 players in so in P = 5 ways
= 10c5 ways Total of such numbers = 5 × 5 × 3 = 75
1 captain can be selected from 6 players in req no. = 30+75 = 105
= 6c1 ways Q.27.(2) Since delegates from two multinational companies
will sit together, so considering these two delegates
Total ways = 10c5 × 6c1 = 189 × 6 = 1134 ways
as one unit, there will be 13 + 1 = 14 delegates who
Q.21.(1) 6c1 × 9c6 +6c2 × 9c5 +6c3 × 9c4 = 4914 can be arranged in a circular table in 14! ways.
Q.22.(2) Here, the order of arrangement of digits does The two delegates from the MNCs can be
matter. arranged among themselves in 2! ways.
Using the product rule, the required no. of ways
n
Pr = n! / (n-r)!
= 14!×2!
n
Pr = 4! / (4-3)!
Q.28.(5) No. of non-relative friends = 12-7 = 5
4
P3 = 4! / 1!
He may invite 6 friends in following ways:
 4P3 = 4!
I : 4 relatives + 2 non-relatives Þ7C4 × 5C2
 4P3 = 24
II : 5 relatives+ 1 non-relativesÞ 7C5 × 5C1
142 QUANTITATIVE APTITUDE
www.mahendras.org
III : 6 relatives + 0 non-relativesÞ 7C6
= 3 ×15 + 3 × 6 + 1
Required number of ways = 45 + 18 + 1

7 5
= C4 × C2 + C5 × C1 + C6
7 5 7
= 64
= 35×10+21×5+7=350+105+7=462 Q.34.(3) There are 6 letters in the given word, out of which
Q.29.(4) Since first letter C and last letter Y are fixed. Therefore there are 3 vowels and 3 consonants. Let us mark
we have to permutate only six letters between C and Y, these positions as under :
C O U R T E S Y (1) (2) (3) (4) (5) (6)
Therefore, total number of words = 6! Now, 3 vowels can be placed at any of the three
= 720  places out 4, marked 1, 3, 5.
Q.30.(3) Arranging 8 balls in 8 places Number of ways of arranging the vowels
Total number of ways = 8!  = 3P3 = 3! = 6.
Therefore,  Also, the 3 consonants can be arranged at the
remaining 3 positions.
Number of ways in which 3
Number of ways of these arrangements
given balls are together= 6!.3!
= 3P3 = 3! = 6
Therefore,
Total number of ways = (6 × 6) = 36
Number of ways in which 3 given balls are not all
together = 8! - 6!.3! Q.35.(3) 'LOGRAITHMS' contains 10 different letters.

Q.31.(4) We may have (2 men and 3 women) or (4 men and Required number of words = Number of
1 women) or (5 men only) arrangements of 10 letters, taking 4 at a time.
= 10P4= (10 × 9 × 8 × 7) = 5040
Required number of ways
Q.36.(4) We are to choose 11 players including 1 wicket
= ( 7
C3 ×6 C2 +) ( 7
C4 ×6 C1 + ) ( 7
C5 ) keeper and 4 bowlers  or, 1 wicket keeper and 5
bowlers. 
 7× 6×5 6×5  Number of ways of selecting 1 wicket keeper, 4
= ×
 3× 2×1 2×1 
+ ( 7
C3 ×6 C1 + ) ( 7
C2 ) bowlers and 6 other players in 
2
C1 x 5C4 x 9C6 = 840 
 7 × 6 × 5   7 × 6 Number of ways of selecting 1 wicket keeper, 5
= 525 +  ×6 +
 3 × 2 × 1   2 × 1  bowlers and 5 other players in 2C1x5C5x9C5=252
Total number of ways of selecting the team = 840
= (525 + 210 +21)= 756. + 252 = 1092
Q.32.(3) Number of ways of selecting (3 consonants out of Q.37.(2) Hence, the total number of possible choices is 52C18
7) and (2 vowels out of 4) × 35C2 + 52C19 × 35C1 + 52C20
 7 × 6 × 5 4 × 3 Q.39.(4) Hence, the required number of ways in which the
( 7
C3 ×4 C2 ) = ×
 3 × 2 × 1 2 × 1 
= 210.
four vowels do not come together
9 4× 6
Each group, each having 3 consonants and 2 = 2 2 2− 2 2 2
vowels = 210.
= 45360 - 2160 = 43200
Each group contants 5 letters.
Q.40.(1) Hence, the total number of ways = 4! × 5!
Number of ways of arranging = 5!
= 24 × 120 = 2880
= 5×4 ×3 × 2 × 1 = 120
Q.41.(5) 4!×2! = 48
Required number of ways = (210 × 120)
Q.42.(2) The 5 letter word can be rearranged in 5!=120
= 25200 Ways without any of the letters repeating.
Q.33.(3) 2 W, 3 B, 4 R The first 24 of these words will start with A.

Required probability = 3C1 × 6C1 + 3C2 × 6C1 + 3C3 Then the 25th word will start will CA _ _ _. 

QUANTITATIVE APTITUDE 143


www.mahendras.org
The remaining 3 letters can be rearranged in 3!=6 Now the last one gift can be given to any of
Ways. the  8  GF hence the total number of ways of
i.e. 6 words exist that start with CA. distributing,

The next word starts with CH and then A, i.e., CHA =9×8!×8=9×8!×8=72 × 8! ways.
_ _.  Q.47.(4) case (i): As Mr. Y agrees to be in committee only
The first of the words will be CHAMS. where Mrs. Z is a member.

The next word will be CHASM. Now we are left with (6−1) men and(4−2) ladies
(Mrs. X is not willing to join).
Therefore, the rank of CHASM will be 24+6+2= 32
case (ii): If Mr. Y is not a member then we left
Q.43.(1) There are 7 letters in the word Bengali of these 3 with (6+4−1) people.
are vowels and 4 consonants.
So total ways
There are 4 odd places and 3 even places. 3
vowels can occupy 4 odd places in  4P3 ways and 7
C1 + 9C3 = 91
4 constants can be arranged in 4P4 ways. Q.48.(2) Step I:  Two male members can be selected
Number of words =4P3 x 4P4= 24 x 24 = 576 in 9C2=36
Q.44.(2) First we take books of a particular subject as one Step II:  Having selected two male members, 2
unit. Thus there are 4 units which can be arranged female members can be selected in 7C2=21 ways.
in 4! = 24 ways. Now in each of arrangements, Step III: Two male and two female members can
mathematics books can be arranged in 3! ways, arranged in a particular game in 2 ways.
history books in 4! ways, chemistry books in 3!
Total number of arrangements,36×21×2=1512 ways.
ways and biology books in 2! ways. Thus the total
number of ways = 4! × 3! × 4! × 3! × 2! = 41472 Q.49.(1) Total  number of balls = 6 + 2 + 4 + 3 = 15.
Q.45.(1) There are 4 bus routes from A to B and 3 routes Number of non-blue balls = 15 - 6 = 9
from B to C. Therefore, there are 4 × 3 = 12 ways Number of ways of selecting 5 non-blue balls
to go from A to C. It is round trip so the man will
travel back from C to A via B. It is restricted that = 9C5
man can not use same bus routes from C to B and Number of ways of selecting 5 balls out of 15 balls
B to A more than once. Thus, there are 2 × 3 = 6 = 15C5
routes for return journey. Therefore, the required Number of ways for selections such that atleast one
number of ways = 12 × 6 = 72. ball is blue.
Q.46.(4) As every girl friend should get one gift.
15c5 – 9c5 = 3003 − 126 = 2877
The number of ways  88  distinct gifts can be
selected is: 9C8= 9ways. Q.50.(1) For each combination of 4 out of 8 boxes,
The number of ways each GF gets one gift each the box with the greatest number has to be
out of these 8 selected gifts 8!. given to the first person, the box with the
second highest to the second person and so on.
Total number of ways 8 gifts can be distributed
The number of ways of giving 4 boxes to the 4
is 9×8!
person is: 8C4= 70

144 QUANTITATIVE APTITUDE


www.mahendras.org

CHAPTER

18 PROBABILITY

Some useful facts


If one dice is rolled randomly, the set of total possible 3
Probability of the even number =
outcomes are {1, 2, 3, 4, 5, 6} and the set of all 6
outcomes even number is 2,4,6. 1
P (E) =
From the above example we design some definitions 2
Results :
I. Experiment : An activity which when performed any
number of times under identical condition gives the (i) P (S) = 1...........(maximum probability is 1)
same (unique) outcome.
(ii) P (φ) = 0..........(minimum probability is 0)
Ex. When we measure the three angles of any triangle we
always get their sum as 1800. (iii) 0 < P (E) < 1
II. Random Experiment : An experiment is called a (iv) P(E) + P ( E ) = 1
random experiment when it satisfies the following ie. P (E) = 1- P ( E ) ............Where E denotes not E
two conditions:
Ex. In a question there are 5-options in which one is right
(1) It has more than one possible outcome.
and remaining 4 are wrong, if one option is to be
(2) It is not possible to predict the outcome in advance. selected randomly, what is the probability that-
Ex. Tossing a coin; tossing a dice. (I) The question will be right
III. Sample space : The set of total possible outcomes
(II) The question will be wrong
in an random experiment is known as sample space,
denoted by S. 1
Sol. P (E) = .............(I)
- For rolling a dice S = {1, 2, 3, 4, 5, 6} 5
- For tossing a coin once, S = {H, T} 4
P (E) = ..................(II)
5
- If two coins are tossed, then
Adding these equation (I) & (II), we get-
S = {HH, HT, TH, TT}
IV. Event : Any subset of a sample space is called an

bg ej
P E +P E =
1 4
+
5 5
event, denoted by E.
In the above, E = {2, 4, 6} is an event and E ⊆ S bg ej
i.e. P E + P E = 1 ..........(Remember that)
V. Probability of occurrence of an event(E) : Ex. Find the probability of getting head when single coin

Probability of any event(E) =


b g i.e.
nE is tossed.
nbSg Sol. For single coin
nbEg
PbEg = n(E) = 1 {H}
nbSg
n(S) = 2 {H, T}
Where n (E) ⇒ Total number of favourable outcomes
n (S) ⇒ Total number of possible outcomes. 1
P(E) =
P (E) ⇒ Probability of Event. 2
Ex. Find the probability of getting 1 head when two coins
For rolling a dice are tossed simultaneously.

QUANTITATIVE APTITUDE 145


www.mahendras.org
Sol. For two coins Ex. Find the probability that sum is divisible by 4 when two
dice are thrown simultaneously.
n(S) = 4 {(H, H) (T, T) (H, T) (T, H)}
Sol. P(E) =
nE bg
n(E) = 2 {(H, T) (T, H)} nS bg
2 1
=
LM1,3 2,6 2,2 OP
P(E) =
4 2

MM 3,1 6,2 6,6 PP 9 1
5,3 4,4 , =
Ex. Find the probability of getting at least one head when
two coins are tossed simultaneously.
MN3,5 PQ 36 4

Sol. For two coins Ex. Find the probability that sum is a prime no. & less
than 8 when two dice are thrown simultaneously.
n(S) = 4 {(H, H) (T, T) (H, T) (T, H)}
1 + 2 + 4 + 6 13
Here, n(E) = 3 {(H, H) (H, T) (T, H)} Sol. P(E) = =
36 36

3 Note : Sum Ways No. of results


P(E) = 2 (1, 1) 1
4
Ex. Find the probability of getting at most one head when 3 (1,2) (2,1) 2
two coins are tossed simultaneously.
5 (3,2) (2,3) (1,4) (4,1) 4
Sol. n(S) = 4 {(H, H) (T, T) (H, T) (T, H)}
7 (1, 6) (6, 1) (4, 3) (3, 4)
n(E) = 3 {(T, T) (H, T) (T, H)} (2, 5) (5, 2) 6
Cards

3
P(E) = 4 DICE CARDS (52)
Ex. Find the probability of getting a multiple of 3 when
one dice is thrown once. Red (26) Black (26)

Sol. n(S) = 6 {1, 2, 3, 4, 5, 6}


Diamond Heart Club Spade
n(E) = 2 {3, 6} 13 13 13 13
2, 3, 4, 5, 6, 7, 8, 9, 10 – no. cards
2 1
P(E) = = → 9×4 = 36
6 3
Ace, King, Queen; Jack - Honour cards
Ex. Find the probability of that number which is a multiple
of 2 when one dice is thrown once. → 4×4 = 16
Sol. n(S)=6 {1, 2, 3, 4, 5, 6,} King, Queen; Jack - Face cards
n(E) = 3 {2, 4, 6} → 3×4 = 12
3 1 Ex. If from a pack of 52 cards, 1 card is drawn at random.
P(E) = = Find the probability that the card is an Ace card.
6 2
Ex. Find the probability that sum on both faces is 9 when Solution :
two dice are thrown simultaneously. n(E) 4
P(E) = = C1 4 1
Sol. 6 = 36
2 = =
n(S) 52
C1 52 13
n(S) = 36 Ex. If from a pack of 52 playing cards, 1 card is drawn at
LM6, 3OP random. What is the probability that it is either a king
or queen?
MM3, 6PP
MN5,4, 45PQ C1 + 4 C1
4
8 2
Sol. P(E) = 52
= =
C1 52 13
4 1
n (E) = 4, P(E) = =
36 9

146 QUANTITATIVE APTITUDE


www.mahendras.org
EXERCISE
Q.1-5. Study the given information carefully to answer the questions that follow.
An urn contains 4 green, 5 blue, 2 red and 3 yellow marbles.
Q.1. If four marbles are drawn at random, what is the probability that two are blue and two are red ?
10 9 17 2
(1) (2) (3) (4) (5) None of these
1001 14 364 7
Q.2. If eight marbles are drawn at random, what is the probability that there are equal number of marbles of each colour?
4 361 60 1
(1) (2) (3) (4) (5) None of these
7 728 1001 1
Q.3. If two marbles are drawn at random, what is the probability that both are red or at least one is red?
26 1 199 133
(1) (2) (3) (4) (5) None of these
91 7 364 191
Q.4. If three marbles are drawn at random, what is the probability that at least one is yellow ?
1 199 165 3
(1) (2) (3) (4) (5) None of these
3 364 364 11
Q.5. If three marbles are drawn at random, what is the probability that none is green ?
2 253 10 14
(1) (2) (3) (4) (5) None of these
7 728 21 91
Q.6. A person teaches to three children. What is the probality that no one among them will be a girl.
1 1 1 1
(1) (2) (3) (4) (5) 0
2 16 3 8
Q.7. The letter I, G, L, N & C are arranged to form the word CLING. What is the probability?
1 1 1 3
(1) (2) (3) (4) (5) None of these
120 64 36 5
Q.8. A bag contains 10 white balls and 16 black balls. Two balls are drawn in succession without replacement. What is
the probability that first is white and second is black?
18 17 35 37
(1) (2) (3) (4) (5) None of these
145 29 134 145
Q.9. Four boys and three girls sit in a row for interview. The probability that they will sit in alternate position is-
1 1 1 1 1
(1) (2) (3) (4) (5)
34
23 35 15 65
Q.10. What is the probability that number selected from number 1,2,3......100 is a prime number, when each number is
equally likely to be related?
9 8 11 1 10
(1) 30 (2) 35 (3) 30 (4) 4 (5)
30
Q.11. A bag contains 6 black, 5 white and 8 yellow balls. If four balls are picked at random, what is the probability that
all four of them are either black or any two out of the four are white?
3 925 359 11
(1) (2) (3) (4) (5) None of these
1292 3876 1927 387
1 1 3
Q.12. Ramesh play 3 sets of a badminton match. The probability of his winning the three sets are , , respectively.
7 5 4
What is probability that he will win atleast 1 set of the game?
1 7 13 23
(1) (2) (3) (4) (5) None of these
24 24 24 24
Q.13. What is probability of a head and even number when a coin is tossed and a dice is thrown?
1 1 1 1
(1) (2) (3) (4) (5) None of these
2 3 4 5
QUANTITATIVE APTITUDE 147
www.mahendras.org
Q.14. What is probability of getting a sum greater than 15 if three dice are rolled simultaneously?
(1) 5/54 (2) 15/216 (3) 11/216 (4) 17/216 (5) None of these
Q.15. The probability of sucess of A, B, C in an interview is 1/5, 1/4, 1/3 respectively. Find the probability of sucess of
atleast two?
(1) 1/6 (2) 3/5 (3) 3/4 (4) 2/5 (5) None of these
Q.16. Two team x and y are participating in quiz contest. The probability that team x will win is 5/8 and the probability
that team y will win is 1/5. What is the probability that either team x or team y will win the contest?
(1) 3/5 (2) 5/7 (3) 6/7 (4) 33/40 (5) 21/30
Q.17. A 5 digit number is formed by the digits 1, 2, 3, 4 and 5 without repitition. What is the probability that the number
formed is a multiple of 4?
(1) 1/2 (2) 1/3 (3) 1/4 (4) 1/5 (5) 1/6
Q.18. Two unbiased coins are tossed. What is the probability of getting at most one tail?
(1) 1/2 (2) 1/3 (3) 2/3 (4) 3/4 (5) 1/4
Q.19. Three unbiased coins are tossed, What is the probability of getting atleast 2 tail?
(1) 1/2 (2) 1/3 (3) 1/6 (4) 1/8 (5) 1/10
Q.20-22. There are 3 bags containing 3 colored balls – Red, Green and Yellow.
Bag 1 contains: 24 green balls. Red balls are 4 more than yellow balls. Probability of selecting 1 red ball is 4/13.
Bag 2 contains: Total balls are 8 more than 7/13 of balls in bag 1. Probability of selecting 1 red ball is 1/3. The ratio
of green balls to yellow balls is 1 : 2
Bag 3 contains: Red balls is equal to total number of green and yellow balls in bag 2. Green balls is equal to total
number of green and red balls in bag 2. Probability of selecting 1 yellow ball is 3/14.
Q.20. 1 ball each is chosen from bag 1 and bag 2, What is the probability that 1 is red and other is yellow?
(1) 15/128 (2) 21/115 (3) 17/135 (4) 25/117 (5) 16/109
Q.21. Some green balls are transferred from bag 1 to bag 3. Now probability of choosing a yellow ball from bag 3 becomes
3/16. Find the number of remaining balls in bag 1.
(1) 60 (2) 58 (3) 5 (4) 48 (5) 44
Q.22. Green balls in ratio 4: 1 from bags 1 and 3 respectively are transferred to bag 4. Also 4 and 8 red balls from bags 1
and 3 respectively. Now probability of choosing green balls from bag 4 is 5/11. Find the number of green balls in
bag 4?
(1) 12 (2) 15 (3) 10 (4) 9 (5) 11
Q.23-24. There are 3 people – A, B and C. Probability that A speaks truth is 3/10, probability that B speaks truth is 3/7
and probability that C speaks truth is 5/6. For a particular question asked, at most 2 people speak truth. All people
answer to a particular question asked.
Q.23. What is the probability that B will speak truth for a particular question asked?
(1) 7/1 (2) 14/33 (3) 4/15 (4) 9/28 (5) 10/33
Q.24. A speaks truth only when B does not speak truth, then what is the probability that C does not speak truth on a ques-
tion?
(1) 11/140 (2) 21/180 (3) 22/170 (4) 13/140 (5) None of these
Q.25. Four persons are chosen at random from a group of 3 men, 3 women and 4 children. What is the probability that
exactly 2 of them will be men?
(1) 1/9 (2) 3/10 (3) 4/15 (4) 1/10 (5) 5/12
Q.26-29. Rahul and Saif appear for an interview for two vacancies. The probability of Rahul's selection is 1/3 and that of
Saif's selection is 1/5 .

148 QUANTITATIVE APTITUDE


www.mahendras.org
Q.26. Find the probability that both of them will be selected.
(1) 1/15 (2) 8/15 (3) 8/9 (4) 7/8 (5) None of these
Q.27. Find the probability that none of them is selected.
(1) 1/15 (2) 8/15 (3) 8/9 (4) 7/8 (5) None of these
Q.28. Find the probability that at least one of them is selected.
(1) 1/15 (2) 8/15 (3) 8/9 (4) 7/15 (5) None of these
Q.29. Find the probability that only one of them is selected.
(1) 1/15 (2) 2/5 (3) 8/9 (4) 7/8 (5) None of these
Q.30. A committee of five persons is to be chosen from a group of 10 people. The probability that a certain married couple
will serve together is?
(1) 54/199 (2) 52/195 (3) 53/186 (4) 51/126 (5) None of these
Q.31. A man can hit a target once in 4 shots. If he fires 4 shots in succession, what is the probability that he will hit his
target?
(1) 1 (2) 1/256 (3) 81/256 (4) 175/256 (5) None of these
Q.32. An anti-aircraft gun can fire four shots at a time. If the probabilities of the first, second, third and the last shot hitting
the enemy aircraft are 0.7, 0.6, 0.5 and 0.4, what is the probability that four shots aimed at an enemy aircraft will
bring the aircraft down?
(1) 0.84 (2) 0.916 (3) 0.036 (4) 0.964 (5) None of these
Q.33-37.In a bilateral cricket series between India and Australia, the probability that India wins the first game is 0.4. If India
wins any game, the probability that it wins the next game is 0.3; otherwise the probability is 0.2.
Q.33. Find the probability that India wins the first two games.
(1) 0.08 (2) 0.32 (3) 0.18 (4) 0.12 (5) None of these
Q.34. Find the probability that India wins at least one of the first two games.
(1) 0.48 (2) 0.32 (3) 0.56 (4) 0.52 (5) 0.58
Q.35. Find the probability that India wins the first three games.
(1) 0.028 (2) 0.030 (3) 0.032 (4) 0.036 (5) 0.044
Q.36. Find the probability that India wins exactly one of the first three matches.
(1) 0.416 (2) 0.396 (3) 0.096 (4) 0.404 (5) 0.214
Q.37. Find the probability that India wins exactly one of the first two games.
(1) 0.20 (2) 0.40 (3) 0.44 (4) 0.36 (5) 0.28
Q.38. A number is to be chosen from the number - 4, - 3, -2, 0, 1, 2, 3, 4. What is the probability that the number is less
than 3?
5 3 3 1 3
(1) (2) (3) (4) (5)
7 7 4 3 5
Q.39. Two dice are rolled, find the probability that the sum is less than 13.
1
(1) (2) 1 (3) .75 (4) 1.25 (5) 0
2
Q.40. Two dice are tossed. The probability that the total score is a odd number-
1 5 1 7
(1) (2) (3) (4) (5) None of these
6 12 2 9

QUANTITATIVE APTITUDE 149


www.mahendras.org
EXERCISE Explanation
5
C2 × 2 C2 10 6 29
Q.1.(1) Required probability = = Probability of winning atleast one set= 1- =
14
C4 1001 35 35
Q.13.(3) Probability of getting a head = 1/2
4
C2 × C2 × C2 × C2
2 3 5
When a coin is tossed, Probability of getting a even
Q.2.(3) Required probability = 14
C8
number when dice is tossed = 1/2
180 60
= =
3003 1001 1 1 1

Q.3.(1) Required probability =


2
C2 L
+ M1 −
12
C2 OP Required probability = × =
2 2 4
14
C2 MN 14
C2 PQ Q.14.(1) Sum of 18 - {6, 6, 6} - 1 way
Sum of 17 - {6, 6, 5} - 3 way
1  66  1 25 26
== + 1 −  = + = Sum of 16 - {6, 6, 4}{6,5,5} - 3+3 = 6 way
91  91  91 91 91
11
Sum of 15-{6, 6, 3}{6,5,4}{5,5,5}-3+6+1= 10 way
C3
165 199
Q.4.(2) Required probability =1- 14 = 1- = Total way = 20 ways
C3 364 364
Required probability = 20/216 = 5/54
10
C3 30 Q.15.(1) Required probability
Q.5.(5) Required probability = =
14
C3 91
 4 1 1  1 3 1  1 1 2   1 1 1
5 × 4 × 3 + 5 × 4 × 3 + 5 × 4 × 3 +  4 × 3 × 5
Q.6.(4) Probability of the child of not a girl = 1/2        

1 1 1 1 4 3 2 1 10 1
Required probability = × × = = 60 + 60 + 60 + 60 = =
2 2 2 8 60 6
Q.7.(1) Favourable case = 1
Q.16.(4) Winning probability of team x = 5/8
Total case = 5!
winning probability of team y = 1/5
1 1
Probability = = required probability = 5/8 + 1/5 = 33/40
5! 120
Q.17.(4) Favourable event : To be a multiple of 4, the last
10 16
C C1 2 digits of the number have to be divisible by 4 is
Q.8.(5) Required probability = 26 1 ×
C1 25
C1 they must be 12, 24, 32 or 52. and semaining place
can be filled in 3! ways
10 16 16 So total favourable event = 4 × 3!
= × =
26 25 65 Total event = 5!
4!× 3!
Q.9.(2) Required probability = 4 × 3! 1
7! Required Probability = =
5! 5
4 × 3 × 2 × 3 × 2 ×1 1 Q.18.(4) Favourable cases = [HH,TH,HT] = 3
= =
7 × 6 × 5 × 4 × 3 × 2 × 1 35 Total cases = [HH,TT,TH,HT] = 4
Q.10.(4) Favourable cases = 2,3,5,7,11,13,17, 19,23,29,31, Required probability = 3/4
37,41,43,47,53,59,61,67,71,73,79, 83,89,97.
Q.19.(1) Favourable cases = [TTH,THT,HTT,TTT] = 4
Total case = 100, Required probability = 25/100 = 1/4
Total cases=[TTT,HHH,THT,HTT,HTH,HHT] = 8
6
C4 + 5 C2 × 14 C2
Q.11.(2) Required probability = 19 Required probability = 4/8 = 1/2
C4
Q.20-22.
6 × 5 × 4 × 3 5 × 4 14 × 13 19 × 18 × 17 × 16 925
= + × / = Red Green Yellow Total
1× 2 × 3 × 4 2 2 ×1 4 × 3 × 2 ×1 3876
Bag-1 16 24 12 52
Q.12.(5) Probability of not winning all the three sets Bag-2 12 8 16 36
 1  1  3 6 4 1 6 Bag-3 24 20 12 56
=  1 − 7  ×  1 − 5  ×  1 − 4  = × × =
7 5 4 35
150 QUANTITATIVE APTITUDE
www.mahendras.org
Q.20.(4)  16 16   12 12  Q.28.(4) 8 7
 52 × 36  +  52 + 36  1− =
    15 15
16 1 25 1 2 4 1 2
+ = Q.29.(2) × + × =
117 13 117 5 3 5 3 5
Q.21.(5) 12 3 , x=8
= Q.30.(4) Five persons is to be chosen from a group of 10
56 + x 16 people = 10C5 = 252
Couple Serve together = 8C3 x 2C2 = 56
Number of balls in bag 1 = 52-8 = 44
Probability = 102/252 = 51/126
Q.22.(3) Let number of green balls be 4 k and k
Q.31.(4) (1−81/256) = 175/256
5k 5
= Q.32.(4) The probability that none of the four shots hit the
5 k + 12 11
aircraft is given by: 
12
k= =2 (1−0.7)(1−0.6)(1−0.(5) (1−0.
6
(4)=0.3×0.4×0.5×0.6=0.036
Number of green balls = 5×2 =10 So, the probability that at least one of the four
Q.23.(4) In any case B speaks truth. Now at most 2 people hits the aircraft: =1–0.036=0.964
speak truth for 1 question Q.33.(4) P(Win first game)× P(Win second game)
So case 1: B and A speaks truth Probability = 0.4×0.3=0.12
3 3  5 3 Q.34.(4) P(won at least 1 game)= 1- P(won no games)
= × × 1 −  =
7 10  6  140 =1- [(1-0.(4) × (1-0.(2) ] (0.(2) = 0.52
Case 2: B and C speaks truth Probability Q.35.(4) (4) 0.4× 0.3 × 0.3= 0.036
3  3 5 5 Q.36.(4) Case1- India wins first game and loses second and third
= × 1 −  × =
7  10  6 20 Case 2= Lose + Win + Lose
Case 3: Only B speaks truth Probability Case 3= Lose + Lose+ Win P
3  3   5 1 = P1+P2+P3 = 0.404
= × 1 −  × 1 −  =
7  10   6  120
Q.37.(2) Case 1= Won first × Lost Second

3 1 5 9 = 0.4 × (1-0.(3) = 0.4 × 0.7=0.28


Add the three cases = + + =
140 20 20 28 Case 2= Lost First × Won second
Q.24.(1) Case 1: B does not speak truth, A speaks truth So A = (1-0.(4) × 0.2 = 0.6×0.2=0.12
speaks truth here Probability that C does not speak P= 0.28+0.12=0.40
truth = (3/10) × (1 – 3/7) × ( 1- 5/6) = 1/35 Q.38.(3) Favourable case = 2,1,0,-2,-3,-4
Case 2: B speaks truth So A does not speak truth Total cases = -4,-3,-2,-1,0,1,2,3,4
here Probability that C does not speak truth
6 3
= ( 1- 3/10) × 3/7 × ( 1- 5/6) = 1/20 Probability ==
8 4
So total = 1/35 + 1/20 = 11/140 Q.39.(2) Favourable case = 36
Q.25.(2) 2 men means other 2 woman and children So prob. Total case = 36
C2 ×7 C2 3
3
36
= 10
= Probability = =1
C4 10 36
Q.26.(1) Q.40.(3) Favourable case = (1,2) (1,4) (1,6) (4,1) (4,3) (4,5)
1 1 1 (2,1) (2,3) (2,5) (5,2) (5,4) (5,6) (3,2) (3,4) (3,6)
× = (6,1) (6,3) (6,5)
3 5 15
Total cases = 36
4 2 8 18 1
Q.27.(2) × =
5 3 15 Probability = =
36 2
QUANTITATIVE APTITUDE 151
www.mahendras.org

CHAPTER

19 DATA
SUFFICIENCY
I. Sum of their digits is 10.
Introduction
II. The difference between the number and the
Data sufficiency has recently become a favourite topic
number obtained by interchanging the po­sition
for many of the recent examinations. In this type of
of the digits is 36.
questions, usually a question is given followed by two or
three statements. These two or three statements contain Sol. Let the two-digit number be 10x+y
data or some pieces of information, using which, the
Then, y-x = 4..........(i)
question can possibly be solved. You are required to
judge whether the data given is sufficient to answer the I. x + y = 10 .........(ii)
question or not.
Solving (i) with (ii), we get
Format of Study : Data sufficiency questions are not new
x = 3, y = 7
topics in themselves. They may be covering any of the topics
already covered; for example : percentage, time and work, Number = 10 × 3 + 7 = 37
algebra, time and distance profit and loss, S.I., C.I., average II. |10x+y-10y-x| = 36
etc. Hence you should treat these questions as old-type only.
Only these questions are asked in a different pattern and not or, |9x - 9y| = 36 or, |x-y| = 4
the conventional pattern. we wouldn’t get the value of x and y.
Helping Hands: When you are attempting a question of data Therefore, only statement I alone is sufficient to
sufficiency you should follow a systematic approach as laid answer the question.
down below. This approach being a systematic one, will save
your time. Also in case you are stuck up at any point, it will Ex. What is the value of m - n ÷ 37 ?
help your chances of guessing a correct answer because it I. m is the largest possible six-digit number and n
narrows down the possible answers from 5 to 3 or 2.
is the smallest possible six-digit number.
To understand this approach let us first look at the way in
which such questions are usually asked : (1) Two statements II. The difference between m and n is known.
D.S. (2) Three statements D.S. Sol. I. m = 999999, n = 100000 ∴ ? = m - n ÷ 37
Direction : The questions given below contain two = 999999 - 100000 ÷ 37
statements giving certain data. You have to decide whether
the data given in the statements are sufficient for answering = 999999 - 2702.70 = 997296.30
the question ? Mark answer- II. m - n = known, but neither the value of ‘m’
(1) If statement I alone is sufficient but statement II is known nor the value of ‘n’ is known. So,
alone is not sufficient. we cannot find the value of m-n ÷ 37 by this
statement.
(2) If statement II alone is sufficient but statement
I alone is not sufficient. Ex. What is the average daily wages of a worker who
works five days if he made Rs. 80 for the first
(3) If each statement alone (either I or II) is day?
sufficient.
I. The worker made a total of Rs. 400 for the first
(4) If statement I and II together are not sufficient. four days of work.
(5) If both statement together are sufficient, but II. The worker made 20% more each day than he
neither statement alone is sufficient. did on the previous day.
Ex. In a two-digit number, the digit at unit's place is 4 more Sol. Only II statement is sufficient.
than the digit at tens place.Find the two digit number.
152 QUANTITATIVE APTITUDE
www.mahendras.org
Ex. What is the difference between the present ages of Ex. A train crosses another train running in the op­posite
mother and her daughter? direction in x seconds. What is the speed of the train?
I. Ratio of the age of the daughter eight years hence I. Both the trains are running at the same speed.
to the present age of the mother is 3 :4.
II. The first train is y cm long.
II. Ratio of the present age of the daughter to that
Sol. The length of the other train is not given in any of the
of the mother is 11 : 20.
statements.
Sol. Statements I and II will give us the separate equations
Both the statements are not sufficient.
for mother and daughter. Therefore, both statements
together are sufficient. Three Statement Data Sufficiency
Ex. What is the population of state A? Direction.The questions given below contain three
I. After an increase in the population of state A by statements giving certain data. You have to decide whether
12% it becomes 627.20 lakhs. the data given in the statements are sufficient for answering
the question ?
II. Ratio of population of state A to that of state B is 4:5.
Ex. What is the average of three numbers?
Sol. I. Population of State
I. The largest number is 20 more than the smallest
627.20 number.
A= × 100 = 560 lakhs
112
.
II. The sum of the largest and the smallest number
II. Ratio never determine value therefore only I
is twice the middle number.
statement is sufficient.
III. The difference between first two numbers is 10.
Ex. What is the rate of simple interest per annum?
I. The sum trebles in 20 years at simple interest. (1) Only I and III

II. The difference between the sum and the simple (2) Only II and III
interest earned after 10 years is Rs. 1000. (3) Any two

Sol. I. b g
R = 3 −1 ×
100
20
= 10%
(4) All I, II and III
(5) Answer can not be given even using all the
II. Here the sum is not given.
three statements.
Therefore, statement I alone is sufficient.
Sol. Answer can not be given even using all the three
Ex. What is the speed of a running train which takes 6 statements.
seconds to cross a signal post?
Ex. What will be the sum of ages of mother and daughter
I. The length of the train is 90 m. after 6 years?
II. The train takes 18 seconds to cross a platform180 I. Mother's present age is 24 years more than the
m long. daughter's present age.
90 18 II. 4 years ago, the ratio of ages of mother and
Sol. I. Speed of the train = × = 54 km/hr
6 5 daughter was 5:1.
II. Quicker Method : Length of train III. Five times the daughter's present age is 16 years
more than the mother's present age.
Length of platform
= × Time
diff. in time (1) Any two (2) Only I and either II or III
taken to cross a signal post (3) Only I and II (4) Only II and III
180 (5) None of these
= × 6 = 90m
18 − 6
Sol. From statement (I) and (II),
90 18
Speed of the train = × = 54 km/hr. M = D + 24 _________(i)
6 5
Either I or II is sufficient. M = 5 D ___________(ii)

QUANTITATIVE APTITUDE 153


www.mahendras.org
From equation (i) and (ii), III. The train crosses a pole in 14.4 seconds.
D = 10 years (1) All I, II and III
M = 34 years (2) Any two
From statement (I) and (III), (3) Only II and either I or III
M = D+24 __________(iii) (4) Only II and III
5D = M + 16 _________(iv) (5) None of these
From equation (iii) and (iv), Sol. From statement (I) and (II),
D = 10 years 5 25
Relative speed = 25+5 = 30 × = m./sec.
M = 34 years 18 3
25
Therefore, only I and either II or III is sufficient. Distance = ×12 = 100 meter
3
Ex. What is the length of the platform? Length of train = 100 m.
I. A train running at the speed of 25 km./hr. crosses Length of train + length of platform
a platform in 18 seconds.
25 × 5
II. The train crosses a man moving in opposite = × 18
18
direction at the speed of 5 km./hr. in 12 seconds.
Length of platform = 25 m.

Notes

154 QUANTITATIVE APTITUDE


www.mahendras.org
EXERCISE
Q.1-15. The questions given below contain two statements giving certain data. You have to decide whether the data given
in the statements are sufficient for answering the question ? Mark answer-
(1) If statement I alone is sufficient but statement II alone is not sufficient.
(2) If statement II alone is sufficient but statement I alone is not sufficient.
(3) If each statement alone (either I or II) is sufficient.
(4) If statement I and II together are not sufficient.
(5) If both statements I and II together are sufficient, but neither statement alone is sufficient.
Q.1. Is 12 is one of the number in 13 consecutive even numbers.
Statement I. Difference between first and least numbers is 24.
Statement II. Average of the set is zero.
Q.2. In a school, 35% girls and 19% boys participated in singing. Number of students in the school is 700. How many
boys are there in the school?
Statement I. In school number of boys are 200 more than number of girls
Statement II. In singing 45 more boys participated than girls
Q.3. Is a>0?
Statement I. (a+b)² < (a-b)²
Statement II. 2a+2b < 2a-2b
Q.4. What is the rate of interest per annum?
Statement I. A sum becomes double to itself at SI in 4 years
Statement II. The difference between CI and SI on an amount of Rs. 10,000 in 2 years is Rs.625
Q.5. What is the circumference of a circle?
Statement I. Area of circle is half of the area of square.
Statement II. Ratio of the perimeters of square and circle is 2:1
Q.6. What is the present age of Vijay?
Statement I. Present age of Vijay and his wife are in the ratio of 13:11
Statement II. After 5 years his age is 6 years more than his wife.
Q.7. The average weight of A, B, C, D and E is?
Statement I. Average weight of A, B and C is 30 kg.
Statement II. Average weight of C, D and E is 40 kg.
Q.8. Is c >0?
Statement I. (b² -4ac) > 0
Statement II. 4ac <0
Q.9. A car dealer sold 80% of the cars in its inventory during September. What was the total revenue from the sales of cars?
Statement I. All but 60 cars in the showroom inventory were sold during September.
Statement II. Cars were sold for an average price of Rs.4.5 lakh during September.
Q.10. Did Incumbent P get over 60% of the vote?
Statement I. Challenger M got 39% of the vote
Statement II. Incumbent P got 40000 of the total votes cast
Q.11. How much money did X invest?

QUANTITATIVE APTITUDE 155


www.mahendras.org
Statement I. An increase in the Simple Interest from 39/8 % to 41/8 % per annum increases his yearly income
by 25%.
Statement II. The sum invested gets doubled, when invested at 8% per annum for 25/2 year.
Q.12. For a certain bottle and cork, what is the price of the cork?
Statement I. The combined price of the bottle and the cork is Rs.95
Statement II. The price of the bottle is Rs.75 more than the price of the cork.
Q.13. A man sales mobile sets as profit of 20%. How much total amount he gains in profit?:
Statement I. He sales 20 mobile sets.
Statement II. He sales each mobile set on Rs. 18000
Q.14. What is the ratio of investments of P, Q and R ?
Statement I. Total investments is Rs.12000 in which P’s contribution is Rs.4500.
Statement II. Ratio of investments of P to Q is 4 :3 and Q to R is 3:1.
Q.15. What is Ravi’s present age ?
Statement I. Ravi is three years older than Sita.
Statement II. The ratio between Sita and Renuka’s age is 3:4.
Q.16-28. Which information are necessary to answer the questions asked-
Q.16. The following information is given about a four sided polygon. Which of the above facts are sufficient to deter-
mine the dimensions of the polygon?
I. The polygon is a rectangle
II. The area of the polygon is given to be 100 m2.
III. One side of the polygon is 8 m.
IV. All the adjacent sides are at right angles to each other.
(1) Any two (2) All together are not sufficient
(3) II, III and (I or IV) (4) (I or IV) and (II or III)
(5) None of these
Q.17. What is the length of the train X?
I. Two trains X and Y running in opposite directions and cross each other in 25 seconds.
II. The length of the train Y is 320m and the difference between the speeds of trains X and Y is 14 km/h.
III. The ratio of the speed of the trains is 19 : 26.
(1) Only II and III (2) Only I and III (3) Any two
(4) All three together are not sufficient (5) None of these
Q.18. Find the two digit number.
I. The product of the two digits of the two digit number is 20.
II. The difference between the two digits of the two digit number is 1.
III. The sum of the two digits of the two digit number is 9.
(1) None of these (2) II and III together are sufficient
(3) III and (I or II are sufficient) (4) All three together are sufficient
(5) All three together are not sufficient
Q.19. How much is monthly profit of a company?
I. Company decided to give bonus with the salary to its employees. Bonus is 12% of the monthly profit of the
company.
156 QUANTITATIVE APTITUDE
www.mahendras.org
II. On the day of salary distribution, company changed its mind and gave a total of Rs.24000 as bonus which
was 125% of what the company had decided earlier.
III. Company gives an amount of Rs.15000 as bonus which is 3/4th of total monthly profit.
(1) None of these (2) II and III together are sufficient
(3) III and (I or II are sufficient) (4) All three together are sufficient
(5) All three together are not sufficient
Q.20. Is integer x divisible by 6?
(I) x is the product of three consecutive positive integers
(II) x2 is divisible by 36
(III) x2 is divisible by 4
(1) I or II (2) II and III (3) None of these
(4) All three together are sufficient (5) All three together are not sufficient
Q.21. If a = 5, what is value of b + c?
(I) a is the average of the set {a, b, c}
(II) a is the median of the set {a, b, c}
(III) ba + bc = 5
(1) Only II (2) II and III (3) Only I
(4) All three together are sufficient (5) Any one
Q.22. What is the second number out of three numbers?
(I) Sum of the three numbers is 182
(II) The ratio of first and second number is 3 : 4 and second to third number is 2 : 3.
1
(III) Middle number is 33 % greater than the first number and lesser by the same percent from third number.
3

(1) Only II and III (2) Only II (3) Only III (4) Only I (5) None of these
Q.23. x, y and z together will finish the work in how many days?
(I) x does half as much work as y and y does half as much work as x and z does together in one day.
(II) y does the whole work in 40 days.
(III) x and z do the whole work in 20 days.
(1) Only I or II or III (2) Either I and II or I and III (3) Only II and III
(4) All are necessary (5) None of these
Q.24. In how many days 9 men and 15 women can the work be done ?
(I) 6 men and 5 women can complete the work in 6 days.
(II) 3 men and 4 women can complete the work in 10 days.
(III) 18 men and 15 women can complete the work in 2 days.
(1) Only I (2) Only II (3) Any two (4) All I, II and III (5) None of these
Q.25. Which is the smaller of two numbers?
I. The difference between these two numbers is one-fourth of the larger number.
II. The sum of these two numbers is 49.
III. The difference between these two numbers is 7.
(1) Only I and II (2) Only I (3) Either I or II and III (4) All (5) None of these
QUANTITATIVE APTITUDE 157
www.mahendras.org
Q.26. What is the ratio of the present ages of Rohan and his father?
I. Five years ago Rohan’s age was one-fifth of his father’s age.
II. Two years ago the sum of ages of Rohan and his father was 36 years.
III. The sum of the ages of Rohan, his mother and his father is 62 years.
(1) Only II and III (2) Only I and II (3) Only I and III (4) All (5) None of these
Q.27. What is the rate of interest per annum?
I. The amount becomes Rs. 11025 with compound interest after 2 years.
II. The same amount with simple interest becomes Rs. 11000 after two years.
III. The amount invested is Rs. 10000
(1) Any two (2) III and either I or II (3) I and either II or III
(4) II and either I or III (5) All
Q.28. In how much time will the train cover the distance between city ‘Y’ from city ‘X’?
I. The train crosses another train of equal length of 200 m running in opposite direction in 15
seconds.
II. The train leaves city ‘Y’ at 7.15 am for City ‘X’ situated at a distance of 560 km.
III. The 200 m long train crosses a signal pole in 10 seconds.
(1) Only II and III (2) Only I and III (3) Only I and II (4) All (5) None of these

EXERCISE Explanation
Q.1.(2) Average of the set is zero: in that case 12 must be Q.7.(4)
there in the set. Q.8.(4) If the data given in Both Statements I & II are not
Q.2.(1) 700= B+G---------(i) sufficient to answer the question Statement I
B - G = 200 ---------(ii) b² > 4ac
adding (i) and (ii) c could be either positive or negative
B = 450 Q.9.(5) Let total no of cars in September = x
Q.3.(5) If the data given in both statements I & II together From Statement I – 20% of x = 60 (Cars that are
are necessary to answer the question not sold)
From II b<0 x = 300
And from I ab<0 then a>0 80% 0f 300 =240
Q.4.(3) If the data either in Statement I alone or Statement From Statement II – Price of a car
II alone are sufficient to answer the question = Rs.4.5 lakh
R =25% can be determined from either of the From Statement I & II – The total revenue from
statements the sales of cars = 240 × 4.5 lakh = 1080 lakh
Q.5.(4) If the data given in Both Statements I & II are not Q.10.(1) If the data in statement I alone is sufficient to
sufficient to answer the question answer the question.
No values of radius and side can be find out from From Statement I, P got 61% of the vote which is
above statements over 60%.
Q.6.(5) If the data given in both statements I & II together Q.11.(1) If the data in statement I alone is sufficient to
are necessary to answer the question answer the question.
V/W = 13/11 Invested amount – x
V = w+5 From Statement I, Increased Rate of interest =
V=39. (41/8) – (39/8) = 2/8 =1/4

158 QUANTITATIVE APTITUDE


www.mahendras.org
x×(1/4)×(1/100) = 25 ⇒ x=10000 x>y=1
From Statement II, we can’t find x. From statement (3),
Q.12.(5) if the data in both the statements I and II together x+y=9
are necessary to answer the question.
Here its seems that answer can be find but on
From Statement II, Let the price of cork be x, then
the price of the bottle = (x+75) solving we get two 2 digit numbers that is 54 and
45. Hence all three together are not sufficient
From Statement I, combined price bottle and cork
= Rs.95 Q.19.(1) From statements (I) and (II),
x+(x+75) = 95 ⇒ 2x + 75 = 95 125% of 12% of profit = 24000
⇒ x = 10
From statement (III)
Q.13.(3) SP = 18000, Profit = Rs. 3000
3/4 of profit = 15000
CP = 15000
Hence statement III or (I and II) together are suf-
Total profit = Rs. 3000 × 20 = Rs. 60000
ficient.
Q.14.(2) Only II sufficient
Q.20.(1) From statement (I)
4:3:1=P:Q:R
Q.15.(4) Nither I nor II are sufficient Any product of three consecutive positive intigers
will be always divisible by 6.
From I and II we cannot find the exact Sita’s
present age From statement (II)

Q.16.(3) From the statement IV, it is clear that the poly- If x2 is divisible by 36 then x will be always divis-
gon is a rectangle. Thus, statements I and IV are ible by 6.
similar. We can not reach any certain conclusion from
To find the dimensions of the polygon, either I, statement (III)
II and III or II, III and IV are sufficient. Q.21.(3) From statement (I)
Q.17.(5) From statement (1), b+c
a=
x+y 2
V1+V2 = b+c =2a = 10
24
From statement (2), but from statement (II) and (III) we can not reach
V1 V2 = 14 any certain conclusion.
Q.22.(5)
From statement (3)
Q.23.(2)
V1 = 19
Q.24.(3)
V2 26
Q.25.(3)
Now we have 3 variables and 3 equations so all Q.26.(2)
3 statements together are sufficient. Q.27.(4)
Q.18.(5) From statement (1), Q.28.(1) From III, we can get the speed of train and in
statement II distance is given. Therefore we can
xy = 20 find the required time from II and III.
From statement (2)

QUANTITATIVE APTITUDE 159


www.mahendras.org

CHAPTER
DATA
20
INTERPRETATION
but, suppose another student got 64% marks out of
Data Interpretation
maximum marks 75. What is the marks obtained by
By interpretation of data we mean understanding, that student ?
organising and drawing appropriate conclusions from
the given data. In these days, Data Interpretation is an It is difficult when we find 64% of 75 but if we find
important aspect of almost every competitive examination. 3
( 75% = )
Usually, a table or a bar diagram or a pie-chart or a graph 4
is given and candidates are asked questions that test their 3
75% of 64 = × 64 = 3 × 16 = 48
ability to analyse the-data given in those forms. Through 4
these questions, examiner makes an attempt to check your We easily get the required answer.
ability to calculate faster and to comprehend complex and (4) % profit of a company x in various years are given
voluminous data. below in the form of line graph-
Unorganised and haphazard data does not make any sense 30
more so to top management for whom time is a very valuable
and rare commodity. Hence, any data, be it daily production 25 25%
figures, daily sales Figures, financial performance or
20
Profit Percent

20%
productivity, will have to be presented in a concise manner
at the same time being precise so that top management can 15
study it with least of effort and time thus also facilitating
10 12%
faster decision making,
10% 7%
Types of D.I. : 5
5%
(1) Tabulation (2) Bar Graphs 0
(3) Pie- charts (4) Line-Graphs 2011 2012 2013 2014 2015 2016

(5) Mixed Graph (6) Missing DI


Ex. In which year company x got maximum profit ?
(7) Puzzle DI
Sol. (According to the question, most of the candidate
Useful Tips : answered year 2016 (25%) but it is not necessary)
(1) To solve D.I., first we learn some useful topic; that 25% is maximum out of all given years then
percentage profit & loss, Ratio, Average, because the value of 25% is also maximum (it may be),
these topics are used in D.I. because (% P is always calculated on its expenditure,
(2) In, income- expenditure type of D.I. (expenditure is not given).
Expenditure is equivalent to C.P. Ans. CND
& Income is equivalent to S.P. Ex. In which year, company x got maximum % profit?

25 Sol. 2016 (25%)


(3) In percentage- 25% of 80 = × 80 = 20
100 Solved Examples :

it is equivalent to- 80% of 25 =


80
× 25 = 20
Table
100 A table is one of the simplest and most convenient means of
For example : Suppose a student got 80% out of summarising data and presenting them in a meaningful way,
maximum mark 75. What is the marks obtained by as all the numerical values are given directly and any errors
that student ? that may arise due to discrepancies in analysing or interpreting
4 4 the data expressed in graphical form are eliminated. Here,
80% of 75 = × 75 ...(∴ 80% = ) = 60
5 5 it should be borne in mind that data expressed in the tabular
160 QUANTITATIVE APTITUDE
www.mahendras.org
form may be expressed in most of the other graphical forms. =15+19+24+21+34+27 =140 (thousand)
So, we can express or summarise the voluminous data in any Number of tickets sold of movie B in Hyderabad
one of the forms as we wish. So, the ways of expression do =34 (thousand)
not matter. The various graphs can be drawn from the original
34
piece of data which is in the form of table. Required percentage= × 100 = 24
140
Q.1-5. Study the following table carefully and answer
the questions given below. Q.2.(5) Number of tickets sold of movie D in Kolkata
=21000
Number of tickets sold in a week of five movies
in the multiplexes in six different cities Number of tickets sold of movie B in Lucknow
(number in thousands) =27000

Movie® Required difference=27000-21000=6000


City¯
A B C D E Q.3.(4) Total number of tickets of movie C sold in all
Mumbai 20 15 35 26 18 cities=35+21+19+32+26+20=153 (thousand)
Delhi 17 19 21 25 28 153000
So, required average= = 25500
Kolkata 32 24 19 21 17 6
Q.4.(1) Number of tickets sold of movie E in Chennai
Chennai 18 21 32 28 34
Hyderabad 16 34 26 29 22 = 34 (thousand)
Lucknow 15 27 20 35 26 Number of tickets sold of movie A in Mumbai
= 20 (thousand)
Q.1. The number of tickets of movie B sold in 34
Hyderabad was approximately, what per cent of required percentage = × 100 = 170
20
the total number of tickets of the same movie sold
Q.5.(1) Total number of tickets sold in Mumbai
in all the cities together?
= 114 (thousand)
(1) 15 % (2) 18 % (3) 12 %
(4) 20 % (5) 24 % Total number of tickets sold in Delhi=110 (thousand)
Q.2. What is the difference between the number of Total number of tickets sold in Kolkata
tickets of movie D sold in Kolkata and the number = 113 (thousand)
of tickets of movie B sold in Lucknow?
Total number of tickets sold in Chennai
(1) 7500 (2) 7000 (3)14000
(4) 9000 (5) None of the above = 133 (thousand)

Q.3. What is the average number of tickets of movie C Total number of tickets sold in Hyderabad
sold in all the six cities? = 127 (thousand)
(1) 15500 (2) 2550 (3) 24000 Total number of tickets sold in Lucknow=123
(4) 25500 (5) None of the above (thousand). Minimum tickets sold in Delhi.
Q.4. The number of tickets of movie E sold in Chennai Bar graph
is what per cent of number of tickets movies A sold
Bar graph is probably the most widely used method of
in Mumbai?
diagrammatic representation of data. A bar chart is a graph
(1) 170 (2) 70 (3) 30 that consists of a number of rectangles (called bars) whose
(4) 130 (5) None of the above length or height varies with the magnitude repsresented but all
Q.5. In which city was the total number of tickets of all bars are of equal width. The bars may be arranged vertically
the five movies together sold the minimum? or horizontally.
(1) Delhi (2) Chennai (3) Lucknow Q.1-3. Read the following graph carefully and answer the
(4) Kolkata (5) None of these questions given below:-

Explanation : Q.1-5. Profit earned by three companies in different years


(in crore Rs.)
Q.1.(5) Total number of tickets of movie B sold in all the
cities together

QUANTITATIVE APTITUDE 161


www.mahendras.org
[Since the angle at the centre of the circle is 360°,
the total magnitude of the various components is
taken to be equal to 360°]
Total magnitude of the various components is equal
to 100%. Therefore, 100% = 360°
o o
 360   18 
1% =  = 3.6o =  
 100   5
Hence, the percentage of the component parts can
be converted into degrees by multiplying each of
them by (3.6)°.
Percentage Part of Total Value
Q.1. In which of the following years was the difference
=  
Central Angle
× 100 %
between the profit earned by company B and  360 o

company A maximum?
(1) 2006 (2) 2004 (3) 2008 Central angle
(4) 2005 (5) None of these o
 Percentage Part of Total Value 
Q.2. What was the percent increase in the profit earned =  × 360
 100 
by company C from 2006 to 2007 ?
Q.1-5. Study the information carefully and answer the
(1) 40 (2) 45 (3) 56 questions that follow:
(4) 50 (5) None of these
The following pie-chart shows the percentage of
Q.3. In which of the following years was the total profit people in Lucknow who are interested to buy
earned by all the three companies minimum? different company mobiles.
(1) 2004 (2) 2003 (3) 2006 Total Number of peoples = 65000
(4) 2005 (5) None of these
Explanation : Q.1-3.
Redmi Vivo
Q.1.(1) In 2006 20% 21%

150
Q.2.(4) Percentage increase = × 100 = 50% Motorola
300 12%
Oppo
14.50%
Q.3.(2) In 2003
Apple I
Pie-Chart phone, 3%
Samsung
30%
Pie-Charts (also called Circle-Charts) are used
to represent the relative sizes of component in Q.1. What is the difference between the number of
an aggregate. Pie-Chart is a circle broken down people interested to buy Samsung and the total
into component sectors. Therefore, in pie-chart number of people interested to buy Oppo, Motorola
different data are represented by sectors. Pie- and Apple i Phone together?
Chart is generally used on a percentage basis (1) 750 (2) 650 (3) 605
and not on an absolute basis. Different sectors of (4) 625 (5) 675
a Pie-Chart represent various component parts.
Q.2. The number of people interested to buy Apple
Each component value is expressed either as a
i Phone is what percentage of the number of
percentage of respective total or as the central
people interested to buy Motorola? (Calculate
angle of the respective total.
approximate percentage)
o
 Value of the sector  (1) 12% (2) 30% (3) 37%
Central angle =  × 360o 
total Value  (4) 21% (5) 40%
Q.3. What is the ratio of people interested to buy Apple
Central Angle i Phone to the number of people interested to buy
Value of the Sector = × Total Value
360o Oppo?
162 QUANTITATIVE APTITUDE
www.mahendras.org
(1) 5 : 29 (2) 17 : 5 (3) 6 : 22
X Y
(4) 7 : 29 (5) 29 : 5
180
Q.4. The number of people interested to buy Redmi 160 159
is approximately what per cent of the number of 140 139 141
148
128
people interested to buy Oppo? 120 119 120 120
100 107
100
(1) 148% (2) 118% (3) 130% 99
80 78
(4) 125% (5) 138% 60
Q.5. The number of people interested to buy Motorola 40

is approximately what per cent of the people 20


0
interested to buy Vivo, Redmi and Oppo together? 1997 1998 1999 2000 2001 2002
(1) 14% (2) 32% (3) 22% YEAR

(4) 10% (5) 28%


Q.1. What is the difference between the two companies
Explanation : Q.1-5. in the given years ?
65000 (1) 16000 (2) 26000 (3) 28000
Q.1.(2) Required difference= × (30 − 29) = 650 (4) 30000 (5) None of these
100
Q.2.(4) Number of people interested to buy Apple i Phone Q.2. What is the difference between the number of
65000 × 2.5 vehicles manufactured by Company Y in 2014 and
= = 1625 = 1625 2015 ?
100
(1) 21000 (2) 22000 (3) 23000
Number of people interested to buy Motorola
(4) 24000 (5) None of these
65000 × 12
= = 7800 Q.3. What is the average number of vehicles
100 manufactured by Company X over the given
1625 period?
Required % = 7800 × 100 = 20.83% = 21%
(1) 119133 (2) 119233 (3) 119333
(4) 119433 (5) None of these
2.5 25 25
Q.3.(1) Required ratio = = = = 5 : 29 Q.4. In which of the following years, the difference
14.5 145 145
between the production of Companies X and Y was
Q.4.(5) Number of people interested to buy Redmi
the maximum among the given years ?
= 65000 × 20/100 = 13000
(1) 2011 (2) 2012 (3) 2013
Number of people interested to buy Oppo (4) 2014 (5) None of these
= 65000 × 14.5/100 =9425 Q.5. The production of Company Y in 2014 was
Required % = 13000/9425 × 100 = 137.93 approximately what percent of the production of
Company X in same year ?
= 138%
(1) 163% (2) 164% (3) 165%
Q.5.(3) (4) 166% (5) None of these
Line Graph Explanation : Q.1-5.
The line graph is one of the simplest ways of
Q.1.(2) Please note that line graph values are given in
expressing the data. There could be one or more
thousands.
lines depicting different sets of data. X-axis is
usually the category axis and Y-axis is usually the Total production of Company X from 2011 to 2016
value axis. Therefore, in line graph there are two =119000+99000+141000+78000+120000+15900
axes representing two different variables. It is also 0 = 716000
known as XY-Chart or -XY Graph.
Total production of Company Y from 2011 to 2016
Q.1-5. Study the following line graph and answer the
questions based on it. = 139000+120000+100000+128000+107000+148
Vehicles made by two companies 000 = 742000
2011 2012 2013 2014 2015 2016 Difference = 742000 - 716000 = 26000

QUANTITATIVE APTITUDE 163


www.mahendras.org
Q.2.(1) Required Difference = 128000-107000 = 21000 Q.1. What is the population of City C which is above
poverty line? (in lakhs)
Q.3.(3) Average number of vehicles manufactured by
Company X (1) 8.44 (2) 10.24 (3) 9.15
(4) 7.96 (5) None of these
= 1\6(119000+99000+141000+78000+120000+15
9000) = 119333 Q.2. What is the difference between the population of
City E which is below poverty line and that which
Q.4.(4) For 2011 = (139000 - 119000) = 20000 is above poverty line? (in lakhs)
For 2012 = (120000 - 99000) = 21000 (1) 2.96 (2) 3.24 (3) 3.96
For 2013 = (141000 - 100000) = 41000 (4) 2.24 (5) None of these
For 2014 = (128000 - 78000) = 50000 Q.3. What is the ratio of the population of City A which
is above poverty line to the population of City D
For 2015 = (120000 - 107000) = 13000 which is below poverty line?
For 2016 = (159000 - 148000) = 11000 (1) 3:2 (2) 2:3 (3) 4:5
Clearly the difference was maximum in year 2014. (4) 5:4 (5) None of these
Q.4. The population of City G which is above poverty
Q.5.(5) Required Percentage
line approximately what per cent of the population
128000 of City A which is below poverty line?
= ×100 = 164.1%
780000 (1) 88% (2) 94% (3) 123
Combined Graph (4) 80% (5) 73%
Here combination of two or more than two graphs Q.5. The population of City B which is below poverty
is given like table-line chart, pie chart with table, line is approximately what percent more/less than the
line chart with bar graph and double pie chart with population of City D which is below poverty line?
table graph etc. (1) 73% (2) 79% (3) 67%
Q.1-5. Read the following informations carefully to (4) 52% (5) 85%
answer the questions asked-
Explanation : Q.1-5.
Given pie-chart shows the percentage distribution
Q.1.(5) Population of City C which is above poverty line
of population and the table shows the percentage
population below poverty line in these cities. 8 65
Total Population = 180 Lakhs = 180 × × = 9.36
100 100

A Q.3.(5) Population of City A which is above poverty line


F 10
18 10 52
= 180 × × = 9.36
B 100 100
20
Population of City D which is below poverty line
E
22 13 40
C
= 180 × × = 9.36
D
8
100 100
13
 
Population below Required ratio=1:1
City
poverty line Q.4.(2) Population of City G which is above poverty line
A 48% 9 50
B 45% = 180 × × = 8.10
C 35%
100 100
D 40% Population of City A which is below poverty line
E 55% 10 48
F 45% = 180 × × = 8.64
G 50% 100 100

164 QUANTITATIVE APTITUDE


www.mahendras.org
8.10 Company M was 10 : 7, what was the number of
Required percent= × 100% ≈ 94% Arts graduate employees in M?
8.64 (1) 294 (2) 266 (3) 280
Q.5.(1) Population of City B which is below poverty line (4) 308 (5) 322
Q.4. The total number of employees in Company N
20 45 increased by 20% from December 2017 to December
= 180 × × = 16.20
100 100 2018. If 20% of the total number of employees
Population of City D which is below poverty line in Company N in December 2018 were Science
graduates, what was the number of Science graduate
13 40 employees in company N in December 2018?
= 180 × × = 9.36
100 100 (1) 224 (2) 266 (3) 294
(4) 252 (5) 168
16.2 − 9.36
Required percent= × 100% ≈ 73% Q.5. The total number of employees in Company P was
9.36 3 times the total number of employees in Company
O. If the difference between the number of Arts
Missing DI graduate employees in Company P and that in
This type of DI is mostly in table form where some Company O was 180, what was the total number of
values are missing and some values are given with employees in Company O?
some instruction along the table. (1) 1200 (2) 1440 (3) 720
Q.1-5. Study the table and answer the given questions. (4) 900 (5) 1080
Data related to the number of employees in five Explanation : Q.1-5.
different companies in December 2017.
Q.1.(3) Total number of employees in company N = 700
Percent Percent of Percent Percentage of Science graduate employees
Total
Company of Science Commerce of Art = [100 – (31 + 40)] = 29%
Employee
Graduates Graduates Graduates
Now, percentage difference between Arts graduate
M 1050 32% – – and science graduate employees
N 700 – 31% 40% 11% of 700 = 77
O – 30% 30% –
P – – 40% 20% Therefore, difference = 77
Q – 35% 50% – Q.2.(2) The percentage of Arts graduate employees in
Note: Company Q = 100 – 35 – 50 = 15%
(I) Employees of the given companies can be Now, the percentage of Arts graduate employees
categorised only in three types: Science graduates, and Commerce and Arts = 50 + 15 = 65%
Commerce graduates and Arts graduates Average = 312
(II) A few values are missing in the table (indicated –). Therefore, the total number of employees in
A candidate is expected to calculate the missing commerce and Arts = 2 × 312
value, if it is required to answer the given question,
Let the total employees in Company Q be x
on the basis of the given data and information.
Then, 65% of x = 2 × 312
Q.1. What is the difference between the number of
Arts graduate employees and Science graduate x = 960
employees in Company N? Q.3.(1) The percentage of commerce graduate and Arts
(1) 87 (2) 89 (3) 77 graduate employees in company M
(4) 81 (5) 73 = 100 – 32 = 68%
Q.2. The average number of Arts graduate employees and Now, the percentage of Arts graduate employees
commerce graduate employees in Company Q was 312.
What was the total number of employees in Company Q? = =28%
(1) 920 (2) 960 (3) 1120
(4) 1040 (5) 1080 the percentage of Commerce graduate employees
Q.3. If the ratio of the number of Commerce graduate = =40%
employees to that of Arts graduate employees in
QUANTITATIVE APTITUDE 165
www.mahendras.org
The number of arts graduate employees in company less than the number of male employees who work
for ‘Ops’ in the same company by 75%. Out of the
M= =294
remaining female employees, 1/4 work in ‘Admin’.
Q.4.(5) The number of employees in company N in Q.1. What per cent of the total number of male employees
December 2017 = 700 in company A work in ‘other’ departments?
The number of employees in company N in (1) 45 (2) 25 (3) 30
December 2018 (4) 35 (5) 40
Q.2. What per cent of the total number of female
=
employees in company B work in administration
Number of Science graduate employees in department?
company N in December 2018 (1) 18.5 (2) 8.75 (3) 14
(4) 16 (5) 19
=
Q.3. What is the total number of female employees who
Q.5.(4) The percentage of Arts graduate employees in work on Ops in Company A and B together?
company O = 100 – 30 – 30 = 40% (1) 681 (2) 781 (3) 689
The percentage difference between Arts graduate (4) 649 (5) 788
employees in company O and P Q.4. What is the difference between the average number
= 40 – 20 = 20% of males working in ‘Admin’ in both the companies
Now, let the number of employees in company O together and average number of females working
be x in ‘Other Departments’ in both the companies
Then, x × 20% = 180 together?
x = 900 (1) 26 (2) 36 (3) 16
(4) 24 (5) 14
Puzzle DI Q.5. In company B, what is the respective ratio between
A puzzle is a set of information that is given in the total number of employees (both male and
paragraph form. In a puzzle, no tables or graphs are female) who work in ‘Admin’ and the total number
given. You have to read the information and draw of employees (both male and female) who work in
a table or caselet accordingly by means of which ‘Other Department’ in the same company?
organize the given data to solve the questions. (1) 2 : 3 (2) 1 : 3 (3) 1 : 4
Q.1–5. This data is regarding total number of employees (4) 3 : 5 (5) 1 : 5
working in Administration (admin), Operations
(Ops.) and other departments of corporate divisions Explanation : Q.1-5.
of Companies A and B. For company A , Total = 2000
The total number of employees working in both the Male (1400) Female (600)
companies together is 4800. The respective ratio Ops 840 285
of number of employees in Companies A and B is
Admin 70 144
5 : 7. Each employee works in only one of the 3
Departments i.e. “ops”, “Admin” and “others”. Other 490 171
In company A, 70% of the total employees are For Company B –, Total = 2800
males. 60% of the total male employees work Male (2240) Female (560)
in ‘Ops’ out of the remaining male employees,
Ops 1456 364
1/8th work in ‘Admin’. Out of the total female
employees, 24% work in ‘Admin’ and 5/8th of the Admin 196 49
remaining female employees work in ‘Ops’. Other 588 147
In company B, 80% of the total employees are 490
Q.1.(4) Required % = × 100 = 35%
males. 65% of the total male employees work in 1400
‘Ops’. Number of male employees who work in
other departments in Company B is 20% more 49
Q.2.(2) Required % = × 100 = 8.75%
than the male employees who work in ‘Other 560
Departments’ in company A. Number of female Q.3.(4) Required no. of female = 285 + 364 = 649
employees who work in Ops in Company B are
166 QUANTITATIVE APTITUDE
www.mahendras.org
171 + 147 70 + 196 Q.5. The speed of Boat B and D in still water together
Q.4.(1) Required difference = − is approximately how much percent more than the
2 2
= 159 – 133 = 26 speed of stream of the same boats together?
Q.5.(2) Required Ratio = (196 + 49) : (588 + 147) (1) 125% (2) 80% (3) 150%
= 245 : 735 = 1 : 3 (4) 190% (5) None of these
DI Based on Topics Explanation : Q.1-5.
Now a days in exams new type of DI questions are asked in
which we have to apply the fundamentals which we learned Q.1.(3) 96 288
=
in previous chapters. a − 12 a + 12
Q.1-5. Study the data carefully and answer the questions On solving a=24 Km/h.
given below.  similarly for others we can get this table
The table shows the distance travelled by five dif- Boat Speed of Speed of
ferent boats upstream and downstream in same time boat stream
and the speed of stream.
A 24 12
Down- B 24 8
Upstream Speed of
Boat stream C 40 15
distance stream
distance D 25 10
E 40 20
A 96 288 12
Ratio = (24 + 24) : (25 + 40)= 48 : 65
B 120 240 8
Q.2.(5) Speed of Boat C in still water
C 100 220 15
= 40 × 110% = 44 km/h
D 150 350 10
Speed of stream = 15 × 120% = 18 km/h
E 180 540 20 Time taken by Boat C to cover the distance of 91
Q.1. Find the ratio of the speed of Boats A and B together km upstream
in still water to the speed of Boats D and E together
in still water. 91
= = 3.5 hours
(1) 23:25 (2) 34:41 (3) 48:65 44 − 18
(4) 13:17 (5) None of these
210 210
Q.2. If the speed of Boat C in still water is increased by Q.3.(4) Required answer= + = 14 hours
40 − 20 40 + 20
10% and the speed of stream is increased by 20%,
Find the time taken by Boat C to cover the distance Q.4.(1) Speed of the Boat B in still water = 24 km/h
of 91 km upstream.
(1) 2.5 hours (2) 3 hours (3) 4.5 hours 5
Speed of the Boat F in still water= 24 × = 30
(4) 6 hours (5) None of these 4
Q.3. The distance between point P and point Q is 210 Let the speed of stream = x km/h
km. Boat E travels from point P to Q and comes According to the question,
back. What is the time taken by Boat E to cover the 126 81 15
total distance? + =
30 + x 30 − x 2
(1) 10 hours (2) 12 hours (3) 12.5 hours
On solving, Speed of stream = 12 km/h
(4) 14 hours (5) 15 hours Q.5.(5) Speed of Boat B and D in still water together
Q.4. The ratio of the speeds of the Boat B to the Boat F
= (25 + 24) = 49 km/h
in still water is 4 : 5. If the Boat F travels 126 km
distance downstream and 81 km distance upstream Speed of stream of Boat B and D together
in 7 hours 30 minutes. What is the speed of stream = 18 km/h
of Boat F? Required answer
(1) 12 Km/hr. (2) 14 Km/hr. (3) 15 Km/hr. 49 − 18
= × 100% ≈ 170%
172
(4) 10 Km/hr. (5) None of these 18

QUANTITATIVE APTITUDE 167


www.mahendras.org
Q.1-5. Study the following information carefully and Q.5. Three pipes A, D and F together can fill the cistern
answer the given questions : in 8 hours. Find the time taken by F alone to fill the
The following table represents time taken (in hours) cistern?
by different pipes to fill a cistern. Some values are (1) 20 hours (2) 15 hours
missing. (3) 18 hours (4) 12 hours
Pipe Time taken to fill the cistern (5) None of these

A 24 Explanation : Q.1-5.

B -- 4 4 5 5
Q.1.(4) + + + =1
24 x x 30
C --
27 1
D 30 x= = 13 h
2 2
E 45
Q.2.(4) x + x + 20 =1
F -- 30 45 45

Q.1. If A and C are kept open for 4 hours then A is on solving above equation, we get
replaced by D and kept open for 5 more hours, the x = 10
tank is filled. In how many hours pipe C alone can Q.3.(4) Ratio of C, F and B to fill the cistern=1 :2 :4
fill the cistern?
1 1 1 1
3 1 + +
(1) 12 hours (2) 11 hours x 2x 4x = 48
4 4
x = 84
1 1
(3) 9 hours (4) 13 hours Required answer=2×84=168 hours
2 2
(5) None of these 1 1 3
Q.4.(2) Part of tank filled in 2 hours = + =
Q.2. Two pipes D and E are opened simultaneously to fill 24 30 40
the cistern. After how much time should D be closed
so that E alone can fill the cistern in another 20 hours? 3 39
Part of tank filled in 26 hours = × 13 =
(1) 8 hours (2) 15 hours (3) 12 hours 40 40
(4) 10 hours (5) None of these 39 1
Remaining part = 1 − =
Q.3. If C takes half of the time taken by F to fill the 40 40
cistern and F takes half of the time taken by B to fill Time taken by A to fill remaining part
the cistern and all of them working together can fill
the cistern in 48 hours, What is the time taken by F 1 3
to fill the cistern? = × 24 =
40 5
(1) 120 hours (2) 144 hours (3) 180 hours
3 3
(4) 168 hours (5) None of these Total time = 26 + = 26
5 5
Q.4. Two pipes A and D can fill the cistern. If they are
opened on alternative hours and if pipe A is opened 1 1 1
Q.5.(1) + +
first, in how many hours will the cistern be full? 24 30 x

5 3 1
(1) 24 hours (2) 26 hours =
8 5 8

1 1 on solving above equation, we get
(3) 27 hours (4) 26 hours
3 2
x = 20
(5) None of these

168 QUANTITATIVE APTITUDE


www.mahendras.org
EXERCISE
Q.1-5. Study the following table and answer the questions that follow:
The table represents the total sales value (in lakhs) of five books P, Q, R, S and T across eight bookstores A, B,
C, D, E, F, G and H.
BOOKS BOOK STORES
A B C D E F G H
P 560 590 210 670 560 680 420 460
Q 550 560 890 230 820 610 520 230
R 450 290 540 530 500 520 560 410
S 230 240 560 400 430 200 210 480
T 230 400 410 240 200 360 500 470
Q.1. What is the ratio the total sales values of Book P across all the bookstores and Book S across all the bookstores?
(1) 83 : 55 (2) 85 : 53 (3) 83 : 53 (4) 89 : 55 (5) None of these
Q.2. The sale of the book R from Bookstore H is what per cent of the total sale of Book R across all the bookstores
(rounded off to two places after the decimal)?
(1) 21.12 (2) 17.19 (3) 7.43 (4) 3.04 (5) 10.79
Q.3. What is the average sale (in lakh) of Bookstore E?
(1) 502 (2) 504 (3) 512 (4) 540 (5) None of these
Q.4. Book Q constituted approximately what per cent of the total sales of Bookstore C?
(1) 34 (2) 39 (3) 32 (4) 23 (5) 28
Q.5. What is the total sales value (in lakh) of Bookstore D?
(1) 2510 (2) 2670 (3)2900 (4) 2070 (5) None of these
Q.6-10. The table shows the Cost Price of 5 products divided in 3 costs: Production Cost, Transportation Cost and Pack-
aging Cost, the selling price, profit/loss and profit%/loss%. Some values are missing. Find the answers based on
information in table.

Transportation Selling
Product Production cost Packaging cost Profit/loss Profit/loss %
cost price
A 80 16 -- 300 -- 5%
B 100 20 8 -- -- 30% profit
C 90 -- 20 -- 100 --
D 60 12 30 -- -- --
E 120 20 -- 110 -- 10% loss

1
Q.6. If the percentage of profit on selling product A is 9 % , then what is its cost of packaging?
11
(1) 174 (2) 164 (3) 179 (4) 156 (5) None of these
Q.7. What is the difference between the selling price of products B and C, if the cost of transportation of C is Rs 16 and
both are sold at profit?
(1) 54.90 (2) 61.12 (3) 69.44 (4) 59.60 (5) None of these
Q.8. Suppose all the prices are given for per kg of a product. What amount of product B will have to add to 54 kg of
product E such that the resultant product has cost price of Rs 125.
(1) 60 (2) 45 (3) 50 (4) 40 (5) None of these
Q.9. What is the percentage profit (approximate) on selling product D if its selling price is 80% of the selling price of
B?
QUANTITATIVE APTITUDE 169
(1) 20%
www.mahendras.org (2) 30% (3) 25% (4) 22.5% (5) 33%
Q.10. If 2 kg of A, 3 kg of C and 4 kg of E are sold, then what will be the final profit/loss% (approximate) on selling these
given. Transportation cost of C as Rs 10 and profit of 5% on selling A?
(1) 15% (2) 18% (3) 12% (4) 20% (5) 10%
Q.11-14. Study the following information carefully and answer the given questions.
The following bar graph shows the selling price of an article (In Rs.) and the profit % earned by selling the article.
700 650 45

600 40
550
35
500

(In Percentage)
30
384 392
(In Rupees)

400 25
288 290
300 20
15
200
10
100 5
0 0
P Q R S T U
Axis Title

Selling Price P%

Q.11. The cost price of article P is approximately what percentage more/less than the cost price of article U?
(1) 23 % less (2) 14 % less (3) 14 % more (4) 8 % more (5) 23 % more
Q.12. Manoj sold an article S to Kamran who again sold it at 25 % profit. Find the difference between the profits earned
by Manoj to that of Kamran?
(1) Rs. 14 (2) Rs. 18 (3) Rs. 24 (4) Rs. 30 (5) None of these
Q.13. Rajesh marked an article Q, which is Rs. 60 above the cost price. How much discount percentage should be given
on marked price to earn the given profit?
(1) 5% (2) 8% (3) 12% (4) 17.5% (5) None of these
Q.14. Find the average cost price of the article P, Q, R and S together.
(1) Rs. 250 (2) Rs. 350 (3) Rs. 300 (4) Rs. 330 (5) None of these
Q.15-19. Read the given bar graph and answer the following questions.

Number of students taking fresh education loan from different


Banks
6000

5000

4000 2008
2009
3000
2010
2000 2011
2012
1000

0
SBI PNB BOB UCO OBC

170 QUANTITATIVE APTITUDE


www.mahendras.org
Q.15. If 23% from UCO in 2009 and 20% from PNB in 2010 have defaulter then, find the total number of defaulter of
UCO and PNB taken together?
(1) 630 (2) 650 (3) 600 (4) 750 (5) 840
Q.16. In 2007, no of defaulters in SBI was 5%. However each year no of defaulters increases by 10% in number. What
will be the difference between the number of defaulters of SBI in the year 2009 and 2012?
(1) 1500 (2) 2000 (3) 1325
(4) 1456 (5) Cannot be determined
Q.17. In which of the following years, the difference in number of students taking loan from Bank BOB from the previ-
ous year is highest?
(1) 2008 (2) 2009 (3) 2010 (4) 2012 (5) None of these
Q.18. If on average, Rs. 175000 per student's education loan sanctioned by OBC bank all over the year. What will be
total amount sanctioned by OBC in all given years?
(1) Rs. 1055600000 (2) Rs. 1055800000 (3) Rs. 1620000000 (4) Rs. 1050000000 (5) None of these
Q.19. What is the ratio of Number of students taking Education Loans from SBI and BOB together in all the years and
the total no of students taking Education loans in 2010 and 2011 together?
(1) 8 : 5 (2) 5 : 7 (3) 7 : 5 (4) 9 : 7 (5) None of these
Q.20-24. The following Line chart gives the ratio of the amounts of imports by a Company to the amount of exports from
that Company over the period from 2011 to 2017. Answer the following questions based on following Line graph.
Ratio of value of imports to export

2011 2012 2013 2014 2015 2016 2017

Q.20. In how many of the given years were the exports more than imports ?
(1) 1 (2) 2 (3) 3 (4) 4 (5) None of these
Q.21. The imports were minimum proportionate to the exports of the Company in the year
(1) 2013 (2) 2011 (3) 2012 (4) 2016 (5) None of these
Q.22. If the imports of a company in 2012 was Rs. 272 crores, the exports from the company in 2012 was
(1) Rs 120 Crores (2) Rs 220 Crores (3) Rs 320 Crores (4) Rs 420 Crores (5) None of these
Q.23. What was the percentage increase in imports from 2013 to 2014 ?
(1) 70 (2) 72 (3) 74 (4) Data Inadequate (5) None of these
Q.24. If the imports in 2014 was Rs. 250 crores and the total exports in years 2014 and 2015 together was Rs 500 crores,
then the imports in 2015 was
(1) 320 Crore (2) 420 Crore (3) 520 Crore (4) 620 Crore (5) None of these
QUANTITATIVE APTITUDE 171
www.mahendras.org
Q.25-28. Study the following graph carefully and answer the questions given below it.
Profit percent earned by two companies in various years.

Profit percent of two companies in


various years

60
55 55
50
45 45 45
40 40
35
30
25

2011 2012 2013 2014 2015 2016

A B

Q.25. In the year 2013 the income of company A was Rs. 261000 and income of company B was Rs. 256500. Find the
difference of their expenditures.
(1) 15000 (2) 12000 (3) 10000 (4) 17500 (5) None of these
Q.26. Company A spent Rs. 185000 in the year 2011. Find out the profit of company A in that year after 18% tax payment
on total income?
(1) 28580 (2) 32450 (3) 27380 (4) 25290 (5) None of these
Q.27. If the expenditure of both the companies A and B in the year 2015 was same, then what was the ratio between the
incomes of company A to Company B?
(1) 8 : 7 (2) 7 : 8 (3) 5 : 4 (4) 4 : 5 (5) None of these
Q.28. In which of the following years was there maximum percentage of growth/decline with respect to the previous year
in case of company A?
(1) Years 2012 (2) Years 2015 (3) Years 2011
(4) Years 2016 (5) Can not be determined
Q.29-33.Study the graphs carefully to answer the questions that follow.
Total number of children in 6 different schools and the percentage of girls in them
3500

3000
Number of childrens

2500

2000

1500

1000

500

0
P Q R S T U
School

172 QUANTITATIVE APTITUDE


www.mahendras.org
50
45
40
Percentage of girls

35
30
25
20
15
10
5
0
P Q R S T U
School

Q.29. What is the total percentage of boys in schools R and U together? (rounded off to two digits after decimal)
(1) 78.55 (2) 72.45 (3) 76.28 (4) 75.83 (5) None of these
Q.30. What is the total number of boys in school T?
(1) 500 (2) 600 (3) 750 (4) 850 (5) None of these
Q.31. The total number of students in school R, is approximately what per cent of the total number of students in school
S?
(1) 89 (2) 75 (3) 78 (4) 82 (5) 94
Q.32. What is the average number of boys in schools P and Q together?
(1) 1425 (2) 1575 (3) 1450 (4) 1625 (5) None of these
Q.33. What is the respective ratio of the number of girls in schools P to the number of girls in school Q?
(1) 27 : 20 (2) 17 : 21 (3) 20 : 27 (4) 21 : 17 (5) None of these
Q.34-38. The annual rate of interest offered by the two Companies P and Q over the years are shown by the line graph
provided below.

Rate of interest
12
10
9.5
10 9 9
8 8 8 8
7.5
8 7
6.5 6.5
6
6
4
4

0
2006 2007 2008 2009 2010 2011 2012

Company P Company Q

QUANTITATIVE APTITUDE 173


www.mahendras.org
Q.34. A sum of Rs. 4.75 lakhs was invested in Company Q in 2009 for one year. How much more interest would have
been earned if the sum was invested in Company P?
(1) 8400 (2) 9200 (3) 9500 (4) 10250 (5) None of these
Q.35. If two different amounts in the ratio 8:9 are invested in Companies P and Q respectively in 2012, then the amounts
received after one year as interests from Companies P and Q are respectively in the ratio?
(1) 5:2 (2) 2:5 (3) 4:3 (4) 3:4 (5) None of these
Q.36. In 2010, a part of Rs. 30 lakhs was invested in Company P and the rest was invested in Company Q for one year.
The total interest received was Rs. 2.43 lakhs. What was the amount invested in Company P? (in lakhs)
(1) 15 (2) 17.5 (3) 16 (4) 12 (5) 18
Q.37. An investor invested a sum of Rs. 12 lakhs in Company P in 2008. The total amount received after one year was
re-invested in the same Company for one more year. The total appreciation received by the investor on his invest-
ment was?
(1) 225600 (2) 248500 (3) 292000 (4) 265000 (5) None of these
Q.38. An investor invested Rs. 5 lakhs in Company Q in 2006. After one year, the entire amount along with the interest
was transferred as investment to Company P in 2007 for one year. What amount will be received from Company
P, by the investor?
(1) 520450 (2) 580425 (3) 575655 (4) 624500 (5) None of these
Q.39-43. Study the following bar graph and answer the given questions.

Ratio between production and consumption of wheat, rice and


pulses in different years in india
2
1.8
1.6
1.4
1.2
1
0.8
0.6
0.4
0.2
0
2012 2013 2014 2015 2016

Wheat Rice Pulses

Q.39. If in the year 2013 the production of rice is 15 ton and in the year 2014 the consumption of pulses is 14 ton. Find
the ratio between consumption of rice in the year 2013 and production of pulses in the year 2014.
(1) 7 :5 (2) 5 : 7 (3) 4 : 5 (4) 9 : 7 (5) 7 : 9
Q.40. If in the year 2014 the consumption of rice and wheat is same. Find the ratio of production of rice and wheat in the
same year.
(1) 3:7 (2) 4 : 7 (3) 7 : 4 (4) 7 : 3 (5) Data inadequate
Q.41. If in the year 2012 the production of pulses is 21 ton and consumption of rice is 20 ton. How much % the consump-
tion of pulses in 2012 is more/less than production of rice in same year.
(1) 24% (2) 25% (3) 45% (4) 60% (5) 55%
Q.42. If  export=production-consumption then find the % change of export of rice in the year 2015 as compare to previ-
ous year.
(1) 100% (2) 50% (3) 50% (4) No change (5) Data inadequate
174 QUANTITATIVE APTITUDE
www.mahendras.org
Q.43. If production of wheat and consumption of Rice in 2012 is 17.5 ton and 12.5 ton. Find the diiference between
consumption of wheat and production of rice in same year.
(1) 2 ton (2) 3 ton (3) 3.5 ton (4) 2.5 ton (5) 1.5 ton
Q.44-48. Read the following information carefully and answer the questions given below.
In a city, the account holders have their accounts in 5 different banks viz. Allahabad Bank, Dena Bank, Andhra
Bank, Corporation Bank and UCO Bank. The total number of account holders is 2050. 24% of total account hold-
ers have account in Allahabad Bank. One-fifth of the total account holders have account in Corporation Bank. 16%
of total account holders have account in Andhra Bank. Remaining account holders have account in either UCO
Bank or Dena Bank. The number of accounts in Dena Bank is 182 less than the number of accounts in UCO Bank.
Q.44. What is the total number of accounts in UCO Bank, Andhra Bank and Corporation Bank?
(1) 1027 (2) 1157 (3) 1057 (4) 957 (5) None of these
Q.45. What is the respective ratio of number of accounts in Corporation Bank and that in UCO Bank and Dena Bank
together?
(1) 2 : 1 (2) 1 : 2 (3) 3 : 4 (4) 7 : 6 (5) None of these
Q.46. The number of accounts in Andhra Bank is approximately what percent of the total number of accounts in Alla-
habad Bank and Corporation Bank together?
(1) 63 (2) 26 (3) 46 (4) 56 (5) 36
Q.47. What is the total number of accounts in Dena Bank and UCO Bank?
(1) 820 (2) 800 (3) 840 (4) 860 (5) None of these
Q.48. The number of accounts in Andhra Bank is approximately what percent more than that in Dena Bank?
(1)10 (2) 8 (3) 12 (4) 3 (5) 5
Q.49-53. There are five students who appeared for RBI Grade B exam. Paper consists of 100 questions with 1 mark for
each correct answer and 0.25 marks for each wrong answer.
Students Questions attempted Right Questions Wrong Questions Marks obtained
Aditya 78 – – 70.5
Puskar 92 76 – –
Anshuman 98 – 36 –
Alka – 30 – 27.25
Avanish 56 – – 53.50

Q.49. Difference between total right number of questions of all students together and total wrong no. of questions of all
students together is
(1) 141 (2) 161 (3) 223 (4) 156 (5) None of these
Q.50. Marks obtained by Aditya and Puskar together is what % of the marks obtained by Anshuman, Avanish and Alka
together? (rounded off to 2 decimal places)
(1) 106.54% (2) 91.16% (3) 95.20% (4) 96.71% (5) 101.71%
Q.51. If the penalty of wrong answer is 0.33 then marks obtained by Aditya, Anshuman and Puskar together is
(1) 192.21 (2) 224.19 (3) 190.86 (4) 219.14 (5) 194.22
Q.52. If the passing % marks in the exam is 50 marks than at least how many questions has to be answered right by
Puskar? (He attempted 92 questions)
(1) 58 (2) 56 (3) 59 (4) 55 (5) 60
Q.53. What is the percent of marks obtained by all of them together?
(1) 59.03% (2) 53.15% (3) 52.53%
(4) 45.05% (5) 55.25%
QUANTITATIVE APTITUDE 175
www.mahendras.org
Q.54-58. Study the following information and answer the questions asked:

The premises of an office are to be renovated. The renovation is in terms of flooring. Certain areas are to be floored
either with tiles or wooden flooring. All rooms/halls and pantry are rectangular. The area to be renovated comprises
of a hall for customer care executive measuring 24 m by 18 m, team leader’s room measuring 12 m by 15 m, a
pantry measuring 18 m by 16 m, a server room measuring 21 m by 14 m and cafeteria area measuring 26 m by 22
m. The total area of the office is 1950 square meters. The cost of wooden flooring is 175/- per square meter and
the cost of tiles flooring is 160/- per square meter. The cafeteria area, server room and pantry are to be floored with
tiles. Team leader’s room and the hall for customer care executive are to be floored with wooden flooring. No other
area is to be renovated in terms of flooring.

Q.54. What is the difference between the total cost of wooden flooring and total cost of tiles flooring (in Rs.)?

(1) 120800 (2) 84375 (3) 77540 (4) 62500 (5) None of these

Q.55. If the four walls and ceiling of the team leader’s room (the height of the room is 8 meter) are to be painted at the
cost of 180/- per square meter, how much will be the total cost of renovation of the team leader’s room including
the cost of flooring? (Note-you have not to consider any left area for door or window)

(1) 142560 (2) 112800 (3) 108600 (4) 132000 (5) None of these

Q.56. If the remaining area of the office is to be carpeted at the rate of 175/- per square meter, how much will be the
increment in the total cost of renovation of office premises?

(1) 32200 (2) 30450 (3) 39750 (4) 44800 (5) None of these

Q.57. What is the approximate percentage area of the office that is not to be renovated?

(1) 11.11% (2) 10.52% (3) 9.44% (4) 6.94% (5) 12.35%

Q.58. What is the total cost of renovation of the hall for customer care executive and the cafeteria area?

(1) 162820 (2) 167120 (3) 159080 (4) 175200 (5) None of these

Q.59.-63. Read the following pie-chart carefully and answer the questions asked.
       Population distribution of states in two years
       Total population in 2016=50 Lakhs

Tamilnadu
Karnataka
12%
15%

Andhra
Pradesh
9% Uttarpradesh
20%
Maharastra
11%

Kerala
8% Bihar
25%

176 QUANTITATIVE APTITUDE


www.mahendras.org
         Total population in 2017=56 Lakhs

Karnataka Tamilnadu
15% 20%

Andhra
Pradesh
8%
Uttarpradesh
Maharastra 12%
12%

Kerala
10% Bihar
23%

Q.59. What is the respective ratio of population of Tamil Nadu in 2016 to that of 2017?
(1) 8:13 (2) 17:29 (3) 15:28 (4) 15:31 (5) None of these
Q.60. What was the approximate difference between percent increase in population of Bihar and Kerala from 2016 to
2017?
(1) 32% (2) 41% (3) 28% (4) 37% (5) 51%
Q.61. What is difference between population of Tamil Nadu  in 2016 and Karnataka in year 2017?
(1) 280000 (2) 240000 (3) 180000 (4) 320000 (5) None of these
Q.62. If in 2018 population of Bihar is increased by 15% whereas the population of Uttar Pradesh is decreased by 12%
as compared to previous year, Find the sum of population of these two states in 2018.
(1) 2072560 (2) 2284320 (3) 3122450 (4) 1844620 (5) None of these
Q.63. Find the difference of central angle of Uttar Pradesh and Kerala in 2016.
(1) 17.20 (2) 32.40 (3) 44.40 (4) 30.80 (5) None of these
Q.64.68.Study the following information carefully to answer the questions asked.
The following bar graph represents the exports (in Cr Rs.) and imports (in Cr Rs.) of five countries in 2016.
Import and Export of five countries

Import Export

350
296
300 280
270 264
240
250
200
200
156 150
144
150
100
100

50

0
Pakistan Bangladesh China Bhutan India

QUANTITATIVE APTITUDE 177


www.mahendras.org
Country Ratio of import (2016 to 2017) Ratio of export (2016 to 2017)
Pakistan 27:25 2:3
Bangladesh 39:41 12:11
China 37:32 5:3
Bhutan 10:11 18:19
India 15:16 7:5

Q.64. Find the respective ratio between Import of China in 2017 and export of Bhutan in 2017.
(1) 32:17 (2) 32:19 (3) 35:23 (4) 19:17 (5) None of these
Q.65. Imports of all the countries together in 2016 is what percent of the exports of all the countries together in 2016?
(1) 112.57% (2) 108.59% (3) 106.22% (4) 123.15% (5) 119.62%
Q.66. Find the sum of the imports of the Pakistan and Bangladesh in 2017 and exports of the China and Bhutan in 2017.
(1) Rs. 626 crore (2) Rs. 616 crore (3) Rs. 606 crore (4) Rs. 636 crore (5) None of these
Q.67. Find the difference between the sum of Import and Export of  India in 2016 and that in 2017. (in crore)
(1) 70 (2) 62 (3) 74 (4) 80 (5) None of these
Q.68. Import of China in 2017 is approximately what percent more/less than the export of the same country in same
year?
(1) 221% (2) 427% (3) 369% (4) 327% (5) 411%
Q.69-73. Read the following informations carefully to answer the questions asked-
There are three sections in class 12 and students are given two choices weather to choose Science or Art as one of
optional subject. Related informations are given as below-
4
(1)  In section C there are total 66 girls out of which  45 %   girls choose Arts which is 20 less than number of
9
boys choosing Arts in same section.
(2)  Ratio of boys choosing Science to boys choosing Arts in section C is 24:25.
(3)  In section B, 12 more boys and 9 more girls choose Science as compared to section C.
(4)  In section B, 40 boys choose Arts which is 20% less than number of girls choosing Arts in same section.
(5)  Overall 188 boys and 121 girls choose science.
(6)  Total number of students taking all three sections together is 604.
(7)  Ratio of boys and girls choosing Art in section C is 11:14 respectively.
Q.69. Find the total number of girls who choose art.
(1) 120 (2) 145 (3) 150 (4) 175 (5) None of these
Q.70. Find the total number of students in section B.
(1) 175 (2) 195 (3) 205 (4) 185 (5) None of these
Q.71. Find difference of boys who choose Science in section A and number of girls who choose art in section B.
(1) 20 (2) 25 (3) 30 (4) 15 (5) None of these
Q.72. In section A number of boys is approximately how much percent less/more than the number of girls in same sec-
tion?
(1) 22.7% (2) 30.2% (3) 25.6% (4) 21.2% (5) 24.4%
Q.73. Find the ratio of girls choosing Art and Science respectively.
(1) 135:111 (2) 150:121 (3) 73:51 (4) 19:13 (5) None of these

178 QUANTITATIVE APTITUDE


www.mahendras.org
EXERCISE Explanation
Q.1.(1) Total sales values of Book P across all the Q.8.(3) CP of B = 100+20+8 = Rs 128
bookstores
For CP of E:
= 560 + 590 + 210 + 670 + 560 + 680 + 420 +
460 = 4150 SP = 110, loss% = 10%
Total sales value of Book S across all the
bookstores So CP of E = 100/90 × 110 = Rs 122 (2/9)
= 230 + 240 + 560 + 400 + 430 + 200 + 210 Using method of alligation
+480 = 2750
Ratio = 4150: 2750= 83 : 55 (x kg)…………………………(54 kg)
Q.2.(5) Total sales values of Book R across all Rs 128 Rs 122 (2/9)
bookstores
= 450 + 290 + 540 + 530 + 500 + 520 + 560 + Rs 125
410 = 3800
(25/9) (3)=25:27
Sales of book R from Bookstore H is 410.
percent of books from Bookstore H So
410 Here 27=54
= × 100 = 10.789 =10.79
3800
1=2
Q.3.(1) Average sales of Bookstore E
25=50
= (560+820+500+430+200)/5= 502
Q.4.(1) Total sales of Bookstore C= 210 + 890 + 540 + Q.9.(2) SP of B = 130/100 × 128= Rs 166.4
560 + 410 = 2610
SP of D = 80/100 × 166.4 = Rs 133.12
Sales value of Q in Bookstore C = 890
890 CP of D = 60+12+30 = Rs 102
percent of Q in C = 2610 × 100 = 34%
So profit% = 31.12/102 × 100 = 30%
Q.5.(4) Total value of Bookstore D = 670 + 230 + 530 +
Q.10.(4) CP (2 kg of A + 3 kg of C + 4 kg of E) = 2×286
400 + 240 = 2070
1 + 3×120 + 4×122 = Rs 1420
Q.6.(3) SP = 300, profit% = 9 %
11
Similarly SP of (2 kg of A + 3 kg of C + 4 kg of
11 E) = 2×300 + 3×220 + 4×110 = Rs 1700
So CP = 300 × = 275
12
So cost of packaging = 275– (80+16) = Rs 179 So profit %
280
Q.7.(4) CP of B = 100+20+8 = Rs 128 = × 100% ≈ 20%
1420
30% profit Q.11-14.

So SP of B = 130/100 × 128 = Rs 166.4 Article CP SP


P 500 650
CP of C = 90+16+20 = Rs 126 Q 240 288
R 300 384
Profit = Rs 100
S 280 392
So SP of C = 126+100= Rs 226 T 250 290
U 440 550
So difference = 226 –166.4 = Rs 59.6

QUANTITATIVE APTITUDE 179


www.mahendras.org
19000+19500 38500 7
500 − 440 = 13500+14000 = 27500 = 5
Q.11.(3) Required answer = × 100% ≈ 14%
440
Q.12.(1) Profit earned by Manoj =392-280=112 Q.20.(4) Clearly the exports are more than the imports
implies that the ratio of value of imports to
Profit earned by Kamran =490-392=98 exports in less than 1.
So years are 2011, 2012, 2013 and 2016. So
Required answer=112-98=14 Rs.
these are four years
Q.13.(5) CP=240 Q.21.(1) Clearly from the line graph we can judge it is
minimum in year 2013.
MP=240+60=300
Q.22.(3) We are given with the ratio of imports and
SP=288 exports in the line graph.

300 − 288 Let the exports from the company in 2012 was x
Discount % = × 100% = 4% then,
300
Q.14.(4) Required answer 272 272
= 0.85 Þ x = Þ x = 320
x 0.85
500 + 240 + 300 + 280 1320
= = = 330
4 4 Note: Please not that we are given the ratio of
Q.15.(1) Students taking loan from UCO in 2009 = 1000 imports to exports, so export will in denominator
Defaulters (UCO) = 23% of 1000 = 230 Q.23.(4) For calculating the percentage we will need
value of exports, imports etc. We are only given
Person taking loan from PNB in 2010= 2000
with the ratio. So data in Inadequate
Defaulters (PNB) = 20% of 2000 = 400
Q.24.(2) The Ratio of imports to exports for the years
Total desired defaulters = 230 + 400 = 630 2014 and 2015 are 1.25 and 1.40 respectively.
Q.16.(5) Cannot be determined because no. of students Let the exports in the year 2014
taking a loan from SBI in 2007 is unknown.
= Rs. x crores
Q.17.(2) From graph, it is clear that in 2009, difference
Then, the exports in the year 2015
between no. of students taking a loan is highest
as compared to previous year. = (500-x) crores
Q.18.(5) No. of students taking education loan from OBC 250
⇒ 1.25 =
bank all over the year x
= 1000 + 1000 + 1500 + 2000 + 1500
[because 1.25 is 2014 ratio]
= 7000
250
Total loan amount sanctioned over the years ⇒x= = 200 crore
1.25
= 7000 × 1,75,000
Thus the exports in the year 2015 were 500 -
= Rs. 1,22,50,00,000
200 = 300 crore
Q.19.(3) SBI : 2500 + 3000 + 4500 + 4000 + 5000
Let the imports in the year 2015
= 19000
= Rs y crore
BOB : 2500 + 3500 + 4000 + 4500 + 5000
y
= 19500 Then 1.40 =
300
Total no. of students taking loan in 2010
=> y = 1.40×300 = 420 crore
= 13500
Q.25.(3) Required answer
Total no. of students taking loan in 2011 = 14000
100 100
Desired ratio = 256500 × – 261000 ×
135 145
=190000-180000 = 10000
180 QUANTITATIVE APTITUDE
www.mahendras.org
Q.26.(3) Income=140% of 185000=259000 Then, interest received after one year from

Income after tax deduction=82% of 48


Company P=6% of 8x = x
259000=212380 100
and interest received after one year from Company
Required answer=212380-185000=27380

Q.27.(2) Required ratio = 7 : 8 Q = 36 x


100
Q.28.(5) Can not be determined
48
Q.29.(5) Number of boys in school R and U together x
Required ratio = 100 = 4
2000 × 77.5 87.5 36 3
+ 1000 × x
100 100 100
= 1550 + 875 = 2425 Q.36.(5) Let Rs. x lakhs be invested in Company P in 2010,
∴Required percentage the amount invested in Company Q in 2010 = Rs.
(30 - x) lakhs.
2425
= × 100 = 80.83% Total interest received from the two Companies
3000
after 1 year
Q.30.(3) Number of boys in school T = Rs. [(7.5% of x) + {9% of (30 - x)}] lakhs
1250× 60
= = 750 2.7–1.5% of x = 2.43
100
1.5% of x = 0.27
Q.31.(1) Total number of students in school R
x = 18
= 2000 Q.37.(1) Amount received from Company P on investing
Total number of students in school S Rs. 12 lakhs after one year
= 2250 = Rs. [12 + (8% of 12)] lakhs
∴Required percentage
= Rs. 12.96 lakhs.
2000
×100 ≈ 89 Amount received from Company P on investing
2250
Rs. 12.96 lakhs after one year
Q.32.(2) Required average
= Rs. [12.96 + (10% of 12.96)] lakhs
= 1  25000 + 60 + 3000×55 
  = Rs. 14.256.
2 100 100 
Appreciation received on investment during the
period of two years
1 1
= (1500 + 1650) = × 3150 = 1575 = Rs. (14.256 - 12) lakhs
2 2
Q.33.(3) Required ratio = Rs. 2.256 lakhs
2500× 40 3000× 45 = Rs. 225600.
= :
100 100
Q.38.(2) Amount received from Company Q after one year
= 25 ×40 : 30 × 45 on investment of Rs. 5 lakhs
= 100 : 135 = 20 : 27 = Rs. [5 + (6.5% of 5)] lakhs
Q.34.(3) Required difference=(10% of 4.75)-(8% of 4.75)
lakhs=9500 = Rs. 5.325 lakhs.

Q.35.(3) Let the amounts invested in 2012 in Companies Amount received from Company P after one year
P and Q be Rs. 8x and Rs. 9x respectively. on investment of Rs. 5.325
QUANTITATIVE APTITUDE 181
www.mahendras.org
= Rs. [5.325 + (9% of 5.325)] lakhs Andhra Bank - 328

= Rs. 5.80425 lakhs Coporation Bank - 410


UCO Bank - 501
= Rs. 580425
Total = 501 + 319 = 820
3 Q.48.(4) Allahabad Bank - 492
Q.39.(2)
15 : 14 × = 5:7 Dena Bank - 319
2
Andhra Bank - 328
Coporation Bank - 410
Q.40.(3) 1.4 : 0.80 = 7 : 4
UCO Bank - 501
Q.41.(2) PP : PC = 7 : 10
9
RP : RC = 6 : 5 Percent increase = × 100
319
For pulse For rice = 3 (approx)
7 = 21 5 = 20 Q.49.(3) Required difference = (72 + 76 + 62 + 30 + 54)
– (6 + 16 + 36 + 11 + 2)
10 = 30 6 = 24
= 294 – 71 = 223
Required percent pulse more
Q.50.(1) Required %
30 − 24
× 100 = 25% 70.5+72
24 = × 100
53+27.25+53.50
Q.42.(5) Data inadequate = 106.54%
Q.43.(4) Difference = 17.5 - 15 = 2.5 ton Q.51.(3) Required marks = (72 + 76 + 62) – 0.33 (6 + 16
+ 36) = 190.86
Q.44.(5) Allahabad Bank - 492
Q.52.(3) By options
Dena Bank - 319
Let right Questions = 59
Andhra Bank - 328
marks =92−14(92−59)=50.75
Coporation Bank - 410
UCO Bank - 501 Q.53.(5) Required %

Required number = 501 + 328 + 410 70.5+72+53+27.25+53.50


= × 100
500
= 1239
= 55.25%
Q.45.(2) Allahabad Bank - 492
Q.54-58.
Dena Bank - 319
Andhra Bank - 328 Type of
Portion Dimension Area
Coporation Bank - 410 flooring
UCO Bank - 501 Hall (C.C.E.) 24×18 432 Wooden
Ratio = 410 : 820 = 1 : 2 T.L. Room 12×15 180 Wooden
Q.46.(5) Allahabad Bank - 492 Pantry 18×16 288 Tiles
Dena Bank - 319 Server Room 21×14 294 Tiles

Andhra Bank - 328 Cafeteria 26×22 572 Tiles


Q.54.(3) Required answer = 1154×160 - 612×175=77540
Coporation Bank - 410
Q.55.(5) Required answer = 180×175+{2(12+15)×8
UCO Bank - 501
+180}×180=141660
328
Percent = × 100 = 36
902 Q.56.(1) Required answer = 175 × (1950 - 1766) = 32200
Q.47.(1) Allahabad Bank - 492
Q.57.(3) Required answer = 1950 − 1766 × 100 ≈ 9.44%
Dena Bank - 319 1950

182 QUANTITATIVE APTITUDE


www.mahendras.org
Q.58.(2) Required answer = 432×175+572×160=167120 Import of Bangladesh in 2017 = 164
Q.59.(3) Required answer 12% of 50 : 20% of 56 41
= ×156 = 164
Ratio = 15 : 28
39

1288 − 1250 3
Q.60.(4) Bihar = ×100 ≈ 3% Export of China in 2017 = ×100 = 60
1250 5

560 − 400 19
Kerala = ×100 = 40% Export of BHUTAN in 2017 = ×144 = 152
400 18
Required sum = 250 + 164 + 60 + 152
Required answer = 40 - 3 = 37 = Rs. 626 crore
Q.61.(2) Required answer 15% of 56 lakhs - 12% of 50 Q.67.(1) Sum of Import and Export of INDIA in 2016
lakhs=240000
= 430
Q.62.(1) Required answer (115% of 23% + 88% of 12%)
Sum of Import and Export of  INDIA in 2017
of 56 lakhs=2072560
= 360
Q.63.(5) Required answer
Required difference = 430 – 360= 70
18
(20-8) × =43.20 Q.68.(3) Import of CHINA in 2017 = 256
5
Q.64.(2) Export of CHINA in 2017 = 60
Country Import Export Required percentage
2016 2017 2016 2017 256 − 60
= ×100% ≈ 327
Pakistan 270 250 240 360 60
Bangladesh 156 164 264 242
China 296 256 100 60 Q.69.(3)
Bhutan 220 242 144 152 Boys Girls
India 150 160 280 200 Section
Science Art Science Art
Import of China in 2017= 256
A 80 55 40 70
Export of Bhutan in 2017= 152
Required ratio = 32:19 B 60 40 45 50

Q.65.(3) Imports of all the countries together in 2016 C 48 50 36 30


= 1092 Required answer = 70+50+30 = 150
Exports of all the countries together in 2016 Q.70.(2) Required answer=195
= 1028 Q.71.(3) Required answer=80-50=30
Required percentage Q.72.(1) Required answer
1092
= × 100 = 106.22 135 −110
1028 =  ×100% = 22.7%
110
25 Q.73.(2) Required answer=150:121
Q.66.(1) Import of Pakistan in 2017 = 270 250
27

QUANTITATIVE APTITUDE 183


www.mahendras.org

CHAPTER

21 MENSURATION
Regular Ploygon :- A polygon that has all sides and all
Mensuration
interior angles equal.
Mensuration is the branch of mathematics which deals with
the study of different geometrical shapes, their area and Convex Polygon :- A convex polygon is defined as a polygon
volume. In the broadest sense, it is all about the process of with all its interior angle less than 180o.
measurement. It is based on the use of algebraic equations B
and geometric calculations to provide measurement data
regarding the width, depth and volume of a given object or A C
group of objects. While the measurement results obtained by
the use of mensuration are usually considered very accurate. D
There are two types of geometric shapes :
Concave Polygon :- A polygon that has one or more interior
1. 2D : Those figures have two dimension is called 2D figures. angle greater than 180o.
like- square, rectangle, parallelogram, rhombus,
triangle, trapezoid, circle etc. D C
2. 3D : Those figures have three dimension are called
3D figures. like- cube, rectangular prism (cuboid),
cylinder, cone, sphere, hemisphere, prism, pyramid A B
etc.
Properties of polygon :-
Polygon :- A polygon is a closed, plane figure bounded by
‘n’ straight lines (n > 3). Each of the n line segments forming (n = number of sides in polygon)
the polygon is called its sides. 1. Sum of interior and exterior angle is 180o.
C
2. Sum of interior angles of polygon is (n–2)×180o.
B D
3. Sum of the exterior angles of polygon is 360o.
4. Each interior angle of regular polygon is
A E
Type of polygon No. of side (n − 2 ) × 180
Triangle 3 n
Quadrilateral 4 5. Each exterior angle of regular polygon
Pentagon 5
Hexagon 6 3600
=
Heptagon 7 n
6. Number of diagonals of a polygon
Octagon 8
Nonagon 9 n (n − 3 )
=
Decagon 10 2

184 QUANTITATIVE APTITUDE


www.mahendras.org

[Plane Figures]
Plane figures are flat two dimenstional shape. A plane figure can be made of straight lines, curved lines, or both straight and
curved lines. Ex. Square, Rectangle and Triangle etc.
In mathematics there are many plane figures which is listed below along with formulas.

S. NO. Name Figure Nomenclature Area Perimeter


l → length
Rectangle b → breadth l×b = lb 2l+2b = 2(l+b)
1. d b d → diagonal
l 2 2
d = l +b
a
2. Square a → side (i) a×a = a2 a+a+a+a = 4a
a d → diagonal d2
a d d = a 2

(ii)
2
a
1
3. Triangle a, b and c are three (i) 2 ×b×h a+b+c = 2s
(Scalene) sides of triangle and
a c s is semiperimeter, (ii) s (s − a)(s − b)(s − c )
h
where (Heron’s formula)
 a + b + c
b s =  2

b is the base and h
is the altitutde of tri-
angle
h = height or altitude

Equilateral a → equal 1 3a
4. (i) ×a×h
triangle sides 2
h → height or
a h a altitude 3 2
(ii) a
3 4
h = a
a 2


Isosceles a → equal 2a+b
5. 1
triangle sides (i) ×b×h
2
b → base
a a h → height or 1
× b × 4a2 − b2
h (ii) 4
altitude
4a2 − b2
b h =
2
Right angled b → base 1 b+h+a
6. ×b×a
triangle a → altitude/height 2
a h
h = h y p o t e n u s e
900 h = a2 + b2
b

QUANTITATIVE APTITUDE 185


www.mahendras.org
S. NO. Name Figure Nomenclature Area Perimeter

a → equal sides 2
7. Isosceles right angled a b
b → other side 1 2 b 2a + b
triangle a =
b = a 2 2 4
900
a

D C
AC is the diagonal and
8. Quadrilateral h1 h1, h2 are the altitudes on AB+BC+CD+AD
1
h2 AC from the vertices D × AC × (h1 + h2 )
and B respectively. 2
A B

9. Parallelogram a and b are side adjacent


to each other. 2(a+b)
h→ distance between a×h
the parallel sides.

a
10. Rhombus D C a- length of each side of
d1 900 rhombus
d1 and d2 are the diago- 1
a 900 900 a × d1 × d2 4a or 2 d12 + d22
nals
d2 90
0 2
d1 → BD
A a B d2 → AC
1 2 2
a= d1 + d2
2

11. Trapezium D b C
a and b are parallel sides  a + b  × h AB+BC+CD+AD
to each other and h is the  2 
h perpendicular distance
between parallel sides.
A a B

a
12. Regular hexagon a 3 3 2 6a
a
a
a → length of each side 2
a a
a

a
13. Regular
a
a
a a → length of each side (
2a2 1 + 2 ) 8a
octagon
a a
a a

14. Circle
πr2
r → radius of the circle 2πr (called as cir-
cumference)

186 QUANTITATIVE APTITUDE


www.mahendras.org
S. NO. Name Figure Nomenclature Area Perimeter

15. Semicircle 1 2 36
r → radius of the circle 2 πr r (π+2) = r
7

16. Quadrant r → radius 1/4 πr2 1 25


πr+2r = r
2 7

17. Ring or circlular path (shaded


region) R→ outer
R radius
π(R2 – r2) (outer) →2πR
r →inner radius
r (inner) →2πr

18. Sector of a circle O→ centre of the


circle
r → radius  θ 
O
l→ length of the arc πr2  360o  l+2r
θ
r θ→ angle of the sec-
tor
A B  θ 
l l → 2πr 
 360o 

19. Segment of a circle θ → angle of the sec-


tor
O r → radius Area of segment
r
 AB → chord ACB (minor seg- 2r  πθ + sin  θ  
 360o   
A B ACB → arc of the ment)  2 
C circle  πθ sin θ 
= r2  360o − 2 

20. Pathways running parallel l l → length


inside to sides of a rectanlge W b → breadth 2(l+b) - 4w
(l+b – w)w
W
w → width of the path
W
(road)
b
W

21. Outer path W


l → length
l
(l+b+2w) 2w (inner) →2 (l+b)
bW
b → breadth (outer) →2 (l+b+4w)
w → width of the path

W
22. Inner path l → length
W
b b → breadth (Other)→2(l+b)
w → width of the path (l+b–2w) 2w (inner)→2(l+b-4w)
l

QUANTITATIVE APTITUDE 187


www.mahendras.org
Solid Figures
Solid figures are three dimensional objects what this mean is that solid figures which have a length a width and a height (depth).
Ex. Computer, Laptop, phone etc.
In mathematics there are many solid figures among few are is listed below along with formulas.
S. NO. Name Figure Nomenclature Volume Curved/ Lateral Total surface
surface area area
1. Cuboid l → length lbh 2(l+b)h 2(lb+bh+hl)
h
b → breadth
b
l h → height

a
2. Cube a→ e d g e / s i d e a3 4a2 6a2
a
a
r

3. Right circular h r→ radius of πr2h 2πrh 2πr(h+r)


cylinder base
h→ height of
the cylinder

l h r→ radius
4. Right circular 1 2 πrl πr(l+r)
h→ height πr h
cone 3
r
l → slant height
l = r 2 + h2
r
r → smaller radiusπ 2
(r +Rr+R2) π(r+R)l lateral sur-
Frustum of a h l R→ layer radius 3
5. face area +
cone l→ slant height π[R2+r2]
h→ height
R
l= ( R − r ) + h2
2

4 3
6. Sphere r→ radius πr 4πr2 4πr2
r 3

7. Hemisphere r 2 3
2πr2
r→ radius πr 3πr2
3

4 inner = 4πr2 4π[R2 + r2]


r→inner radius
8. Spherical shell π[R3 – r3] outer= 4πR 2
R r R→ outer radius 3

188 QUANTITATIVE APTITUDE


www.mahendras.org

area of base × perimeter of lateral surface


h—height
height base × height area + 2(area
9. Right triangular of base)
prism

1 lateral surface
10. Right pyramid ×area of 1
Slant 3 ×perimeter area + area of
height
2 base
base × height of base × slant

height

3 ( side)
1 2
(h ) = × 6 × side 3 3 2
(side)
1
× 2 × (side)
3
3
12
11. Tetrahedron 1 4
r= × 6 × side
12
1
R= × 6 × side
4
(Where h height, r
inradius R circum-
radius)

Ex. A cylinder whose base of circumference is 6 m can Sol. 4 a = 2 (l+b) = 2 (14+20) = 68


roll at a rate of 3 rounds per second. How much
a = 17 = r
distance will the cylinder cover in 9 seconds?
1 22
Sol. Distance covered in one round = 2πr = 6 m A= × × 17 × 17 = 454.14
2 7
Distance covered in 1 second = 3 × 6 = 18 m
Ex. What is the minimum amount of cardboard required
So distance covered in 9 seconds = 18×9= 162 m
to make a closed box of dimension 26 cm × 20 cm
Ex. If a square, circle and rectangle has same perimeter × 7 cm?
then which one of them has the maximum area?
Sol. Area of required cardboard
Sol. In such case the area in descending order is: Circle>
Square> Rectangle = 2 [26 × 20 + 26 × 7 + 20 × 7]

Ex. The perimeter of a square is equal to the perimeter = 2 (520+ 182 + 140)
of a rectangle of length 14 cm and breadth 20 cm. = 2 × 842
Find the area of a semicircle (approx.) whose radius
is equal to the side of the square. (in sq.cm.) = 1684 cm2

QUANTITATIVE APTITUDE 189


www.mahendras.org
EXERCISE
Q.1. What will be the area of trapezium whose parallel sides are 22 cm and 16 cm long, and the distance between them
is 11 cm?
(1) 209 cm2 (2) 282 cm2 (3) 265 cm2 (4) 179 cm2 (5) 302 cm2
Q.2. The perimeter of a rectangle is 42 m. If the area of the square formed on the diagonal of the rectangle as its side is
1
1 % more than the area of the rectangle, find the longer side of the rectangle.
12
(1) 19 m (2) 16 m (3) 9 m (4) 5 m (5) 12 m
Q.3. At the rate of Rs. 2 per sq m, cost of painting a rectangular floor is Rs 5760. If the length of the floor is 80% more
than its breadth, then what is the length of the floor?
(1) 25 m (2) 72 m (3) 67 m (4) 56 m (5) 46 m
Q.4. A 7 m wide path is to be made around a circular garden having a diameter of 7 m. What will be the area of the path
in square metre?
(1) 298 (2) 256 (3) 308 (4) 365 (5) 387
Q.5. The perimeter of a rectangle of length 62 cm and breadth 50 cm is four times perimeter of a square. What will be
the circumference of a semicircle whose diameter is equal to the side of the given square?
(1) 36 cm (2) 25 cm (3) 29 cm (4) 17 cm (5) 16 cm
Q.6. If the four equal circles of diameter is 6 meter touches its adjacent circle, then find the area of the region bounded
by the four circles.
(1) (35 - 9π) m2 (2) (36 - 8π) m2 (3) (36 - 9π) m2 (4) (25 - 9π) m2 (5) None of these
Q.7. The respective ratio of curved surface area and total surface area of a cylinder is 4:5. If the curved surface area of
the cylinder is 1232 cm2. What is the height?
(1) 14 cm (2) 28 cm (3) 7 cm (4) 56 cm (5) 24 cm
Q.8. A rectangular sheet of paper, when halved by folding it at mid-point of its longer side, results in a rectangle, whose
longer and shorter sides are in the same proportion as the longer and shorter sides of the original rectangle. If the
shorter side of the original rectangle is 2 unit, what is the area of the smaller rectangle?
(1) 1 2 sq. unit (2) 2 2 sq. unit (3) 4 2 sq. unit (4) Data inadequate (5) None of these
Q.9. Perimeter of a square and rectangle is equal. If length and breadth of rectangle are 12 cm and 10 cm respectively.
The area of square is what % more than that of rectangle?
(1) 5/6 % (2) 3/4 % (3) 2/3 % (4) 1% (5) None of these
Q.10. What is the volume of a cylinder whose curved surface area is 1408 cm2 and height is 16 cm?
(1) 7715 cm3 (2) 9340 cm3 (3) 8722 cm3 (4) 7346 cm3 (5) 9856 cm3
Q.11. The radii of two cylinders are in the ratio 3 : 2 and their curved surface areas are in the ratio 3 : 5. What is the ratio
of their volumes?
(1) 8 : 11 (2) 5 : 9 (3) 7 : 4 (4) 9 : 10 (5) 13 : 7
Q.12. A square park has a 2 m wide cross road in middle of it. If the side of park is 100 m then find the remaining area
of the park.
(1) 9650 m2 (2) 9596 m2 (3) 9600 m2 (4) 9604 m2 (5) None of these
Q.13. A car has wheels of diameter 70 m. How many revolutions can the wheel complete in 20 minutes if the car is
travelling at a speed of 110 m/s?
(1) 550 (2) 580 (3) 630 (4) 640 (5) 600
Q.14. A clock has its minute hand of length 7 cm. What area will it swept in covering 10 minutes?
(1) 32.17 cm2 (2) 25.67 cm2 (3) 45.45 cm2 (4) 41.23 cm2 (5) None of these
Q.15. The height of the cone is 24 cm and radius of cone is 7 cm . Find its volume.
2

190 QUANTITATIVE APTITUDE


www.mahendras.org
(1) 1200 cm3 (2) 1232 cm3 (3) 1240 cm3 (4) 1260 cm3 (5) 1262 cm3
Q.16. If the radius of a sphere increases by 4 cm then the surface area increases by 704 cm2 . The radius of the sphere
initially was?
(1) 5 (2) 4 (3) 6 (4) 8 (5) 10
Q.17. If the perimeter of square, circle, rectangle are equal. Then whose area is largest?
(1) Circle (2) Square (3) Rectangle
(4) All are equal (5) Can not determined
Q.18. A rectangular plot of grass is 50 m long and 40 m broad. From the center of each side a path of 3 m wide goes
across the center of the opposite side. Find the area of path?
(1) 270 (2) 280 (3) 251 (4) 261 (5) None of these
Q.19. The circumference of a circular garden is 1320 m.Find the area. Outside the garden, a road of 2 m width runs
around it. What is the area of this road and calculate the cost of gravelling it at the rate of 50 paise per sq. m .
(1) 2500.15 m2, Rs.1500.15 (2) 2652.57 m2, Rs.1326.285 (3) 2541.14 m2, Rs.1600.47
(4) 3245.78 m2, Rs.2000 (5) 4157.12 m2, Rs.1452.11
Q.20. A square shape of park of area 23,104 sq. m is to be enclosed with wire placed at heights 1,2,3,4 m above the
ground . Find required length of the wire ,,if its length required for each circuit is 10% greater than the perimeter
of the field ?
(1) 2675.2 m (2) 2145.12 m (3) 2750 m (4) 2478.11 m (5) 2400.5 m
Q.21. If the radius of the circular field is equal to the side of a square field .If the difference between the area of the
circular field and area of the square field is 5145 sq. m ,then calculate the perimeter of the circular field?
(1) 421 m (2) 315 m (3) 310 m (4) 308 m (5) 300 m
Q.22. A rectangular plot has a concrete path running in the middle of the plot parallel to the parallel to the breadth of the
plot. The rest of the plot is used as a lawn ,which has an area of 240sq. m. If the width of the path is 3m and the
length of the plot is greater than its breadth by 2m ,what is the area of the rectangular plot(in m )?
(1) 410 m (2) 288 m (3) 250 m (4) 300 m (5) 320 m
1
Q.23. A rectangular tank of length 37 m internally , 12 m in breadth and 8 m in depth is full of water.Find the weight
3 one cubic metre of water weighs 1000kg.
of water in metric tons, given that
(1) 3584 metric tons (2) 4500 metric tons (3) 4101 metric tons (4) 3870 metric tons (5) 5721 metric tons
Q.24. The length and the breadth of a rectangular table are increased by 1 m each and due to this the area of the table
increased by 27 sq. m. But if the length is increased by 1 m and breadth decreased by 1 m, area is decreased by 7
sq. m. Find the perimeter of the table.
(1) 45 m (2) 52 m (3) 60 m (4) 72 m (5) None of these
Q.25. The perimeter of a square is twice the perimeter of a rectangle. If the perimeter of a square is 140cms and the
length of the rectangle is 20cm. Find the breadth of the rectangle?
(1) 18 (2) 20 (3) 15 (4) 12 (5) None of these
Q.26. A room is 7.5 m long, 5.5 m broad and 5 m high. What will be the expenditure in covering the walls by paper 40
cm broad at the rate of 80 paise per metre ?
(1) Rs. 255.5 (2) Rs. 260 (3) Rs. 282.25 (4) Rs. 244 (5) None of these
2
Q.27. Total area of a circle and a square is equal to 6350 m . The radius of a circle is 35 m. What is the difference of the
circumference of the circle and area of square ? 
(1) 2260 m2 (2) 1280 m2 (3) 3280m2 (4) 2180 m2 (5) 2280 m2
Q.28. If the length of a rectangular plot is increased by 20% and the breadth of the plot is reduced by 20%, the area of
the plot is decreased by 8 m.2. What is the original area of the rectangular plot?
(1) 184 m.2 (2) 196 m.2 (3) 200 m.2 (4) 225 m.2 (5) None of these

QUANTITATIVE APTITUDE 191


www.mahendras.org
Q.29. The ratio between the radius and height of a cone is 3:4.What is the curved surface area of the cone?
(1) 16 m2 (2) 15 m2 (3) 12 m2 (4) Data inadequate (5) None of these
Q.30. If the perimeter of a circle is increased by 30%. What will be the effect on the area of the circle?
(1) 48% increase (2) 52% increase (3) 50% decrease (4) 69% increase (5) None of these

EXERCISE Explanation
Q.1.(1) Area of a trapezium = 1/2 (sum of parallel sides) So a = 14
× (perpendicular distance between them) Diameter, d of the semicircle = 14 cm
1
= ( 22 + 16) × 11 = 209 cm2 Radius = 7 cm.
2
Circumference of the semicircle=πr + 2r= r(π+2)
Q.2.(5) Let the sides of the rectangle be l and b 22
respectively. 7( + 2 ) = 36 cm
7
From the given data, Q.6.(3)
 1

( ) = 1 + 112  lb
2
l2 + b 2
3m 3m

 13 
=> l2 + b2 = 1 +  lb ............(1)
12
l+b = 21 .................(2)
3m 3m
On solving above equations we get

lb = 108
Again l + b = 21 Area of shaded region = 6×6 – π (3)2
Hence longer side = 12 m Area of shaded region = (36 - 9π) m2
Q.3.(2) Let the length and the breadth of the floor be l m Q.7.(2) 4x = curved surface area = 1232
and b m respectively.
x = 308
l = b + 80% of b = l + 0.8 b = 1.8b
5760 5x = total surface area = 1540
Area of the floor = = 2880 sq m
2 curved surface area = 2πrh
l×b = 2880 total surface area = 2πr(r + h)

i.e., l ×
l 2πr(r + h) = 1540
= 2880
1.8 2πr2 + 2πrh = 1540
l = 72 2πr2 = 1540 – 1232
Q.4.(3) Area of the path = Area of the outer circle – Area r = 7; h = 28
of the inner circle
Q.8.(2) According to the question,
2 2
7  7 2x : 2 : : 2 : x
= π  + 7 − π   = 308 sq m
2  2 x= 2
Q.5.(1) Let the side of the square be a cm. Perimeter of Area of the smaller rectangle
the rectangle = 2(62 + 50)
= 2 × 2 = 2 2 = 2 sq. unit
= 224 cm
Perimeter of the square = 56 cm i.e. Q.9.(1) Perimeter of rectangle= 2(l + b) = 22 × 2 = 44
4a = 56 Perimeter of square= perimeter of rectangle
192 QUANTITATIVE APTITUDE
www.mahendras.org
= 44 Q.17.(1) Circle
Side of square= 11 Q.18.(4)
50 m
Area of square= 121
Area of rectangle= 12 × 10 = 120
 121 − 100  5
Required% =  120  × 100 = 6 % 3m 40 m
Q.10.(5) 2πrh = 1408, h = 16
so r = 14
Volume=π r2h =(22/7) × 14 × 14 × 16= 9856 cm3
area of road = 3×50 + 3×40 −3² = 270 −9=261
r1 3 3
Q.11.(4) = or 1= × rr22 Q.19.(2) Circumference of the garden = 2×p×R = 1320
r2 2 2
R= 210m
Again
Outer radius = 210 +2= 212 m
CSA1 2π r1h1 3
= Area of the road = p×(212)2-p×(210)2
CSA2 2π r2 h2 =
5 = p×422×2 = 2652.57 m2
So h1/h2 = 2/5 Therefore , cost of gravelling
Volume1/ Volume2 = 2652.57 × 0.5 = Rs.1326.285
= πr12h1/ πr22h2 Q.20.(1) Perimeter = 23,104 × 4 = (152 × 4) m
= 9/10 152  110 
Length of each circuit = × 
Q.12.(4) Remaining area of the park 4  100 
= 100×100-(100×2+100×2-2×2) =9604 cm2 The wire goes around 4 times ,so the total length
110 × 20 × 60 of the wire required
Q.13.(5) Number of revolutions= = 600
22  110 
70 ×
= 152 × 4 ×  × 4 = 2675.2 m
7
 100 
Q.14.(2) Length will be the radius,
so r = 7cm Q.21.(4) Let the radius of the circular field and the side of
the square field be r Then,
Minute hand covers 360o in 60 minutes So in 10
minutes it covers = 60o π × r 2 − r 2 = 5145
θ => r 2  ( 22 − 7 )  = 5145

r= × πr2  7 
360
=> r = 49 m

So area covered = 60 × 22 × 7 × 7 = 25.67 cm2 Therefore , circumference of the circular field
360 7
= 2×p×r = 308m
Q.22.(2) Let width be x m and length be (x+2)m
1 2
Q.15.(2) Volume= π×r × h Area of path = 3x sq. m
3
x(x+2) – 3x = 240
1 22
× × 7 × 7 × 24 = 1232cm3 => x2 – x – 240 = 0
3 7
=> x(x – 16) +15 (x – 16) = 0

=>(x – 16)(x + 15) = 0
Q.16.(1) 4π (r + 4)2 − r 2  = 704
=>x = 16
On solving above equation
Length = 16 + 2 = 18m
r = 5cm
QUANTITATIVE APTITUDE 193
www.mahendras.org
Therefore , Area of plot = 16 × 18 = 288sq. m ⇒ Cost of paper at 80 paise per meter
 1 3 = 325 × 0.80 = Rs.260
Q.23.(1) Volume of water = 37   × 12 × 8m
3 Q.27.(5) πr 2 + a 2 = 6350
 112 
Weight of water =   × 12 × 8 × 1000 22
× (35) + a 2 = 6350
2
3
7
= 3584metric tons. a 2 = 6350 − 3850
Q.24.(2) Let original length = l a = 2500
breadth = b, a = 50 meter
So area = lb
Required difference = a2- 2pr
When l and b increased by 1: (l+1)(b+1)
= lb + 27 22
2500 - 2 × ×35
Solve, l + b = 26 7
= 2280
When l increased by 1, b decreased by 1: (l+1)
(b-1) = lb – 7 Q.28.(3) Let original length and breadth be l and b
Solve, l – b = 6 respectively.
Now solve both equations, Original area = l × b
l = 16, b = 10 New area = 1.2 l × 0.8 b
Perimeter = 2(16+10)=52m
= 0.96 l × b
Q.25.(3) 4 a = 2 × 2(l+b)
According to question
140 = 4 (20+b)
60 = 4 b 0.04 l × b = 8
b = 15 cm l × b = 8 ÷ 0.04
Q.26.(2) Area of four walls = 200 m.2
= 2 × 5 (7.5 + 5.5) = 130 m 2
Q.29.(4) Data inadequate
Area of required paper = 130 m2
30 × 30
Breadth of the paper = 40 cm = 0.4 m Q.30.(4) Req.% = 30+30+
100
⇒ Length of the paper = 69% increase
=130/0.4= 325 m
Notes

194 QUANTITATIVE APTITUDE

You might also like